Sei sulla pagina 1di 402

\ {~.

:\.... (
. ,_______ ---
r-----.,....-"( ---
'.J-
I ,...-/
/'/ u···\
',.t.-
o

Otolaryngology Clinical Case Studies:


Oral Board Exam Review
~V\C\~
Peter S. Roland, MD
Professor and Chairman
Otolaryngology-Head and Neck Surgery
~
Professor Neurological Surgery
Chief of Pediatric Otology
UT Southwestern Medical Center

With contributing authors:


Robert Adelson, MD -David Osterhus, MD
Amy Brenski, MD Lance Oxford, MD
Orval Brown, MD David Parry, MD
C. Spencer Cochran, MD Allison Pontius, MD
Debbie Eaton, MD Daniel Ratcliff, MD
Romaine Johnson, MD Ravi N. Samy, MD
Joseph Leach, MD Barbara Schultz, MD
Amber Luong, MD, PhD Robert J. Sinard, MD
Michelle Marcincuk, MD Jesse E. Smith, MD
John McClay, MD Bryan Tigner, MD
Neelesh Mehendale, MD John M. Truelson, MD
Brett Miles, DDS, MD D.J. Verret, MD
Alan Murray, MD Joseph R. Williams, MD
Larry L. Myers, MD Michael Yium, MD

Anadem Publishing , Inc.


3620 North High StreeUPO Box 14385
Columbus, OH 43214
800-633-0055/614-262-2539/Fax: 614-262-6630
anadem@anadem.com/www.anadem.com
Using This Book

This book is based upon information from sources believed to be reliable. In


developing this book the publisher, author, contributors , reviewers and editors
have made substantial efforts to make sure that the regimens, drugs and
treatments are correct and are in accordance with currently accepted standards.
The material in this book is presented to professionals for educational and
informational purposes and is not meant to be, and should not be relied upon for,
recommendations regarding diagnosis or treatment for any actual cases. Further,
in compiling a resource such as this there is the possibility of human error. The
publisher, author, contributors, reviewers and editors do not guarantee success
on any examination and disclaim any liability, loss or damage as a result, directly
or indirectly, from using or applying any of the contents of this book.

Publisher's Note

The publisher would like to thank Pam Henderson from the Department of
Otolaryngology-Head and Neck Surgery at the University of Texas Southwestern
Medical Center for her efforts in working with the contributors and our staff to
develop the contents of this book into the final published edition.

Copyright © 2007 by Anadem Publishing , Inc. All rights reserved , including


translation. This book is protected by copyright. No part of this book may be
reproduced or transmitted in any form or by any means, electronic or mechanical,
including photocopying, or by any information storage or retrieval system without
prior written permission from copyright owner.

ISBN: 1-890018-52-X

ii
Otolaryngology Clinical Case Studies:
Oral Board Exam Review

Table of Contents

Abbreviations ... ...... .................... .... ........ ...................... ix

Chapter 1 - Otology

~ 1. Congenital Sensorineural Hearing Loss ............ ..... 1


(Amy Brenski , MD)

"'-.} 2. Aural Atresia with Microtia ......... ... ...................... ...... 6


(RaviN. Samy, MD, FACS)

~'-.1 3. Conductive Hearing Loss .. ................................... . 14


(D.J. Verret, MD)

···~ 4. Sudden Sensorineural Hearing Loss ... .... ......... ... : 21


(David Osterhus, MD)

' ,.J 5. BPPV ... : .................. .... ..... .............. ........ ... ...... ... ..... 25
(Debbie Eaton, MD)

·"'--16. Memere
· · · ·s D.1sease ................ ................................ . 29
(Debbie Eaton, MD)

··,··J 7. Cholesteatoma ................. ........................... ...... .... 34


(David Osterhus , MD)

'"-l 8. Petrous Apicitis ..................... ................... .............. 39


(Peter S. Roland, MD)

'-'9. Temporal Bone Osteomyelitis ...... .......................... 42


(David Osterhus, MD) ·

""'v 10. Paragangliomas of the Temporal Bone .. .............. 47


(RaviN. Samy, MD, FACS)

·".J11. Temporal Bone Fracture .......................... ............. 52


(David Osterhus, MD)

~ 12. Bell's Palsy ....... ........ ...... ............ .......... ................. 57


(Neelesh Mehendale, MD)

"'-J 13. Chronic Suppurative Otitis Media ........... ........ ...... 64


(David Parry, MD)

14. Otitis Media with Meningitis ........................ ........... 70


(Robert T. Adelson, MD)

15. Tympanic Membrane Perforation ...... ... ... ... ........... 79


(D.J. Verret, MD)

16. Autoimmune Inner Ear Disease ............................ 86


(Joseph R. Williams, MD)
iii
Chapter 2 - Head and Neck

1. Nasopharyngeal Carcinoma ............................... ... 91


(Robert J. Sinard , MD)

2. Tonsil Carcinoma ...................... ............................. 94


(Robert J. Sinard, MD)

3. Carcinoma of the Larynx .................... ... .. .............. 97


(Larry L. Myers, MD)

4. Floor of Mouth Carcinoma ............................... .... 102


(John M. Truelson , MD)

5. Salivary Gland Mass ....... ..................................... 105


(Robert J. Sinard, MD)

6. Para pharyngeal Mass .... .... ................................. 108


(Michelle Marcincuk, MD)

7. Angiofibroma ................................ .. ....................... 11 5


(Robert T. Adelson, MD)

Chapter 3 - General Otolaryngology

1. Chronic Middle Ear Effusion .................. .............. 123


(RaviN. Samy, MD, FACS)

2. Unilateral Nasal Mass .......................................... 127


(Amber Luong, MD, PhD)

3. Dysphagia - Pharynx (Zenker's) .............. ....... ... 134


(Neelesh Mehendale, MD)

4. Dysphagia ................................ ............................ 138


(Robert J. Sinard, MD)

5. Epistaxis ........ ................. .......... ............................ 141


(Michelle Marcincuk, MD)

6. Epiglottis ......... ................... ............... ................... 145


(Amy Brenski, MD)

7. Obstructive Sleep Apnea .................................. ... 149


(John M. Truelson, MD)

8. Mandibular Fracture ................... ......................... 153


(Jesse E. Smith, MD)

9. Maxillary Fracture ...................................... ........ .. 158


(Jesse E. Smith , MD)

10. Nasal Fracture ..................................................... 163


(Robert T. Adelson, MD)

11. Orbital Floor Fracture ..... .... ............. ........ ........... 168
(Jesse E. Smith, MD) ·

iv
12. Penetrating Neck Trauma ................................... 173
(Michelle Marcincuk, MD)

13. Blunt Neck Trauma ..... .... ..................................... 178


(Jesse E. Smith, MD)

14. Sialolithiasis ......................................................... 182


(Robert T. Adelson, MD)

15. Transection of Stenson's Duct.. .......................... 188


(Robert T. Adelson, MD)

16. Ludwig's Angina with Tracheotomy .... ................. 193


(Larry L. Myers, MD)

17. Necrotizing F asciitis of the Neck ......................... 196


(Barbara A. Schultz, MD)

Chapter 4 - Laryngology

1. Unilateral Vocal Cord Paralysis ........................... 201


(Daniel Ratcliff, MD)

2. Vocal Cord Nodule ............................................... 204


(Michael Yium, MD)

3. Spasmodic Dysphonia ......................................... 208


(Daniel Ratcliff, MD)

4. Laryngotracheal Trauma ...................................... 211


(Alan D. Murray, MD)

5. Acquired Subglottic Stenosis ............................... 216


(Larry L. Myers, MD)

Chapter 5 - Pediatric

1. Stridor .................................................................. 22 1
(Romaine F. Johnson, MD)

""2. Subglottic Stenosis .............................................. 224


(Alan D. Murray, MD)

3. Foreign Body in the Airway ................................. 229


(Alan D. Murray, MD)

4 . Craniofacial Anomaly ........................................... 233


(Brett A. Miles, DDS, MD and John E. McClay, MD)

5. Lymphatic Malformation/Lymphangioma ............. 243


(Brett A. Miles, DDS, MD and John E. McClay, MD)

6. Tracheoesophageal Fistula ...... ............. ........ ..... 251


(John E. McClay, MD)

v
7. Tracheomalacia ...... .... .......... ............................... 258
(Romaine F. Johnson, MD)

8. Juvenile Papilloma .............................. ...... ........... 262


(Michelle Marcincuk, MD)

~ 9. Choana! Atresia ......................... .......................... 267


(Brett A. Miles, DDS, MD and Orval Brown, MD)

Chapter 6 - Rhinology

1. Recurrent Sinusitis ............ .................................. 273


(Amber Luong, MD, Ph.D)

2. Chronic Sinusitis .................................................. 279


(Amber Luong , MD, Ph.D)

3. Sphenoid Sinusitis ............................................... 286


(Daniel Ratcliff, MD)

4. Acute Frontal Sinusitis ......................................... 290


(Amber Luong, MD, Ph.D)

· 5. Cerebrospinal Fluid Rhinorrhea ......................... 296


(Neelesh Mehendale, MD)

6. Maxillary Squamous Cell Carcinoma ................... 301


(Lance Oxford , MD)

7. Ethmoid Adenocarcinoma .................................... 305


(Lance Oxford, MD)

8. Wegener's Granulomatosis ................................ 309


(C. Spencer Cochran, MD)

9. Allergic Fungal Rhinosinusitis ............. ................ 312


(Amber Luong, MD, Ph.D)

10. Invasive Fungal Rh inosinusitis ............................ 319


(Amber Luong, MD, Ph.D)

11. Allergic Rhinosinusitis ..................... ..................... 324


(Joseph R. Williams, MD)

12. Vasomotor Rhinitis .............................................. 327


(Joseph R. Williams, MD)

13. Frontal Sinus Fracture ....................... :................ 330


(Joseph Leach, MD)

14. Orbital Abscess (Subperiosteal Orbital Abscess) 334


(C. Spencer Cochran, MD)

15. Exophthalmos from Thyrotoxicosis ..................... 337


(Michael Yium, MD)

vi
16 Nasal Dermoid Sinus Cysts ................................. 343
(Bryan Tigner, MD)

Chapter 7 - Cosmetic Facial Surgery

1. Rhinophyma ...... ......... ... ...................... ... ............. 34 7


(Allison Pontius·. MD)

2. Facial Reanimation ........ ........................ .... .......... 353


(Allison Pontius , MD)

3. Complex Laceration Involving the Eyelid


and Lacrimal System ........................................... 360
(Allison Pontius, MD)

4. Complex Lip Laceration ................. ...................... 368


(Joseph Leach, MD)

5. Blepharoplasty ......... ............................................ 371


(Joseph Leach, MD)

6. Rhytidectomy ................. ............................ ...... .... 375


(Allison Pontius, MD)

7. Nasai .Septal Reconstruction (Sertoplasty) ......... 382


(Joseph Leach, MD)

8. Rhinoplasty .......................................................... 386


(Joseph Leach, MD)

vii
viii
Abbreviations

AAOHNS American Academy of Otolaryngology Head and Neck Surgery


ABG arterial blood gas
ABR auditory brainstem response
AC air conduction
ACE angiotensin-converting enzyme
AD right ear; autosomal dominant
AE Aryepiglottic
AFRS allergic fungal rhinosinusitis
AI ED autoimmune inner ear disease
AJCC American Joint Committee on Cancer
ANA anti-nuclear antibodies
AOM acute otitis media
AP action potential
AR autosomal recessive
AS Left ear
ASCVD atherosclerotic coronary vascular disease
AU both ears

BAHA bone-anchored hearing aid


BC bone conduction
BOT botulinum toxin
BP blood pressure
BPPV benign paroxysmal positional vertigo
BUN blood urea nitrogen

CABG coronary artery bypass graft


C-ANCA antineutrophil cytoplasmic antibody
CAP central auditory processing
CBC complete blood count
CF cystic fibrosis
CFTR cystic fibrosis transmembrane regulator
CHL conductive ·hearing loss
CN cranial nerve
CNS central nervous system
COPD chronic obstructive pulmonary disease
COR coronary
CPA cerebellopontine angle
CPAP continuous positive airway pressure
CPR cardiopulmonary resuscitation
CRS chronic rhinosinusitis
CSF cerebrospinal fluid
CSOM chronic suppurative otitis media
C-spine cervical spine
CT computerized tomography
CVA cerebrovascular accident
CXR chest x-ray

DJD degenerative joint disease


OM diabetes mellitus

ix
DPTE direct puncture tumor embolization

EA esophageal atresia
EAC external auditory canal
EBV Epstein-Barr virus
ECG/EKG electrocardiogram
EMG electromyography
ENOG electroneuronography
EOM eosinophilic otitis media
EOMI extraocular muscles intact
ER emergency room
ESR erythrocyte sedimentation rate
ESWL extracorporeal shock wave lithotripsy
ETOH ethanol

FDL flexible direct laryngogoscopy


FESS functional endoscopic sinus surgery
FH family history
FNA fine needle aspiration
FOM floor of mouth
FT full term
FTA-ABS fluorescent treponema! antibody absorbed test

GA general appearance
GBAT · genial bone advancement trephine
GCS glycoconjugates
GER gastroesophageal reflux
GERD gastroesophageal reflux disease

HIH · hearing handicap


HB House-Brackmann
HCTZ hydrochlorothiazide
HEENT head, eyes, ears, nose, throat
HHT hereditary hemorrhagic telangiectasia
HIS Haemophilus influenzae type B
HIV human immunodeficiency virus
Hosp hospitalization
HPI history of present illness
HR heart rate
HTN hypertension

lAC internal auditory canal


ICA internal carotid artery.
ICU intensive care unit
IMF internal maxillary fixation
INAD inadequate
IV intravenous
IVDA intravenous drug abuse
IVN inferior vestibular neNe

JNA juvenile nasopharyngeal angiofibroma

X
kD kiloOalton
KTP potassium titanyl phosphate

LAD left anterior descending artery


LP lumbar puncture
LPR laryngopharyngeal reflux disease
LR lactated Ringer's solution
LTC laryngotracheal cleft

MOO mandibular distraction osteogenesis


MEE middle ear effusion/middle ear exploration
MOE malignant otitis externa
MR mental retardation
MRA magnetic resonance angiography
MRI magnetic reson ance imaging
MSL microsuspension laryngoscopy

NAD no acute distress


N/C non-contributory
NC/AT normocephalic/atraumatic
NG nasogastric
NICU neonatal intensive care unit
NIDDM noninsulin dependent diabetes mellitus
NKDA no known drug allergies
NKFA no known food allergies
NP nasopharynx
NPC nasopharyngeal carcinoma
NPO nothing by mouth
NSAID non-steroidal anti-inflammatory drug
NSVD normal spontaneous vaginal delivery

oc oral cavity
OCP oral contraceptive pill
OE otitis externa
OME otitis media with effusion
OP oropharynx
OR operating room
OR IF open reduction with internal fixation
OSA obstructive sleep apnea
OSAS obstructive sleep apnea syndrome
OTC over the counter

PACU post-anesthesia care unit


PE physical exam
PERRLA pupils equal, round, reactive to light and accommodation
PET pressure equalizing tube
PLM periodic limb movements
PMH past medical history
POD post-operative day
ppd packs per day
PPI proton pump inhibitor
PSH past surgical history

xi
PTA peritonsillar abscess/pure tone average
PTFE polytetrafluoroethylene (Teflon)
PTT pure tone thresholds
PTU propylthiouracil

RAST rad ioallergosorbent test


RDI respiratory disturbance index
ROS review of systems
RR respiratory rate
RRP recurrent respiratory papilloma
RRR regular rate and rhythm

sec squamous cell carcinoma


SCCA squamous cell carcinoma antigen
SDS same day surgery
SH surgical history
SocHx social history
SLE systemic lupus erythematous
SMAS subcutaneous musculoaponeurotic system
SNHL sensorineural hearing loss

SOF superior orbital fissure


SP summating potential
SRT speech reception threshold
sse superior semicircular canal
SSNHL sudden sensorineural hearing loss
SVN schwannoma of the vagus nerve

TAE transarterial embolization · .


TB mycobacterium tuberculosis
TEF tracheo-esophageal fistula
TIA transient ischemic attack
TM tympanic membrane
TMJ temporomandibular joint
TSH thyroid stimulating hormone
TVC true vocal cord

UA urinalysis
UGI upper gastrointestinal
UICC International Union Against Cancer
UPPP uvulopalatopharyngoplasty
URI upper respiratory infection
us ultrasound
LNA ultraviolet A light
lNB ultraviolet B light

VACTERL vertebral/vascular, anorectal, cardiac, tracheoesophageal ,


radial/renal , limb

WBC white blood count


WDWN well developed, well nourished
WF white female

xii
WM White male
WHO World Health Organization
WNL within normal limits

XRT x-ray therapy

xiii
xiv
Cl1apter 1 - Otology Otolaryngology Clinica l Case Studies

CHAPTER l!CASE 1 - CoNGENITALSENSORINEURAL HEARING Loss NoTES


Amy Brenski, MD

Patienr is an 8 week-old white male infant with a history of a failed infant


hearing screening. His m01her serves as his informant.

HPJ: state that you would:

Obtain a detailed med ical history beginning with a history of present illness

What additional history would yot~ seek fr·om this patient? State that you would
ask about:

Prenatal history:
What is the birth history?
Did morn receive prenatal care?
Were there any complications with the pregnancy?
Was the child FT?
Was there any perinatal hypoxia?
Was the child intubated or ventilated?
~id he require admission to the NICU?

History of present illness:


What type of hearing test did he have?
Has he ever had an ear infection or fluid in his ears?
Did he receive any ototoxic medications or have any infections?
Does he use a pacifier or take a bottle?

Family/social factors:
Has mother ever had a sexually transmitted disease?
Is there any illicit drug use?
What is the home situation?
ls the child in daycare?
Are there smokers in the home or daycare?
Does anyone in the fami ly have a hearing loss?
Did either parent have a history of ear infections as children?
Are there any sibl ings?
Does anyone have a history of sudden death at a young age?
Does anyone have a thyroid condition or goiter?

The child :S mother states that he was born at 39 weeks via urgent C-sec-
tion delive1y due to failure to progress and infant bradycardia. His Apgars
were 7 at 1 minute and 9 at 5 minutes. He was cyanotic at delivery second-
ary to nuchal cord. He responded to stimulation, but did require supple-
mental oxygen for 24 hours. He had slightly decreased tone, but this im-
proved. There was a question of aspiration of meconium at the time of
delivery. He therefore received antibiotics intravenously for 2 days. He
was in the NICU for 3 days and underwent a circumcision prior to dis-
charge home. He had a screening ABR the day of admission, which he
Oto laryngology C linical Ca se Studies Chapter 1 - Otology

NOTES failed in both ears. He has not had any major illnesses or high fe vers nor
has he had any hospiral admissions.

Mothers uncle has a hearing loss that requires a hearing aid since about
30 years of age. Her mother had a goite1: The child's father was adopted.
There are no siblings. The remainder of your questions are answered nega-
lively. Mother repon s that she had pelvic inflammatory disease at the be-
ginning of the pregnancy. She admits to smok11zg marijuana in the past.
but denies illicit drug use during the pregnancy. She does not smoke ciga-
rettes. but her mother •vith whom they live smokes. Mother cares for the
baby herself He is not yet in daycare. He does not use a pac[fier and is
breast fed. His father does not live with them.

Immunizations: Up-to-date.

ROS: His birth weight was 6 pounds 3 ounces. He has gained weight since
he came home. He spits up infrequently He feeds well without tiring. He
snores softly, but does not have apnea. He has no history of intraventricu-
lar hemorrhage, seizures, heart murmur or congenital heart disease. He
does not have bronchopulmonm~v dysplasia or asthma. He seems to re-
spond to sound and looks at faces. He has normal muscular tone.

What would you look for on PE? State that you would:

Examine for facial anomalies or signs of the presence of a syndrome


Otologic examination to assess for MEE, TM integrity, EAC stenosis
. Examine for signs of developmental delay

PE:

Afebrile, HR: 110, Oxygen saturation 99% on room ail; BP


deferred, normal facies, in no distress, sleeping quietly
Ears: Normal, no effusions, no retraction
Nose: Patent bilaterally, no congestion, no discharge
OC/OP : Normal palate and uvula
Neck: No masses or adenopathy
Chest: Clear to auscultation
COR: RRR, no murmur
Extremities: Well perjitsed, normal digiLs, normal palmer creases
Abdomen: Non-tende 1~ no masses

What is your differential diagnosis?

1. Congenital sensorineural hearing loss


2. OME

2
Chapter 1 - Otology Otolaryngology Clinical Case Studies

What diagnostics would you request? State that you would request: NoTES

ABR testing (should provide the most information)


Transient evoked otoacoustic emissions can be helpful as welL document-
ing normal cochlear function at the level of the hair cells
Imaging of the inner ear structures wil l rule out a congenital inner ear mal-
formation. The best method of imaging these structures is MRJ with fast-
spin echo. In children with congenital hearing loss, this test will demonstrate
some abnormality of inner ear structures approximately 20% of the time.
This test provides the highest yield in the evaluation of sensorineural hear-
ing loss. It is often helpful to image the brain and brainstem at the same
time. Large vestibular aqueduct or endolymphatic duct can be associated
with Pendred's syndrome. This may demonstrate other sites of hypoxic
injury to the CNS and lend suppo1i to hypoxia as a cause of hearing loss
Treponema! antibody assessment with either FTA-ABS or syphilis IgG
CBC can be helpful in identifying anemias or lymphopenias. In an older
child this might also rule out a leukemic infiltrate as a cause for hearing loss
.UA should be obtained to rule out hematuria and one ofthe many presenta-
tions of A!port's syndrome
Renal US to evaluate for congenital anomalies of the kidneys, if there are
any abnormalities on UA or MRI
Thyroid function tests, EKG, electrolytes, BUN and creatinine.·In the ab-
sence of other systemic manifestations of hypothyroid ism, this is an ex-
tremely unlikely cause of the hearing loss. However, there is a fam ily his-
tory of goiter, and a family history of hearing loss. EKG will rule out Jervell
and Lange-Neilsen syndrome. Also, with half of the parental history un-
available, one might consider this in the interest of being thorough and ulti-
mately preventing sudden death by V-fibrillation-preventable with B-block-
ing agents
Consider evaluation by an ophthalmologist to determine if there is any
visual impairment, which is sometimes associated with congenital hearing
loss. Electroretinography may be abnonnal prior to any signs of retinitis
pigmentosa on physical exam. This is present in some forms of Usher syn-
drome

Test results:

ABR is pe1jormed after the baby is sedated with chloral hydrate and falls
asleep. It shows bilateral moderate to severe loss for both click and tone-
burst stimuli. There was no response to BCs at the limits of the equipment.
Otoacoustic emissions are absent bilmerally.

Diagnosis:

Bilateral congenital sensorineural hearing loss, moderate to severe. Refer


the child for a trial of amplification.

3
Otolaryngology Clinical Case Studies Chapter 1 - Otology

NoTES What is the most liliely etiology?

The child has a number ofrisk factors for acquired hearing loss:

I . Perinatal hypoxia
'">Bradycardia drug delivery, possibly secondary to hypoxia
3. Perinatal cyanosis
4. Poor muscle tone after del ivery suggestive ofCNS insult
5. Admission to NJCU

During his NICU admission, it is probable that be received ototox ic medica-


tions, as Gentam icin is often used in this age group
He also has a possible history of intrauterine exposure to illicit drugs and
subsequent malformations
He has a second degree.family member with hearing loss requiring amplifi-
cation at a young age, raising the question of a possibl e hereditary etiology
ofthe hearing Joss
There are several facts in his mother's history wh ich suggest possible in-
trauterine exposure to a sexually transm itted disease, such as syphilis

i t is important to inform the child's caregivers that the etiology of senso-


rineural hearing loss remains unkno'vvn in most children. Reassure them
thar you have ruled our many of the treatable causes of hearing loss..

What are the treatment options and their complications? . What_would you do
and why?-

If the child has received ampl ificati on during this interval you can assess if
he is receiving any benefit. This is reassuring to most parents
• The importance of speech and language therapy should be stressed
The parents should be counseled that the child may be at risk for continued
loss of hearing
_The child's hearing should be monitored at least every 6 months
Consider genetic testing. Nearly 1/3 of congenitally deaf chi ldren born to
hearing parents have a mutation in 1 oftheir gap j unction proteins, usually
Connexin 26. The most common mutation in the American population is the
35 del G

it is recommended that the caregivers receive genetic counseling prior to


obtaining these test results. It is important to inform them of their right to
have or not have genetic testing if they choose.

They must also be counseled on whether their child should be told the re-
sults. It is important that they understand that this test does not allow us an
opportunity to treat hearing Joss at this time. There may be some form of
genetic therapy in the future. The significance of these results may not be
apparent at this time. They should be informed of the chances for ha'ving a
second child with hearing loss and for the child passing the trait to his or her
offspring. Some famil ies may opt not to participate

4
Chapter 1 - Otology Otolaryngology Clinical Case Studies

If there are any sti gmata ol cet1ain syndromes. there may be testing avail- NoTES
able to identify a specific chromosomal abnormality. such as Pendred 's syn-
drome or Usher's syndrome. for example, with characteristic mutations

The child reLums IO your office at 11 months of age. He has had 3 epi-
sodes of otiris media in the pasr 6 months. He has acquired no language.
His speech therapist says he has made no progress.

What would you do next?

Recommend bilateral myringotomy and tube placement for control of otitis me-
dia and prevention of additional CHL associated with MEE. He should have his
hearing assessed again

How would you follow this individual~ State that you would :

Re-evaluate every 6 months with audiometry, including aided threshold testing


• If after control of his OME, he still makes no progress with his language ski lis,
he may be a candidate for cochlear implantation. Begin this evaluation

5
Otolaryngology Clinical Case Studies Chapter 1 -Otology

NOTES CASE 2 - AURAL ATRESIA WITH MICROTIA


RaviN. Samy, MD, FACS

A 42 year-old female presents wilh a hislor_r of a "misshapen right ear


since birth. ..

HPI: state that you would:

Obtain a detai led medica l bistory beginning with a history of present illness

What additional historical information would you seek? State that you would
ask about:

History of drainage from the involved ear


Hearing Joss in the misshapen ear
History of otalgia in the involved ear
• History of facial weakness
• History of hearing loss in the uninvolved ear
Sudden
- Fluctuating
Progressive
FH of congenital ear anomalies (e.g., Treacher-Collins, CHARGE, etc.)
History of prior surgeries involving the involved ear
History of the use of hearing aids

The patient stales thar she has a life-long histOJy of hearing loss in the right
e01: She feels thQI the he01·ing loss is swble and-is neither progressive nor
fluctuating. She has never had any ~_rauma w her head or em: She does not
complain of any hearing loss in the left e01: She does not have a history of
noise exposure. She does not have· a history of recurrent ear infections,
otalgia, or owrrhea involving either ear. There is no FH of hearing loss or
congenital ear anomalies. She has not had any type of ear surgery. She has
never used hearing aids. She does not have any drainage from her ea1: She
also denies dizziness. tinnitus, and facial weakness or numbness.

P MH: Obesity, anemia. allergies, urinary tract infections


PSH: None
Allergies: NKDA
Medications: None
FH: HTN, diabetes
SH: The patienT is a secrerary. She is married. She has 1 child.
The patient does not smoke. She rarely drinks. She denies
any HIV risk factors
ROS: Negative

What would you look for on PE? State that you would:

Perform an appropriate head and neck PE focusing upon the fo llowing:


Assessment of fac ial symmetry
CN examination

6
Chapter 1 - Otology Otolaryngology Clinical Case Studies

Ophthalmic evaluation NoTES


Otologic examination
Detailed evaluation of auricular remnant
Relationship of auricle to TMJ (palpation)
Evaluation ofEAC
I! Complete atresia
" Stenosis
Tuning fork examination
State of contralateral ear
Oral examination
Mandibular-maxillofacial relationsbips

PE:

Y.i1.als_: Temp: 98.6 F, HR: 74, Wt: 255 lbs, Ht: 5 '4"
GA: Well-developed, well-nourished f emale, in NAD
Neck: Normal. No lymphadenopathy, parotid masses.
or thyroid masses
Nasal: No masses, polyps, or pus
OCIOP: No leukoplakia, no bleeding, no masses
Ears: Binocular otomicroscopy is pe1jormed. First, the
lefi ear (normal side) is examined. The leji auricle,
ear canal, and TM are all completely norrna/

On the right side, you notice an auricle that is slightly smaller than the leji
and has a mild cup deformity to it, but otherwise looks normal

Figure 1 - Right Ear

7
Otolaryngology Clinical Case Studies Chapter 1 - Otology

NoTES Figure 2 - Left Ear

You notice that the right ear canal is significantly narro·wed: you are barely
able to see the ear drum .

Tuning Fork: Weber lateralizes to the right ear at 256 and 512 Hz. Rinne
was positive in the left ear at both frequencies . Rinne was negative at 256,
512, and 1024Hz but positive at 2048Hz in the right em:

Neurologic examination: All CNs are normal on close examination.. Facial


nerve function is grade 1/VI HB . Corneal reflex is intact. Cerebellar testing
is performed for completeness and is also normal (normal Romberg, nor-
mal Fukuda, normal gait, normal tandem gait, f!Ormal finger-to-nose, nor-
mal heel-to-shin) .

What is your next step in further assessing this condition?

Audiologic testing: The patient has normal hearing in her left ear (SRT 15
dB, 100% discrimination).. On the right side, the patient has a SRT of 85
dB, 80% discrimination.. CHL of 50-60 dB (maximal CHL) noted at all
frequencies . Tympanograms are Type A bilaterally..

8
Chapter 1 -Otology Otolaryngology Clinical Case Studies

Audio Diagram NOTES

What is your differential diagnosis?

1. Congenital aural atresia


2. Other types of congenital ossicular abnormalities
3. Tympanosclerosis (unlikely since no prior history of ear infections)
4. Otosclerosis (unlikely based on history of stable hearing loss and PE)
5. Neoplasm (e.g., facial neuroma impinging on ossicular chain)

What do you do next?

The most important step in diagnosing the abnormality present and confirm-
ing your thought of congenital aural atresia is by obtaining a CT scan. The
scan should be fine cut (1 mm or 0.5 mrn slices), axial and coronal, with
bone windows. Contrast will not be needed
The CT is used to assess:
1. The location and course of the facia l nerve
2. Status of the ossicles
3. Size of the EAC
4. Mastoid and middle ear pneumatization (compare with non-involved
side)
5. Presence of oval and round w indows
6. Status of the cochlea
7. Relationship of middle ear space to glenoid fossa
8. Presence of cholesteatoma
9. Severity of atresia and relationship to ossicular mass and facial nerve
Other abnormalities (e.g., aberrant carotid artery, cochlear dysplasia).
Please see Figures 3A and 3B on page 13

9
Otolaryngology Clinical Case Studies Chapter 1 - Otology

NoTES Diagnosis:

Your clinical diagnosis is confirmed on the CT scan; the patient has con-
genital aural atresia with an abnormal ossicular chain. Jahrsdoe1jer RA
et a!. have published a grading system based on the CT scan to assist in
deciding who would benefit from surgical correction. The maximum pos-
sible score is 10 points. Two points are given for ' 1:e stapes being present;
one additional point is given for each of the following.~ An open oval win-
dow; normal round window; middle ear space present,· normal facial nerve:
malleus-incus connection,· incus-stapes connection; well-pneumatized mas-
toid,· and normal appearing external em~ Patients with a rating of 6 points
or greater are considered to be reasonable surgical candidates.

Discuss options:

Observation- Since the patient has· good hearing in 1 ear and has done well
with I ear for a long time, she may choose to do nothing
Hearing aid -A BAHA can be used. A reasonable option is to trial a bone
conducting hearing aid prior to considering surgical correction. Hearing aids
are very important in children born with bilateral hearing loss (e.g., bilateral
atresia). Such children need bone conduction hearing aids placed as·soon as
possible. Surgically implanted BAHA's are usually not placed before 5 years
of age due to risk of dural or sigmoid sinus tears ---
Congenital aural atresia (atresiaplasty) and microtia repair- The timing of
surgery is important. In this case, since the patient is an adult and has 0!11Y
unilateral atresia, she can undergo surgical correction at any time she wishes.
She must have realistic expectations and realize that this is an elective pro-
cedure. If a child is born with bilateral atresia, correction is typically not
performed before 5 years of age (with microtia repair being perfom1ed
prior to atresia repair)

Surgery for unilateral atresia in children is more controversial; some feel


that surgery should be done only when the patient can consent to the proce-
dure (or give assent to the procedure if a minor), while others believe that
early correction will provide improved hearing and avoid the social stigmata
of microtia and bone-conduction amplification. However, surgery is indi-
cated if the patient develops chronic otorrhea or other signs or symptoms
worrisome for atresia-related cholesteatoma formation. Some patients are
considered poor surgical candidates based upon the results of the CT scan
of the temporal bone (Jahrsdoerfer criteria < 6).

10
Chapter 1 -Otology Otolaryngology Clinical Case Studies

Discuss your surgical technique for atresiaplasty and tympanoplasty: NOTES

As described above, it is important to carefully choose the appropriate cand i-


dates for surgical repair to minimize risks and maximize hearing results. One
also needs close coordination between the otologic surgeon and the facial plas-
tic surgeon to achieve the best possible hearing and cosmesis.

Obtain infom1ed consent. Patient needs to have realistic expectations


Intraoperative facial nerve monitoring is used
Post-auricular incision used. Be careful as one approaches the stylomastoid
foramen to prevent accidental damage to fac ial nerve
Cutting burrs and coarse and fine diamond burrs are used to carefully cre-
ate an EAC and remove the atretic plate. Avoid exposing the TMJ or many
mastoid air cells. Care should be taken to avoid the course of the facial
nerve as this may lead to facial nerve injury. Avoid hitting the ossicular
chain with the bUIT, which can cause sensorineural hearing loss
• The ossicular mass should be assessed after removal of the atretic plate
with respect to its mobility and continuity with the stapes superstructure.
Decision is then made as to whether it is left intact or removed and replaced
with an ossicular prosthesis
Neotympanic membrane is created
Skin grafts are then used to epithelialize the newly fanned EAC

What are the potential complications?

• J Facial nerve injury (I% risk in experienced hands). However, this is a major
concern due to the POTENTIALLY aberrant course of the facial nerve.
Intraoperative facial nerve monitoring is standard of care in this procedure
• 1 Sensorineural hearing loss (2%)
o 3 Dizziness-usually transient

• 4 Taste changes due to injury to chorda tympani nerve-usually resolves in


about 3 months
o J The ~ost common complications are EAC stenosis, lateralization of the

TM, and persistent CHL

How would you follow this individual?

The patient requires close (i.e., every 2 weeks for the first 2-3 months)
follow-up during the immediate postoperative period to perfom1 debride-
ment and make sure the skin graft is healing well
Post-operative audiometry is defen·ed until adequate healing has provided
for a stable examination (i.e., 3-6 months following surgery)
Long-tenn follow-up (i.e., every 1- 2 years) is perfonned to make sure that
hearing is stabilized and to make sure that some of the complications listed
above do not occur

11
Otolaryngology Clinical Case Studies Chapter 1 - Otology

NOTES figure 3A- Axial. non-contrast CT scans demonstrating


right EAC atresia and ossicular mass

Figure 3B

12
Chapter 1 - Otology Otolaryngology Clinical Case Studies

Figure 4- Coronal. non-enhanced CT scan demonstrating ossicu lar abnom1ality as NoTES


well as posterior displacement of condyle of mandible

References :

I. Jahrsdoe1fer RA, et al. Grading system for the selection of patients with
congenital aural atresia. Am J Otol. 1992; 13( 1):6-12.
2. Lambert PR. Congenital aural atresia: stability of surgical results.
La1yngoscope. 1998; 108(1 2):1801-5.
3. Chandrasekhar SS, De Ia Cruz A, Garrido E. Surgery of congenital aural
atresia. Am J Otol. 1995; 16(6):713-7.
4. .Shih L, Crabtree JA. Long-term surgical results for congenital aural
atresia. Laryngoscope. 1993; 103(1 0): 1097- 102.

13
Otolaryngology Clinical Case Studies Chapter 1 - Otology

NoTES CASE 3 - CoNDUCTIVE HEARING Loss


D. J. Verret, MD

A 30 year-old WF presems with a chief complaint of decreased hearing in


her right ear

HPI: state that you would:

Obtain a detailed medical history beginning with a history of present illness

What additional historical information would you seek? State that you would ask:

How long has the patient noticed the hearing loss?


Was the hearing loss abrupt or gradual in onset?
Is the hearing loss persistent or fluctuating?
Are there any associated otologic symptoms:
Vertigo or dizziness?
Aural fullness?
Otalgia?
Tinnitus?
Otorrhea?
Has the patient had a recent upper respiratory tract infection?
Is the patient pregnant or did the hearing loss worsen during the patient's ·
pregnancy?
Is there a personal history of noise (occupational or recreational) exposure,
trauma, previous ear surgery, previous hearing aid use, recent air travel, or
water exposure?
ls there a history of recurrent ear infections as a child?
Is there any hearing loss on the contralateral side?
FH of hearing Joss?

The patient states she has not been hearing well for the last yecn~ She does
not have any vertigo, dizziness, or fluctuation in her hearing. She does not
experience ear pain. She has some mild aural fullness and high pitched
ringing. She denies any otorrhea. She has no history of recurrent ear
infections as a child. Hearing in the contralateral ear is reported as normal.

PMH: N one
PSH: Appendectomy 10 years ago
Allergies: NKDA
Medications: None (Ask about a possible history of ototoxic medication
use, such as antibiotics or antineoplastic agents if one is
concerned about pre-existing hearing loss)
FH: There is no FH of hearing loss
SH: The patient does not drink, smoke, or use illegal drugs. She
works as an accountant and has no noise exposure hist01y
ROS: Negative

14
Chapter 1 - Otology Otolaryngology Clinical Case Studies

W hat would you look for on PE? State that you would: N oTES

Perfonn a thorough head and neck examination and an otoscopic examination

PE:

Temp: 98.6 F. BP: 12.:/183, HR. 76, Pain: None,


Wt: 60kg
GA: The patient is silting in the exam chair in NAD. She
is WDWN

ln doing aPE for hearing loss, a complete head and neck examination should be
performed including examination ofthe neck, OC/OP, and nasal cavities. An
otoscopic examination is perfom1ed, followed by tuning fork examination. The
tuning fork examination should include the Rinne and Weber tests. The degree
of hearing loss can be estimated by Rinne testing as indicated below.

Rinne test result Estimated conductive loss


BC>AC at 256 H:: Conductive loss of approximately 10- 15 dB
BC>AC a! 512Hz Conductive loss of approximately 15-20 dB
BC>AC at 1024 H::. Conductive loss of approximatel_v 20- 25 dB
BC>AC a1 2048 Hz Conductive loss of approximately 30 dB

HEENT:

Left ear: EAC without erythema, TM intact, mobile, and transparent.


No fluid behind TM ·
Right ear: EAC open w/o erythema or discharge, Tlvf intact, mobile,
and transparent. No fluid behind TM
TiVeber test: Lateralizes lO right ear at 256 and 512Hz
Rinne test: BC>AC on right at 256 and 512Hz, AC>BC on left at 256
and 512 Hz
Nasal: Patent nares, no septal deviation, nor.mal-appearing mu-
·cosa, no rhinorrhea ·
OCIOP: No OC masses, mobile tongue, no tongue masses, good den-
tition, no mucosal color changes, uvula midline and not bi-
fid, 2+ tonsils without erythema or exudates
Neck: Supple, no lymphadenopathy, no thyromegaly, midline tra-
chea, no neck masses

What diagnostics would y()u request? State that you would:

• Obtain audiogram (See figure on page 9)

15
Otolaryngology Clinical Case Studies Chapter 1 - Otology

NorEs Test results:

The amount of hearing loss fi-om middle ear and TM abnormalilies is


roughly dependent on the component of the conduction system which
is disrupted
The expecred loss and size of disruption are summarized below

Site o( lesion bpected loss


"
TM p e1joration Proportional to s iz e of
TM, should not be greater
than 30- 35 dB
TM pe1joration with ossicular chain disruption 38. 3 dB
Total loss of Jj\;f and ossicular chain 50 dB
Ossicular interruption with an intact TM 55- 60 dB

·What is your differential diagnosis?

I . Otosclerosis - Given this patient's history of hearing loss and a normal ear
exam, this should be high on the differential:
Male:Female l :2
Incidence: Caucas ians > Asians > African-Americans > Native
Americans
Lesions begin by resorption of stable otic capsule bone in adults, fol-
lowed by a reparative phase with bone deposition
Genetic factors, measles virus infection, and autoimmunity may con-
tribute to the disease process
Presenting symptom is usually slowly progressive conductive (and pos-
sibly mild sensorineural) hearing loss beginning by age 20
. Progresses more rapidly at times due to hom1onal factors, such as
pregnancy
Conductive loss maximum at 50- 60 dB
Schwartze's sign: Hyperemia ofthe promontory mucosa from increased
vascularity is sometimes present
Audiometry reveals Carhart's notch, a decrease in BC at 2kHz induced
by stapes fixation
Patients usually have excellent discrimination
2. Tympanosclerosis- Conservative management is warranted as the under-
lying disease process will often cause the tympanosclerosis to recur. While
possible in this patient, this is unlikely given her lack of previous ear infec-
tion history
3. Ossicu lar chain atresia (congenital conductive abnom1alities): While pos-
sible, this is also unlikely given that the patient has a relatively recent onset
of hearing loss. Atresia would be more likely if the patient were younger or
the hearing loss began at a young age:
The malleus and incus are derived from a bridge between the first and
second branchial arch
The malleus head and the short process and body of the incus are
derived from the first arch
The long processes of the malleus and incus are derived from the sec-
ond arch
Stapes develops from the second branchial arch.

16
Chapter 1 - Otology Otolaryngology Clinical Case Studies

The following causes ofCHL are unlike!~, in this patient given her PE findings NOTES
and age. but are included for completeness of the differential diagnosis of CHL.

I. Foreign body in EAC- Remove the foreign body: in children this may need
to be done in the OR under general sedation
OME:
Most patients have a 16-40 dB hearing loss if at alL 20% have a >35
dB hearing loss
Spontaneous clearance of effusion occurs in 90% of patients within 3
months
3. Genetic abnom1alities (only a limited number are Iisted for illusu·ative purposes):
Osteogenesis lmperfecta (numerous subtypes)- 50% of fami lies have
conductive or mixed hearing loss beginning in the late teens and leading
gradually to profound deafness, tinnitus, and vertigo by the end of the
fourth to fifth decade
Beckwith-Wiedemann syndrome (BWS) - 2 patients have been re-
p011ed with stapes fixation
Brachiootorenal syndrome - Mixed hearing loss associated with a
Mondini-type cochlearmalfom1ation (hypoplasia of cochlear apex shown
by tomography) and stapes fixation, cup-shaped, anteverted pinnae, bi-
lateral prehelical pits, bilateral branchial cleft fistulas. and bilateral renal
dysplasia with anomalies of the collecting system. The EYAl gene has
been implicated in the disease
Zunich neuroectodennal syndrome- Early-onset migratory ichthyosi-
fom1 dennatosis, bilateral ocular coloboma, CHL, seizures, MR, and
remarkably similar facial features
Down's syndrome- 90% of all Down's syndrome patients have a sig-
nificant hearing loss, usually of the conductive type
Klippel-Feil syndrome- Ve11ebral fusion at various levels and hearing
Joss that can be conductive, sensorineural or mixed
4. Cholesteatoma:
See discussion in Chapter 7
5. OE:
Approximately 10% of people have an episode of acute OE duringtheir
lifetime with 90% being unilateral
Often referred to as "swimmer's ear"
Cultures often grow Pseudomonas aeruginosa, Peptostreptococcus. and
Staph aureus
Lumen ofEAC can become occluded secondary to inflammation
Patient may complain of pain with chewing or tragal manipulation and
there may be marked periauricular edema and .adenopathy
Presenting complaint is often pruritus
6. Tumors (benign and malignant):
sec
Exostoses - Usually seen in people with repeated exposure to cold
climates- especially swimmers in northern climates
Osteomas - Bony growths at the tympanomastoid suture line
Most osteomas and exostoses require no treatment- surgical exci-
sion is warranted when continued buildup results in canal blockage
and hearing loss

17
Otolaryngology Clinical Case Studies Chapter 1 - Otology

NOTES 7. Glomus tympani cum and Jugulare tumors:


Female/male ratio 5: I
Derived from special neural crest elements, the paragangl ion cel ls.
wh ich. with autonomic ganglion cells. form the paraganglia
Second most common nem·otologic tumor after acoustic neuroma
An autosomal dominant predispositi on has been identified
Most frequent in Sth and 6th decade
Most common PE findi ng is a vascular midd le ear mass
- .. 80% of the time patients present with tinn itus fo llowed by 60% with
hearing loss
Late symptoms include TM erosion and bleeding
Symptoms related to lower CN involvement can also be seen including
dysphagia, hoarseness, aspiration, tongue paralysis, shoulder drop, and
voice weakness. CN VII involvement has a poor prognosis
8. Iatrogenic (i .e., ear surgery)

Diagnosis:

. This patient has an audiogram and PE is most consistent with otosclerosis.

What are the treatment options and their complications? What would you do
and why?

Med ical management should be considered for those patients who:


Are poor surgical candidates
Are not interested in surgical therapy
Are anacoustic in their contralateral ear
Are suspected to have Meniere's disease

Medical management includes:


Sodium fluoride - 20-120 mg/day may serve to stabi lize hearing loss
Vitamin D- 400 U/day
Calcium- I0 mg/day

Medical therapy enjoys varied success. Generally, 50% of patients will note
stabilization of hearing, while 30% may improve slightly. The remaining 20%
will continue to worsen.

Possible side effects of medical therapy are minor but may include nausea,
vomiting, fatigue, and joint or muscular pain. These side effects are generally
self-limited and are J:e!ieved by discontinuation oftherapy.

Amplification:
Patients with otosclerosis should be offered a trial of amplification as
an alternative to surgery
Hearing aid amplification (air conduction)
Cochlear implantation in patients with bilateral severe to profound sen-
sorineural hearing loss due to otosclerosis

18
Chapter 1 - Otology Otolaryngology Clinical Case Studies

Surgical management: NoTES


1. Preoperative counse Iing:
a. The patient should understand that the diagnosis of otosclerosis may
not be definitely made until MEE is perfonned and stapes footplate
fixation is confim1ed via palpation
b. Risks of surgery should be explained. These include sensorineural hea1ing
loss (risk< 2%). TM perforation (2-3%), facial nerve pa•· lysis (< 1%).
Symptoms may also be attributable to stretching or trauma incurred to
the chorda tympani nerve including alteration oftaste and xerostom ia~
These symptoms are nom1ally self-limiting and will resolve over? pe-
riod of weeks to months
c. The patient should also realize that her hearing may not improve, even
after surgery
2. MEE with possible stapedotomy, stapedectomy, or footplate mobilization:
a. Hemostasis and local anesthesia - canal injection with 1% lidocaine
with I :50,000-100,000 epinephrine
b. Tympanomeatal flap is raised
c. Curette posterior-superior bony annulus to expose the pyramidal process
. d. Inspection of middle ear and palpation of stapes suprastructure to con-
finn footplate fixation
e. Stapes mobilization-At this point, if a very small focus of otosclerosis
is encountered the stapes can be mobilized. Caution - refixation is
common, rendering this procedure undesirable by most surgeons
f. Creation of control hole
g. Disarticulation of the incudostapedial joint
h. Lysis ofthe stapedial tendon
1. Harvest of graft material (vein, tragal perichondrium, etc.)
J. Removal of suprastructure:
1. Iftotal stapedectomy is to be performed, then footplate is elevated
and graft is applied
n. If stapedotomy is to be performed, then a hole in the footplate of
the stapes just large enough to accommodate the prosthesis is cre-
ated with a drill or laser
k. Placement of prosthesis
I. Closure of the tympanomeatal flap

Complications:
1. Ve1tigo:
a. Mild to moderate vertigo for 2-3 days following stapedectomy is com-
mon and will resolve without treatment
b. Severe vertigo lasting for greater than 3 days should be evaluated to
rule out the presence of a perilymphatic fistula, reparative granuloma,
and sterile or bacterial labyrinthitis
c. If severe vertigo is accompanied by sensorineural hearing loss (as de-
termined by tuning fork test or BC audiometry), serious consideration
should be given tore-exploration of the middle ear
2. Facial nerve paresis:
a. Facial nerve injury is uncommon, but can occur in an immediate or
delayed fashion

19
Otolaryngology Clinical Case Studies Chapter 1 -Otology

NOTES b. For incomplete and delayed paralysis (i.e.. nerve known to be intact),
high dose steroid treatment shou ld be employed
c. Iffac ial nerve paralysis is immediate and complete. then surgical explo-
ration and decompression ofthe facial nerve shou ld be performed
d. If facial nerve paralysis is delayed and complete. then it should be fol-
lowed with ENOG and treated appropriately
3. Sensorineural hearing loss:
a. Should be treated with high dose corticosteroids immediately after it is
identified
b. Appropriate antibiotics are to be initiated in the event that bacterial
labyrinthitis is suspected
c. Re-exploration should be considered if peri lymphatic fistula is suspected,
or if CT scan identifies that the prosthesis has migrated medially into
the vestibule

20
Chapter 1 - Otology Otolaryngology Clinical Case Studies

CA.SE 4- SuDDEN SENSORINEURAL HEARING Loss NOTES


David Osterhus, MD

A -14 year-old male presents with sudden hearing loss in the left ear of 24
hours duration.

HPI: state that you would:

Obtain a detailed medical history and more information about the hearing loss

What additional historical information would you seek? State that you would ask:

Has th is ever happened before?


Is there associated tinnitus. otalgia, vertigo, or ototThea?
Is there a history of trau rna or recent otologic surgery?
Did this occur while swimming, diving, or flying?
Have there been recent changes in the patient's medications or recent
hospita! izati ons?
Does the patient have other medical problems?

The patient reports sudden onset of significant left-sided hearing loss yes- ·
terday. There is an associated low-pitched roaring in that em~ The patient
has no otalgia, no otorrhea, and is not vertiginous. There has been no
trauma, medication changes, hospitalizations, foreign travel, or recent ow-
logic surgery.

PMH: Negative
PSH: Appendectomy-age 19
Allergies: NKDA
Medications: None
FH: Father wore a hearing aid, died at age 72
SH: Investment banker

What would you look for on PE'? State that you would:

Perform a complete head and neck examination

PE:

~.. Temp: 98.9 F, HR: 90, BP: 135/80, RR: 16


GA: Healthy-appearing adult male, slightly anxious
HEENT Eyes normal with normal extraocular movements
Anterior rhinoscopy: Clear
OC: Clear
Left ear: CleaT~ mobile TM, no effusion, no pe1joration
Right ear: Clem; mobile TM, no effusion, no perforation
Weber to the right
Air conduction > BC bilaterally
CN: Intact and symmetric Il-Xll
Neck: No lymphadenopathy, no bruit, no masses

21
Otolaryngology Clinical Case Studies Chapter 1 - Otology

N OTES What diagnostics would you t·equest? State that you would :

Obtain an audiogram

Test results:

Audio shows left-sided sensorineural hearing loss with slightly more than
30 dB loss at 500 H::, 1000 H::. and 2000 H:: as compared lO the right. SRT
is 33 on the left and 5 on the right. Speech discrimination is 50% on tht!
left and 100% on the right. Tympanograms are normal.

What is your diagnosis?

SSNHL

What is included in the differential of SSNHL?

· • Viral etiology- Viral etiology is thought to be the most common cause of


SSNHL. A variety of upper respiratory viruses are believed capable of
causing SSNHL. Herpes zoster infection is a special case and may present
with a combination of sensorineural hearing loss, cutaneous vesicles, and
facial nerve weakness as part of the Ramsey Hunt syndrome
Vascular occlusion - Vascular occlusion is probably not very common. It is
. ..J.
more likely in patients with other risk factors for cerebrovascular athero-
sclerotic disease. Males and older patients are at increased risk
First attack of Meniere's disease- Some patients with Meniere's disease
may have episodes of fluctuating hearing Joss prior to the development of
symptoms of vertigo. These episodes are usually brief in duration (24-36
hours) and most often manifest as low frequency hearing loss. Symptoms
of vertigo may not develop until years later
AIED- Autoimmune hearing loss is more often rapidly progressive, bilat-
eral, and associated w ith vestibular symptoms
Syphilis - Syphilitic sensorineural hearing loss is also more likely to be rap-
idly progressive and bilateral, but asynm1etric
Perilymph fistula - Perilymph fisrula may present with SSNHL. They are
usually associated w ith a history of either trauma or internal (weight Iifting)
or external (scuba diving) pressure changes
Acoustic neuroma- Approximately 10% of patients with acoustic neuroma
initially present with isolated SSNHL. Only 1% of patients with SSNHL,
however, are found to have an acoustic neuroma. CT scans and unenhanced
MRI scans are inadequate in detecting even quite large acoustic tumors.
Gadolinium-enhanced MRI scan with dedicated images through the lAC is
the imaging modality of choice
Other- Other, less common, causes of SSNHL include toxoplasmosis,
meningococcal or cryptococcal meningitis, Lyme disease, I-llY encephali-
tis, vasculitis syndromes, other cerebella-pontine angle tumors, multiple scle-
rosis, neurosarcoidosis, and ototoxic medications

22
Chapter 1 -Otology Otolaryngology Clinical Case Studies

What, if any, treatment would you prescribe at this time? State that you would: NoTES

Treat with high-dose steroids- High-dose, burst conicosteroids (pred nisone,


l mg:/kg) for I 0-14 days have been shown to be helpfu I in the treatment of
SS HNL, especially with moderate losses of mid to low frequenc ies. Treat-
ment should be fo llowed with audiometry in order to monitor response

What are some contraindications to systemic treatment with corticosteroids?

I. Active peptic ulcer disease


2· Acute bacterial infection
.3· Pregnancy
4 • Granu lomatous disease
S• Brittle diabetes

Are antiviral agents helpful in the treatment of SSNHL?

State that, although they are believed by some physicians to be beneficial if


administered early, the efficacy of antiviral agents is unproven

Without treatment, what is the prognosis for this patient?

There is a high rate of spontaneous recovery, estimated to be approximately


65%

What factors are associated with a poorer prognosis?

1. Profound hearing loss (greater than 90 dB at all fi·equencies)


:;? • Flat or downsloping audiograms
.3 • Presence of vertigo
'I • Older age
.S • Increased time from initial onset

What diagnostics would you request? State that you would order:

High-resolution, gadolinium-enhanced MRI. Although MRI of the entire


brain may be helpful in the diagnosis of other pathology such as toxoplas-
mosis and multiple sclerosis, MRI scans dedicated to the lAC are sufficient
for the diagnosis of acoustic neuroma and may be less expensive
Laboratory tests. Laboratory testing for SSNHL is largely directed by pa-
tient history, PE findings, and physician biases. There is no reliable in vivo
testing for inner ear viral infections. Serum ESR may be helpful in the diag-
nosis of autoimmune disorders. More specific testing for suspected AIED
may be obtained with an immunoassay for 68~kD protein. Suspected syphi-
lis may be tested for with rapid plasma reagin, fluorescent treponema! anti-
body absorption, or microhemagglutination-Treponema pallidum

23
Otolaryngology Clinical Case Studies Chapter 1 - Otology

NOTES Test results:

in this patient, these tests are all normal


MJU in this patient is normal

If your patient had a complete response to corticosteroid therapy, would you


cancel the order for the MRl?

"NO"

There have been many documented cases of acoustic neuroma presenting vvith
steroid-responsive SSNHL.

If your patient's health maintenance organization requested that you perform a


CT scan o( the head or an ABR instead of an MRI, what would you say?

"NO"

CT scans will only pick up bony expansion of the lAC when the tumor has
become large enough. ABR was used extensively in the past for the diagnosis
of acoustic neuroma. However, there have been numerous reports of false-
negative ABR tests in these patients. Additionally, in those patients in which
ABR testing is positive, imaging with MRJ is subsequently needed.

Suppose your patient has profound SSNHL in the right ear with no return of
function at 6 months. The MRI is normal. What tasks will your patient have the
most difficulty with?

Hearing sounds that originate from his right side


Localization of sounds
Hearing in a noisy environment

What advice would you offer?

In classroom settings, sit to the right of the speaker


Decrease background noise during conversation (turn off TV)
Notify physician immediately with any loss of hearing on the left

What kind of hearing aid might benefit this patient?

State that a contralateral routing of sound hearing aid would allow the pa-
tient to more easily hear sounds on his right side and aid in the localization of
sounds

24
Chapter 1 - Otology Otolaryngology Clinical Case Studies

CASE 5 - BPPV NoTES


Debbie Eaton, MD

A 56 year-old man presents with episodic dizziness.

HPI: state that you would:

o Obtain a detailed medical history beginning with a history of the present


illness

What additional historical information would you seek? State that you would:

Describe the dizziness without using the word "dizzy"- is it a feeling of


movement when no movement exists, a feeling of being light-headed, or a
feeling of being off-balance?
~ How long do the episodes last?
Are they associated with sweating, nausea, or vomiting?
Are they exacerbated by certain positions?
Is there any associated hearing loss, aural fullness, tinnitus, otalgia, weak-
ness, visual changes, dysarthria, or numbness?
Are there any associated symptoms of an upper respiratory tract infection?
ls there any recent history of trauma?

He states that several times per day, for the past ·week, he has brief epi-
sodes of vertigo that typically occur when he lies down and turns over (to
the lef() in bed or extends his neck in the showe1~ Each episode lasts about
30 seconds. This vertigo consists of a feeling that the room is spinning,
and is associated with nausea. He denies hearing loss, tinnitus, aural full-
ness, dysarthria, numbness, or weakness. He does recall falling off a step
ladder about a week ago, though he did not sustain any injuries.

PMH: HTN
Allergies: None
Medications: Lisinopril
FH: Noncontributory
SH: Denies tobacco or alcohol use, stockbroker
ROS: As above

What would you look for on PE? State that you would:

Perfonn a complete head and neck PE including vital signs and microscopic
ear examination

PE:

~: Temp: 98. 6 F, BP: 120180, Pulse: 80


GA: WDVVN WM in NAD
HEENT AUWNL
New-a: EOMI, facial nerve vVNL. no motor or sensory deficit, normal
gait

25
Otolaryngology Clinical Case Studies Chapter 1 - Otology

NOTES What is your differential diagnosis?

Differential should include peri'pheral vestibular disorders including BPPV.


Meniere's disease, perilymph fi stula. or labyrinthitis due to either inflamma-
tion or ischemia/stroke

What would you do next?

Perfom1 a Dix-Hallpike maneuver. This is done by quickly moving the pa-


tient from the sitting to reclining position with the head hanging slightly. The
patient's head is first turned 45 degrees toward the side being tested, with
the examiner positioned behind the patient. The examiner holds the patient's
head and the patient is instructed to fall backward quickly. The head is held
stationary and hanging for at least 20 seconds, and the eyes are monitored
for nystagmus. The patient is then returned to the sitting position with the
head still turned and the eyes are again assessed for nystagmus (which
occurs in the opposite direction)

In this patient, Dix-J-Jallpike maneuver to the left produces vertigo and


geotropic rotary nystagmus after a latency of 20 seconds. The response
fatigues after several maneuvers. ·

Diagnosis:

The diagnosis is ·BPP V

What 6 characteristics of nystagmus induced by the Dix-Hallpike maneuver


are pathognomonic for BPPV?

• 1 Latency ·
• .::Z Paroxysmal
· 3 Accompanied by ve1tigo
• ~ I Geotropic rotary
• 3 Direction reverses upon sitting up
• 6 Fatigable

BPPV is most commonly a disorder of the posterior semicircular canal, caused


by the presence of displaced otoconia! debris freely floating within the canal or
attached to the cupula under the influence of gravity. Most cases are idiopathic.
Trauma and viruses are the most common identifiable causes.

What are the treatment options and their complications? What would you do
and why?

The disease is usually se1f-limited


Initially, treatment consists of canalith repositioning maneuvers (i.e., Epley,
Semont, etc.) designed to liberate the clot of material from the posterior
semicircular canal
Rarely, patients may have prolonged intractable BPPV that is unresponsive
to conventional therapy. For these patients, surgical treatment is considered

26
Chapter 1 - Otology Otolaryngology Clinical Case Studies

The Epley (as an example of a canalith repositioning maneuver) maneuver is a N oTES


positional maneuver designed to remove debris from the posterior sem icircular
canal. A Dix-Hal lpike maneuver is performed and the patient maintains that
position fo r 2-3 minutes foll owing resolution of nystagmu s. The patient's head
is then rotated to the opposite side followed by rol ling of the patient's body
laterally until the patient is lying on his side. This is accomplished in 1 smooth
motion, with the head extended throughout. This position is ma intained, again,
for l- 2 minutes after resolution of nystagmus. The patient is then slowly retumed
to a sitting<position. With a successful maneuver, no nystagmus occurs when the
patient retums to the upright position. Maneuvers are repeated as needed e,ve1y
2-3 weeks. When appropriately administered. repositioning maneuvers resolve
BPPV in 95% of cases. Patients with incapacitating symptoms which do not
resolve with 3 repositioning maneuvers are offered surgica l treatment.

What are the surgical options for tr·eatment?

• Posterior semicircular canal occlusion


• Posterior ampullary nerve section (singular neurectomy)

What are the highlights and potential pitfalls of each procedure?

Posterior semicircular canal occlusion


Highlights:
Mastoidectomy approach, posterior semicircular canal is blue-lined
Occlusion is performed using a plug of bone chips and tissue glue, in
order to compress the membranous labyrinth
> 95% control of vertigo
Pitfalls:
Most patients develop temporary hearing loss post-op
Hearing loss may be pem1anent
Most patients have 2-4 week period of vertigo post-op

Posterior ampullary nerve section (singular neurectomy)


Highlights:
Transcanal procedur.e
Singular canal is identified by drilling inferiorly in round window niche,
then nerve tissue is destroyed and bone dust is placed into singular
canal
96% control of vertigo in some series
Pitfalls:
Risk of hearing lossJ%
--~
This procedur~ has been largely replaced by posterior canal occlusion
Singular nerve may be inaccessible in some patients

27
Otolaryngology Clinical Case Studies Chapter 1 -Otology

NoTES How would you follow this individual?

Postoperative patients should be followed up clinically at I week. A postop-


erative audiogram should be conducted after approx imately I month Rou-
tine fo llow-up thereafter is dictated by patient symptoms, but may be at6-
12 month intervals
Patients may develop vertigo postoperatively. This may be caused by in-
complete transection of the singular nerve, BPPV involving the contralat-
eral ear, or incomplete vestibular compensation. Patients with persistent
ve1tigo shou ld be carefully reevaluated using the Dix-Hallpike maneuver to
detem1ine the presence of persistent or contralateral BPPV. Those patients
with dysequilibrium due to incomplete compensation should be offered ves-
tibularrehabilitation. An individualized vestibular rehabilitation program. with
emphasis on balance exercises, should be designed by a vestibular thera-
pist. Follow-up should be on a regular basis until compensation and habitu-
ation are optimized
Patients who develop hearing loss postoperatively should be counseled that
the hearing loss may be permanent. A course of steroids may be offered to
the patient, but has not been proven to have significant benefit in this setting

28
Chapter 1 - Otology Otolaryngology Clinical Case Studies

CASE 6- MENIERE' S DISEASE N OTES


Debbie Eaton, MD

A 38 year-old fe male prese111s with episodic di::::::iness.

HPJ: state that you would:

Obtain a detailed medical hist01y beginnjng with a hist01y of the present


illness

What additional historical information would you seek? State that you would ask:

To describe the dizziness without using the word "dizzy"- is it a feeling of


movement when no movement exists, a fee ling of being light-headed, or a
feeling of being off-balance?
How long do the episodes last?
Are they associated with sweating, nausea, or vomiting?
Are they exacerbated by certain positions?
Is there any associated hearing loss, aural fullness , tinnitus, otalgia, weak-
ness, visual changes, dysarthria, or numbness?
Are there any associated symptoms of an upper respirat01y tract infection?
Is there any recent history of trauma?

The patient reports daily episodes of vertigo that last from minutes lO hours.
occurring over the last 5 years. She describes the vertigo as a sensation
that the room is spinning. The episodes typically begin with a feeling of
pressure and a maring noise in her right em~ followed by vertigo associ-
ated with nausea. She f eels that her hearing in her right ear has become
much worse than the left over the past few years. The ·episode~ can occur
in any position, but are exacerbated by rapid head movements and loud
noises. She denies any history of trauma.

PMH: Unremarkable
Allergies: NKDA
Medications: Oral contraceptives, multivitamin
FH: Noncontributory
SH: Denies tobacco use, drinks 2- 3 glasses of wine on the week-
ends, medical transcriptionist, married
ROS: As above

What would you look for on PE? State that you would:

Perform a complete head and neck PE including vital signs and microscopic
ear examination

29
Otolaryngology Clinical Case Studies Chapter 1 -Otology

NOTES PE:

b..l.als.: Temp: 98. 6 F BP: ]]0180, HR: 80


GA: WDWN WF in NAD
HEENT AUWNL
Neuro: EOMJ, facial nerve W}lL, Weber 51] Hz rzming fork lareralizes
lO the left. wirh AC>BC in both ears, 110 nystagmus

What is your diff~rential diagnosis?

• Meniere's disease
BPPV
Perilymph fistula
Labyrinthitis
.Orthostatic hypotension
Central vertigo (i.e., multiple sclerosis, ischemia, vertebrobasilar insufficiency,
neoplasm, etc.)

What diagnostics would you request? State that you would request:

Audiogram
Electrococh leography
MRI brain and lACs with contrast
Electronystagmography

Test results:

Audiogram revealed low-freque ncy SNHL AD with a PTA of 40 dB,


WNLAS
• Electrocochleography: Elevated S PIAP ratio AD (0.55). The SP/AP
ratio AS was normal
No mass lesion or evidence of ischemia on MRl
N ormal electronystagmography

Diagnosis:

The diagnosis is Menih·e s disease, which is a constellation of symptoms


due to endolymphatic hydrops. This is supported by the patients typical
history, the classic audiogram of low frequency SNHL, and the abnormal
SP/AP raiio (SPIAP ratio of greater than 0.50 is suggestive of an inner ear
fluid imbalance) . •

What are the treatment options and their complications? What would you do
and why?

Initially, treatment consists of vestibular suppression for the acute spells of


ve11igo and long-term treatment with salt restriction and diuretics
Dyazide 1 tab po q day
Valium 2.5- 7.5 mg po q day pm severe vertigo

30
Chapter 1 - Otology Otolaryngology Clinical Case Studies

Clonazepam 0.25- 1 mg po q day NOTES


Phenergan 25 mg po q 6 hours pm nausea
Salt intake should be less than 2 g per day - the patient should be
counseled thoroughly on the sodium content of foods and given specific
guidelines to follow. A dietary consult may be helpful
For lifestyle-limiting Meniere's disease that is refractory to medical man-
agement, surgical treatment is an option

What are the surgical options for treatment in a patient who fails maximum
medical therapy? What are the highlights and potential pitfalls of each technique?
Include indications and success rates for each:

Endolymphatic sac surgery (decompression or mastoid shunt)


Highlights:
Nondestructive procedure ·
Indicated in patients.with serviceable hearing who have failed medi-
cal therapy, and patients with bilateral Meniere. s disease
Somewhat controversial
After simple mastoidectomy with wide decompression of the sig-
moid sinus, the endolymphatic sac is incised and a silastic splint is
placed into the sac to decompress this space into the mastoid
Approximately 70% successful in control of vertigo, 50% success
in improvement of tinnitus or pressure
Pitfalls:
Risk of profound hearing loss post-op 3-5%
CSF leak upon entering posterior fossa dura - This can be treated
intraoperatively by packing with tissue and/or absorbable packing
Injury to the posterior semicircular canal
Facial nerve injury
Vestibular nerve section
Highlights:
Destructive procedure
Indicated in patients with serviceable hearing. unilateral vertigo,
and a normal central vestibular system to allow nom1al compensa-
tion postoperatively
Hearing preservation successful >80%
85-99% success rates for control of ve11igo
Different approaches
11 Middle fossa (selective vestibular nerve section)
- Gain access to lAC through removal of supralabyrinthine
bone
- lAC is identified within 60 degree angle to the blue-lined
superior SCC. The SVN and lVN are sharply sectioned.
- Advantages over posterior fossa approaches: Division of
vestibular nerve fibers in distal lAC, less CSF leak, no
cerebellar retraction
• Retrolabyrinthine or retrosigmoid
- Gain access to vestibular nerve through posterior cranial
fossa

31
Otolaryngology Clinical Case Studies Chapter 1 - Otology

NOTES Mastoidectomy is performed and sigmoid is retracted


anteriorly in the retrosigmoid approach, posterior fossa dura
is entered and CP angle is accessed. The retrolabyrinthine
procedure is pe1fom1ed anterior to the sigmoid. If no
cochleovestibular cleavage plane is present. the lAC is
opened and nerve section is performed there
- Advantages over midd le fossa approach: No temporal lobe
retraction
a T·ranslabyrinthine
Gain access to lAC through labyrinthectomy
- Hearing is sacrificed
Advantage over middle fossa approach: Less chance of
facial nerve injury
Pitfalls:
a Hearing loss (rates vary depending on approach, from 20--l 00%)
a CSF leak (6-1 0%)
a Transient facial nerve weakness (3-5%)
• Permanent facial nerve injury (< 1%)
Labyrinthectomy
Chemical
Highlights:
• Destructive procedure, indicated in patients who have operative
risk factors or are not interested in surgery
• Intratympanic gentamicin placed into middle ear using rieedle
puncture or through PE tube
• Presumed efficacy due to diffusion of aminoglycoside into
labyrinth via the round window membrane
11 80- 90% success rate for control of vertigo
Pitfalls:
• Hearing Joss (30%) - requires monitoring with serial
audiograms during the course of treatment
• Usually requires more than one treatment
Transcanal
Highlights:
a Indicated in patients with high anesthetic risk and non-
serviceable hearing
a Under local anesthesia, the promontory is drilled (via a
transcanal approach) until the round and oval windows are
connected and neuroepithelium is removed from the vestibule
and semicircular canals
a Rarely perfom1ed
Pitfalls:
'- Poor exposure
Transmastoid
Highlights:
• Indicated in patients with non-serviceable hearing
• 88% success rate in control of vertigo
• A transmastoid, translabyrinthine vestibular nerve section is
perfom1ed by sectioning the vestibular nerve in the lAC after
completing the labyrinthectomy. This gives vertigo control rates
of90+%
32
Chapter 1 -Otology Otolaryngology Clinical Case Studies

Pitfalls: N OTES
a Facial nerve is at ri s ~ at the ampullated ends of the posterior
and horizontal sec
a CSF leak

H ow would you follow this individual? State that you would order:

Vestibular rehabilitation for assistance with compensation is essential and


should be initiated immediately postoperatively
Outpatient follow-up for examination of wound, hearing status, and f<;~-cial
nerve function, and follow-up on symptom control

Postoperative patients should be followed up clinical ly 1-2 weeks after hospital


discharge for a wound check. A postoperative audiogram should be checked
after approximately one month. Routine follow-up thereafter is dictated by pa-
tient symptoms, but may be at 3-6 month intervals

Vestibular rehabilitation should be initiated postoperatively, while the patient is


still in the hospital. Specifically, balance and habituation exercises should be
used and patients should be encouraged to ambulate with assistance as soon as
possible. Severe postoperative ve11igo may be treated successfully with benzo-
diazepines, such as valium 2.5 mg IV q 30 min pm vertigo/nystagmus, with a
maximum dose of I 0 mg in 24 hours. Nausea may be treated with Phenergan
12.5-25 mg IV q 4-6 hours as needed

Outpatient vestibular rehabilitation should be arranged after discharge from the


hospital, and follow-up should be on a regular basis until conipensation and
habituation are optimized

Patients who develop hearing Joss postoperatively should be counseled that the
hearing loss may be pe1manent. A course of steroids may be offered to the
patient, but has not been proven to have significant benefit in this setting

References:

1. Baloh RW. Vertigo of peripheral origin. In: Canalis RF, Lambert PR, eds.
The ear comprehensive otology. Philadelphia: Lippincott Williams &
Wilkins; 2000, 647-64.
2. Brackmann DE, Shelton C, Arriaga MA, editors. Otologic surgery. 2"d
ed. Philadelphia: Saunders; 2001.
3. Antonio SM, Friedman R. Meniere's disease. In: Jackler RK, Backrnann
DE, eds. Neurotology, St. Louis: Mosby; 2005, 621 - 38.

33
Otolaryngology Clinical Case Studies Chapter 1 - Otology

NoTES CASE 7- CHOLESTE;\fOMA


David Osterh us, MD

A 35 year-old immigrant woman presents to your office with a chief com-


plaint of right-sided hearing loss and intermittent, yellow drainage ji·01n
her righr ear

HPI: state that you would:

Obtain a detailed medical history and more information about the patient's
chief complaint

What additional historical information would you seek? State that you would
ask:

How long has there been hearing loss?


Was it sudden or progressive?
How long and how frequent has the drainage been present?
Has the drainage responded to any therapy?
Is there any history of trauma?
Is there any associated tinnitus or vertigo?
Have there been fevers or otalgia?
· Have there been any previous surgical procedures?

The patient reports that she had had multiple ear infections as a child in
both ears. She reports noticing progressively worsening hearing loss since
the age of 18 in the right ear only The drainage has been intermittent for
the past 7 years and usually responds briefly to a variety of ototopical
antibiotics that she has been prescribed by various physicians. She has no
pain or other associated otologic symptoms.

PMH: Multiple ear infections as a child, hypothyroidism


PSH: Negative
Allergies: NKDA
Medications: Levothyroxine 0.125 mg QD
FH: Negative
SH: No ETOH, no tobacco: housewife

What would you look for on PE? State that you would:

Perform a complete head and neck exam including a microscopic ear exam

PE:

YiWJ.s.: Temp: 98.6 F, BP 125170, HR: 80, RR: 16


GA: Well developed adult female in NAD
CN 2-12 grossly intact
HEENT Anterior rhinoscopy clear. OC clem: Neck with no lymphaden-
opathy

34
Chapter 1 - Otology Otolaryngology Clinica l Case Studies

Lefi ear: Clem~ Tlv/ intact. mobile, no effusion NOTES


Right ear: Slighr amoum of purulent. yellow drainage in posrerior quad-
ram (~f Ti'vf that is easily suc1ioned. There is a posrerio1~ mar-
g inal perforarion ll'ith granulation tissue. There is mild scuta!
erosion

What diagnostics would you request'! State that you would request:

Audiogram
Tympanogram
o CT scan

The use of imaging studies in this clinical scenario is not widely agreed upon.
Many otolaryngologists reserve pre-operative imaging studies for revision cases
or for cases in ~hich abnormal anatomy is suspected. In any case, there is little
doubt that the appropriate use of imaging studies can provide valuable informa-
tion about pre-operative risk assessment and the presence of complicating fac-
t~rs. If a CT is to be ordered, the imaging study of choice is a non-contrasted,
fine-cut, axial and coronal, temporal bone CT scan. Although the CT scan will
not be able to distinguish granulation tissue from cholesteatoma, it will provide
useful infonnation about bony anatomy, and delineate the extent of soft tissue
involvement.
..
What information would be valuable to you on a CT scan't

1. Extent of soft tissue involvement


2. Presence of a defect in the tegmen mastoid or tegmen tympani with or
without intracranial soft tissue extension
3. Presence of a labyrinthine fi stula .
4. Anatomic course and bony integrity of the fallopian canal
5. Presence of a high-ridingjugular bulb
6. Status of the ossicular chain
7. Degree of mastoid pneumatization

Test results:

Audio shows bilateral symmetric, sloping, high-frequency sensorineural


hearing loss. There is also right-sided CHL with an average air-bone gap
of about 25-30 dB.
Tympanogram shows type A on the left and type B on the right.

In the present case, the CT scan shows opacification of the epitympanum.


antrum, and some mastoid air cells. There is scuta! erosion. There is no
evidence of a tegmen deject, labyrinthine fistula, or erosion of the fallo-
pian canal.

35
Otolaryng ology Clinical Case Studies Chapter 1 - Otology

NOTES What is your differential diagnosis?

CSOM
MOE
Congenital or acquired cholesteatoma
Neoplasm

Diagnosis:

Secondary. acquired cholesteatoma wirh scuta! erosion· and CHL.

What are the treatment options and their complications? What would you do
and why?

o State that you would treat disease in the EAC- While awaiting surgery, it is
often helpful to treat purulent drainage with ototopical antibiotics and de-
bride any granulation tissue that is easily accessible. Systemic corticoster-
oid therapy may also decrease the amount of granulation tissue prior to
surgical intervention
State that you would recommend surgery - Discuss the risks, benefits, and
alternatives to tympanomastoid surgery and reasonable expectations of hear-
ing outcome. Discuss the necessity of a second stage procedure both for
hearing reconstruction (if needed) and for removal of residual or recurrent
cholesteatoma · ·
State that you would discuss potential complications- Complications may
result from progression of cholesteatoma or from surgical intervention. In
relation to surgery, risks include:
Total, permanent hearing loss:. The risk is about 1-2%
'Tinnitus
Prolonged balance disturbance:·This may last only a few days but may
be. more severe and last several months
Alteration in taste: The risk is about 10% and may last several months,
but usually resolves
Facial paralysis: This may be permanent or temporary and the risk is
about l-3%, respectively
Dural tear and CSF leak: The risk is Jess than I%
TM perforation: Graft failure rates vary widely, but are generally Jess
than 10%
Recurrence of cholesteatoma: This also varies, but increases with the
number of previous operations

Would you perform a canal wall-up (intact canal wall) or canal wall-down (open
cavity) procedure?

State that, unless there are specific indications to perform a canal wall-
down procedure, you would likely perfonn a canal wall-up tympanoma-
stoidectomy. Canal wall-up procedures are often performed at the first
operation with the expectation that there wi II be a second-look procedure to
examine for residual disease. Although there is a slight increased risk of

36
Chapter 1 - Otology Otolaryngology Clinical Case Studies

residual cholesteatoma with leaving the canal wall up. these procedures NOTES
have several advantages:
I. The EAC and auricle have a normal appearance
1
Hearing aids are easily fitted and wom
3. tvlinin1al n1aintenance is required

I n which instances might you perform a canal wall-down procedure?

State that there are a variety of relative ind ications for canal wall-down
procedures. These include:
- ) Several. documented cases of cholesteatoma recurrence
- :::! Cases in which close follow-up is not possible or desired
- 3 Congenital abnormalities such as a stenotic EAC
- ~ In an only hearing ear
-5 Cases in which cholesteatoma involves difficult-to-reach areas such as
the sinus tympani and the hypotympanum
- 6Cases in which a large portion of the bony canal has been destroyed
- 1 Cases of labyrinthine fistula in which exteriorization of involved cho-
lesteatoma matrix is desired

Are hearing results affected by the decision to perform canal wall-up or canal
wall-down procedures'?

State that comparable results are achieved with either procedure. Canal
wall-up techniques, however, more often require delaying ossicular chain
reconstruction until a secondary procedure. During the interim, a maximal
conductive loss can be expected

Describe your surgical approach and the goals you wish to achieve:

Consent: Review expectations of the procedure with the patient, answer


questions, and mark surgical site
• Antibiotics: Adequate gram-positive coverage (i.e., cefazolin) is usually suf-
ficient. If there is a particular concern about CSOM, pseudomonal cover-
age is desired (i.e., ceftazidime)
• Facial nerve monitor: Although use of a facial nerve monitor has not been
shown to improve facial nerve outcome, it can provide valuable feedback in
difficult tympanomastoid surge1y. While not required, its use is probably
warranted if it is available at your institution
• Anesthesia: The use of nitrous oxide, in theory, can lead to graft displace-
ment and its use should be avoided. The use of neuromuscular agents should
be litnited if the facial nerve is to be monitored
• Goal # I: State that your primary goal is to eliminate all potentially erosive
and invasive disease and to achieve a safe and dry ear
Goal # 2: State that your secondary goal is to preserve residual hearing and,
possibly, to improve hearing
Approach: State that you would make post-auricular incision, harvest
temporalis fascia, perform complete mastoidectomy, remove all choleste-
atoma, achieve adequate communication with the middle ear space, deter-
mine and deal with ossicular involvement, reconstruct the TM with fascia,
and close with a mastoid dressing
37
Otolaryngology Clinical Ca se Studies Chapter 1 - Otology

NoTEs Cholesteatoma often in volves the epitympanum and eff011s should be made to
eradicate all disease in this area. Ot)}er areas tbaJ are at in crp~sed risk of re-
siR;ual cholesteatoma incl ude the sim'fs' tympani,~t~cial recess, '!iypotympanum.
a]'l'(l:.sequestrated tegmental and sinodural cells.

You would perform a facial recess, if needed, to better visualize and eradicate
disease or to achieve adequate communication or "flow" with the middle ear
space. Note that some surgeons perform a faci al recess in all cases of choleste-
atoma.

In most cases of cholesteatoma, seconda1y ossicu lar chain reconstruction is


preferred. Advantages of staged operations include:

• I Presence of a heal ed TM
• .1. Adequate room and aeration of the middle ear space
o 3 Limitation of disruptive middle ear adhesions
• .:( Confidence in eradication of cholesteatoma

What technique can you use to limit post-operative middle ear adhesions?

State that the use of Gelfoam or si lastic sheeting in the middle ear space
can help to reduce post-operative adhesions. These materials are particu-
larly needed in cases in whi ch there are large areas of denuded mucosa

How would you follow this patient? State that you would:

Remove packing ·1- 2 weeks after surge1y - At the first post-operative visit,
surgical packing should be removed and the post-auricular incision should be
examined for signs of infection. In general, surgical packing after canal-wall
down procedures is left in place longer than after canal-wall up procedures-
• Schedule audiogram - Informal hearing testing with a Weber tuning fork
exam may be performed in the immediate post-operative period. Formal
audiologic testing should be performed at about 6 weeks post-operatively
for best results. Tympanograms, however, are usually not performed at that
time
• Schedule second-look procedure - Second-look surgical procedures for
hearing reconstruction and removal of residual cholesteatoma should be
performed 6-12 months after the first procedure
Recommend regu lar follow-up - Patients with a history of cholesteatoma
should be examined on a regular basis with audiogram testing and binocular,
microscopic ear examination and cleaning if needed. Patients with canal
wall-down procedures often require regular removal of debris that collects
within the mastoid bowl. Follow-up visits should be scheduled frequently
(every 3 months) in the initial post-operative period and less fi·equently (yearly)
as suspicion of recurrent cholesteatoma decreases

38
Chapter 1 - Otology Otolaryngology Clinical Case Studies

CASE 8 - PETRO !JS APICITIS NoTES


Peter S. Roland, MD

A 45 year-old male presents -.vith a chronic drainage from his right ear of
6 months· duration.

HPJ: state that you would:

Obtain a detai led medical history beginning with a history of the present
illness

What additional histor·ical information would you seek? State that you would ask:

Is drainage continuous or intermittent?


Watery, mucoid, sanguineous or pw:ulent?
Has there been fever or otalgia?
Hearing loss, dizziness or tinnitus?
Facial twitching or weakness, visual change or loss?

Subject reports he has had a right-sided pe1joration for many years. Drain-
age began after Florida vacation with water sporrs. He has received sev-
eral courses of treatment with otic drops and systemic augmentation and
cefuroxime. Over last 4 weeks has developed increasing rt. retro-orbital
pain and episodic diplopia.

PMH: Negative
Allergies: . NKDA
Medications: Amoxici!lin 500 mg po tid
FH:- Negative
SH: ( - ) tobacco ( - ) ETOH: stockbroker
ROS: Negative

What would you look for on PE? State that you would:

Perfom1 a complete head and neck PE including vital signs and microscopic
ear examination

PE:

Yi.Lcils.: Temp: 98.6 F, BP: 130190, Pulse: 82


GA: WDW'N WM in NAD
HEENT None
Left ear: WNL
Right ear: 50% posterior marginal pe1joration with moderate mucopu-
rulent drainage. Moderate granulations at posterior mar-
gin of pe1joration. EA C WNL
New-o: EOM- Normal exam but with subjective diplopia on lat-
eral gaze. Remainder of CN exam, gait and station,
Rhomberg, cerebellar tests, muscle strength and sensation
WNL

39
Otolaryngology Clinical Case Studies Chapter 1 - Otology

NOTES What is your differential diagnosis?

Differential should include CSOM. MOE and petrous apicitis and neoplasm

What diagnostics would you request? State that you would obtain:

Sed. rate and white count


Fine cut CT scan of temporal bones with and without contrast
Culture of drainage

The sed rate was normal. Fine cut CT scan demonstrated a lytic area in the
pell·ous apex on right with an enhancing periphery. Mastoid and middle
ear were opacified bur otherwise normal. Septate structure of the mastoid
was intact. Contralateral ear was entirely normal. Culture showed a mix-
ture of Pseudomonas and Staphylococcus aureus-both sens itive to
quinolones.

Diagn.osis:

The diagnosis is petrous apicitis. The patient has Gradenigo s syndrome.


Petrous apicitis is an abscess of the petrous apex that occurs as a compli-
cation of otitis media (often partially or inadequately treated). A petrous
apicectomy is r~quired.

What are the surgical routes to the petrous apex?

• Retro labyrinthine
Supra labyrinthine
Infra labyrinthine
• Through arch of SSC

Re-review of CT shows opacified air cells from the sinodural angle to pe-
trous apex, posterior to the labyrinth.

Surgical technique:

Highlights and pitfalls:


• Obtain consent, mark surgical site, etc.
Start appropriate antibiotics (in this case, oral quinolone)
Discuss whether or not you would use a facial nerve monitor
Use q, post-auricular incision to perform a tympanomastoidectomy for evacu-
ation of mucopurulence and removal of granulomatous tissue
Use small cutting/diamond burrs to follow cells posterior to be SCC out to
petrous apex. Dura may be exposed, but great care should be taken to
avoid entering the subarachnoid space
Reculture for bacteria, fungi, and TB
Irrigate extensively
Repair TM perforation with fascia
Consider a post auricular drain

40
Chapter 1 - Otology Otolaryngology Clinical Case Studies

What are the potential complications? N OTES

Facial nerve injury - If injury does not require surgical repair. treat with
steroids. Stmt Prednisone I kg/day and taper over l 0- 14 days
Injury to SCCs with violation of membranous labyrinth may lead to signifi-
cant neurosensory hearing loss. Many surgeons would treat this injury with
a tapering do~e of systemic steroids
Laceration of sigmoid or petrosal sinus can almost always be controlled by
placing a piece of Gelfoam completely over the injured area. Bleeding vvill
stop but blood flow through the sinus will continue
Dural laceration should be repaired with homograft or all ograft (i.e ..
AlloDem1)

H ow would you follow this individual? State that you would:

Che~k surgical cultures - Pseudomonas and Staph are most common


Continue culture directed antibiotics 10-14 days
F/U CT scans on first post op day and in 2 weeks

41
Otolaryngology Clinical Case Studies Chapter 1 - Otology

NOTES CASE 9- TEMPORAL BONE OSTEOMYELITIS


David Osterhus, MD

A 47 year-old f emale presems with a one-week history of worsening left-


sided otalgia.

HPI: state that you would:

Obtain a detailed medica l history and more information about the chief
complaint

What additional historical information would you seek? State that you would ask:

ls there associated drainage?


• How severe is the pain and how long has it been present?
Has there been any recent water exposure or swimming?
• ls there associated hearing loss, tinnitus, or vertigo?
Is there any recent history of surgery or trauma?
Has ~my treatment been initiated?
What are the patient's other medical problems?

The patient reports that she has never had any problems with her ears
other than periodic difficulty with wax impaction. At a regularly scheduled -
visit with her primary care physician last week, she had both of her ears
irrigated for wax removal. Since then, she has had an increasing, throb-
bing pain in her left eaT: She denies any hearing loss, drainage. tinnitus or
vertigo. She has been taking Tylenol and Motrin with little relief
/

PMH: Diabetes (type !I)- diet controlled, hypothyroidism


PSH: Hysterectomy
Allergies: NKDA
Medications: Levothyroxine 100 meg· QD, Tylenol, Motrin
FH: Diabetes
SH: 3 children, husband deceased, works in convenience store.
She is a smoker,· 1 ppd

What would you look for on PE? State that you would:

Perfom1 a complete head and neck exam as well as a microscopic ear


exam

PE:

l'1.tals_: Temp: 99.9 F, HR: 80, BP: 120160, RR: 16


GA: Well developed female in NAD
HEENT: Eyes: extraocular movements intact
OC: Clem; no masses, 2+ wnsils
Anterior rhinoscopy clear

42
Chapter 1 - Otology Otolaryngology Clinical Case Studies

Righr ear: Normal, TM clem: mobile, intact NoTEs


Lej! ear: Slight edema of the EAC. small amoum of Lhin purulem fluid
mixed 1-Filh some retained cerumen. TM is clear and without
effusion. Maswid is non-tender
Neck: No lymphadenopathy. 110 masses
CN II-XII intacr

Differential diagnosis:

Acute OE
Chronic OE
TMJ
Osteomyelitis of the temporal bone (MOE)
RefeiTal of pain to the ear from distant source
. .
What is your diagnosis?

• Acute diffuse external otitis

What are the treatment options and their complications? What would you do
and why?

Suction debris from EAC under binocular microscopy


Prescribe ototopical antibiotic drops
Recommend dry ear precautions
Schedule return appointment 5-7 days later

She returns to your office 5 days later with a persistent, deep boring pain
in her left ea7~ Exam shows persistent edema of the EAC with a small amount
of thin purulent jluid. When you are suctioning the canal under binocular
microscopy, you notice a small amount of granulation tissue along the
floor of the canal. There is also increased tenderness of the peri-auricular
soft tissues.

Differential diagnosis:

Acute 0£
Chronic 0£
TMJ
Osteomyelitis of the temporal bone (MOE)
Referral of pain to the ear from distant source

Diagnosis:

MOE is a clinical diagnosis rhat encompasses a spectrum of disease pro-


cesses ranging from necrotizing OE to osteomyelitis of the temporal bone
01~ possibly, the entire skull base. The term originates from Chandlers
original description of an aggressive infectious disease process in which
many of his patients had involvement of multiple CN and poor outcomes.

43
Otolaryngology Clinical Case Studies Chapter 1 - Otology

NOTES What diagnostics would you request? State that yo u would:

Obtain a culture - Cultures and sensitivities may help direct therapy


Take directed biopsies - Biopsy of the granulation tissue is needed to rule
out malignancy
Send appropriate labs - Appropriate laboratory testing in this patient would
include a complete blood cell count. fasting glucO"" leveL and ESR. The ESR
will be very higb (90-110) in MOE but normal in acute OE and malignancy
Technetium-99m scan -<Tecbnetium-99m scans are very sensitive for de-
tecting increased osteoblastic activity which occurs in osteomyelitis as well
as bony dysplasias, trauma, and malignancy. They, however, remain posi-
tive long after resolution of the inflammatory process as new bone forma-
tion occurs
Gallium-67 scan or Indium- Ill WBC scan - Gallium scans are more spe-
cific indicators of inflammation as Gallium-67 is preferentially absorbed by
macrophages and reticular endothelial cells. These scans revert to normal
when ·inflammation has resolved and are most useful in determining dura-
tion oftherapy. lndium-11 1 labeled white blood cell scans are, also, highly
specific for inflammation and are used in place of gallium scans in some
centers. They are, however, not widely available

Test results:

Cultures are negative in this patient. Biopsy shows acute and chronic in-
flammarion. The WBC = 9.9, H;H = 12.1136.5, platelets = 450, glucose =
200, ESR = 95. The technetium-99m scan and the gallium-67 scan were
srrongly positive in the left temporal bone. ·

Diagnosis:

Specifically, this patient has temporal bone osteomyelitis.

Skull base osteomyelitis involves the temporal bone as well as other bones of
the skull base. As nuclear medicine scans are anatomically imprecise, the dis-
tinction is based mostly on clinical criteria of CN involvement. Involvement of
CN other than the facial nerve denotes skull base osteomyelitis. CT scans and
MRl imaging may also help to distinguish isolated temporal bone involvement
from skull base osteomyelitis.

What would you do next? State that you would:

• Continue to treat the patient with ototopical drops and that you would add
high-dose, oral ciprofloxacin therapy

The majority of external ear and associated infections are caused by Pseudomonas
aeruginosa. Empiric therapy should include systemic and ototopical antibiotics
directed at this organism. Systemic therapy is needed to address the soft tissue
component. The fluoroquinolones have excellent activity against Pseudomonas
and have the added benefit of high oral bio-availability in soft tissue and bone

44
Chapter 1 - Otology Otolaryngology Clinical Case Studies

after oral administration. Uncompl icated. temporal bone osteomyelitis can be NoTEs
effectively treated with outpatient. high-dose ciprofloxacin therapy with very
close follow-up.

What else, in the patient's history, should be addressed?

State that you would attempt to achieve strict glucose control. Hyperglyce-
mia in these patients does not necessarily coincide with disease onset but
may be exacerbated by the infectious process. Temporary adjustment (or
initiation) of insulin coverage may require inpatient admission and the assis-
tance of other medical services
State that you would encourage the palient to stop smoking. Aside from the
deleterious effects of smoking on general health, smoking can impair white
blood cell functi on and resolution of the infectious process

She returns to your clinic 10 days later feeling much bette1~ Her pain is
significantly reduced and the otorrhea has resolved. Ho1veveJ: she now
has left-sided facial weakness.

What do you do next? State that you would:

Admit the patient to the hospital and give 6 weeks of double-coverage,


antipseudomonal antibiotic therapy. While patients who suffer from stage l
disease (see below), this patient's development of facial nerve paralysis has
failed outpatient therapy with an appropriate oral fluoroquinolone and re-
quires more aggressive treatment. Double-coverage for pseudomonas is
favored in this scenario to limit the development of antibiotic resistance.
Specific antibiotic choices vary but an appropriate regimen would usually
. include IV ciprofloxacin or ceftazidime in conjunction with gentamicin
therapy. Ototopical antibiotic drops containing polymyxin, an aminoglycoside,
or a quinolone are also recommende~
Obtain cultures. At this point, repeat culture becomes important and efforts
should be made to secure the organism. Culture of otorrhea fluid and biopsy
and culture of granulation tissue should be perfom1ed. Culture and sensitiv-
ity results should be used to direct systemic therapy
Perform regular aural cleaning. Regular aural toilet is essential in the treat-
ment of all forms of OE. Suction and careful debridement should be rou-
tinely performed, and there should be judicious use of wicks
Aggressively treat and control the underlying disease process. As indicated
earlier, strict glucose control is required. In other patients, correction of
underlying immunodeficiency may be needed

Are there any further imaging tests that you would like to order?

Repeat Gallium scan: As indicated above, Gallium scans provide informa-


tion about active inflammatory conditions and revert to nom1al with resolu-
tion of the disease process. When used at regular intervals, they are useful
in monitoring disease progression and response to therapy

45
Otolaryngology Clinical Case Studies Chapter 1 -Otology

NoTES Contrasted, high-resolution CT scan or MRI of the temporal bone. Both CT


ai1d MRI are usefu I in determining anatomic extent of soft tissue disease and
inflammation. They are both useful in disease staging and detennination of
any surgical indications. CT scans are especially useful in identifying abscess
fonnation that might benefit from surgical drainage. Bony detail is superior on
CT scan. Bony destruction, however. is a late finding in temporal bone osteo-
myelitis and is not useful in the early diagnosis of this disease process

The CT scan in this patient shows some opacification of the mas10id air
cells and middle ear space with no bony destruction and no abscess. There
are diffuse inflammatory changes with enhancement in the peri-auricular
soft tissues and within the infratemporal fossa.

What are the various stages of MOE and what are their prognoses?

Gates proposed a staging system in 1992 as fo llows:


Stage I: Infection with limited soft tjssue involvement with or without
facial nerve involvement
Stage II: Multiple CN involvement
Stage III: Intracranial involvement

Facial nerve weakness in MOE most commonly is the result of extratemporal


involvement or involvement at the stylomastoid foramen. Isolated involvement
of the facial nerve does not portend a worse prognosis. Prognosis ~enerally
worsens as more CN are involved. Involvement of the ninth, tenth, and eleventh
CN should raise suspicion of lateral sinus thrombosis and involvement of the
fifth and sixth CN should raise suspicion of cavernous sinus thrombosis. Both of
these developments are commonly fatal.
( '
'·'
' When is surgical intervention warranted?
J'

State that surgery is rarely indicated but is mandated by progression of infection


~ ./.
despite maximal medical therapy. Surgery should be limited to debridement of
necrotic bone and drainage of any abscesses

How would you follow this individual? State that:

This patient will receive at least 6 weeks of therapy and that antibiotics will be
continued until there is a combination of:
• Clinical response: Absence of pain, granulation, otorrhea, etc.
Decreasing ESR: To normal levels
Nonnalization of Gallium or indium scans: Indicating resolution of active
inflammation

What are the fissures of Santorini?

They are fissures within the bony portion of the EAC through which, in addition
to the bony-cartilaginous junction, infection is thought to spread to gain access
to the i11fratemporal fossa and skull base. Infection is then thought to spread
along vascular and fascial planes. Infection may also reach the skull base di-
rectly through the temporal bone via the haversian system of compact bone

46
Chapter 1 - Otology Otolaryngology Clinical Case Studies

CAsE 10 - PARAGANGLIOMAS oF THE TEMPORAL BoNE NoTES


RaviN. Samy, MD, FACS

A 55 year-old African-American female presents ·with a chief complaim of


pu!smions in the left ear for rhe lasr 6 mnnrhs.

HPI: state that you would :

Obtain a more detailed otologic and medical history beginning with the chief
complaint

What additional historical information would you seek? State that you would ask:

Does the pulsation sound like her heartbeat? Does it coincide with her pulse?
Is it constant or intenn ittent?
Does she have any problems in the other ear?
Does she have any hearing loss, otalgia, aural fu llness, or otorrhea?
Does she have any complaints of dizziness?
Does she have a history of prior ear disease or ear surgery?
Has she developed any difficulty swallowing, hoarseness. or coughing dur-
ingmeals?
Does she complain of headaches, visual changes, facial twitches. or
hoarseness?
Does she have any relatives who have tumors of their head or neck?

The patient states that the pulsOLions are constant and coincide with her
pulse. It sounds like her heart beating iri her left ear: She has no com-
plaims of hearing loss. She has mild aural fullness in the ·left em: Her right
ear is normal. She has no otalgia, otorrhea, dizziness, headaches, vision
changes, facial twitching, swallowing difficulties, coughing paroxysms or
hoarseness. She has no prior history of ear disease or ear surgery.

PMHIPSH: · Breast cancer treated by mastectomy and radiation therapy


5 years ago
Allergies: Penicillin - Hives
Medications: Multivitamins, acetaminophen prn
FH: Father died from a myocardial infarction; otherwise, negative
SH: Librarian. Married. No children. No HJV risk factors.
No history of significant noise exposure. No histOJ:V of
tobacco use. Social drinker
ROS: Negative

What do you look for on PE? State that you would:

Perform a complete head, neck, and neurotologic PE, including vital signs
and otomicroscopy (with pneumatic insufflation)

47
Otolaryngology Clinical Case Studies Chapter 1 - Otology

NOTES PE:
11uili_: Temp: 99.3 F BP: 125 70. HR: 85. RR: 19. Wr: 155 lb.
GA: Middle aged African-A merican fe male in NAD and appear-
ing healthy
HEENT AD - No retraction packer. no fluid. normal mobilirv
AS - TM intact, reddish mass in inf erior aspecr of middle
ear: not all of the mass can be seen through the TM: insuf-
flation shows the TM LO have decreased mobility: blanch-
ing of the mass is noted (+ Brown :S· s ign) (see picture at
end of case)
Tun ing fo rk
examination: Weber midline, Rinne + A U (51 2H::.)
OCI OPI
nasal cavities: No masses. lesions, or other abnormalities
Eyes: No ery thema or conjunctival edema
Neck: Supple with no nodes or bruits
Neuro: CN normal. Normal gait, normal tandem gait, nonnal Rom-
berg, normal Fukuda

What is your differential diagnosis? State the following:

Glomus tympanicum/jugulare (paraganglioma)


Dehiscentlhigh-ridingjugular bulb
Malignant tumor (metastatic from recunent breast cancer)
Blood in middle ear (from barotrauma)
Abenant ICA/ICA aneurysm

What do you do next? State that you would order the following:

Detailed audiometric evaluation including PTA, SRI, SDS, tympanograms


• Fine cut (1 mm) CT scan of the temporal bones (axial and coronal views,
bone windows) to assess size, location, and type of lesion

Test results:

• Audiometry shows bilateral normal hearing. The PTA is 25 in the right


ear with an SRT of 20 and discrimination score of 96%. The left ear
has a PTA of 25, an SRT of 25, and discrimination score of 94%. Type
A tympanogram in the left ear and type A in the right ear. No prior
hearing test ava ilable for comparison
CT scan: Soft tissue mass noted in mesotympanum and portion of hypo-
tympanum; jugular bulb intact and separate from mass; carotid canal
intact and separate from mass; no mastoid disease: no bone erosion

Diagnosis:

Paraganglioma of the temporal bone (glomus tympanicum)

48
Chapter 1 - Otology Otolaryngology Clinical Case Studies

W hat treatment would yo u begin? State that yo u would tell the patient the NOTES
following:

There are 3 options available: Observat ion, radiation therapy, and surgical re-
section. The risks and benefits of each are discussed thoroughly with the pa-
tient. Due to the small size of the tumor. relatively young age and good health of
the patient, and low surgical risks, you recommend surgery. Surgical resection
would also al low pathologic examination of the lesion, which is imp011ant in a
([·)patient with a history of fllll al_ignancy. J~e ri sks of su r?i~a l resection inc lud ~
tumor recuJTence, TM per'forat1p,n, taste changes (due to tJlJUred chorda tympani
nerve)~facial nerve paralysis, 'tomplete hearing loss, ih]ury to carotid. artery,
~.:.~ excessive blood loss requ iring blood transfusions, as well as risk of anestheticl:' .
complications. Observation would not be recommended since the patient is rela-
tively young, has a history of malignancy, and has a small tumor. Radiation is not
recommended ~or the same reasons. The patient needs to understand that the
tum.or will grow with time at an unpredictable rate. Observation or radiation are
acceptable options for those refusing surgery, elderly patients, and patients with
significant comorbidities. After discussin g the above, the patient agrees with
you and would like to undergo surgical resection.

Would you need any additional radiographic imaging prior to proceeding with
.surgery? State the following:

Additional imaging options include an angiogram, MRA, and MRI with gado-
linium. Because this tumor is a glomus tympanicum, an angiogram is an unnec-
essarily invasive procedure. MRA scan would not provide any additional signifi-
cant information. State that you would obtain an MRI with gadolinium to assess
for any synchronous paragangliomas of the head and neck (e.g., carotid body
tumor, contralateral glomus vagale, etc.). MRJ was obtained in this patient and
was otherwise nonnal. ·

Since some paraganglioma secrete vasoactive substances, do blood tests need


to be done prior to surgery? If so, what types of tests? State the follo.wing:

Most paragangliomas do not secrete honnones. Symptoms and signs of hor-


mone secretion include tachycardia/palpitations, elevated BP, flushing, sweat-
ing, and diarrhea. Since there have been no known cases of hormone secreting
glomus tympanicum tumors and since this patient has no systemic symptoms,
you state that you will not order any ayl9jtional tests. However, if you had con-
cem about honnone secretion, serum catecholamine levels would be obtained
as would \J-I~j·nary metanephrine and vanillylmandelic acid. V I~
(~ • . (} -· . '! ~
Explain how you plan to remove this tumor. State the following:

Glomus tympanicum tum ors can be removed via transcanal, endaural, or


postauricular route with a mastoidectomy. In thi s patient, due to limited
hypotympanic extension. you decide to use an endaural approach with a
hypotympanotomy flap (superiorly based tympanomeatal flap). Bone would be
curetted until the entire mass is visualized. Cup forceps. bipolar cautery, and the
argon laser can al l be used to resect tumor and assist with control of bleeding.

49
Otolaryngology Clinical Case Studies Chapter 1 - Otology

NOTES Surgical technique:

Highlights and pitfalls:


Size ofthe tumor: One must determine the size of the tumor both preopera-
tively and intraoperatively. The tumor may extend into the hypotympanum,
mastoid, protympanum, middle cranial fossa floor, or posterior fossa. This
will allow for proper preoperative patient counseling for planning of the
incision, and the determination of the amount of bone removal needed for
complete resection. Furthermore, one needs to keep in mind the location of
the petrous ICA at all times in relation to the tumor. Avu lsing the carotid
artery can cause significant morbidity and potential mortality
TM/ossicular chain: During tumor removal, it is easy to damage the ossicu-
lar chain and cause sensorineural hearing loss, CHL, or mixed hearing loss.
Intraoperatively, one can plan to disrupt the incudostapedial joint to mini-
mize inadvertent cochlear damage. The incus may also be removed to im-
prove visualization. After tumor removal, the ossicular chain can b~ recon-
structed at the same time or during a 2"d stage procedure. If the ear drum is
damaged during tumor removal, it can be reconstmcted easily with temporal is
fascia
Facial nerve function: In all except the smallest tumors, it is wise to use a
facial nerve monitor during surgery. The. monitor is helpful in cases of tu-
mors that abut the facial nerve or erode the fallopian canal. The use of the
monitor may minimize the risk of facial nerve injury and is prudent

How would you follow this patient?

Follow-up at one week post-op for wound check


Follow-up at 8 weeks post-op audiogram
Follow-up every 6 months to 1 year for recurrence. The overall risk of
recurrence in resection of glomus tympanicum tumors is 5%. State that the
main method to assess for recurrence is via follow-up clin ical exams (on a
yearly basis) and with audiograms. In addition, one can assess for recur-
rence with radiologic scans (CT, MRJ) or radionuclide scans (octreotide
scintigraphy). State that you plan to obtain an MRJ scan one year postop-
eratively. MRl scans can pick up tumors as small as 5 mm; octreotide scans
typically pick up tumors 1 em or larger

50
Chapter 1 -Otology Otolaryngology Clinical Case Studies

N OTES

References:

1. Heth J. The basic science of flomus jugulare tumors. Neurosurg Focus.


2004; 15(2):E2.
2. Noujaim SE, Pattekar MA, Cacciarelli A, Sanders WP, Wang AM.
Paraganglioma of the temporal bone: role of magnetic resonance imaging
versus computed tomography. Top Magn Reson Imaging.
2000;11(2):108-22.

51
Otolaryngology Clinical Case Studies Chapter 1 - Otology

NOTES CASE 11- TEMPORAL BONE FRACTURE


David Osterhus, MD

A 31 year-old male construction worker presents to the ER after having


fallen from a scaffold. He has a headache, decreased hearing on the left,
and left-sided bloody otorrhea.

HPI: state that you would:

With the assistance of the trauma service, follow Advanced Trauma Life
Support protocol. (Given the fact that the patient has sustained a poten-
tially significant traumatic injury, must assume that other injuries may have
occurred)
Following initial assessment and stabilization if necessary, you would obtain
a detailed account of the events surrounding the trauma and a detailed
medical history

What additional historical information would you seek? State that you would:

• Establish adequate airway, breathing, and circulation


Determine any major disability and expose the patient for examination
• Ask questions regarding the nature of the fall , loss of consciousness, and
mental status
How much bleeding is present?
Is there CSF otorrhea?
Is otorrhea bloody?
Is there any bruising?
What is the CN exam?
Is there facial nerve weakness? Has facial nerve paralysis been present in
the past?
What is the status of the TM?
What is the nature of the hearing loss?

Patient reports falling from a second floor scaffold to the ground which
was hard dirt. He reports hitting his head but does not recall the events
surrounding his injury. He has been alert and oriented throughout trans-
fer and arrival in the ER. He denies drinking alcohol or using other in-
toxicants. He reports having had normal hearing but now says that his left
ear feels "blocked up. "

PMH: Negative
PSH: Negative
Allergies: NKDA
Medications: None
FH: Diabetes
SH: Tobacco; no ETOH; construction worker

52
Chapter 1 -Otology Otolaryngology Clinical Case Studies

What would you look for on PE? State that you would : NoTES

Perfom1 a complete head and neck PE including CN exam and microscopic


ear exam

PE:
J!i1a/.s.: Temp: 98.6 F, BP: 1-15185. HR: 95. RR: 20
GCS: 15, breathing easily. Cervical collar in place

CN 2-12 grossly intact. Facia/ nerve symmetric, HB !IV!.


Tender to palpation over left mastoid with no discoiOT-arion.
Non-clotted blood in left ear canal. Left TM poorly visualized. Right exter-
nal canal and TM are normal.

Tuning fork examinarion: Weber lateralizes to the left


Rinne reveals AC>BC AD: BC>AC AS

What is your differential diagnosis?

Laceration of the ear canal


TM. perforation
Temporal bone fracture
Carotica-tympanic fistula

What diagnostics would you request? State that you would:

Order a non-contrasted, head CT scan. In this patient who has sustained


_acute head trauma and a possible loss of consciousness, a non-contrasted
CT scan of the head is needed to evaluate for an intracranial hemorrhage.
Although MRJ may also detect the presence of acute intracranial blood, it is
Jess useful in the trauma patient as it is more time-consuming and generally
poor at diagnosing skull fractures. Bony detail is far superior on CT scans
of the head. Dedicated CT scans of the temporal bone are usually not
required unless there is facial nerve paralysis and surgical intervention is
anticipated
If a traumatic vascular fistula is clinically suspected, then a contrasted CT
scan should be considered (auscultation of the mastoid may reveal a bruit in
this case)

The head CT scan shows a longitudinal fracture of left temporal bone


extending from the area anterior to otic capsule through tympanic cavity o

and posteriorly through mastoid p ortion. Opacification of mastoid air cells


and middle ear space. No evidence of intracranial blood or other acute
injury.

53
Otolaryngology Clinical Case Studies Chapter 1 - Otology

NOTES What would you do next? State that you would:

o Perform serial examinations - ' Re-evaluate patient's mental status. facial


nerve exam. and otOIThea at regular intervals. The patient remains stable
Perfom1 a microscopic ear exam - Clean and suction blood from the ear
canal. The TM shows a simple laceration with hemotympanum. Some phy-
sicians advocate the use of aura I iTTigation with either acetic acid or antibi-
otic drops to hasten the clearance of debris within the EAC. These prepa-
rations. however, can be ototoxic in the presence of a TM perforation and,
moreover, may camouflage the presence of a CSF fi stula
Obtain an audiogram - Although fom1al audio logic testing is often not pos-
sible in the acutely injured patient, attempts should be made to obtain an
audiogram as soon as possible. Results of audiologic testing will aid in diag-
nosis, patient counsel ing, and consideration of surgical intervention. The
patient's audiogram shows that the right ear has normal bearing. The left
ear shows a 25-30 dB CHL. The bone lines are symmetric

What do you do if your patient develops clear drainage from the affected ear?

State that you would test the fluid for CSF. Persistent, thin or clear otorrhea
should raise suspicion for a CSF fistula. Although demonstrating a halo sign
or testing the fluid for glucose are methods that have sporadically been used
to test for tb~ presence of CSF, neither method is very sensitive or specific.
Detection of B2-transferrin within the· sample, however, is both sensitive
and highly specific. At least 1.0 cc of fluid is usually required to perfom1 this
test

Diagnosis:

Longitudinal temporal bone fracture with hemotympanum and CHL. No


evidence of facial nerve injury or CSF otorrhea.

What are the treatment options and their complications? What would you do
and why? ·

Clinically observe the patient &nd provide reassurance- Most non-compli-


cated fractures of the temporal bone require no surgical intervention. The
CHL is most often a result of the hemotympanum (and possibly the TM
laceration) and should return to nonnal with resolution of the blood within
the tympanic cavity
• Give strict wamings- The patient should notify a physician or return to the
ER with the development of facial nerve weakness of any degree. Delayed
facial nerve paralysis has a generally good prognosis but may benefit from
the administration of systemic corticosteroids
Recommend dry ear precautions- Wet blood in the ear canal is a wonder-
ful medium for bacterial growth. As indicated above, antibiotic or other
topical preparations should be avoided as they may be ototoxic in the pres-
ence of a TM perforation. D1y ear precautions and regular microscopic
cleansing of the canal should suffice in resolution of debris within the canal

54
Chapter 1 - Otology Otolaryngology Clinical Case Studies

Recommend against prophylactic antibiotics- They are not likely to pre- NoTES
venl infection and they may result in the selection of a resistant organ ism if
an infection occu rs
AtTange for close follow-up - Repeat examination with formal audiologic
testing

What if your patient develops facial paralysis? What features portend a worse
prognosis?

State that the development of a complete facial paralysis would indicate a


worse prognosis. Delayed facial nerve paralysis, however, generally has a
better prognosis than immediate paralysis

What medications should be administered with the development of complete


facial nerve paralysis? State that you would:

Administer. high-dose, systemic corticosteroids- If not contraindicated by


the patient's other injuries, 1 mg/kg/day or greater of prednisone or equiva-
lent may decrease facial nerve edema and improve outcome
Initiate measures to protect the affected eye - Hourly saline eye drops or
artificial tears and nightly ophthalmic ointment should be used for corneal
protection. Nocturnal taping of the upper lid and plastic shields may also be
used
Consider administering antivirals-Antiviral medications are clearly benefi-
cial in the treatment of herpes zoster oticus and are often used empirically
for the trea~ment of Bell's palsy. The benefit of antiviral medications in the
treatment of traumatic facial paralysis is less clear

Are there any other adjunctive procedures that you might consider in this setting?
~tate that you would:

Consider placing a gold weight in the upper lid for improved eyelid closure
and corneal protection. Gold weight placement is a simple, outpatient pro-
cedure that may be performed under local anesthesia and is easily revers-
ible. It should be considered early after the onset of complete paralysis

What diagnostics would you request? State that you would:

Order ENOG and EMG. ENOG is an objective test of facial nerve function
that measures compound muscle action potentials that are elicited by elec-
tric stimulation. EMG is used to measure the electrical activity of striated
muscle

ENOG is most useful if perfom1ed between day 6 and day 21 after the onset of
paralysis. Three to five days are required to pass prior to the use of this modality
in order to allow Wallerian degeneration to occur. After 21 days, differential
neural regeneration leads to dyssynchronous neural discharge and overestima-
tion of neural degeneration. EMG is most useful beyond 21 days, when ENOG
is no longer infom1ative. On EMG testing, the presence of fibrillation potentials
indicate neural degeneration, while the presence of polyphasic action potentials
indicate neural regeneration.

55
Otolaryngology Clinical Case Studies Chapter 1 - Otology

NOTES What do you do if your patient has greater than 90'Yo degeneration? State that
you would:

Recommend surgical intervention with facial nerve decompression. Patients


with this degree of degeneration have a less favorable prognosis, with only
50% of patients achieving Hause-Brackman grade I-ll. Facial nerve de-
compression may improve outcome in this subset of patients and should be
offered. While controversy surrounds performance of facial nerve decom-
pression in the case of Bell"s palsy, there is much less disagreement in the
setting of trauma (i.e., it is logical to reestablish nerve continuity in the case
of disruption)

Patients who demonstrate less than 90% degeneration within the first 3 weeks
have an 80-100% chance of favorable recovery (House-Brackman I- II).

What other information on a high-resolution CT scan of the temporal bones


may also lead you to perform facial nerve exploration?
0
The presen~ of diastasis of the fracture segments along the course of the
facial nerv&r the presence of a bony spicule within the fallopian canal may
also lead you to perform facial nerve exploration

What surgical approach should be used for facial nerve decompression?

State that if hearing is present, a middle cranial fossa approach to decom-


press the labyrinthine segment and the geniculate ganglion should be per-
fom1ed. The remaining tympanic and descending segments may be decom-
pressed through a standard post-auricular, trans-mastoid approach.lfhear-
ing is absent, the entire nerve may be decompressed through a trans-mas-
toid approach with the addition of a labyrinthectomy

Decompression involves exposing 180 degrees of the nerve along its entire
course and opening the epineural sheath.

What is the narrowest portion of the fallopian canal?

e State that the labyrinthine segment is the narrowest

What is your plan if your patient develops CSF otorrhea? State that you would:

• Clinically observe you patient and recommend head of bed elevation anci
stool softeners. Most cases of traumatic CSF fistulae resolve spontane-
ously and do not require surgical intervention
While opinions vary, there is no evidence to suggest that prophylactic antibi-
otics in this setting decrease the risk of meningitis. Again, they may result in
the selection of resistant organisms if an infection occurs ·

56
Chapter 1 - Otology Otolaryngology Clinical Case Studies

CAsE 12 - BELL 's PALSY NoTES


Neelesh Meh endale, MD

A 19 year-old Caucasian fe male stmes that she cannor move the lefi side
of her f ace.

HPJ: state that you would:

Obtain a detailed medical history beginning with the hist01y of present illness

What additional historical infor·mation would you seek? State that you would ask:

When did the present illness begin?


Was the paralysis sudden onset or gradual?
Has the patient experienced any similar episodes in the past?
Was there any antecedent trauma?
ls ther~ any associated hearing loss, vertigo, or tinnitus?
Was there any otalgia?
Is there any weakness of the extremities or trunk?
• Was there any facial pain?
Do you have altered sensation in your face?
Have you had any facial rash or vesicles?
Have you had any rash on other part of your body?
Have you been exposed to tick or insect bites? History of recent camping
or outside activities that could increase risk of insect or tick bite?
Is there hypersensitivity to noise?
Has there been any alteration in your sense of taste?
Are you having any difficulty with swallowing?

The patient reports that she was in her usual state of health until she awoke
this mornin·g and found she was unable to move the left side of her face.
This has never happened before to the patient. She feels like the right side
of h~r face is movingfine and she denies any extremity or trunk weakness.
She denies any history of trauma over the last 5 years. She has not noticed
any change in her hearing 'and denies experiencing vertigo, tinnitus. or
otalgia. She denies the presence of a facial rash or rash anywhere else on
her body. She also denies any hyperacusis, facial hypesthesia, facial pain,
dysgeusia, or swallowing difficulties. The patient also denies any recent
insect or tick bites.

PMH: Gl, at age 17


PSH: Septoplasty at age 16
Allergies: NKDA
Medications: None
SH: Occasional alcohol; ~ ppd smoking for 4 years: No JVDA or
illicit drug use: student at local junior college
ROS: Patient denies any preceding ear infections or upper respi-
ratory tract infections. The remainder of the ROS is normal

57
Otolaryngology Clinical Case Studies Chapter 1 - Otology

NOTES What would you look for on PE? State that you would:

Perform a complete head and neck PE with specific attention to the oto-
logic exam and CN exam

PE:

l!lw.Ls_: Temp: 98.3 F. BP: JJ 2164. HR: 72. RR: 12


GA: Well-developed, we/1-neurished fe male with ob-
vious facial asy mmetry ar rest. Normal tone of
voice
Scalp: No obvious deformiTies
Eves: EOMJ, vision grossly intact. Patient demonstrates
Bells Phenomenon (superior-temporal rotation of
the globe on attempt to close ipsilateral lid). Cor-
neal sensation is intact
Ears: Auricles and concha! bowls demonstrate normal
appearance without evidence of vesicular erup-
. tion. EAC's and TM's clear AU, no hemotympanum,
no perforation
Weber to midline, Rinne test shows air conduc-
tion > BC bilaterally
Facial skeleton: No palpable abnormalities. no rash or evidence
of vesicles
Nose: Anterior rhinoscopy reveals slightly boggy infe-
rior turbinazes and a septal spur to the left side.
Otherwise, there are no abnormalities
OCIOP: Normal dentition, class 1 occlusion, tonsils 1+ bi-
laterally, no palpable or visible lesions
Neck: No palpable adenopathy
CN: II-XII intact bilaterally with the exception of left
CN VII
She is unable to completely close her left eye, but
she lias the slightest movement in all divisions of
her left facial nerve. Sensation in all divisions of
CN V is intact bilaterally
Extremity: Strength is 515 bilaterally
Skin: No abnormalities seen
Cerebellar exam: WNL by Romberg and alternating finger-to-nose

What is your differential diagnosis?

This patient presents with acute facial nerve paresis without a history of
trauma or infection. At this point the differential diagnosis would include:
Herpes Zoster (Ramsay Hunt syndrome)
CVA
Guillain-Barre, autoimmune disease. sarcoidosis
Lyme disease
HIV

58
Chapter 1 - Otology Otolaryngology Clinical Case Studies

Kawasaki disease NarEs


Betrs palsy (idiopathic facial nerve paralysis). It is impot1ant to remem-
ber that Bell's Palsy is a diagnosis of exclusion but can usually be dif-
ferentiated from other causalities with a thorough history and PE

What diagnostics would you request? State that you would:

Not perfom1 any adjunctive tests at this time and would follow the patient's
clinical status

History and PE identif/ an acute left facial paresis without history of trauma
or infection. At this point the patient meets the minimum diagnostic criteria
for Bell~· palsy (spontaneous idiopathic facial paralysis) which are: (1)
paralysis or paresis of all muscle groups of one side of the face, (2) sud-
den onset, (3) r;zbsence of otolog ic or cerebellar disease, and (4) absence
of CNS disease. Electrophysiologic testing can be considered in the work-
up of idiopathic facial nerve paralysis if the paralysis is complete. If the
. faci(ll nerve paralysis was due to trauma, one must wait 3- 5 days after the
onset before electrophysiologic testing will become abnormal (to account
for Wallerian degeneration). Since this patient presents with incomplete
facial paralysis, no further tests need to be performed acutely.

Diagnosis:

Left-sided Bell's palsy with incomplete paralysis (HB IV-VIVJ).

What are the treatment options and their complications? What would you do
and why?

Place this patient on a 1 week course of corticosteroids (1 mg/kg/day) and


antiviral regimen with a steroid taper
Corneal protection -Instruct the patient to use artificial tears during the day
and lubrication in her left eye at night
You may also instruct the patient to ·tape the eye shut at night. Alternatively,
an ocular terrarium can be used to maintain moisture for the corneal surface

The most important acute consideration in Bell's palsy is corneal protection for
incomplete eye closure. The patient should be prescribed artificial tears for use
throughout the day, and ophthalmic lobe for use at night. The patient can also be
instructed to tape the eye closed while sleeping. Current consensus holds that
individuals with Bell's palsy will likely benefit from treatment with a course of
corticosteroids as soon as possible. There is also a growing consensus that anti-
viral medications should also be included, with either Famciclovir or Valacyclovir
preferred over the poorly absorbent parent compound Acyclovir. These medi-
cations are thought to decrease the incidence of synkinesis, Jessen nerve degen-
eration, and may help hasten recovery. The patient should be placed on a 7- 10
day cot1icosteroid ( 1 mg/kg/day) and antiviral regimen with a steroid taper. The
patient should be counseled that the most important prognostic factor is whether
the paralysis is complete or incomplete. If the paralysis is incomplete, there is a

59
Otolaryngology Clinical Case Studies Chapter 1 - Otology

NOTES 95-100% chance that the patient wil l recover with no identifiable sequelae.
Although the overall probabi Iity of acceptable facial nerve recovery (defined as
HB I or II) is 85%, patients over 65 years old and patients with HTN are less
likely to regain full facial function. Their chances of an acceptable recovery are
probably no higher than 50-60%

How would you follow this individual? State that you would:

Schedule a fo llow-up appointment in 3-5 days.lfthe facial nerve function


has improved at that time, then no flllther testing needs to be pe1fom1ed and
the patient can be reassured
•- The patient should still be fol lowed periodically to ensure that worsening of
facial function does not occur
Iffacial nerve function deteriorates, then consider electrodiagnostic testing
at the point that complete paralysis occurs (electrodiagnostic testing is of
limited value while facial function is present)

The patient arrives in your office 4 days later for her follow -up appoint-
ment. She has been taking the medicine as prescribed Yow' PE now reveals
that she has complete flaccid facial paralysis with no tone (HB VI/VI).

What do you do next? State that you would:

Order ENOG and EMG to be performed within 1- 2 days

At this point, electrodiagnostic testing is the most appropriate step and ENOG
and EMG should be performed. If available, the most accurate tests to evaluate
for extent of degeneration are ENOG and EMG Patients who retain I 0% of
functioning nerve fibers du_ring the first 3 weeks of injury are likely to regain 80-
100% of normal facial function. However, once 950/o offunctioning nerve fibers
are lost, th~ chance for a favorable recovery (HB I or II) drops to 50%. If
ENOG and ENG are not available, facial testing may also be performed with a
Hilger nerve stimulator. lfthe differential of3.5 mA between the minimal cur-
rent" that is required for threshold stimulation of facial movement on each side
exists, then it is likely that progressive degeneration has occurred. Facial nerve
testing should be repeated every 3-4 days until facial function returns or the
response to electrical stimulation is lost. Imaging of the facial nerve can also be
performed using MRJ. Enhancement of the facial nerve can be a common
finding in healthy individuals and can make the interpretation difficult. MRI can,
however, be useful in differentiating inflammatory from neoplastic changes in
the facial nerve.

Test results:

ENOG - Compound action potential for the left side is 15% of the normal
side
EMG - Two detectable voluntary motor units

60
Chapter 1 - Otology Otolaryngology Clinical Case Studies

What do you do next? NoTEs


The patient should be brought back for retesting with ENOG and EM G >
4 days later

Test results:

ENOG - Compound potemial f or the left side is 5% of the normal side


Elv!G - No detectable vo/wuarv mowr units

What is your next step?

Since the ENOG and EMG show >90% nerve degeneration, this patient
should now be counseled that she has a 50% change that significant re-
sidual fac ial weakness and synkinesis will occur following regeneration of
the facial nerve
This patient can be offered decompression of the labyrinthine segment of
the facial nerve, but opinions differ regarding facial nerve decompression
for Bell's palsy. If the patient elects to have this done, it is important to note
that Fisch has documented that the labyrinthine portion of the fallopian ca-
nal is the most likely point of nerve compression and therefore should be
included in the decompression. Further, the procedure theoretically should
be performed within I ~-2 1 days from the onset of facial nerve paralysis

What are the surgical options for Bell's palsy?

The role of surgical management for Bell's palsy is controversial. Surgery


involves decompression of the meatal foramen of the facial nerve at the
distal end of the lAC. Decompression of the tympanic and mastoid seg-
ments ofthe facial nerve is ineffective. Some authors have shown statisti-
cally significant improvements in long-term facial recovery of patients with
greater than 90% degeneration by ENOG who underwent surgical decom-
pression within 3 weeks as compared to those who were managed medically
The patient should be counseled as to the risks of the surgical procedure
that most commonly involves a middle cranial fossa approach. Although
complications are unusual in the hands of an experienced surgeon, the risks
can include vertigo, bearing loss, facial nerve paralysis, stroke, and death

Surgical technique: State that you would:

Perfom1 a left-sided middle, cranial fossa approach for decompression of


the labyrinthine segment of the facial nerve
Highlights:
General anesthesia
Incision in temporal skin from root of helix onto temporal cranium
Divide temporalis muscle to expose temporal cranium. zygoma, and
roof ofEAC
Rectangular craniotomy centered;/; anterior and !13 posterior to EAC
Inferior aspect of craniotomy is at zygomatic root

61
Otolaryngology Clinical Case Studies Chapter 1 - Otology

NOTES Elevate dura from bone flap and expose floor of middle fossa
Place self-retaining middl e-fossa retractor
Identify arcuate eminence and locate lAC by relationship to greater
superficial petrosal nerve, geniculate ganglion. ru1d arcuate eminence
Expose peri geniculate region with drill
Skeletonize the roof of the lAC from geniculate to the meatus
Remove additiona l bone from lAC with microcurene
Use microblade to incise dura from meatus to geniculate ganglion
Seal roof of lAC with free fat or muscle graft
Replace bone plate and close subcutaneous and cutaneous layers
Pitfalls:
Complications include stroke, CSF leak, conductive and sensorineural
hearing loss
Labyrinthine segment runs between the upper basal turn of co-
chlea and ampulla of sse
Prevent fenestration of cochlea by identifying blue line of basal
tum of cochlea immediately anterior to labyrinthine segment
Occlude inadvertent fenestrations immediately with bone wax
Minimize dural trauma by using the proper size burr and use dia-
mond burrs when possible
Repair dural tears primarily or with temporal is fascia patch
Reduce temporal lobe injury by maintaining gentle traction and re-
duce intracranial pressure with medical management (redu.cing
PC0 2, mannitol, corticosteroids)
Prevent CSF leak with occlusion of roof of lAC with free fat or
muscle graft

What are complications of Bell's palsy?

Exposure keratitis/corneal abrasion. This is perhaps the most feared conse-


quence ofBell's Palsy resulting from incomplete eye closure. Therefore, it
is important to stress to the patient the importance of eye lubrication and
eye care. If the patient is sloyv to recover facial function, surgical proce-
. dures can help address incomplete eye closure. These procedures include
an upper lid gold weight, tarsal pillars, tarsorrhaphy, and lower lid lateral
canthopexy
• Persistent synkinesis. This can be a problem for patients recovering from
severe facial nerve degeneration. Some authors are now advocating the
use of selective BOT injection for unwanted facial movement
Persistent facial asymmetry. For gross persistent facial asymmetry, the pa-
tient has options for static facial slings or reanimation procedures (Xll-Vli
crossover, free flaps, etc.) to attempt to achieve some degree of facial
symmetry

62
Chapter 1 - Otology Otolaryngology Clinical Case Studies

References: NoTES

1. Fisch U. Surgery for Bell·s pal sy. Arch Otolarnygol Head Neck Surg.
1981 ;107:1 - 1I.
') Fisch U, Felix H. On the pathogenesis of Belrs palsy. Act Otolaryngol.
983;95:532-8.
3. K.imtush JM, Linstrom CJ , McCann P, et al. Early gold weight implantation
for facia l paralysis. Otolaryngol Head Neck Surg. 1990:103: 1016- 23.
4. Mattox DE: Clinical disorders of the facial nerve. In: Cummings CW,
Fredrickson JM, Harker LA, Krause CJ, Schuller DE, Richardson MA,
editors. Otolaryngology-head and neck surgery. 3'd ed. St. Louis: Mosby-
Yearbook; 1998,2767-84.
5. May M, Klein SR, Taylor FH. Ind ications for surgery for Bell's palsy. Am J
Otol. 1984;5:503-12.
6. Murphy TP, Teller DC. Magnetic resonance imaging of the facial nerve
during Bell's palsy. Otolarygol Head Neck Surg. 1991; 105:667-74.
7. Peiterson E. The natural history of Bell's palsy. Am J Otol. 1982;4: 107-11.
8. Selesnick SH, Patwardhan A: Acute facial paralysis: evaluation and early
management. Am J Otol. 1994; I 5:387-408.
9. Stankiewicz JA. Steroids and idiopathic facial paralysis. Otlaryngol Head
Neck Surg. 1983;92:672-7.

63
Otolaryngology Clinical Case Studies Chapter 1 - Otology

NoTES CASE 13 - CHRONIC SuPPURATIVE OTITIS MEDIA


David Parry, MD

A 38 year-old female presents wirh chronic intermittenr drainage fi"om her


left ear for 12 years.

HPI: state that you would:

Obtain a detailed medical history beginning with a history of the present


illness

What additional historical information would you seek? State that you would ask:

What is the consistency and color of the drainage?


What relieves it?
What brings it on?
What were the circumstances surrounding the onset of the drainage?
Has the affected ear been operated on?
Hearing Joss, otalgia, dizziness, or tinnitus?
Associated facial weakness, headache, or ocular complaints?
Prior treatment if any?

She states that the onset occurred after a scuba diving trip in the Bahamas
when she heard a "pop '' and fell water fill her ear on ascent from a dive.
The ear began to drain a clear otorrhea and was treated with otic
Cortisporin which provided transient relief The drainage recurred, how-
eve!; about a month later at which time it was described as "thick and
smelly..'' Again, it cleared with otic drops. It continued to drain periodi-
cally several times per year without known inciting factors. Drainage would
consistently respond to treatment with otic drops until the last yeC/1; at which
time the drainage became persistent (jar the past 3 months) despite vari-
ous otic and oral antibiotic preparations. She denies associated facial, or
other neurologic complaints except that she notices she has lost some hear-
ing in the affected ea1: She had no prior ear problems, and has not been
operated on.

PMH: G3P2, Obese. and type 2 DM controlled with diet. She states
her blood glucose is well controlled
PSH: 2 C-Sections
SH: Denies tobacco use, occasional ETOH. and no illicit drug
use. Married. Works as a baker
M edications: Oral contraceptive
Allergies: NKDA
FH: Dad and mom alive. Mom with DM and COR artery disease

64
Chapter 1 - Otology Otolaryngology Clinical Case Studies

What would you look for on PE? State that you would: NoTES

Perform a complete head and neck exam including vital signs and a micro-
scopic ear examination

PE:

Wt: 245 fbs, Ht: 5jt. 8 in. , HR: 88, BP: 123167, RR: 12
J!.iLals.:
GA: Pleasant. NAD.
Face: Symmetric without lesions. Hause/Brackman 116 bilaterally
OPIOC: WNL
FOM/BOT WNL
Eyes: EOMI bilaterally, PERRLA
Ears: Under the microscope
AD - Normal pinna, EAC. TM mobile. No evidence of MEE.
AS- Normal pinna. EAC with purulent drainage. TM with 45%
posterior inferior perforation. Granulation tissue visualized
in middle ear through perforation extending onto medial EAC.
Ossicles not visualized. No squamous debris is noted
Weber: To left
Rinne with 512Hz tuning fork: Air > bone AD/Bone > A ir AS
Neck: No adenopathy, trach midline, no stridor
New·o: CN grossly intact bilaterally. Subjective decreased hearing AS
Nose: Septum midline. Pazent

What diagnostics would you request?

An endoscopic. evaluation of the NP may be considered in a patient with


unilateral ear pathology to assess for neoplasm. A neoplasm in this location
could obstruct the Eustachian tube preventing adequate ventilation of the
middle ear cleft, thus contributing to the chronic otologic problems. This
scenario is more likely in the case of chronic serous otitis media (especially
in adult patients)
An audiogram would be important to obtain in the initial work-up of this
patient

Test results:

This patient did not have any findings in her NP on transnasal endoscopy.
Her audiogram shows a 25 dB low frequency CHL on the left sloping to 35
dB in the high frequencies. Her tympanograms and reflexes do not provide
any additional information.

What is your differential diagnosis at this point?

CSOM
Cholesteatoma
Langerhans cell histiocytosis
Neoplasia foreign body

65
Otolaryngology Clinical Case Studies Chapter 1 - Otology

NoTES Diagnosis:

This patient has CSOM with granulation rissue visuali::ed through a pel:fo-
rarion of the middle ear This palienr does not appear to be suffering from
any of the complications of this disease. Choles1eatoma and neoplasia are
considered as pass ible contributing pathologic processes.

What are the treatment options? What would you do and why?

lf granu lation tissue is present in canal, would biopsy


Follow the patient closely with frequent aural toi let and follow-up of biopsy
results. If the purulent drainage continues, systemic therapy with culture and/
or surgical control of granulation tissue or cholesteatoma will be considered
If the patient fa iled to respond to appropriate topical treatment and aural
toilet, then consider surgery. If squamous epithelial debris, consistent with
cholesteatoma, is noted within the perforation or middle ear, then surgery is
indicated

What may be some complications of this disease process if left untreated?

I Sigmoid sinus thrombosis


• 'J Brain abscess
• 3 Otitic hydrocephalus
~ Extradural abscess
• .5Meningitis
6 Petrositis
• -?Labyrinthitis

CSOM, in the absence of proper treatment, can progress into a variety of mild
to life-threatening suppurative complications. These complications can be sepa-
rated into 2 subgroups:

Intratemporal complications include:


- ; Petrositis
- :;::, Facial paralysis
- ~Labyrinthitis
Intracranial complications include:
-I Lateral sinus thrombophlebitis
_.;z Meningitis
--2, Intracranial abscess

What are the treatment options and their complications? What would you do
and why?

Aural toilet with suctioning of purulent discharge and application of otic drops

Although suppurative otitis media has traditionally been treated with systemic
antibiotics, the use of ototopical antibiotic preparations in uncomplicated cases
offers some unique advantages. The concentrations of antibiotic that can be

66
Chapter 1 - Otology Otolaryngology Clinical Case Studies

ach ieved at the surface of the middle ear mucosa are hundreds to thousands of NoTES
times higher than those Jttainable during system ic therapy. Resistance to these
concentrations is extremely rare. However, if the suppuration continues despite
aggressive aural toilet and topical therapy. systemic medication based on culture
results should be considered.

A steroid containing otic preparation might be of additional benefit in the event


that granulation tissue is present in the middle ear. By controlling the granulation
tissue in the middle ear, the delivery of antibiotic drops can be irnproved. With-
out adequate control of granulation tissue, the antibiotics cannot reach th~ of-
fending organisms and the infection will persist.

Is there anything else you would consider doing at the first visit?

A biopsy ofthe granulation tissue should be done at the first visit to check
for neoplasia or granulomatous disease (such as TB)

When would you consider systemic therapy?

Systemic therapy should be reserved for cases ofCSOM that fail to respond to
topical therapy. Topical therapy presumably fails because antibiotics cannot reach
infected tissues. Systemic therapy is expected to succeed in penetration of the
tissues.

If a focus of infection in the mastoid cannot be reached by topical drops, there


is a reasonable chance that systemically administered antibiotics can penetrate
into these areas in sufficient concentrations to control or elii11ii1ate infection.
Oto~opical therapy is generally continued once systemic therapy has begun.
The ability to perform reliable aural toilet may be as important as the systemic
antimicrobial therapy in eliminating the disease for some patients.

Prior to instituting systemic therapy, a culture should be obtained for culture .and
sensitivity, Sensitivity testing is important when systemic therapy is being con-
sidered. The antibiotics should be selected on the basis of the resulting sensitiv-
ity profile. The narrowest spectrum antibiotic with the fewest adverse effects
and complications should be used.

What further diagnostics could you request?

CT scanning: IfCSOM is unresponsive to medical treatment, a fine-cut CT


scan of the temporal bone may provide an explanation. Possible reasons for
failed treatment include occult cholesteatoma or foreign body
CT scanning is a necessary adjunct to treatment if the clinician sus-
pects a neoplastic process or if the clinician anticipates intratemporal or
intra-cranial complications
A fine-cut CT scan can reveal bone erosion from cholesteatoma, os-
sicular erosion, involvement of petrous apex, coalescent mastoiditis,
erosion of the fallopian canal, and subperiosteal abscess

67
Otolaryngology Clinical Case Studies Chapter 1 - Otology

NOTES MRI: Use MRJ of the temporal bone and brain ifintratemporal or intracra-
nial complications are suspected
By clearly depicting soft tissues. MRJ can reveal dural inflammation.
sigmoid sinus thrombosis. labyrinthitis. and extradural and intracranial
abscesses

The patient is non-responsive to medical therapy and surgery is thought to be


necessar-y. What surgery would you do?

A canal wall-up mastoidectomy with tympanoplasty and possible facial re-


cess dissection are initial options

Surgical technique:

Consent and mark surgical site, remembering to advise patient as to the


possibility of a staged procedure if cholesteatoma is encountered
Facial nerve monitoring can be considered, if available
Post-auricular incision
Elevation·of a tympanomeatal flap
Perfom1 mastoidectomy
Consider dissection of facial recess if additional middle ear exposure is
needed
Culture for fungi , bacteria, and TB
Biopsy any suspicious tissue
Repair of TM defect
Closure of post-auricular incision

Intraoperatively, how would your managemen·t change if cholesteatoma were


found?

In the event that cholesteatoma is encountered, then long tenn safety of the
ear must be considered. Given the high rate of recidivism in this situation,
long term evaluation of the ear must be provided. Possible options would
include:
Performance of a staged procedure in which the patient is retumed for
surgical exploration (second look procedure) at a later date (6-9 months)
to ensure no recurrence of cholesteatoma. If this option is selected,
then ossicular chain reconstruction, if necessary, can be performed at
either stage
Performance of an open cavity mastoidectomy and tympanoplasty (ca-
nal wall-down) can provide for improved clinical assessment in the event
that the patient is a poor candidate for further surgical procedures
In some patients, a single stage intact canal wall tympanomastoidectomy
can be followed with high resolution CT scans of the temporal bones.
In this case, surgical revision would be reserved for those patients in
whom CT findings are suggestive of recunence of cholesteatoma

68
Chapter 1 -Otology Otolaryngology Clinical Case Studies

What are the potential complications? NOTES

• l Facial nerve injury- If injUJy does not require repair. treat with steroids
with aggressive taper over 14 days
• .') Disequilibrium associated with injury to vestibu lar system
•0 Perversion oftaste associated with injury or sacrifice of the chorda tympani
nerve
• '1 Laceration ofsigmoid can almost always be controlled by placing Gelfoam
completely over injured area
• S Hearinab loss
• (:, Intracranial injury

How would you follow t his individual? State that you would:

Check cultures and direct antibiotic therapy appropriately


Check pathology
Plan second look operation if cholesteatoma was found at tl-ie time of surgery
Repeat audiogram in 4-8 weeks

69
Otolaryngology Clinical Case Studies Chapter 1 -Otology

NoTES CASE 14 - Onns MEDIA WITH MENINGITIS


Robert T. Adelson, MD

A 4 year-old male is referred to you for management of an ear infection


refractory to medical management.

HPI: state that you would:

Obtain a detailed medical history beginning with a history of the present


illness

What additional historical information would you seek? State that you would ask:

For how long has the patient had symptoms ascribed to an ear infection?
What are those signs/symptoms (ear-pull ing, otalgia, fever, irritability. sleep-
lessness, headache, otoJThea, anorexia, cough, sneezing, nasal congestion I
rhinorrhea, conjunctivitis, "dizziness" or clumsiness)?
What treatment has the patient received?
What has been the response to therapy?
Have any new or more worrisome symptoms developed (lethargy, neck
stiffness, photophobia, altered mental status)?
What is the patient's otologic history (with regard to prior infections, hear-
ing loss, and surgery)?
Does the patient have any congenital or acq uired disorders that may put
him at risk for complications of otitis media?

The patients mother reports that her son had been complaining of a head-
ache and pullbzg at· his left ear for 2 days when she took him to his pedia-
trician. The boy was diagnosed with a left A OM, started on amoxiciilin (45
mg/ kg!day, div ided BID), and told to return to the pediatricians office if
he developed a fever or failed to improve over the next 2- 3 days. The
patient has continued to run a low grade feve r (38.0 C) dnd complain of
ear pain; however, the patients mother became very concerned this morn-
ing when the patient was nearly impossible to arouse. The patient had
been unable to "sleep for 2 days and, though his mother initially believed
him simply to be exhausted, she notified their pediatrician when his tem-
perature was found to be 40.0 C. The patient has had approximately 4-5
ear infections during his life, his first at the age of 8 months-old, but none
for the last 6 months. The patient had a normal newborn hearing screen-
ing test, has readily understandable speech, communicates appropriately
in short sentences and has met milestones in other categories of develop-
ment without difficulty. The patient is otherwise healthy, has never suf-
fered bacterial infections of the paranasal sinuses or lungs and takes no
medication other than that prescribed for the present illness. The patient
has never had ear surgery.

After speaking with both the referring pedian·ician and the patient~ mothe1~
plans are made to evaluate the patient in the nearest emergency depart-
ment with experience in managing acutezv ill pediatric pacients.

70
Chapter 1 - Otology Otolaryngology Clinical Case Studies

PMH: A OM. as described above NOTES


Rece111 hisror_v of URI, 2 weeks p rior to present illness
(treated with a::irhromycin)
PSH: Jngui11al hernia repair at age 3 years old: no complications
Allergies: NKDA
MedicaTions: Amoxici/lin (45 mglkg!day, divided BID)
Immunizations: Up to date, including H. influenza, rype B
FH: No family members with either congenital or early onset
hearing loss
A paternal grandfather has had two operations for a "skin
cyst" of the right ear~ and is nearly deaf in the opermed on
ear
SH: The patient is in daycare 5 days a week, he was breastjed
for the first 7 months of life, he is never exposed to cigarette
. smoke, the family has no pets, the patient has no siblings
ROS: (With assistance from patients mother)
,The patient has .left otalgia and /eft-sided headache, anor-
exia, and sleeplessness and he has been somewhat more
irritable over the last fevl' days. No nausea, vomiting or
diarrhea. The patient was recenlly ill with an URI, but at
this time has no cough, nasal congestion, rhinorrhea or
sneezing. The patient has never experienced otorrhea.
Though he is lying prone and sleeping nOH·; he previously
demonstrated some clumsy movements while walking but
has not shown any unusual or rigid positioning of the neck
or back. The patient is sleeping in a darkened room: the
television has been turned off as the patient was complain-
ing that it hurt to look at the screen

What would you look for on PE? State that you would:

Perfonn a complete head and neck PE including vital signs and microscopic
ear exam, in addition to any other appropriate tests

PE:

l!lliJLs.: Temp: 40.0 C, HR: 105, BP: II5170, RR: 26


GA: WDWN WM, sleeping. The patient is difficult to arouse, but will
respond to pain and when his mother speaks loudly. The patient
keeps his eyes shut at all times and is breathing quietly without
stridor or stertor
Head: NCIAT, Normal facies
Nose: Erythematous mucosa, no lesions, no secretions
OPIOC: 2+ tonsils, symmetric,· uvula midline, symmetric gag reflex. No
lesions noted
Neck: Shotty cervical lymphadenopathy bilaterally (levels I, 11, 111).
No other masses
AD: EAC normal. TM shape and color is normal, pneumatic otos-
copy demonstrates a mobile TM, no MEE. The pinna and mas-
toid region are normal

71
Otolaryngology Clinical Case Studies Chapter 1 - Otology

NoTEs AS: EAC normal. TM is bulging and erythematous with white M.EE.
No granulation tissue. No TM peJ:foration. The EAC is dr_r. The
pinna and mastoid region are normal
Tuning fo rks are deferred (pat ienr age and general medical
condition)
New·o: The patient is very resistant to szanding up and refuses to walk
- instead he holds his head and cries. Facial motion is normal
and symmetric. Gag reflex and bilateral corneal reflexes are in-
tact. Pupils are equal and reactive to light, extraocular fnotion is
grossly normal, with a few beats of spontaneous leftward nystag-
mus. Ophthalmoscope exam finds no papilledema bilaterally
Brudzinski sign is negative (flexion of the neck does nor result
in .flexion of the hip and !-.:nee)
Kernig sign is positive (lying supine, the patient cannot extend
the leg at the knee while the thigh is jlexed toward the abdomen)
The neck is felt to be stiff as it is painful to palpation and can-
not be flexed such that the chin may touch the chest

What is your differential diagnosis?

The patient's otologic history, time-course of disease and PE favors a complica-


tion of AOM. These complications are classified as either intratemporal or
extratemporal, and are listed below:

lntratemporal complications ofAOM:


Acute mastoiditis is unlikely in .the absence of mastoid pain or tender-
ness; however, a masked ·mastoiditis (subacute mastoiditis) origina1ing
during the pat_ient's earlier URI is possible
Labyrinthitis is possible (serous vs. suppurative), given his nystagmus
and clumsy movements; however, it is difficult to test this patient appro-
priately
Petrositis is unlikely in the absence of otorrhea and with preservation of
normal extraQcular motion
·Intracranial complications ofAOM:
Meningitis is the most common intracranial complication of acute men-
ingitis, and must be the chief consideration for any acutely ill patient
with otitis media, fever, and meningeal signs. Upwards of 86% of pa-
tients with meningitis resulting from otitis media had at least l additional
intratemporal or intracranial complication, and these must be consid-
ered in the evaluation of suspected otitic meningitis
Extradural abscess, subdural empyema, focal otitic encephalitis, brain
abscess and lateral sinus thrombosis are intracranial complications that
may occur in conce1i with meningitis
Otitis hydrocephalus is Jess likely, as the patient does not have papilledema
Other CNS infections must be considered, as the otitis media may not
be the etiology of his neurologic symptoms. Acute bacterial meningitis
(N. meningococcal meningitis must be considered in a patient with a
recent URJ), acute aseptic meningitis and encephalitis are all valid dif-
ferentials for this patient's presentation

72
Chapter 1 - Otology Otolaryngology Clinical Case Studies

What are the treatment options and their complications? What would you do NoTES
and why?

Initial stabilization of the patient requires obtaining IV access. hydration .


assessment and management of electrolyte balance in addition to seizure
precautions
Once the patient is stabi Iized, one should undertake an expedient diagnostic
work-up includ ing both CSF analysis and radiologic imaging

Should radiologic imaging precede CSF analysis?

Yes. Both CT and MRl of the brain can identify mass effect as well as risk for
hemiation, both of which are contraindications to LP.

Which radiologic study should be used for this patient?

CT and MRJ can reliably identify mass effect that could result in herniation.
CT scans will provide an accurate study of the inflammatory state of the
temporal bone as well as identify congenital ear malfom1ations that can
predispose a patient to intratemporal and intracranial complications of otitis
media. MRI is superior to CT in demonstrating intracranial suppurative pro-
·cesses and depicting inflammatory conditions of the brain and meninges

What diagnostics would you request?

Though it would be ideal to have access to both imaging modalities, a con-


trast-enhanced· CT brain and high resolution CT temporal bone are suffi-
cien! in most cases, with MRI reserved for indeterminate cases or for evalu-
ation of coexistent infection

This patient undergoes the aforementioned contrast-enhanced CT scans


of the brain and temporal bone.

Test results:

CT brain: "No mass effect, no mass lesions, 'questionable ' meningeal


enhancement "
CT temporal bone: "Opacified left middle ear 'With air-fluid levels through-
out the mastoid air cells. No mastoid bone erosion or cortical erosion noted"

Should an MRI be obtained?

No. Although a contrast-enhanced MRJ is superior to CT in the diagnosis of


meningitis, the patient's history, PE findings and radiographic clearance for
LP make CSF analysis a more important study for the patient with sus-
pected meningitis

73
Otolaryngology Clinical Case Studies Chapter 1 -Otology

NoTES What further measures should be taken?

Now that meningitis is very likely. can you initiate empiric antibiotics? When
should antibiotic therapy begin?
Antibiotics are not given until CSF samples have been secured via aLP
LP may be undertaken when concern for herniation is alleviated by aCT or
MRl, demonstrating no intracranial mass effect
CSF should be sent for aerobic and anaerobic cultures as well as Gram
stain, cell count with differential, glucose and protein

Test results:

The initial LP results for this patient are as follows:


Color = xanthochromic
WBC = 10,000 (PMN= 80%)
Glucose = 35 mgldL [Serum Glucose = JJO mgldL}
Protein = 100 mgldL
Gram positive cocci in pairs and chains

Diagnosis:

The patient has PE and laboratory evidence consistent with ac·ure bacte-
rial meningitis caused by S. pneumococcus. In addition, the patien.t has a
serous or suppurative labyrinthitis. The most likely eriology is an AOM.

What is your choice of antibiotics?

Third generation cepbaJosporinsthat provide broad-spectrum coverage treat


most beta-lactamase producing organisms and demonstrate good CSF pen-
etration. Expanded coverage (with aminoglycosides or metronidazole) may
be necessary when culture and drug sensitivity results are available

This patient is started on Rocephin as the Gram stain, culture and drug
sensitivity results are pending. ·

How would the empiric antibiotic therapy differ if this patient had a history of
chronic otitis media?

Empiric therapy would cover anaerobic organisms in the patient suspected to


have a complication of chronic otitis media. Expanded coverage of anaerobes
in the pediatric pppulation would include aminoglycosides or metronidazole

Are any other medications helpful at this point in the patient's management?

Yes. Dexamethasone is given prior to the first dose of antibiotic, as corti-


costeroids are helpful in diminishing the neurologic consequences of rapid
bacteriolysis. This protective effect is usually associated with H injluenzae
meningitis of otologic origin (a CSF Gram stain result showing small, Gram
negative, bipolar rods)

74
Chapter 1 - Otology Otolaryngology Clinical Case Studies

How does an AOM lead to bacterial meningitis? NOTES

There are 3 generally accepted routes by which otitis media may cause
meningitis. Listed below, they are:
1- Hematogenous spread: Either a generalized bacterem ia or a more lo-
calized retrograde thrombophlebitis is responsible for seeding the
memnges
~- Direct extension: This route is more common in sub-acute or chronic
otitis media with bony destruction, or in post-traumatic temporal bone
lesions. The pathogen spreads through dehiscent bone directly to the
memnges
Labyrinthine route: Children with congenital malfom1ations (Mondini
dysplasia) are at a higher risk for spread of pathogens through the oval
or round window, crossing the labyrinth to reach the meninges. This is
the most likely route in this case. given his nystagmus and vestibu lar
symptoms, in the absence of temporal bone destruction on CT scan

What is the surgical therapy for this condition?

Surgical intervention should begin with myringotomy with or without grom-


met placement. The aspirated fluid should be sent for culture and sensitivity,
and ototopical antibiotics should then be initiated

How does a temporal bone CT scan help direct surgical therapy in this patient
with meningitis resulting from AOM'?

In a patient with an unopacified mastoid, a myringotomy and gronunet place-


ment is sufficient surgical therapy. Patients witl1 opacified mastoids, hi sto-
ries consistent with temporal bone trauma or congenital abnmmalities of the
temporal bone should undergo mastoidectomy, both to remove infected bone
and to identify and repair CSF leaks

For this patient, surgery would be limited to an antero-inferior quadrant


myringotomy, suction-trap evacuation of middle ear purulence, and place-
ment of a grommet.

What are the potential iatrogenic complications?

Surgical management with myringotomy and grommet placement is a re-


markably safe procedure. Complications are rare and are listed below:
Chronic perforation, fracture or dislocation of ossicles, canal skin lac-
eration, vascular inj ury (jugular bulb, ICA), labyrinthine injury (penetra-
tion of oval or round window)

What are the possible complications of AOM?

Compl ications of otitis media are classically divided into lntratemporal and
Intracranial processes. Many patients suffer from more than one complica-
tion, as one disease process often leads to another (i.e., apical petrositis
leading to intracranial abscess)
lntratemporal complications ofAOM:
75
Otolaryngology Clinical Case Studies Chapter 1 -Otology

Aa l--"( ~ 11: ·tp. l·c ,. ,~•.po ~ (' C>~ l ~c~ 'rf~ :;. ~
NOTES LabynnthJtts (serous/suppurat1ve) and labynntlutJS osstflcans are
managed with broad-spectrum antibiotic coverage to treat the caus-
ative organism, in addition to myringotomy and ototopical antibiot-
ics. Labyrinthine fistula is usually a result of chronic ear disease or
cholesteatoma; therefore, surgical therapy begins with a complete
mastoidectomy and MEE for cholesteatoma removal. The choleste-
atoma matrix shou ld never be removed from an area in wh ich a
labyrinthine fistula is suspected, as the risk of severe hearing loss is
substantial. These concerning sites can be addressed at a planned
second-look revision procedure
.::)• Mastoiditis management is controversial in the acute setting. AOM
complicated by mastoiditis with either subperiosteal abscesses or
meningitis requires a complete mastoidectomy, as do all cases of
chronic mastoiditis. Acute mastoiditis without other complications
often will respond to myringotomy and ototopical and IV antibiot-
ics, saving mastoidectomy for medical treatment failures ~) ,•.)
J. Petrous apicitis, identified by the triad of retro-orbital pair1, lateral
rectus palsy and'B~oJTbea, demands complete mastoidectomy and
exenteration of all air cell tracts up to and including the petrous
apex. IV antibiotics are not sufficient coverage for this acute process
-:;' . Chronic perforation (present> 2 months) should be treated with
tympanoplasty when the contralateral ear is free of effusion or
~
eardrum retraction
:). Hearing loss: An acute effusion can result in a 15-40 dB CHL:
however, spread of the infection with resultant labyrinth.itis may
produce a ~ 40 dB sensorineural loss
;(/
<-.1 . Facial paralysis is a very rare corppli~ation of AOM, thought to
occur in 0.005% of cases of AOM. Fallopian canal dehiscence is
most common in the tympanic segment (45-50% of dehiscences),
allowing spread of infection, acute neuritis, edema, and nerve com-
pression with resultant neurapraxia. Temporal bone CT scan should
rule out cholesteatoma or other etiology, as well as localize a dehis-
cence. Myringotomy and grommet placement with appropriate an-
tibiotic coverage is usually sufficient therapy. Patients who fail to
recover facial motor function should receive electrodiagnostic test-
ing, with mastoidectomy and facial nerve exploration reserved for
cases in which ENOG demonstrates 90% degeneration within the
first week after onset of paralysis. Mastoidectomy allows for gross
removal of disease, while exploration of the facial nerve should be
1imited to debridement of granulation tissue that has invaded dehis-
cent pmtions of the fallopian canal. The facial nerve sheath should
not be incised and granulation tissue should not be removed from
within the nerve sheath, as these maneuvers are more likely to
spread infection and injure the nerve than provide for decompression
Intracranial complications of AOM:
• (?.:complications of meningitis include hearing loss, suppurative
intracranial processes, encephalitis, MR, seizure disorders and death.
The mortality rate of meningitis associated with AOM is < 8%;
however, this rate rises to 30% for meningitis resulting fi·om chronic
otitis media. In general, the mortality rate for S. pneumococcal
meningitis (25%) is higher than that for H injluenzae meningitis
(8%). The management of otitic meningitis is detailed above

76
Chapter 1 - Otology Otolaryngology Cl inical Case Studies

~. Extradu ral abscesses are largely asymptomatic collections of granu- NOTES


lation tissue and pus located berween bone and dura mater. usuall~'
associated with another (more symptomatic) intratemporal or in-
tracranial process. Often discovered during surgical therapy for
symptomatic di sease (e.g., cholesteatoma. petrositis), these co llec-
tions of granulation tissue shoul d be removed with the surrounding
bone and careful ly trimmed away from the involved dura
Subdural empyemas collect in the potential space between the dura
mater and the arachnoid. This exceedingly rare intracrani al compli-
cation is best diagnosed by MR1, followed by emergent neurosurgi -
cal intervention. Management of the responsible otologic condition
(myringotomy versus mastoidectomy) is completed when the pa-
tient is stable from a neurosurgical standpoint
Otogenic brain abscesses result from direct extension from middle
ear and/or mastoid infection, or as a resu lt of venous thrombophle-
bitis. This complication is more commonly found in cases of chronic
otitis media, cho lesteatoma and in the post-mastoidectomy patient.
Though authors disagree whether the temporal lobe or the cerebel-
lum is the most common location for otogenic brain abscesses, the
majority of cerebellar abscesses are associated with an otologic
source. Polymicrobial infections are the rule, and empiric antibiotic
coverage should treat anaerobes. MRJ scans will provide a better
view of changes in the brain parenchyma and response to antibiot-
ics. Neurosurgic.al consultation is required; however, high-dose an-
tibiotics and close monitoring with MRJ may obviate the need for
an intracranial drainage procedure. Management of ear di sease is
tailored to the specific otologic issue, but is deferred until the pa-
tient is neurologically stab le. Modem management of intrapa-
renchymal abscesses is still associated with a rnortality rate near 25%
Sigmoid sinus thrombosis is a rare complication, yet it remains as-
sociated with a high mortality rate (18%). Contrast-enhanced MRl
and CT will both demonstrate a low intensity triangle surrounded
by high intensity contrast-enhancing dura in the location of the in-
volved sigmoid sinus (the "delta sign"). The Tobey-Ayer test will
confim1 sigmoid sinus thrombosis when 'ipsilateral compression of
the internal jugular vein fails to change the CSF pressure, as mea-
sured duringLP. Broad-spectrum antibiotics are initiated and surgi-
cal therapy includes complete mastoidectomy, sigmoid sinus aspi-
ration and clot removal. Thrombus limited to the sigmoid sinus may
be followed with serial imaging alone, providing anticoagulation only
in cases of thrombus progression, neurologic changes, embol ic events
or persistent fever. Internal jugular vein ligation is strictly reserved
for cases of sepsis refractory to medical and surgical management
or thrombus propagation proximally into the heart
Otitic hydrocephalus is a rare complication, usually associated with
sigmoid sinus thrombosis, and is generally quite responsive to treat-
ment. Imaging studies of the brain are nom1al; however, CSF pres-
sure is elevated, similar to the clinical picture of benign intracranial
HTN. Patients typically present with mental status changes, nau-
sea, vomiting. and visual disturbances. Papilledema and lateral rec-
tus palsy, in addition to ear disease, are the most prominent PE
findings. MRl wil l show the venous sinuses more clearly than will

77
Otolaryngology Clinical Case Studies Chapter 1 -Otology

NoTES CT, and wi ll determine if LP can be perfom1ed safely. Acute man-


agement of this condition includes steroids, mannitol, diuretics and
acetazolamide. Ear disease is most often treated by a complete
mastoidectomy, though some authors advocate sigmoid sinus de-
compression during this procedure. Ophthalmology should be con-
sulted to assess the patient's risk of developing optic atrophy. This
condition can be prevented by decompressing the subarachnoid
space via serial CSF drainage, a ventricu loperitoneal shunt, or by
• fenestration of the optic sheath

How would you follow this individual? State that you would:

This patient should receive an appropriate course of antibiotics, assessment


ofhis hearing, and evaluation for a possible anatomic predisposition to meningitis
Antibiotics: A I 0-day course of culture-directed antibiotics is appropriate
for H injluenzae and S. pneumococcus meningitis. (Highlight: Some
pediatricians recommend a course of Rifampin to eliminate the
nasophatyngeal carrier condition in children with H influenzae meningitis)
Audiology: Residual hearing deficits should be identified as soon as the
ears are free of middle ear fluid. Sound field audiogram will not provide
specific information regarding the auditory capabilities of each ear.
Otoacoustic emissions and ABR testing is the most appropriate study in
this 4 year-old patient. Serial audiograms should be used to follow this
patient, as delayed and progressive hearing Joss may occur ·
Re-assessment of etiology: Congenital malformations, cholesteatoma,
and CSF leaks should be followed by repeat imaging, and surgical re-
pair as indicated

This patient appears to have normal temporal bone anatomy on CT scan,


and should nor require additional imaging.

References:

1. Bluestone CD. Clinical course, complications and sequelae of AOM. Pediatr


Infect Dis J. 2000; 19:S37-46. .
2. Wetmore RF. Complications of otitis media. Pediatr Ann. 2000;29:637-46.
3. Haddad J. Treatment ofAOM and its complications. Otolaryngol Clin N01th
Am. 1994;27:431-41.
4. Levine SC, Souza CD. Intracranial complications of otitis media. In:
Glasscock ME, Julianna A, GulyaAJ, eds. Surge1y of the ear. Slh ed. Hamilton,
ON: B.C. Decker; 2002, 443-61. .
5. Neely JG. lntratemporal and intracranial complications of otitis media. In
Cummings CW, Fredrickson JM, Harker LA, Krause CJ, Schuller DE,
Richardson MA, eds. Otolaryngology-head and neck surgery. 3rd ed. St.
Louis: Mosby-Yearbook:l998; 1759-72.
6. Dew LA, Shelton C. Complications of temporal bone infections. In:
Cummings CW, Fredrickson JM, Harker LA, Krause CJ, Schuller DE,
Richardson MA, eds. Otolaryngology-head and neck surgery. 3rd ed. St.
Louis: Mosby-Yearbook: 1998, 3047- 75.

78
Chapter 1 - Otology Otolaryngology Clinical Case Studies

CASE 15 - T YMPANIC MEMBRANE PERFORATION NoTES


D. J. Verret. MD

A 6 year-old right-handed Caucasian male presenzs with a chief complaint


of righT ear drainage after swimming.

!WI: state that you would:

Obtain a detailed medical history beginning with a history of the present


illness

What additional historical information would you seek'? State that you would ask:

How long has the otorrhea occurred?


Is it clear or colored?
Is the otorrhea foul smell ing?
Does the child have any history of recun·ent ear infections?
Does the child have otalgia?
Does he complain of pain when the auricle is touched?
Is there auricular swelling? Is there any hearing loss?
Does the child have aural fullness?
Is the child experiencing dizziness?
Does the child have tinnitus?
Has the child had any prior ear operations?
Does the child have a previous history ofTM perforation? .
Has the child ever had ear tubes placed?
Has the child recently had instrumentation in his ear including attempted
cerumen removal or q-tip exposure?
Has the child recently had EAC trauma?

The patients mother reports that he was in his usual state of health w1til 2
days prior to seeing you, when he developed a yellowish foul -smelling
otorrhea from his right ear. There are no signs of systemic illness (i.e., no
feveT~ chills, nausea, or vomiting). The mother reports that the otorrhea
developed the day after they went swimming in Lake Ponchartrain. The
child says that he can 't hear very well out of the affected eGT~ The child
also says that he hears a strange \'ringing" sound in his right ear that just
won 't go away. He has no complaints of pain but says that he feels some-
thing is stuck in his em~ He has no dizziness.

PMH: NSVD without complications. No history of allergies, asthma,


or GER. There is a history of multiple episodes of otitis me-
dia between birth and age 2, but without recent episodes of
infection
PSH: The patient had bilateral myringotomy with tympanostomy
tube placement at age 2. Unfortunately, the parents failed
to take the patient for follow-up visits after the tubes were
placed

79
Otolaryngology Clinical Case Studies Chapter 1 -Otology

NOTES A!lergies: NKDA


J\1/edications: None
FH: Non e
SH: The patien1 is in first grade. He is not exposed to smoking.
He was breastfed when he was an infant
ROS: Negative

What would you look for on PE ? State that you would:

Perform a thorough head and neck examination and otomicroscopic ear


examination and a systemic examination as ind icated by the history

PE:

lJ.ulls.: Temp: 98.6 F. BP: 114183, Pulse: 76


GA: The patient is sitting in the exam chaiT~ He appears to be in
NAD and is WDWl'l
Left ear: EAC without erythema; the TM is intact, mobile, and trans-
parent. There is no middle ear fluid
Right ear: No pain was elicited when the auricle was pulled to insert
the speculum. Purulent discharge in the EAC which is
suctioned under otomicroscopy. The pars tensa has a 20%
pe1joration overlying the round window. N o evidence of
cholesteatoma. No pars jlaccida retraction pocket. Purulent
drainage was found to be emanating fi'Oin the middle ear
space
Weber test: Lateralizes to right ear al 512 Hz
Rinne test: BC>AC on right at 256 and 512Hz, AC>BC on left at 256
and 512Hz
Nasal: Patent nares, no septal deviation, normal-appearing mucosa,
no rhinorrhea
OP: No OC masses, mobile tongue, no tongue masses, good denti-
tion, no mucosal color changes, uvula midline and not bifid,
2+ tonsils without e1ythema or exudates
Ne ck: Supple, no lymphadenopathy, no thyromegaly, midline tra-
chea, no neck masses

What diagnostics would you request? State that you would:

Obtain an audiogram

Test results:

Left ear: Pure tones and SRT WNL: normal discrimination : Tympanogram
Type A, static compliance WNL
Right ear: 25- 35 dB air-bone gap for the !ower frequencies, Tympanogram:
Type B, large static compliance

The patient should also be evaluated f or ossicular chain damage. In this

80
Chapter 1 - Otology Otolaryngology Clinical Case Studies

case, his CHL is moderate (;5- 35 dBJ: ossicular chain damage is ques- NoTEs
rionable. Ossicular damage should always be su~pec1ed i11 patiems with a
CHL. especial~l ' if >30 dB.

What is your diffe rential diagnosis?

The patient has a right TM perforation with otorrhea secondary to his recent
episode of swimming. There are several questions: ( 1) What caused the
otorrhea? (2) What caused the perforation? (3) Is there ossicular damage?
The most common etiologies in the differential diagnosis are:
Iatrogenic: In this patient, iatrogenic perforation is possible as the pa-
tient has a history oftympanostomy tube placement. Heal ing of his TM
after tube extrusion was not confirmed. Permanent TM perforations
after tympanostomy tube placement ranges from 0.5-3%
Infection: This patient developed otorrhea after swimming. TM perfora-
tion can result from acute or chronic otitis media and rarely OE. The lack
of pain as a symptom makes AOM or OE Jess likely to be the cause
Barotrauma injury: This patient denies scuba diving or water ski ing,
both of which can cause TM perforation and ossicular chain disruption.
Any trauma with a force of~ 25 lb/in~ to the TM and EAC can cause
TM perforation
Instrumentation: The patient·should be questioned about the use of cot-
ton swabs or placing other objects in .the EAC. He denies putting any-
thing smaller than hi s elbow in his ear
Cholesteatoma: The lack of a retraction pocket or keratinous debris
makes cho lesteatoma unlikely. However, squamous epithelium may be
hidde1i by the otorrhea and not clearly visualized

Diagnosis:

The patient has a posterior TM perforation with otorrhea after swimming.

The patient is at increased risk of cholesteatoma formation from migration


of squamous epithelium as ·the pe1joration is marginal and in the posterior
quadrant.

What are your treatment options and their complications? What would you do
and why?

State that you would first treat the otOJThea and TM perforation
In teqns of immediate care, the patient should be instructed in dry ear pre-
cautions. He should avoid water exposure (including further swimming until
this clears). When he showers, he shou ld place a cotton ball with Vaseline
in the EAC

In this patient, the otorrhea has likely occurred because of contamination


from a preexisting perforation. The likely organisms responsibl e for the
infection in this case include Staphylococcus aureus and Pseudomonas
aeruginosa. Ototopical drugs are appropriate in this situation without the
need for system ic antibiotics. Thought must be given to avoid ototopical
81
Otolaryngology Clin ica l Case Studies Chapter 1 - Otology

NOTES dru gs that have potential ototoxicity (since there is a TM perforation). Ap-
propriate ototopical med ications include Ofloxacin and Ciprofloxacin with
hydroc011isone. Fluoroquinolones have almost no risk of causing sensorineu-
ral hearing. loss after absorption through the round window. (Note should be
made that if this same patient presented with a contralateral ear that showed
evidence of infection in the middl e ear space, systemic antibiotics in add i-
tion to ototopical medications to the affected ear would be warranted.)

In the majority of cases, med ical therapy as stated above is all that is needed
to stop the otoJThea. If the otorrhea does not stop, one nee~s to give consid-
eration to surgical treatment of CSOM.

The patient returns for follow-up I month later with a d1~V ear but a persis-
tent TM pe1joration.

What would you do now? State that you would:

Obtain a fine cut CT scan of the temporal bone to evaluate for possible os-
sicular chain discontinuity and middle ear pathology, especially cholesteatoma

What diagnostics would you request? State that you would request:

Imaging stu.dies: ln this case, CT scan would be helpful. Fine·cut CT scans


of the temporal bone would be useful if there is evidence of cholesteatoma,
ossicular-chain disrupti on, or question of mastoid involvement (especially if
surgery is considered)

What is the next step in your treatment of this patient? State that you have
several options including:

Dry ear precautions - As long as the patient has a TM perforation, ·he


shou ld not dive and should avoid activities which would introduce water into
the middle ear and increase chance of infection
Observation - In this case, the patient's acute infection must be resolved
before any surgical intervention is pondered. In addition, over 90% of trau-
matic TM perforations will spontaneously heal. If true traumatic perfora-
tion is suspected, the patient can be reassured and fo llowed in 4-8 weeks or
sooner if otorrhea returns. Patients who do not swim, have a minimal hear-
ing loss, and who do not have a history of recurrent infections can safely be
observed. If observation is undertaken, a 6 month interval should pass be-
fore surgery is planned to determine if the perforation will spontaneously
heal
Office interventions- These are likely not feasible in a 6 year-old child, but
are included for completeness of discussion. Office interventions include
paper patch, fat-plug, irritant oil application, and silver nitrate cauterization.
If a paper patch is to be used, the edges of the perforation must be freshened
before paper patch application. Conservative office repair is generally not
advised in TM perforation greater than 65% of the TM and contraindicated
in cases of cholesteatoma, chronic drainage, and ossicular disruption

82
Chapter 1 - Otology Otolaryngology Clinical Case Studies

Tympanoplasty- Operations that involve elevation of the tympan ic annulus NOTES


out of the tympan ic sulcus and entrance into the midd le ear. This can in-
clude TM repair and ossicular chain reconstruction. Tympanoplasties are
common ly divided into 4 types with Type I tympanoplasties being opera-
tions that are limited to the TM only
Myringoplasty (i.e ., Type I tympanoplasty)- An operation on the TM alone:
it does not involve elevating the annulus out of the tympanic sulcus or exam-
ining the middle ear space

State that in this case, you would prefer surgical intervention. You should also
state the indications for surgery according to the AAOHNS guidelines for
tympanoplasty, which are:

I or more of the following:


'- CHL dt,~e to TM perforation
2- CHL due to ossicular continuity or necrosis
=> - CHL due to o"ssicu lar ankylosis
'-'
Jj- Chronic or recurrent otitis media
s- Recurrent middle ear infections due to contamination through perfora-
tion ofTM
~~ - Progress.ive hearing loss due to chronic middle ear pathology
":.(
Perforation or hearing Joss persisting more than three months due to
., tr<,~uma, infection, or prior surgery
,; - Inability to safely bathe or participate in water activities due to perfora-
tion ofTM with or without hearing loss
0_ ~
t· Create a safe ear
PE mustbe performed prior to any surgical undertaking and must include:
Description of complete ear examination, including both norn1al and
abnormal findings
The fo llowing tests are required before surgical intervention and should be
done no longer than three months in advance of surgery:
Air and bone - Pure tone audiogram
SRT and discrimination

In this case, the patient has a CHL assumed to be due to the TM perfora-
tion. Although elective, surgery can be performed to attempt to correct his
hearing loss. The PE and testing have been done; surgery can be sched-
uled if the patient :S· parents wish to proceed.

Be sure to mention the contraindications to surgical intervention:

Elective repair of <.1 better hearing ear is a relative contraindication


Elective repair of an only hearing ear is an absolute·contraindication

State that you would explain the risks and complications of surgery to the person
from whom you obtain informed consent:

I0% failure rate of tympanoplasty


5% chance of worsen ing hearing

83
Otolaryngology Clinical Case Studies Chapter 1 - Otology

NOTES 1% chance of sensorineural or profound hearing loss


Facial nerve injury
Loss or alteration of taste due to chorda tympani injury
Ve1tigo and disequilibrium

State that you have several surgical options and which one you would choose:

Surgical options include:


Medial grafting (i.e., underlay technique), the most commonly used technique
Technique:
May be perfom1ed postauricular, transcanal , or endaural
Mucosa around perforation is removed and perforation is rimmed
For the canal work, posterior canal incision through skin at osteocarti-
laginous junction with elevation of a tympanomeatal flap.lncision carried
around until the annulus can be elevated and the middle ear space en-
tered
.:... For a postauricular incision - An incision is made one finger breadth
posterior to the postauricular fold
Elevate the skin and subcutaneous tissues anteriorly and carry the dis-
section in the plane of the temporalis fascia
A small piece of temporal is fascia is harvested
Deep tissues are elevated until the posterior wall of the EAC is revealed
The incision is held open with a Weitlaner retractor
Middle ear packed with Gelfoam
Graft placed under the anterior TM remnant
Gelfoam added medial to the graft to hold it to the underside of the
drum remnant
Vascular stripped, replaced
Graft held in place with Gel foam
Postauricular or endaural incision closed ·
Complications:
Graft medializing and causing failure of repair. This may be minimized
by securing the graft to the lateral wall, anterior annulus, or lateral wall
of the eustachian tube ·
Scar tissue formation in the middle ear causing graft movement

Lateral grafting:
Technique:
Vascular strip created with tympanomastoid and tympanosquamous su-
ture line incisions
Flap elevated medial to lateral
Circumferential incision through canal skin at osteocartilaginous junc-
tion ·with removal of skin
Drum remnant denuded of squamous epithelium
Temporalis fascia harvested through postauricular incision
Middle ear filled with Gelfoam
Graft placed lateral to drum remnant and medial to long process of malleus
Graft must extend to anterior lim it ofTM but not past it
Canal skin placed back in EAC
EAC packed with Gelfoam

84
Chapter 1 -Otology Otolaryngology Clinical Case Studies

Complications: NOTES
1/s; Anterior blunting (2- 5% with minor blunting resulting in a 40 dB or
· more CHL)
r~?.:- Lateral grafts may pull away from the TM and 5% may lateralize: a
fully laterali zed graft is associated with a maximum or near-maximum
CHL. Skin grafting may be needed for repair
.J_ lntratympanic cholesteatoma - Can be excised in the office under
direct microscopy

Fat graft myringoplasty:


o Most common in children after tympanostomy tube removal
Contraindicated in significant infection, cholesteatoma, or large perforations
I •U - ~

Cmtilage graft:
Low metabolic activity and high fiber density
May be used for only the posterior superior quadrant and/or attic or larger
pieces to replace most or all of the TM

Homograft tympanoplasty (not used in the United States any more due to in-
creased medico-legal risks):
May be used for TM and ossicu lar reconstruction
Chance of infectious disease transmission via slow viruses (Creutzfeldt-
Jakob disease)

How would you follow this individual? State that you would:

Instruct the patient according to AAOHNS recommendations to observe


for the following conditions postoperatively:
)- Vertigo and nystagmus - Appropriate medication. If severe, notify
surgeon
;<- Drainage - Reinforce or change dressing
,.''J - Jaw pain·- Reassure
4- Facial motion- Notify surgeon of weakness or paralysis
\_f- Moderate tinnitus- Reassure. Notify surgeon

The AAOHNS recommendations for postoperative follow-up are:

One week- Incision and ear canal should be evaluated for signs of infection.
Tbe patient should continue with dry ear precautions until the TM is well healed.
In addition, ototopical ear drops should be used for 2-4 weeks postoperatively
to decrease the chance of infection .

Beyond 1 month - Patient should receive an audiogram·and the TM should be


viewed to see if it is intact (typically at 3 months).

Outcome:

TM healing - 4-5% lower for children than adults (overall, adults are expected
to have a 90-95% success rate with fasc ia techniques)

Hearing improvement- Less than a I 0 dB air/bone gap is expected if there are


no ossicular chain abnonnalities.
85
Otolaryngology Clinica l Case Studies Chapter 1 - Otology

NOTES CASE 16 - AUTOIMMUNE INNER EAR DISEASE


J oseph R. Williams, MD

A 57 year-old WF presents with a chief complain! of bilazeral hearing loss


fo r the last 2 months. She firsr noticed decreased hearing and mild dis-
equilibrium after a trip to Colorado.

HPI: state that you would :


.
Obtain a more detai led otologic and medical history beginning with the chief
complaint

What additional historical information would you seek? State that you would ask:

• . Did she have a URJ prior to the onset of her bearing loss?
Does her hearing fluctuate? If so,. how often and how much?
Has her hearing "loss progressed?
Which is her better hearing ear?
• Can she still use the phone in either ear?
Does she have otalgia, aural fullness, tinnitus or otorrhea?
Does she experience dizziness? If so, is it ve11igo or disequilibri um? Is the
dizziness accompanied by nausea or vomiting? Does her hearing decrease
when.she is vertiginous?
Does she complain of headaches or vision changes?

The hearing loss was not associated with an upper respirat01y tract infec-
tion. The patient states that her hearing fluctuates unpredictably Her left
ear is usually the better hearing ear. She can use the phone but with diffi-
culty. Her hearing does tend to be worse when she has significant vertigo
or disequilibrium. The episodes of vertigo anrl disequilibrium usually last
fo r minutes and are rarely associated with nausea. Overall, she believes
her hearing is now significantly worse than it was 4-6 weeks ago. She has
mild bilateral tinnitus and aural fullness. She has no otalgia, otorrhea,
headaches, or vision changes. ·

PMH: No known medical problems: no history of hospitalizations


or ER visits
PSH: No prior history of otologic surgery
Allergies: NKDA
Medications : Pseudoephedrine to help with her aural fullness
FH: Mo{her had systemic lupus erythematosus (SLE)
No hearing loss
SH: School teache1~ Married. 2 children. No HIV risk factors.
No history of significant noise exposure. No history of
tobacco use. Social drinker
ROS: Complains of muscle and joint stiffness/soreness
Otherwise, negative

86
Chapter 1 - Otology Otolaryngology Clinical Case Studies

What do you look for on PE? N OTES

State that you wou ld perform a complete head, neck, and neUt·otologic PE
including vital signs and otomicroscopy (with pneumatic insufflation)

PE:

J!ili:d.s..: Temp: 98.6 F, BP: 120175, P: 88. RR: 18, Wt: 145 lb.
GA: Middle aged Caucasian female in NAD lmd otherwise
appearing healthy: she is attemp ting to read lips to better
understand you: pocket talker used to assist with
communication
HEENT AD - Normal- No retraction pocket, norma/mobility, no
MEE
AS -Anterior focus of myringosclerosis, otherwise same
as AD
Tuning fork examination: Weber midline, Rinne + AU
(51 2Hz)
OC!OPI '--.......
nasal cavities: No masses. lesion~~ otf!.er abnormalities
Eyes: No erythema or conjunetjyaJ edema
Neck: Supple with no nodes or br1its
New·o: CN normal. Normal. gait, mild dif.flculty with tandem
." 'rj,,'

gait, sway on Romberg, normal FukUcla

What is your differential diagnosis?


. s.-..._...._____
'• Bllater~J.Meniere's disease
2. Bilateral SSNHL
J· AIED
-<1 • Genetically mediated hearing loss
..S • Neurosyphilis or Cogan syndrome

What do you do next?

Detailed audiometric evaluation: PTA, SRT, SDS, tympanograms


• Serology to rule out AIED: ESR, ANA, 68 kD inner ear antigen test and
rheumatoid factor. Can also consider additional laboratory testing if patient
has not seen a physician in awhile (i.e., CBC, Panel 7, TSH, lipid panel)
Serology to rule out syphilis- FTA-ABS
Serology to assess for genetic defects including Connexin 26 and 30
MRJ with gadolinium to rule out retrocochlear processes

Test results:

• Audiometry shows bilateral moderate to severe sensorineural hearing


loss. The PTA is 65 in the right ear with an SRT of 60 and discrimina-
tion score of 56%. The left ear has a PTA of 55, an SRT of 50, and
discrimination score of 74%. Type A tympanograms. Unfortunately, no
prior hearing test available for comparison.

87
Otolaryngology Clinical Case Studies Chapter 1 -Otology

NOTES 68 kD inner ear amigen 1esting is positive. The ESR is elevated a1 -10
mmlhr: ANA and rheumaToid factor are negaTive
FTA -ABS is negative
Connexin 26 and 30 showed no evidence of muwtion
MRI scan with gadolinium is normal

Diagnosis:

AIED (primary vs. secondary); clinical history and p ositive 68 kD inner


ear antigen test ·with elevated ESR strongly suggests an autoimmune basis
for this patients fluctuating and rapid bilateral neurosenso1y hearing loss.

What treatment would you begin?

Prior to waiting for the lab tests to come back and for the MRJ to be done,
it would be appropriate to begin the patient on a high dose steroid regimen
(60 mg/day or 1 mg/kg/day) for at least one month. The patient should
consent to its use; risks of mood changes, weight/appetite gains, blood sugar
elevation, osteoporosis, aseptic femoral head necrosis, and peptic ulceration
are discussed. To protect against stomach ulceration, a PPI or H2 blocker
is concomitantly prescribed. Steroids need to be prescribed carefully in some,
such as those with poorly controlled diabetes or HTN, psychiatric disorders,
or AIDS. Studies are being conducted to assess whether transtympanic
steroids could provide similar benefits without the systemic risks ·
If hearing improves with steroid therapy, consideration is given to a slow
taper (approximately 10 mg per month) to minimize risks and side effects.
The goal is to stop the steroids, use the lowest possible dose, or use them on
an every other day basis
Consultation with a rheumatologist is obtained. The rheumatologist can help
determine ifthere is any sign of concomitant systemic autoimmune disease
(secondary AIED) vs. AIED alone (primary AIED). Systemic autoimmune
disease could be of an increased possibility in this patient with a FH of SLE
and her complaints of muscle and joint pain
Historically, cyclophosphamide has been used for f\IED. However, there is
concern about significant side-effects and toxicity. Other options include
plasmapheresis, azathioprine, and methotrexate. All of these therapies, if
effective in a patient with AIED, may be best monitored by a rheumatolo-
gist who has experience in these treatments. Unfortunately, a recent double-
blinded multi-institutional study suggests that methotrexate up to a dose of
20 mg/week is no more effective than placebo
Audiometric evaluation should be repeated frequently. Once treatment is
initiated, the first audiogram should be obtained within l-2 weeks. Audio-
grams should be obtained monthly thereafter for the first several months of
treatment. The frequency of subsequent audiograms will be determined by
the patient's clinical course and progress. Ifthe hearing loss has stabilized,
hearing aids are recommended. Ifthe hearing loss progresses to a profound
level, a cochlear implant will be a good option
• 50% of patients with AIED have complaints of dizziness. Sorne patients
have symptoms of Meniere's disease. A low salt diet and diuretic can be

88
Chapter 1 - Otology Otolaryngology Clinical Case Studies

given. A work-up can include electronystagmogram, electrocochleo-graphy. N oTES


and rotary chair testing. Vestibu lar rehabilitation can be helpful for com-
plaints of imbalance/disequilibrium

How would you follow this patient:

The patient should be followed on a monthly basis with audiograms until


hearing has stabilized. Hopefully, significant hearing improvement (both in
terms of PTA and word discrimination scores) shofild be seen in a few
weeks to a couple of months. Once hearing is stabilized, visits can be stretched
out to once every 3 to 6 months depending on how long the patient is treated
The patient should be instructed to notify the otolaryngologists if she notices
a sudden decrease in her hearing. Notified of such an occunence, the oto-
laryngologist should order an audiogram. Oftentimes, steroids need to be
restarted (if the rheumatologist has weaned the patient off steroids) or in-
creased (if steroids are being tapered). If the patient's hearin.g deteriorates
despite high dose·steroids and the appropriate dose of other inm1Unomodulating
agents, the diagnosis should be questioned
At some point after the hearing has stabilized (usually between 6-24 months),
the decision will be made to try to taper therapy. Visits to the otolaryngolo-
gist with repeat audiometry shou ld then be reinstituted on a monthly basis
until the taper is complete and hearing is stable

References:

1. Broughton SS, Meyerhoff WE, Cohen SB.lmmune-mediated inner ear dis-


ease: .] 0 year experience. Semin Arthritis Rheum . 2004;34(2):544-8.
2. Harris JP, Weisman MH, Derebery JM, et al. Treatment of corticosteroid-
responsive autoimmune inner ear disease with methotrexate: a randomized
· controlled trial. JAMA. 2003;290(14):1875- 83.

89
Otolaryngology Clinical Case Studies Chapter 1 - Otology

NOTES

90
Chapter 2 - Head and Neck Otolaryngology Clinical Case Studies

CHAPTER 2/CASE 1- NASOPHARYNGEAL CARCINOMA NOTES


Robert J. Sinard, MD

A -18 year-old male presents with a 3 month history of a right neck mass
and intermiuent right ear discomfort f or the past yem:

HPJ: state that you "vould:

• Obtain a detailed medical histOl)' beginning with a history of the present


illness

What additional historical information would you seek? State you would ask:

• Has the neck mass fluctuated in size, grown steadily, or remained the same?
• Is it painful?
• Any loss of hearing in the right ear?
ls the throat sore?
• Has th~re been any change in voice or swallowing?
• New nasal obstruction or epistaxis?

The mass has not changed in size since it was first noticed. It is neither
painful nor tender. The right ear has been ''popping" episodically, and
the patient reports a subjective sense of hearing loss. He has no com-
plaints of sore throat, dysphonia, or dysphagia. He has experienced no
weight loss; nasal obstruction, or epistaxis. The patient does complain of
persistent post-nasal drainage.

P MH: Otherwise healthy, takes no medicines ·


Allergies: NKDA
Medications: None
FH: Negative
SH: Negative, does not smoke or drink
ROS: NC

What would you look for on PE? State that you would:

• Perform a complete head and neck PE including vital signs

PE:

111£Il.s.: Temp: 98.6 F, BP: 180190, Pulse: 90


GA: WDWN, no respiratory distress
HEENT" Right ear has a non-mobile drum with yellowish fluid in the
middle ear. Web er exam lateralizes to the right. Fiberoptic
laryngopharyngeal exam reveals a submucosal fullness of the
right NP with loss of the contours of the Fossa of Rosenmii/ler
and eustachian tube orifice. Soft palate, nasal septum, and OP
are clear. Normal vocal cord motion. Neck has a 3. 0 em right
upper level 5 mass, mobile, with an additional 1.5 em lower
level 5 mass with the same characteristics
New-o: No CN deficits

91
Otolaryngology Clinical Case Studies Chapter 2 - Head and Neck

NoTES What is your differential diagnosis?

1
• Viral pharyngitis
.2. • Atypical non-viral infection (i.e .. mycobacterial)
OJ • Lymphoma
./~ .
'{ Temporal bone osteomyelitis
~ • Cancer of the NP

What diagnostics would you request? State that you would• request:

• FNA of3 em right neck node


• CT scan of neck with contrast

Test results:

• FNA is pe1jormed and is suggestive of "lymphoepithelioma" (be pre-


pared to discuss· the subtypes o/NPC).
• CT scan reveals a contrast enhancing mass centered in the right supe-
rior NP involving the posterior nasophar.yngeal wall, with enhance-
ment around the eustachian tube orifice, without bone erosion (be pre-
pared to dis·cuss relative value of MRJ, PET).

Diagnosis:

The provisional diagnosis is T2N2bMY WHO type 3 NPC, Stage 3 (be pre-
pared to discuss the staging of NPC)

Tl Tumor confined to NP
T2 Tumor extends to soft tissues of OP and/or nasal fossa
T2a Withou{ parapharyngeal extension
T2b With parapharyngeal extension
T3 Tumor invades bony structures and/or paranasa/ sinuses
T4 Turr:or with intracranial extension and/or involvement ofCN, infratem-
poral fossa, hypopharynx, or orbit

What further studies are needed?

• Biopsy of the primary site, ideally in the office under local, but not possible in
all patients
• CT scan of chest
• EBV serology (anti-EBV IgA levels, be prepared to discuss which NPC
subtypes are related to EBV serology)

Test results:

• Biopsy confirms WHO type 3 NPC


• CT scan reveals no significant abnormalities
• EBV titer elevated

92
Chapter 2 - Head and Neck Otolaryngology Clinical Case Studies

Wh.at are the treatment options and their complications? NOTES

• The cuJTent treatment for advanced stage NPC is concurrent chemoradio-


therapy. Some may argue for radiotherapy alone or induction chemotherapy
fo Ilowed by radiation therapy, but the best data support concomitant therapy

Discuss your complete plan:

• Consu Itation with medical and radiation oncology for treatment


• Consu ltation with oral surgery for pre-radiation dental evaluation
• Consultation with general surgery for gastrostomy tube, MediPon place-
ment (be prepared to discuss swallowing side-effects of therapy)

How would you follow this individual?

• Regular exams (evety 1- 2 months is reasonable)


• Radiology (CT vs. MRI vs. PET)
• Salvage therapy for recuJTence (re-irradiation vs. surgery, possible roles of
brachytherapy and chemotherapy)

93
Otolaryngology Clinical Case Studies Chapter 2 - Head and Neck

NoTEs CAsE 2 - ToNSIL CARCINOMA


Robert J. Sinard, MD

A 64 year-old male presems with a several month history of a sore throat


that is primarily left-sided. He also has a 5 em left neck mass which is
mildly painful.

HPI: state that you would:

• Obtain a detailed medical hi story beginning with a histoly of the present


illness

What additional historical information would you seek? State that you would ask:

• Is there dysphagia or odynophagia; has there been any weight loss?


• Is there trismus?
• Is there otalgia?
• Has the neck mass fluctuated in size or stead ily grown?

It is both difficult and painful to swallow, and he has lost 30 lbs. over the
last 6 months. His left ear hurts but he was told by his family doctor that he
has had an ear infection. He can't bite into a sandwich anymore because it
is too painful to open his mouth very far. The neck mass has never shrunk
in size.

PMH: Emphysema and HTN


Medications: Two inhalers, 2 anti-hypertensive medications, and analgesics
Allergies: NKDA
FH: Negative -
SH: Has smoked 2 ppd for 40 years, drinks 6-8 beers/day
ROS: NC

What would you look for on PE? State that you would:

• Perform a complete head and neck PE including vital signs

PE:

YlliJls.:
Temp: 99.8 F, BP: 190190, Pulse: 72, Wt: 124 lbs
GA: Thin, elderly man, unkempt, no respiratory distress
HEENT· Ears bilaterally clear without fluid. OC and oropharyngeal exam
l{mited by trismus, but one can see scattered carious teeth. On
the left in the area of the tonsil, there is visible ulceration with
cavitation and mounded edges, measw-ing at least 4 em. The ul-
ceration extends superiorly to the lateral aspect of the soft pal-
ate and inferiorly to the lateral aspect of the tongue base. Pal-
pation confirms firmness in this area not extending to the mid-
line. Mass/ulcer moves independent of the mandible_ Fiberoptic

94
Chapter 2 - Head and Neck Otolaryngology Clinical Case Studies

exam shows the vallecula. hypophmynx, and larynx to be nor- NoTES


mal. Neck exam reveals a 5 em level If- Ill mass that is firm but
mobile, and a 2 em firm level IV mass
New·o: Tongue motion is slightly decreased, no other CN deficits

What is your differential diagnosis?

.. sec of the left tonsil (anything else would be quite uncommon)


• "Lymphoma

What diagnostics would you request? State that you would request:

• Office biopsy and/or FNA of left neck mass


• CT scan of neck
• CT scan of chest
• Labs: obtain CBC, electrolytes, liver function tests, TSH, but assume none of
the labs are abnormal enough to prohibit treatn~ent of any kind
• Medical consultation: to obtain better control ofHTN and assure the patient
is on therapeutic doses of the appropriate inhalers .
• Oral surgery consultation: reveals that if patient is to have radiation therapy,
he will need a full mouth extraction

Test results:

Biopsy confirms moderately differentiated SCC.

CT scan of the neck shows an enhancing mass centered at the left tonsil
extending superiorly to the soft palate and inferiorly to involve the lateral
aspect of the tongue base. The mandible appears to be contacted by the
mass, but there is no bone erosion. There is confluent adenopathy in the
level 2-3 area as well as an additional 1.5 em high level IV mass with
central necrosis. The lumen of the adjacent jugular vein is not visible at
the level 2 area but reconstitutes above and below this area.

CT scan of the chest shows no masses or evidence of metastases.

Diagnosis:

The patient has a T3N2bMO squamous carcinoma of the left tonsil. He


appears healthy enough to tolerate any treatment strategy.

TX Primary tumor cannot be assessed


TO No evidence of primmy tumor
Tis Carcinoma in situ
Tl Tumor 2cm or less in greatest dimension
T2 Tumor more than 2 em but not more than 4 em in greatest dimension
T3 Tumor more than 4 em in greatest dimension
T4 (lip) Tumor invades adjacent structures (e.g. , through cortical bone,
inferior alveolar nerve. FOM, skin of face)

95
Otolaryngology Clinical Case Studies Chapter 2- Head and Neck

NOTES T4 (OC) Tumor invades adjacent structures (e.g. , through c01·tical bone.
inzo deep [excrinsic} muscle of tongue, maxillary sinus. skin. Super-
fic ial erosion alone of bone/tooth socket by gingival primary is not
sufficient to classify as T4)

What are the treatment options and their complications? What would you do
and why?

• There are 2 treatment options to consider:


- Surge!)' foll owed by post-operative chemoradiation therapy: Th is is the
traditional treatment. Surge!)' consists of wide resection, neck dissec-
tion, and reconstruction, with appropriate dental extractions and secur-
ing a source of nutrition
- Concomitant chemoradiation therapy with or without radiation chemo-
therapy followed by.surgical salvage, if needed: Th is represents a more
contemporal)' standard of care. Articulation and cosmetics are superior
with chemoradiation as compared with surgel)'/radiation, though swallow-
ing function is probably about the same. Overall survival is equivalent

Be prepared to discuss the differences in treatment philosophy between the 2


strategies, including side effects and complications of treatment, the surgical
specifics of resection and reconstruction, the upper aerodigestive ·assault of
chemoradiation (put in a G-tube before starting treatment), and what to do with
the neck if chemoradiarion achieves a complete response at the prima1y.

• Discuss surgical salvage after failed chemoradiation


Highlights/pitfalls:
- Re-stage with neck CT to see if there has been progression
- Mandibu!otomy vs. mandibulectomy
- Be careful if there is trismus; this symptom requires wide clearance of
the pterygoid musculature
- Fistula rate higher in previously radiated field following surge!)'
Reconstruction should make use of vascularized, non-radiated tissue

How would you follow this individual?

• Routine office PE
• Discuss relative merits of CT vs. MRI vs. PET
• Serology for post-treatment hypothyroidism

96
Chapter 2- Head and Neck Otolaryngology Clinical Case Studies

CASE 3- CARCINOMA OF THE LARYNX NOTES


Larry L. Myers, MD

A 65 year-old man presents with 2 month history of progressive hoarseness.

HPI: state that you would:

• Obtain a.detailed medical history beginning with a history of present illness

What additional historical information would you seek? State that you would ask:

• How long has he been hoarse?


• Dysphagia?
• Odynophagia?
• "Lump in throat" sensation?
• Weight loss?
• Hemoptysis?
• Symptoms of carotid atherosclerotic disease?
• ·Any previous treatment for complaint?

Allergies: NKDA
Medications: ACE-inhibitor
P1\I!H: HTN
PSH: R shoulder surgery in remote past
FH: Negative
SH: . Tobacco: I ppd for 35 years
ETOH: Heavy beer on weekends
Occupation: Machinist
ROS: Shor~ness of breath

What would you look for on PE? State that you would:

• Perform a complete head and neck examination including vital signs

PE:

111£J.Ls.: Temp: 36 C, BP: 130/74, Pulse: 80, RR: 18


GA: WDWN WM in no obvious distress
HEENT Indirect laryngoscopy remarkable for left-sided
laryngeal mass, left vocal fold paralysis
Fiberoptic laryngoscopy: Left true and false vocal fold mass, left vocal
fold is not mobile
Neck: 2 em firm, nontender, mobile mass in level III

97
Otolaryngology Clinical Case Studies Chapter 2- Head and Neck

NoTES

Figure 1

What is your differential diagnosis?

• To generate a broad differential diagnosis, the use of a mnemonic may be


helpful

Mnemonic: KIITTENS
K (congenital) Unlikely
I (infectious) Bacterial, viral, TB, atypical TB,
I (inflammatory) Pseudotumor of larynx, supraglottitis
T (tumor) Benign (papilloma, laryngocele), malignant (SCC, chondro-
sarcoma), metastatic
T (trauma) Unlikely, no history of trauma
E (endocrine) Thyroid malignancy/disorder
N (neurologic) CVA
S (systemic) Wegener's, sarcoidosis, autoimmune, amyloidosis

What diagnostics would you request? State that you would:

• Order a CT scan with IV contrast of the neck


• Other studies that may be ordered:
- CT chest
- Barium swallow (evaluation for synchronous lesions ofupper aerodigestive
tract)
- Pulmonary function test (assess pulmonary status preoperatively if con-
servation laryngeal surgery feasib le)

98
Chapter 2 -Head and Neck Otolaryngology Clinical Case Studies

- Pre-operative speech and swallowing evaluation NoTES


Positron emi ssion tomograph y (PET) scans, usin g 18-F fluoro-
deoxyglucose as a radioactive tracer. theoretically image the increased
metabolic activity of malignant neoplasms. A higher concentration of
radiotracer coJTesponds to higher tissue activity. PET scans generally
are not considered initial imaging modality of choice for untreated tu-
mors of the upper aerodigestive tract

Test results:

CT scan of the neck shows a contrast enhancing mass with irregular bor-
ders in region of left glouis and supraglottic region.

What should be done to confirm diagnosis?

• Direct laryngoscopy with biopsy and FNA biopsy of left neck mass

Biopsy results:

• Both biopsies demonstrate SCC .

Diagnosis: ·

T3Nl SCC of L true vocal fold (Stage Ill)

Be aware of the current TNM classification of glottic cance1~ You may use
the AJCC or the UJCC staging systems. Both systems are the same for
glottic cancer

Tl Tumor limited to the vocal folds (may involve the anterior or poste-
rior commissure with normal vocal fo ld mobility)
Tl a Tumor limited to 1 vocal fold
T 1b Tumor involves both vocal folds
T2 Tumor extends to supraglottis and/or subglottis and/or with im- .
paired vocal fold mobility
T3 Tumor limited to larynx with vocal fold fixation
T4 Tumor invades through thyroid cartilage and/or extends to other
tissue beyond the larynx (such as trachea, soft tissue of the neck,
thyroid, pharynx)

It is common in many institutions that all patients with head and neck tu-
mors are presented at a multidisciplinary conference. If you have been
trained at an institution where such conferences are held, state so.

What are the treatment options and their complications?

• For advanced laryngeal carcinoma (Stage Ill/IV), dual modality treatment is


indicated
• Treatment consists of either: 1) surgery followed by post-operative radiation
therapy, or 2) organ sparing protocols using combined chemotherapy and
radiation therapy

99
Otolaryngology Clinical Case Studies Chapter 2- Head and Neck

NoTES Discuss the surgical options:

• Any procedure that ensures complete tumor extirpation with adequate mar-
gins is acceptable. Examples of acceptable extirpati ve procedures are: total
laryngectomy, left frontolaterallaryngectomy (if tumor 1imited on ly to vocal
fold); supracricoid laryngectomy (if tumor does not invo lve arytenoid). A
complete neck dissection must also be peiform ~J because the patient has
diagnosed cervical metastasis

The patient opts for a total /aryngecwmy/neck dissection and post-opera-


tive radiation therapy.

How do you perform a total laryngectomy?

Highlights:
o Divide strap muscles from inferior sternal attachments (ipsilateral thyroid
lobe is left attached to specimen)
• Release pharyngeal constrictors from thyroid cartilage bilaterally
o · Incise trachea at least 1 tracheal ring from inferior extent of laryngeal tumor

o Release base of tongue muscles from hyoid attachments


o Enter pharynx on contralateral side oftumor
• Preserve pyriform sinus mucosa and post cricoid mucosa
• Remove total laryngectomy specimen with 1.5-2 em margins
o Take circumferential mucosal margins
o Close neogullet with interrupted inverting suture
• Perform watertight seal test (forcibly infusing saline into neogullet) to ensure
water-tight closure
• Reinforce closure with second submucosal layer

Pitfalls:
• Not preserving pyriform sinus mucosa leads to tight closure and pharyngeal
stenosis
o INAD tumor extirpation/positive margins
o Improper spacing of neopharynx sutures leads to salivary leak and likely
fistula

Potential complications:

• Early complications include:


-
1
Hematoma
--< Pharyngocutaneous fistu la
-~ Wound infection/breakdown
• Late complications include:
1
- Pharyngeal stenosis
-( Tracheostomy stenosis

100
Chapter 2- Head and Neck Otolaryngology Clinical Case Studies

How do you avoid/treat complications? NOTES

• Hematoma - Th is complication can be avoided by meticulous intraoperative


hemostasis. If a hematoma occurs, the patient is promptly returned to the
OR to evacuate the hematoma, to obtain hemostasis, and to place new drains
• Pharyngocutaneous fistula - This complication can be avoided by proper
placement of sutures at initial closure of neopharynx. A W?.te1tight seal test
should be perfom1ed intraoperatively prior to closure of second layer of
neopharynx. Generally, a swallow test with dye is performed at 5- 7 days
postoperatively to determine if there is a salivary leak. PC fistulas, when
they occur, are treated with wet to dry gauze packing in fistula cavity twice
daily unti l healed 0 :.. : ,- .
• Wound bre~9;own- A skin closure under tension, posFirradiation state, w'bdnd
infection, ol'-fistula may lead to wound breakdown. Local wound care usu-
ally suffices for treatment
• Pharyngeal stenosis - Adequate pharyngeal wall to close over a 36-F dilator
is sufficient for primary closure without risk of pharyngeal stenOSiS. If there
is insufficient pharyngeal wall mucosa, then a pedicled or free flap should be
used to augment neogullet
• Tracheostomy stenosis - This can be avoided by making the initial, matured
tracheostomy as wide as possible. Avoid excessive adipose tissue from flap
closure that compromises tracheostomy lumen. Scar contracture may also
lead to tracheostomy stenosis. This condition is treated by temporary place-
ment of laryngectomy tube· or by local flaps to enlarge stoma

How would you follow this individual?

• Routine, monthly follow-ups for first year, then bi-monthly for year 2, then
every 6-12 months thereafter

101
Otolaryngology Clinical Case Studies Chapter 2 - Head and Neck

NoTES CAsE 4- FLooR oF MouTH CARCINOMA


John M. Truelson, MD

A 59 year-old white male presenrs wirh chief complaint of sore spar in rhe
mouth for 2 months.

HPJ: state that you would:

• Obtain detailed medical history, beginn ing with history of present illness

What additional historical information would you seek? State that you would ask:

• Is there a mass or ulceration?


• How long has it been present?
o Is it enlarging or getting smaller?
• Is there lip or tongue numbness?
• Is there pain?
• ls there tooth pain or mobility?
• Articulation?
• Swallowing?
• Weight loss?
• Previous treatment?
Exposure to alcohol and/or tobacco?

The patient reports that he has a right-sided lesion in the mouth under his
tongue, which seems to have doubled in size since being noticed two months
ago. His primary care doctor had previously prescribed penicillin which
resulted in no appreciable change in the growth of the lesion. He reports
no particular pain except when he chews on that side, but he has lost
about 10 lbs over the last 10 days. There is no difficulty with articulation.

PMH: DM, HTN, ASCVD - no hiscory of MI, no current chest


p ain and no history of CHF, but has had CABG
PS: CABG 7 years ago, no complications
Allergies: Tetracycline
Medications: Toprol XL, oral hypoglycemic
FH: NC
SH: Salesman, smokes cigarettes 1 ppd x 39 years, contin-
ues to smoke, ETOH: drinks frequently because of job
and on weekends
ROS: Herpes simplex I from time to time, frequent urination
especially the last three weeks, recent malaise

What would you look for on PE? State that you would:

• Perfonn a complete head and neck PE, including vital signs with weight and
height, and fiber-optic examination of the upper aerodigestive tract
• State that you would ascertain the extent of the oral mass (e.g., gingiva,
tongue, teeth, mandible, tongue, movement of the tongue)

102
Chapter 2 - Head and Neck Otolaryngology Clinical Case Studies

PE: NoTEs
~. Temp: 98.6 F. BP: 95170, Pulse: 81. Wt: J 90 lbs, Ht: 6 '0"
GA: WDWN NAD
Oral: Adequate denti1ion, mild periodontal disease. 2.5 em
ulceration right FOM exrending onto and is adherent
ro the gingival. It extends anteriorly to the canine teeth
and posteriorly to the ventral swjace of the tongue
OP: Normal
indirect exam: Normal
Neck: No palpable masses
Fiber-optic exam: Negative

What is your differential diagnosis?

' • Cancer
7. HSV
~ · Giant aphthous ulcer
'--P• Local trauma

What should you do next?

• Order glucose, electrolytes, CBC: (This should be high priority because of


the history of malaise and increased urination x 3 weeks noted in the PMH)
• Biopsy ma~s
• Radiologic imaging: Examiner should be able to discuss the best modality for
the head imd neck: CT, MRI, PET or PET/CT scan
Metastatic evaluation: be able to discuss the usefulness of:
- Blood tests
- Routine imaging for specifically which organs and modality:
• Chest: CT scan vs CXR
• Bone scan: Indications
• Specific evalua~ion of abdominal .viscera
- Endoscopy: wh ich exams are needed? Laryngoscopy, esophagoscopy,
bronchoscopy?

Test results:

• Lab:
- Glucose - 400
.
- No ketosis .
- No leukocytos:s

• Biopsy: Biopsy of the mass reveals invasive SCC

• Imaging: mass abuts upper alveolus, but no bone erosion. Two 1 em


lymph nodes: 1 in Ievell, 1 in level II. Chest: 3 small (< 2 mm) noncalcified
opacities in the lungs which are too small to characterize. His esopha-
gus is mildly edematous but no lesions are seen. A 4 mm mass is identi-
fied on the right adrenal, but is roo sma!l to characterize. (These chest

103
Otolaryngology Clinical Case Studies Chapter 2- Head and Neck

NoTES and adrenal findings are common and should simply be followed. Dedi-
cared scans of the adrenal::; may be done if suspicious.)

• Endoscopy: FOM lesion is noted as on clinical and radiographic ex-


aminmion. The remainder of the examination, including the laryngeal
and esophageal examinations, is otherwise normal. Bronchoscopy was
not pe1jormed because chest CT was negative.

Diagnosis:

Diabetes: hyperosmolar, nonketotic


T2NOMO SCC right FOM The lymph nodes in the neck are not cystic ne-
crotic, and are not large enough to call cancer based on CT scan.

TX Primary twnor cannot be assessed


TO No evidence of primmy tumor
Tis Carcinoma in situ
Tl Tumor 2 em or less in greatest dimei1sion
T2 Tumor more than 2 em but not more than 4 em in greatest dimension
T3 Tumor more than 4 em in greatest dimension
T4 (L ip) Tumor invades adjacent structures (e.g., through cortical bone,
inferior alveolar nerve, FOM, skin of face)
T4 (OC) Tumor invades adjacent structures (e.g., through cortical bone,
into deep [extrinsic} muscle of tongue, maxillary sinus, skin. Super-
fic ial erosion alone of bone/tooth socket by gingival primary is not .
sufficient tiJ classify as T4)

What are the treatment options and their complications? What would you do
and why?

• Treat diabetes by controlling blood glucose levels


• "Lesions" in the lungs and the adrenal are too small to characterize and
should be followed with repeat CT scan in approximately 3-4 months
• Discuss surgical vs. nonsurgical treatment in light of the likelihood of cure
compared to the morbidity of both surgery and nonsurgical treatment. Spe-
cifically discuss dysphagia, articulation, dentition, and salivary output
• Discuss radiation alone vs. chemotherapy/radiation concurrently as primary
treatment
• Discuss surgical treatment
- "Rim" mandibulectomy, FOM resection versus segmental resection
- Reconstructive options: local flaps - tongue, nasolabial; myocutaneous
flap with or without skin paddle; soft tissue free flap reconstruction such
as radial forearm flap with or without bone plate depending on ablation
Mandibular reconstruction with bone plate vs osteocutaneous flap such
as fibula
Potential complications

How would you follow this individual?

• Periodic examinations
• Consider CT or PET/CT scan at 6 months and one year and possible endo-
scopy as indicated

104
Chapter 2 - Head and Neck Otolaryngology Clinical Case Studies

CASE 5- SALIVARY GLAN D MASS NOTES


Robert J. Sinard, MD

A .f2 year-old female presents with a lump in from of her ear on the right
thea she has recently discovered.

HPI: state that you would:

• Obtain a detailed medical history beginning with a history of the present


illness

What additional historical information would you seek'! State that you would ask:

• How long has the mass been there?


• Has it changed in size since first noticed?
• Is there associated pain?
• Any other masses or problems with facial motion?

She noticed the mass about 3 months ago while just feeling her face. It
may have increased a little in size since first detected. There are no other
masses or pain, and no facial motion problems. Additionally she has no
history of skin cancers.

P MH: Essentially negative


Allergies: NKDA
Medications: None
FH: Negqtive
SH: Smoked 1 ppd fo r 10 years but quit 10 years ago, denies
alcohol
ROS: Negative

What would you look for on PE? State that you would: ·

• Perform a complete head and neck PE including vital signs

PE:

~.. Temp: 98.6 F. BP: 150180, Pulse: 72


GA: WDWN female, exam is relevant for a 3.0 em mass in the
right neck just anterior an~ inferior to the tragus. The mass
has limited mobility but does .move independent of the ear
canal. Facial nerve function is intact. Skin of the head and
neck shows no specific lesions. The skin overlying the
lesion is freely mobile.

105
Otolaryngology Clinical Case Studies Chapter 2 - Head and Neck

NoTES What is your differential diagnosis?

• l Infectious or inflammatory process of the parotid gland


• .:<Primary parotid neop lasm
·3 Metastatic neoplasm
• -<(Lymphadenopathy
• 3 Cutaneous les ion (i.e .. ep ithelial inclusion cyst, etc.)
• £ congenital anomaly (i.e. , branch ial cleft anomaiy, etc.)

What diagnostics would you request? State that you would obtain:

• CT or MRJ with contrast of the neck. MR1 wil l show detail s of soft tissue
better, but CT is less expensive and certainly adequate for this case
• FNA
• CXR

Test results:

• CT reveals a moderately enhancing 3 em mass with slightly irregular


borders arising in the parotid gland, not involving the overlying skin.
There are no other neck masses or abnormalities
• FNA of the mass reveals mucoepidermoid carcinoma, intermediate grade
• CXR shows no lung masses or evidence of metastasis

Diagosis:

T2NOMO intermediate grade mucoepidermoid cancel; of the parotid, pos-


sibly T3 if it extends to involve tissue outside of the parotid gland itself

TX Primary tumor cannot be assessed


TO No evidence of primary lumor
Tl Tumor 2 em or less in greatest dimension without extraparenchymal
extension
T2 Tumor more than 2 em but not more than 4 em in greatest dimension
without extraparenchymal extension
T3 Tumor having extraparenchymal extension without seventh nerve in-
volvement and/or more than 4 em but not more than 6 em in greatest
dimension
T4 Tumor invades base of skull, seventh nerve, and/or exceeds 6 em in
greatest dimension

"
What are the treatment options and their complications? What would you do
and why?

• The treatment consists of surgery which may be followed by post-operative


radiation therapy, depending on the fina l pathology. Surgery would consist of
total parotidectomy with possible sacrifice/reconstruction ofthe facial nerve.

106
Chapter 2 - Head and Neck Otolaryngology Clinical Case Studies

One can debate doing a neck dissection, but most otolaryngologists would NoTES
argue that a neck dissection adds little to the morbidity of the operation and
•vould provide valuable prognostic data and therefore should be performed

Discuss your surgical technique:

• Standard modified Blair incision extended forward to provide access for


neck dissection . The di ssection should include levels I b, 2, 3, and 5
Key is detem1ining what to do witb the facial nerve. Given that the tumor is
not fixed to the ear canal, the gland is mobilized by freeing the parotid from
the sternocleidomastoid, digastric, and ear canal. Next the surgeon must
identify the main trunk of the facial nerve
• Follow the nerve peripherally, and if the plane of dissection is clean over the
nerve, preserve it. If not, sacrifice the relevant branches. Send intraopera-
tive frozen section of the tumor if any facial nerve branches need to be
sacrificed to at least confirm the diagnosis of malignancy
• If any major branch needs to be sacrificed, perfom1 a primary grafting with
great auricular or transverse cervical nerve. If the main trunk or an entire
division needs to be removed, graft with medial antebrachial cutaneous nerve
• One can fill the parotidectomy defect with autogenous fat or AlloDerm in
order to decrease the cosmetic defect and the chance of developing Frey's
syndrome, but one can also make the val id argument that tissue augmenting
can confuse post-operative monitoring for recurrence

What are the potential complications?

t• Facial nerve weakness, which may be temporary or permanent


~ • Frey's syndrome. Be able to discuss the pathophysiology of gustatory sweating
(i.e., post-ganglionic parasympathetic nerve fibers that normally supply the
parotid gland are able to innervate subcutaneous sweat glands normally in-
nervated by the sympathetic nervous system following parotidectomy [both
share acetylcholine as a neurotransmitter.] Gustatory stimulation intended to
cause secretion of parotid secretions leads instead to triggering of cutaneous
:) . sweat glands.)
')• Numbness in the distribution of the cervical plexus
1· Temporary shoulder dysfunction (from the level 5 dissection)

How would you follow this individual? State that you would:

• If final pathology confirms intermediate grade mucoepidermoid carcinoma


with negative neck nodes and ultimately negative margins, post-operative~
radiation to the primary and neck is still recommended because of the ag-
gressiveness of the primary tumor
• In general, intermediate grade mucoepidermoid carcinoma is treated in the
same fashion as high grade mucoepidermoid carcinoma with the expectation
that the patient will do better in terms of survival

107
Otolaryngology Clinical Case Studies Chapter 2 - Head and Neck

N oTEs CASE 6 - P ARAPHARYNGEAL MAss


Michelle Marcincuk, MD

A 35 y ear-old woman presents with a lump in her left neck of 2 year :5


duration which is gradually increasing in size.

HPI: state that you would:

• Obtain a detailed medical history beginning with a histOJy of the present


illness

What additional historical information would you seek? State that you would ask:

• Is the mass painfu l?


• . Does the patient experience dysphagia or odynophagia?
• Does the mass contribute to difficulty in breathing?
• Has the patient experienced aspiration?
Has the patient experienced any hoarseness?

Patient reports that the mass was noticed 2 years ago while putting on
maketp. It is painless. She has recently felt .fullness in the left side of her
throat, especially while swallowing. No other symptoms are reported.

P MH: Healthy, distance runner otherwise in good health


Allergies: NKDA
Medications: Oral contraceptives and multivitan1in
FH: Negative
SH: Nonsmoke1~ occasiona l ETOH
ROS: Negative

What would you look for on PE? State that you would:·

• Perform a complete head and neck PE including a complete neurologic exam


and flexible fiberoptic nasal, pharyngeal -and laryngeal exam

PE:

111..aLs.:
Normal
GA: 35 year-old woman in NAD. No stridor
HEENT Ears: Normal. No serous otitis mec{ia
Nose: Normal, including rigid endoscopy of the NP
Throat: r;ullness in the left lateral wall pushing the left palatine tonsil
toward the midline. Tongue protrudes midline. No sign of exu-
dates or erythema. Hypopharynx and larynx normal by mirror
and flexible endoscopic exam. Vocal cord mobility normal
Neck: 2.5 em firm, nontender relatively immobile mass in the left neck
behind the ramus of the mandible. No other masses, lymph nodes,
or thyroid nodules palpable. Slight trismus noted

108
Chapter 2 - Head and Neck Otolaryngology Clinical Case Studies

Eyes: Normal EOM Fundi normal. Left eye miosis and mild ptosis noted NoTES
New·o: CN otherwise intact. Generalnew·o exam includes cerebellar 1ests,
gair and starion, Rhomherg, muscle srrength, and sensation WNL

What is your differential diagnosis?

• Differential sbou ld include the etiologies of a paraph~nge:- t,.~1ass of two


__ year's duration and an ipsilateral Homer's syndrome (miOsis, an1~ydrosis and
~~-¢tosis). In this case, Horner's syndrome suggests involvement of the cervi-
cal sympathetic nerves
1- Pleomorphic adenoma of the deep lobe of the parotid
C"\-
e/. Neurogenic tumors such as paragangliomas, schwannomas, and neu-
rotibromas
;-3- Lyn~phoma
J.(- Metastatjc disease in the superior deep jugu lar lymph nodes

What diagnostics would you request? State that you would obtain:

• CT scan of the neck with contrast


• MRl/MRA of the neck
• Possible carotid angiogram depending on the results of the CT and MRI
• FNA biopsy unless imaging studies suggest a vascular tumor or hypervascular
lesion

Test results:

CT shows a mass in the left posterior (poststyloid) parapharyngeal space.


It is inhomogeneous on contrast- enhanced CT (See Figure 1). Tl-weighted,
pre-contrast lv!Rl reveals a mass, isodense to muscle, of the poststyloid
parapharyngeal space (See Figure 2). Tl -weighted, post-contrast MRI
-- shows enhancement of the mass after administration of gadolinium (See
Figure 3). T2-weighted, pre-contrast MRl shows the inhomogeneous na-
ture of the parapharyngeal space mass. Note that the mass does not have
the high signal characteristics on T2-weighted images which are expected
of a pleomorphic adenoma (See Figure 4). The ICA and jugular veins are
displaced, but patent and uninvolved with tumor on MRA. Carotid angio-
gram was deemed unnecessary.

Diagnosis:

The presumptive diagnosis is neurofibroma of the left cervical sympathetic


chain. Complete surgical excision is required.

What are the surgical routes to the posterior pharyngeal space'?


t
• Transcervical approach with or without mandibulotomy
<")
.,.,. Transparotid
3 infratemporal
<r Transoral
·~"'· Combinations ofthe above 4 approaches
--'

109
Otolaryngology Clinical Case Studies Chapter 2 - Head and Neck

NoTEs The approach used depends on the size and location of the tumor, its relation to
the great vessels, and the index of suspicion of malignancy. When removing
benign tumors, the approach chosen must limit potential morbidity to a minimum

Surgical techniques:

High lights and pitfalls:


• The transcervical approach provides adequate exposure in most cases in a
region of the neck familiar to most surgeons
• The transparotid approach has wide appli cation for many types of
parapharyngeal space tumors, including those located high in the neck, and
can be extended to include a suboccipital craniotomy for intracranial exten-
sion. However, the facial nerve is at risk of injury from this approach
• The infratemporal fossa approach can be extended to encompass a temporal
·craniotomy, and is useful for lesions involving the superior most regions of
the parapharyngeal space and skull base
• The transoral approach is rarely used due to limited visualization of the sur-
gical field, risk to the great vessels, subtotal removal, increased risk of infec-
tion, and tumor rupture

Discuss your surgical technique:

• The transcervical approach was chosen without mandibulotomy due to the


wide exposure provided, the ability to gain proximal and distal control of the
great vessels if necessary, and the ability to increase the exposure by includ-
ing a mandibulotomy if needed. The patient was counseled regarding the risk
to the major vessels and CN IX-Xll, including the risk of stroke, hoarseness,
aspiration, dysphagia, shoulder weakness, and tongue weakness. She was
advised of the probability of persistent postoperative, permanent Homer 's
syndrome. The postoperative scar was outlined for her
The skin incision is placed in a natural crease of the neck midway be-
tween the mandible 1md the clavicles. The marginal mandibular branch
of the facial nerve is identified and preserved by elevating it with the
superior skin flap. The facial artery and anterior facial vein are ligated
and divided. The submandibular gland is retracted anteriorly
- Parotidectomy is not necessary in benign neurogenic tumors of the
poststyloid parapharyngeal space. A midline mandibulotomy with a man-
dibular swing was not necessary for adequate exposure
- Care is taken to preserve the carotid vessels, CN IX- XII, and the cervi-
cal sympathetic plexus if possible
- If the carotid artery is at risk, i.e., with a paraganglioma of the vagus,
nerve proximal to the jugular foramen, the incision is carried up posteri-
orly over the mastoid. In this way, the mastoid cortex may be removed to
permit proximal control of the ICA and internal jugular vein if necessary

Diagnosis:

Frozen section of the tumor at the time of resection confirmed the diagno-
sis of benign schwannoma.

110
Chapter 2 - Head and Neck Otolaryngology Clinical Case Studies

What are the potential complications? NOTES

I • injury to the JCA and intemal jugular vein


? • Injury to CN IX-XU
3 · Persistent Homer's syndrome is likely
How would you follow this individual?

• 1 or 2 postoperative office visits and yearly exams to evaluate for recurrence

Figures:

Figure 1: Axial, contrast-enhanced CT of left parapharyngeal neurofibroma


showing inhomogeneous mass

111
Otolaryngology Clinical Case Studies Chapter 2 -Head and Neck

NOTES Figure 2: AxiaL T !-weighted, pre-contrast MRI reveals a mass, isodense to


musc le of the poststy loid parapharyngeal space

112
Chapter 2 -Head and Neck Otolaryngology Clinical Case Studie s

Figure 3: Axial, TJ-weighted, post-contrast MRJ shows enhancement of the NoTES


mass after administration of gadolinium

113
Otolaryngology Clinical Case Studies Chapter 2 - Head and Neck

NOTES Figure 4: Axial, T2-weighted pre-contrast MRJ showing inhomogeneous mass


of the parapharyngeal space without the high signal characteristics
expected of a pleomorphic adenoma

114
Chapter 2 - Head and Neck Otolaryngology Clinical Case Studies

CASE 7 -ANGIOFIBROMA NoTES


Robert T. Adelson, MD

A /7 year-old white male presents 1-vith persistenr left-sided rhinorrhea,


nasal airway obstruction and lejr maxillary facial pain.

HPI: state that you would:

• Obtain a detai led medical history beginning with a history of the present
il~1ess

What additional historical information would you seek? State that you would ask:

• For how long has the patient had these symptoms?


• What is the nature of the rhinon-hea (watery/purulent)?
• Does he have hyposmia?
Episodes of fever?
• Fatigue?
• Cough?
• Prior episodes of rhinosinusitis?
• Epistaxis?
• Aural fullness/Otalgia?
• Otorrhea?
• Proptosis?
• Exophthalmos?
Diplopia? ·
-
The patient has had the symptoms of left-sided nasal airway obstruction
and mucoid, non-purulent rhinorrhea for about 6-9 months. The left fa-
cial pain has been present for the last -6 weeks. This pain is constant and
low grade with occasional exacerbations. The patient has had 2 episodes
of acute left maxillary sinusitis in the past 4 months: each was re_sponsive
to the medical regimen prescribed by his primary care physician (mucolytics.
decongestants, and 2'"1 generation cephalosporins). Though the patient
reports fever, cough and dentalgia during the episodes of sinusitis, these
are not chronic problems. The patient admits to left aural fullness without
otalgia or otorrhea. The patient has had 3 episodes of left-sided epistaxis
over the past 4 months: each was successfully treated with nasal
oxymetazpline and external pressure. The patient has no hyposmia or vi-
sion changes, but does admit tp decreased hearing in the left ear.

PMH: None
PSH: None
Allergies: NKDA
Medications: Allegra-D. Flonase nasal spray. The patient has been treated
V.•ith Cefiin, Guaifenesin, and pseudoephedrine during pre-
vious episodes of sinusitis
FH: NIDDM
SH: (-) Tobacco, (-) ETOH, (-) illicit drugs, 11'" grade

115
Otolaryngology Clinical Case Studies Chapter 2 - Head and Neck

NOTES Student ROS: In addition 1he findings given above (see HPJ), the palienr
reports occasional leji-s ided non-pulsarile linnitus. He de-
nies odynophagia, dy sphagia, voice changes. weight loss.
verligo. diplopia. and facial paresthesia/ hypesthesia

What would you look for on PE? State that you would:

• Perform a complete head and neck PE including vital signs and symptom-
directed endoscopic examination

PE:

J!iJ.a.JJi.: Temp: 99.2 F, BP: 125175, Pulse: 70, RR: 18


WDWN WM in NAD: NCIAT
AD: TMIEAC normal
AS: Amber-colored MEE. TM moves ·with negative pres-
sure on pneumatic. otoscopy. EAC normal
Nose: Exam of left nasal cavity is obscured by a rounded,
smooth, pale pwple mass which fills the cavity and
pushed the septum toward the patients right. ·The
right nasal cavity is unremarkable, with the excep-
tion of a decreased diameter resulting from the de-
viated septum. Normal mucosa, no lesions, no
polyps _
OPIOC: Normal, no lesions, normal palatal motion, 2+ palatine
tonsils
Salivary glands are palpably normal
No cervical lymphadenopathy
Neuro: CN I-XII are intact. EOM grossly normal. No dys-
topia of globe position
Nasopharyngoscopy: The scope cannot be advanced easily past the left-
sided mass. The scope is advanced through the right
nasal cavity, confirming the impression on anterior
rhinoscopy of normal nasal mucosa without pol-
yps or other lesions. The NP is nearly filled by the
pale-purple smooth mass. The soft palate is not in-
volved. The visualized portion of the tongue base
is normal

What is your differential diagnosis?


I
~ 11
Inflammatory nasal polyposis

1 <-r Angiomatous polyp
0
-~ .- Inverting papilloma Pyogenic granuloma
7 • JNA ( .·
'- '
(' ,
Teratoma
<t • Kaposi sarcoma )'- Rhabdomyosarcoma
s. Hemangioma } Lymphoproliferative lesion
G. sec '-' Mucosal malignant melanoma
7 •· Encephalocele 1..-
'r:-
M.
1 enmgoce e
I
0• Metastatic malignancy ' Minor salivary gland carcinoma
(Renal cell carcinoma)

11 6
Chapter 2 - Head and Neck Otolaryngology Clinical Case Studies

What diagnostics would you request? NoTES

• The fi rst step in the diagnostic work-up of a unilateral nasal/nasopharyngeal


mass should be to obtain imaging.

Why is it important to obtain imaging of unilateral nasal/nasopharyngeal masses?

A contrast-enhanced CT scan of the paranasal sinuses (axial/coronal) should


prevent one from taking a biopsy of a meningocele, encephalocele, heman-
gioma or angiofibroma in the otolaryngology office. The office setting is not
conducive to the control of either a significant hemorrhage or an iatrogenic CSF
leak

The CT scan demonstrates an intensely-staining left-sided nasopharyn-


geal mass filling the NP and left nasal cavity. The pterygopalatine fossa is
widened and the vomer is eroded and largely displaced to the right. The
p osterior wall of the left maxillary sinus is bowed anteriorly (Holman-
--~--
k!!.!.l:_'-·-~ign). No erosion of the cranial base is noted. · :,

Do CT findings provide enough information to make a diagnosis? What is the


lesion classically depicted above?

Yes, CT imaging findings are considered pathognomonic for a JNA. Recently,


authors are recommending MRl as the diagnostic imaging modality of.choice,
as the true territory of a JNA is more clearly depicted and post-treatment tu.ll)or
recurrence is more easily monitored by MRI than CT.

• Is a tissue diagnosis necessary before proceeding with treatment of this


neoplasm? The role of histopathologic diagnosis in the management of a
suspected JNA is controversial. Imaging modalities have obtained such a
high degree of specificity for this lesion over the past 25 years that mcist
otolaryngologists are comfortable proceeding with surgery based on a high-
quality CT or MRl study. Biopsy is reserved for thOse lesions which occur in
people outside of the typical patient profile for INA's, with equivocal CT/
MRI studies, or in whom further confinnatory studies (such as angiography)
are medically contraindicated. Biopsy of the hypervascular lesion should take
place in the OR
• When is angiography employed in the diagnosis of JNA? Angiography is a
time-honored technique for diagnosing JNA; however, advances in CT and
MRI technology and radiologist's experience have relegated angiography to
an ancillary role in diagnosis of suspected JNA. Ambiguous cases can be
investigated with ar.giography, wh ich may identify features classic for a
hypervascular lesion, thereby confim1ing the diagnosis. The main role for
angiography in management ofJNA is the preoperative identification of feed-
ing vessels and subsequent embolization of these arteries
·~ 1\Wha.t are the classic a?giograp&ic findings of JNA: JNA hac'f\characteristic
- feedmg vessels, artenal phase aneurysmal formatiOn, a dense' homogenous
blush of the capillary phase, and the early appearf.ln,ce of large venous drain-
acre channels ~j
"'

117
Otolaryngology Clinical Case Studies Chapter 2- Head and Neck

NOTES Diagnosis:

This young male has a cluster of symp1oms easi~v associated with a mass
lesion located within the left nasal cavity and left side of the NP: long-
standing lejl-sided nasal airway obszruction, history of left-sided maxil-
lary sinusitis, left serous otitis media, and episodes of epistaxis. The imag-
ing studies ore pathognomonic for a JNA.

What is the stage of this JNA?

(Fisch Stage II. Sessions Stage IJA)

JNA staging systems

Sessions classification:
Stage IA - Tumor limited to posterior nares and/or nasopharyngeal vault
Stage lB - Tumor involving posterior nares and/or nasopharyngeal vault with ·
involvement of at least 1 paranasal sinus
Stage IIA - Minimal lateral extension into pterygomaxillary fossa
Stage liB -Full occupation of pterygomaxillary fossa with or without superior
erosion of orbital bones
Stage IliA- Erosion of skull base (i.e., middle cranial fossa/pterygoid base);
minimal intracranial extension
Stage IIIB -Extensive intracranial extension with or without extension into ·cav- _
ernous smus

Fisch classification:
Stage I - Tumo~ limited to nasal cavity, NP with no bony destruction
Stage II - Tumor invading pterygomaxillmy fossa, paranasal sinuses with bony
destruction
Stage Ill - Tumor invading infratemporal fossa, orbit and/or parasellar region
remaining lateral to cavernous sinus
Stage IV -Tumor invading cavernous sinus, optic chiasm, and/or pituitary fossa

What are the treatment options and their complications?

• Counsel the patient and patient's fami ly: what are the available treatment
options: (Given the epidemiology of these tumors, the patients are only rarely
adults [average age of presentation= 14 years], and as such the parents are
included in medical decision-making.) Surgical resection and XRT are the
only acceptable prima1y treatment strategies for these tumors
• Is surgery superior to radiation therapy in the management of INA's? Equiva-
lent treatment results have been reported for both surgery and XRT in the
management of JNA's: 15-20% local recurrence/residual disease with 100%
ultimate control. Studies have found the risks incurred with each treatment
modality to be roughly equal: estimated at a 1% chance for a fatal complica-
tion. Advances in endoscopic surgery allow lesions (through Stage III) to be
resected without facial incisions, while improvements in radiation therapy
decrease the risk of ocular and neural complications. Routinely excellent
control of a stage II lesion has been demonstrated both by XRT and surgery

118
Chapter 2 - Head and Neck Otolaryngology Clinical Case Studies

• What are the major side effects assoc iated with radiation therapy and with NOTES
surgery? In addition to the risks for major complications described above.
both methods have post-treatment side-effects or complications that are
unique to the modality. Each surgical technique has its own inherent compli-
cations (pain, trismus, fac ial paresthesias/anesthesia). XRT introduces risks
for secondary malignancies. cataracts, mucositis and brain irradiation. Lon-
gitudinal stltdies of facial growth have failed to demonstrate aesthetic abnor-
malities resulting from either XRT or surgery
• How is the patient optimized for surgery? A major goal of preoperative man-
agement is devascularization of the JNA. Systemic estrogen therapy . had
been used in the past to decrease tumor vascularity; however, results were
unpredictable
- ls a more predictable method of tumor devascu larization avai !able? The
decision for surgical resection ofthis stage II lesion would also require
angiography to delineate the feeding vessels in addi6on to TAE of the
vascular supply. DPTE has been introduced in an effon to reduce the
risks ofTAE (especially when the JCA is involved). Both methods are
carried out 24--48 hours prior to pIanned resection and each been shown
to decrease EBL as well as the need for intra- and periopen,ttive blood
transfusion. Reduced bleeding in the surgical field improves surgical vi-
sualization of the tumor. The DPTE technique imparts a blue-black stain
to the neoplasm, further enhancing a surgeon's ability to distinguish the
INA from surrounding mucosa

The patient and h~s parents decide to proceed with surgical extirpation of
the JNA.

Surgical technique:

• A small (Stage IIII) lesion is amenable to resection via the transpalatal ap-
proach, lateral rhinotomy, midfacial degloving with LeFort I osteotomy, and
transnasal endoscopic techniques
Highlight: To avoid palatal incisions (and the risk of a palatal fistu la) as
well as facial incisions (with subsequent scarring), the lesion is excised
with transnasal endoscopic techniques
• 24 hours after TAE, the patient was taken to the OR
• intranasal decongestion with topical 4% cocaine
• The tumor was released from the septum and vomer by dividing the
septum anterior to the tumor
• The maxillary sinus ostium was identified and widened until the or-
bital floor and the posterior wall of the maxillary s~nus could be visu-.
alized. The posterior half of the middle turbinate was resected, ex-
posing the sphenopalatine foramen. The posterior wall of the maxil-
lary sinus was removed, widely exposing the tumor. The tumor was
moved medially and elevated away from the pterygopalatine fossa
• Endoscopic inspection of the resection site allowed burrs to remove
residual disease and bipolar cautery to treat any bleeding sites
• The cavity was packed with antibiotic impregnated gauze and the
patient is started on systemic antibiotics to coverS. aureus

119
Otolaryngology Clinical Case Studies Chapter 2 - Head and Neck

NOTES - Pitfall: The first chance to obtain a tissue diagnosis for suspected JNA is
often during resection. A frozen section diagnosis could prevent an in-
sufficient operation if a malignancy is diagnosed

What are the potential complications?

• Hemorrhage: TAE and DPTE can decrease the intraoperative blood loss;
however, the patient must be prepared to receive a blood transfusion and the
attendant risks of donated blood
• Stroke: TAE involves a 2.:1 1% risk of serious vascular complication. The
risks of DPTE are thought to be less, yet this technique is neither as wide-
spread norwell-accepted as TAE. Surgical manipulation and ligation of feed-
ing vessels is subject to similar complication
• Complications of endoscopic sinus surgery - The rare but serious risks of
FESS are considerations in tumor resection: orbital injllly, blindness, diplopia,
epiphora, CSF rhinorrhea, and intracranial injUJy
• Facial growth disturbance: These operations are usually performed on pa-
tients with incomplete development of the facial skeleton. Large retrospec-
tive reviews have not identified clinically significant changes in cosmesis. Of
note, similar studies of patients receiving XRT find no long-term clinically
significant impaim1ent offacial growth

How would you follow this individual? State that you would:

• Remove nasal packing by post-op day #5


• Confirm the diagnosis by permanent histopathology
• Qrder follow-up imaging studies and nasal endoscopy biannually for the first
year, and then annually thereafter
• MRJ with gadolinium may detect recurrence better than contrast enhanced
CT scan

What is the management of residual/recurrent JNA?

• Recurrent (residual) disease detected on clinical exam or radiologic studies


may be closely followed by endoscopy and CT/MRI at more frequent intervals
- No treatment should be proposed until the residual mass becomes
symptomatic
- Long-term follow- up of patients with residual tumor has demonstrated
slow involution and regression to be the expected course
• Residual disease that becomes symptomatic is treated equally well by sur-
gely and XRT
- Intracranial recurrence is usually tre~ted with XRT

. What is the role of chemotherapy in management of JNA?

• Small studies have demonstrated that chemotherapy regimens can produce


cures or asymptomatic disease states in patients with recurrent disease in-
volving vital intracranial structures or those receiving the majority of their
vascular supply from intracranial vessels

120
Chapter 2 - Head and Neck Otolaryngology Clinical Case Studies

References: NOTES

1. Chaloupka JC. Mangla S, Huddle DC. Roth TC, Mitra S, Ross DA, Sasaki
CT. Evolving experience with direct puncture therapeutic embolization for
adjunctive and palliative management of head and neck hypervascular neo-
plasms. La1yngoscope. 1999~ I 09: 1864-72.
2. Cummings BJ, Blend R, Keane T, Fitzpatrick P, Beale F, Clark R, Garrett P,
Harwood A, Payne D, Rider W. Primmy radiation therapy for juvenile na-
sopharyngeal angiofibroma. La!'yngoscope. 1994; 12:1599-05.
3. Cummings BJ. Relative risks factors in the treatment of juvenile nasopha-
ryngeal angiofibroma. Head Neck Surg. 1980;3( 1):21-6.
4. Herman P, Lot G, Chapot R, Salvan D, Huy PT. Long-term follow up of
juvenile nasopharyngeal angiofibromas: analysis of recurrences. Laryngo-
scope.1999;109:1 40-7.
5. Lowlicht RA, Jassin B, Kim M, Sasaki CT. Long-term effects ofLe Fort I
osteotomy for resection ofjuvenile nasopharyngeal angiofibroma: trends and
limits. Arch Otolaryngol. 2002; 128: 923-7.
6. Moulin G, Christophe C, Gras R, Gueguen E, Dessi P, Gaubert JY, Bartoli
JM, Zanaret M, Botti G, Cannoni M. Juvenile nasopharyngeal angiofibroma:
comparison of blood loss during removal in embolized group versus
nonembolized group. Cardiovasc Intervent Radio!. 1995; 18:158-61.
7. Reddy KA, Mendenhall WM, Amdur RJ, Stringer SP, Cassisi NJ. Long-term
results of radiation therapy for juvenile nasopharyngeal angiofibroma. Am J
Otolaryngol. 2001; 11 (3): 172-5.
8. Roger G, et al. Exclusively endoscopic removal ofjuvenile nasopharyngeal
angiofibroma: trends and limits. Arch Otolaryngol Head Neck Surg.
2002; 128:928-35.
9. Scholtz AW, Appenroth E, Kammen-Jolly K, Scholtz LU, Thumfart WF.
Juvenile nasopharyngeal angiofibroma: management and therapy. Laryngo-
scope. 2001;111:681- 7. ·
10. Tewfik TL, Tan AK, al Noury K, Chowdhury K, Tampieri D, Raymond J,
Vuong T. Juvenile nasopharyngeal angiofibroma. Otolaryngol.
1999;28(3): 145-51.
11. Tranbahuy P, Borsik M, Herman P, WassefM, CasascoA. Direct intratumoral
embolization of juvenile nasopharyngeal anigofibroma. Am J Otolmyngol.
1994; 15( 16):429-35.

12 1
Otolaryngology Clinical Case Studies Chapter 2 - Head and Neck

NOTES

122
Chapter 3- General Otolaryngology Otolaryngology Clinical Case Studies

CHAPTER 3/CASE 1- CHRONIC MIDDLE EAR EFFUSION NOTES


RaviN. Samy, MD, FACS

An 80 year-old righ1-handed Caucasian male presents with a his!OIJ' of


left-sided hearing loss.

HPJ: state that you would:


6

• Obtain a detailed medical history beginning with a histmy of the present illness

What additional historical information would you seek? State that you would ask:

• How long has the patient had hearing loss?


• Has the hearing Joss progressed?
• Does he have tlucruating hearing loss?
• Can he sti ll use the phone in the left ear?
• Is there any history of trauma to the ear?
• Does he have any hearing loss in the other ear?
• Did he have a recent upper respiratory tract infection?
• Is there a FH of hearing loss?
• Does he have a history of noise exposure?
Does he have a history of recurrent ear infections?
• Has he had tympanostomy tubes placed or any other otologic procedure
done?
• Has he ever used hearing aids?
• Is there any otorrhea?
Does the patient have any dizziness?
• Does he have any otalgia?
• Does he have any tinnitus?
• Does he have any aural fullness?
• Has the patient noticed any facial weakness or numbness?

The patient states tha( he has a 3-month history of hearing loss in the left
ear that begart after an upper respiratory tract infection. He feels that the
hearing loss is stable and is neither progressive nor fluctuating. Although
he is still able to use the phone in the left ear, he prefers using it in the right
ear. He feels he has lost 50% of the hearing in his left ear. He has never
had any trauma to his head or ear. He does not complain of any hearing
loss in the right ear. He does have a FH of hearing loss in his father. He
also has a history of noise exposure (he used to fly planes in World War
II). He does not have a history of recurrent ear infections. He has not had
any type of ear surgery. He has never used hearing aids. He does not have
any drainage from his eaT: He also denies dizziness, tinnitus, otalgia, and
facial weakness or numbness. However, he does have aural fullness in his
left ear.

PMHIPSH: Prostate cancer (treated by radiation therapy), colon cancer


(treated by surgery and an unknown type of chemotherapeu-
tic agent). cholecystectomy, severe allergies, obesity

123
Otolaryngology Clinical Case Studies Chapter 3- General Otolaryngology

NOTES Allergies: NKDA


Medications: Mulrivitamins
FH: Cancer
SH: The patient is a retired salesman. He is married. He has 2
children who are alive and well. The patient has a heavy
history of smoking (80 pack years) and continues to smoke 1
ppd. He rarely drinks. He denies any HIV risk factors
ROS: Frequent episodes of heartburn

What would you look for on PE? State that you would:

• Perform a complete head and neck PE


• In addition, you would perform a systemic exam (e.g., cardiac, abdominal,
etc.) as you would deem appropriate based on patient symptoms, signs, or
J:>E findings

PE: ·

Yi1.al.s.: Temp: 98.6 F, BP: 123/71, Pulse: 74, Wt: 206lbs., Ht: 5'10"
GA: WDWN elderly Caucasian male, in NAD
Neck: Normal. No lymphadenopathy, parotid masses, or thyroid
masses
Nasal: No masses, polyps, or pus on anterior rhinoscopy OCIOP ex-
amination: No leukoplakia, no bleeding, no masses .
Ears: Binocular otomicroscopy is pe1jormed. First the right ear (nor-
mal side) is examined. The right auricle, ear canal, and TM
are all completely normal. On the left side, you notice that the
auricle and EAC are normal. However, the TM is retracted
and has decreased mobility. Yellow fluid with bubbles is seen
behind the TM Tuning fork tests are pe1jormed: Weber later-
alized to the left ear at 512 Hz. Rinne test revealed BC greater
than air on the left, but air conduction greater than bone on
the right
Neurologic: All CN are normal on close examination. Facial nerve func-
tion is grade !/VI HB

What diagnostics would you request? State that you would:

• Obtain an audiogram with tympanometry

Test results:

Audiologic testing: The patient has mild sensorineural hearing loss in the
right ear (SRT 30 dB, word discrimination of 100%). On the left side, the
patients cochlear function is the same as the right but with a conductive
loss of about 10- 15 dB, depending on the frequency (i.e. , mixed hearing
loss in left ear). Word discrimination is 92%. The patient has a normal
(Type A) tympanogram in the right ear but a flat (Type B) tympanogram in
the left eaT:

124
Chapter 3 - General Otolaryngology Otolaryngology Clinical Case Studies

Before proceeding with discussion and treatment of the left ear, what is the NoTES
most important thing to do for an adult with a unilateral MEE?

• State that you would perfom1 nasopharyngoscopy to rule out a nasopharyn-


geal mass

Test results:

Nasopharyngoscopy was normal.

What are the treatment options and their complications? What would you do
and why?

• Ideally, one would be able to take care of the MEE in a noninvasive manner
(i.e., medically and not surgically)
• Options include observation until the effusion has resolved, a trial of nasal or
systemic steroids (to reduce peritubal edema), course of antibiotics (Amoxicillin
SOOmg po TID x l 0 days), and/or Afrin nasal spray for 5 days ·

If medical treatment fails to eradicate the effusion, when and what type of surgical
procedure should you consider?

• Although a myringotomy with tympanostomy tube placement may be con-


sidered at any time, the patient needs to be made aware that approximately
90% of chronic MEE resolve within 90 days. Tubes can be placed in adults
and teenagers as an office procedure but require general anesthesia for
placement in children

Some complications of tympanostomy tube placement are:

1
• Permanent TM perforation
• Hearing Loss
-• Otorrhea
!
• Mild hearing loss
·· • Iatrogenic cholesteatoma

For these reasons, once a tube is placed, the patient needs follow-up until the
tube comes out and the TM has healed without complication.

Functions of the eustachian tube are: '


·, ' .

To protect the middle ear and mastoid from nasopharyngeal secretions, to aer- \
,/'"\~.
- ,{
' • I
: • ' ""

ate the tympanomastoid compartment, and provide mucociliary clearance of


secretions from the compartment

Due to the more horizontal position ofthe eustachian tube and its shorter length,
children are more prone to eustachian tube dysfunction

125
Otolaryngology Clinical Case Studies Chapter 3- General Otolaryngology

NOTES List some reasons why eustachian tube dysfunction may have occurred in this
patient.

\ • Viral infections: due to decreased ciliary activity, increased peritubal edema,


and increased mucous production. In addition, inflammatory mediators re-
leased due to an infection may also increase the risk of eustachian tube
dysfunction . Some viruses also have an immunosuppressive effect, increas-
ing the risk of bacterial superinfection
• Allergies: patients with allergies may have an increased predispo.sition to
eustachian tube dysfunction. Treating these patients with nasal steroids and
antihistamines may prevent the recunence of eustachian tube dysfunction.
In those with severe or perennial allergies, allergy testing with immunotherapy
may be appropriate
. • Smoking: Another reason to have patients quit smoking! Smoking also de-
creases ciliary activity and increases tubal edema and mucous production
• Laryngopharyngeal reflux: Although relatively newly discovered, this may
be of significant importance in.some patients. Performing a thorough history,
flexible fiberoptic nasopharyngoscopy and laryngoscopy, and dual pH probe
testing could help identify this as a significant factor in this patient. Treat-
ment with conservative measures (elevating head of bed, no eating 2-4 hours
before sleeping, weight Joss, etc.) and proton-pump inhibitors is then indicated
J • CRS: Inflammatory disorders of the nasal cavity and paranasal sinuses may
give rise to similar inflammatory changes of the torus tubarius, resulting in
eustachian tube dysfunction

References:

1. Inglis AF Jr, Gates GA. AOM and otitis media with effusion. In: Cummings
CW, Fredrickson JM, Harker LA, Krause CJ, Schuller DE, Richardson MA,
eds. Otolaryngology- head and neck surgery. 4th ed. St. Louis: Mosby-Year-
book; 2005, 4445-68.
2. Slatter WH III. Pathology and clinical course of inflammatory disease of the
middle ear. In: Glasscock ME, JuliannaA, GulyaAJ, eds. Surgery ofthe ear.
5·th ed. Hmilton, ON: B.C. Decker; 2002, 422-33.

126
Chapter 3 - General Otolaryngology Otolaryngology Clinical Case Studies

CASE 2 - UNILATERAL NASAL MASS NOTES


Amber Luong, MD, PhD

A 52 year-old white male presents wirh a 3-month history of worsening


nasal obstruction.

HPI: state that you would:

• Obtain a more detailed med ical, surgical and rhinologic history beginning
with the chief complaint

What additional historical infm·mation would you seek? State that you would ask:

• Any concurrent nasal symptoms such as rhinorrhea, epistaxis, and anosmia?


• Any allergy symptoms or history of allergies?
• Any history of fevers, chills or evidence of systemic involvement?
• Any recent trauma?
• Any history of foreign body in the nasal cavity?
• Any blurred or loss of vision? Any diplopia?
• Any palpable or visible growth on or near the nose.
• Any neurological complaints or changes?
• Any pulmonary symptoms such as cough or hemoptysis?
• Any unintentional weight Joss?

Initially he noted difficulty breathing through his nose at night when he


was lying down. The obstruction at first oscillated between the right and
left nasal cavity. Then, over the last several months, the obscruction re-
mained right-sided and persisted all day. He admits to recurrent epistaxis,
about 2- 3 times a week. He associates it with his increased stress level. He
denies any history of anosmia. He does suffer from seasonal allergies: but
they have been controlled with antihistamines. There has been no recent
increase in allergy symptoms. He denies any systemic illness. No recent
trauma or history of placing a foreign body in his nose. No change in his
vision or neurological status. Howeve1; he repoi-ts crusting in the corner
of his right eye every morning the last couple of months. He has not had
problems with a cough or hemoptysis. No unintentional weight loss.

PMH: HTN and hyperlipidemia. No DM or heart disease


PSH: Tonsillectomy and adenoidectomy at age 6, ACL repair at
age 41
Allergies: NKDA
Medications: Simvastatin, metoprolol, baby aspirin, OTC vitamins
FH: Father died of lung cancer. Mother alive at 78 years-old
with no H/0 cancers or serious illnesses
SH: Used to smoke a ppd, but quit 6 years ago. Occasional
alcohol. No IV drug use
ROS: No change in hearing, GI function, bowel habits, or
urination. Suffers from no asthma symptoms

127
Otolaryngology Clinical Case Studies Chapter 3- General Otolaryngology

NOTES What do you look fo•· on PE?

• State that you would perfom1 a complete head, neck. and rhinology PE in-
cluding vital signs and nasal endoscopy

PE:

l11JJls.: Temp: 98.6 F. BP: 105152. Pulse: 90, RR: 18, Wt: 191 lbs.
Ht: 70 in.
GA: Well-developed male in NAD
Head: Normal cephalic, no asymmeTry
Ears: AU- normal clear TMs. normal mobility, no MEE, no
masses
Eyes: No proptosisltelecanthus, no restricted gaze, no diplopia,
normal exam. no active epiphora
Nasal cavity: Septum is midline with no crusting or prominent blood ves-
sel on the anterior septum. Right nasal cavity hqs a pale
soft-tissue mass lateral to the middle turbinate. The mass is
causing medialization of middle turbinate. It extends ante- .
riorly to the anterior aspect of the inferior turbinate. Clear
rhinorrhea is present. No purulence present. Left cavity is
clear without masses or lesions
Nasal
endoscopy: After deconges(ant applied, an endoscope could be
passed posterior to the mass. The mass could be followed
to the lateral aspect of the uncinate process. Discomfort
prevented further endoscopic evaluation
OC: No masses, lesions, or other abnormalities, no obvious post-
nasal drainage
Neck: Supple with no nodes or bruits
Neuro: CN normal

What diagnostics would ·you request?

1. CT with contrast of the sinuses


2. MRI with gadolinium of the nasal sinuses
3. Biopsy of nasal mass after the CT and MRl scans completed
4. Labs - CBC, chem. 10

What is your differential diagnosis?

" • Inverted papilloma - These masses are common benign nasal neoplasms.
These tumors consist of proliferation of the squamous epithelium that invo-
lute into the underlying stoma and frequently originate from the lateral nasal
wall. More commonly unilateral, 10% of these tumors present bilaterally.
Although it is a benign tumor, 10% are noted for malignant changes
• Hemangioma- This is a common benign lesion that presents as a unilateral
nasal mass.lt is the most common benign intranasal lesion in children. Similar
to hemangiomas in other sites, these lesions have a predictable proliferative
phase that lasts for several months. This phase is followed by an involution

128
Chapter 3 - General Otolaryngology Otolaryngology Clinical Case Studies

cycle that can last several years. ultimately resolving the lesion. A small N oTES
percentage of hemangiomas do not involute completely and can be present
in adults
Nasal polyposis - Nasal polyps more commonly present as bilateral nasal
masses: however. early disease may affect only one nasal cavity. The polyps
often obstruct the outflow of the sinuses and incite CRS. The polyps consist
of edematous nasal mucosa concentrated with inflammatorv cells
• Esthesioneuroblastoma- This is an uncommon malignant sinonasal neoplasm
that is also refened to as olfactory neuroblastoma. Originating from the spe-
cialized neuroepithelium cells of the upper nasal cavity, these tumors are
differentiated from other neuroendocrine tumors by immunohistochemical
staining and histological characteristics. They present as a unilateral nasal
mass that is locally aggressive. In addition, 10-30% have metastasized via
···-,
the lymphatics or hematogenous system to cervical lymph nodes, lungs, brain
and viscera at the time of presentation
• Encephalocele - An encephalocele is a herniation of CNS tissue through a
skull defect. If the mass includes meninges then it is referred to as a menin-
goencephalocele, and if part of the ventricular system is involved then it is
referred to as meningoencephalocystocele. These are normally congenital
defects affecting l in 3~000 live births. However, iatrogenic or small skull
defects can present in adults. Initial differentiation from other nasal masses
is made on MRI studies that wi II show herniation of CNS tissue through a
skull defect
• Foreign body or blood after trauma- This diagnosis should be su.spected
based on history. The PE will confirm the diagnosis prior to imaging studies.
If facial trauma is suspected, a CT of the facial bones will be necessary to
identify any facial fractures that may need surgical intervention
• Antral choana! polyp- This mass is initially a mucous retention cyst in the
maxillary sinus. Over time, the cyst grows and extends beyond the confines
of the maxillary sinus. It can enlarge the natural os and grow to fill the nasal
cavity
o Other benign and malignant intranasal neoplasms - In addition to the tumors
discussed above, there are a number of other possible neoplasms that can
present as unilateral nasal masses. The differential diagnosis is often nar-
rowed after the imaging studies and biopsy results are available

Test results:

• CT with contrast of the nasal sinuses revealed a right intranasal mass


and opacification of the right maxillary sinus. There is no obvious bony
dehiscence of the skull base. Howeve1~ the medial maxillaTY. wall had
bony erosion
o MRI showed a convoluted cerebriform mass within the right nasal cav-
ity on T2-weighted and on contrast-enhanced TJ-weighted images. There
was no intracranial extension
• The biopsy of this nasal mass histologically revealed hyperplastic epi-
thelium that is invading into the underlying stroma
• The CBC was WllL. There was no elevation of white blood cells. There
was also no evidence of pancytopenia. The chemistries were also 'WNL

129
Otolaryngology Clinical Case Studies Chapter 3 - General Otolaryngology

NOTES Diagnosis?

lnverred papilloma

What treatment would you begin?

• Inverted papillomas are treated by surgical resection. About 10% are noted
for malignant changes. Therefore, postoperative radiation is considered if
the resected tumor has any evidence of malignant changes ·
• Given that a majority of these lesions originate from the lateral nasal wall.
medial maxillectomy is the surgical procedure that is necessaty for excision
ofthese tumors. The methods of surgical resection have undergone an evo-
lutiot1 from external approaches to endoscopic surgical approaches. Cur-
rently, most will attempt to resect the tumor endoscopically and only resoti·to
~n external approach if exposure is not adequate for complete removal

Surgical management of inverted papilloma:

· • External approach: Lateral rhinotomy versus midface degloving



•. • Endoscopic approach

Surgical techniques:

Lateral rhinotomy
Highlights:
• The incision extends along the lateral aspect of the nose to around the ala
from the level of the medial canthus to the base of the nose. The inferior .
incision may be_continued through the upper lip iffurther exposure is neces-
sary. A gingivobuccal incision may be necessary for adequate exposure
• The incision is completed through the periosteum
• The periosteum is elevated off the anterior wall of the maxillary antrum
without injuring the infraorbital nerve. The periorbita is elevated off the lamina
papyracea to expose the anterior and posterior ethmoid arteries. Care should
be taken to avoid dan1aging these vessels. Elevation of the periorbita will
remove the lacrimal sac out of the lacrimal fossa
• The lacrimal sac should be transected as distally as possible
• The periorb ita should be elevated such that the medial aspect of the orbital
floor is exposed .
• Remove the anterior wall of the maxillary sinus while preserving the infraor-
bital'nerve
• Elevate the periosteum offthe lateral nasal wall
• Osteotomies through the maxillary bone
- The superior osteotomy should be inferior to the frontoethmoid suture,
which is marked by the anterior and posterior ethmoid arteries. The
osteotomy should be taken just medial to the superior and inferior orbital
fissure without entering the fissures
- The inferior osteotomy is along the nasal floor to the posterior aspect of
the middle turbinate
- The posterior osteotomy transects the orbital floor and intersects with
the posterior aspect of the superior osteotomy

130
Chapter 3 - General Otolaryngology Otolaryngology Clinical Case Studies

• Remove the lateral nasal wall by excising the soft tissue connections to the NoTEs
posterior aspect of this bone fragment. This should result in the complete
removal of the tumor
• Remove the sinus mucosa from the remaining maxi llary si nus
• Remove all the ethmoid air cells and any mucosa
• Complete a sphenoid sinusotomy and remove sinus mucosa. Removing the
sinus mucosa allows for more accurate survei Ilance for recurrent tumor
• Perform dacryocystorhinostomy
• Pack the sphenoid s'inus and line the remaining maxillary sinus and roof of
ethmoid sinus with Gelfoam. Pack the nasal cavity with antibiotic-impreg-
nated antibiotic
• Resuspend medial canthal ligament
• Close the skin incision with precise reapproximation of the vem1ilion border
if the lip was split on the opening incision. Close the gingivobuccal su lcus

Pitfalls:
• CSF leak- If a skull base defect is noted intraoperatively, this should be
a
repaired immediately. IfCSF rhinorrhea is present postoperatively, lumbar
drain shoul9 be placed. The CSF leak will normally heal with this conserva-
tive approach
• Injury to the orbit - Again, intraoperative violations of the periorbita should
be immediately repaired to prevent fat herniation. It is important during the
osteotomies that the posterior osteotomy should not extend into the poste1ior
one-third of the lamina papyracea. This posterior extension puts the optic
nerve at risk for injury
• Epiphora -The intraoperative dacryocystorhinostomy should prevent this
postoperative complication. However, if epiphora is noted, then a revision
dacryocystorhinostomy or stenting of the dud may be necessary

Midfacial degloving:

• Local anesthesia is injected into the nasal vestibule. The nasal cavity is. de-
congested with cottonoids soaked with a vasocontrictor decongestant such
as oxymetazoline
• Make a transfixion incision and bilateral intercattilaginous incisions. Connect
these incisions
• Make a gingivobuccal incision that spans from one maxillary tubercle to the
other
• Elevate the soft tissue in a subperiosteal plane from the anterior maxillary
sinus and from around the pirifom1 aperture without injuring the infraorbital
nerve
• The same medial maxillectomy is perf01med once all the periosteum and
periorbita are elevated to allow for the osteotomies
• The nasal cavity is packed as described above and the incisions are closed

Pitfalls:
• Limited exposure at the frontal ethmoid recess and ethmoid areas - This
approach should be used only if the tumor is limited to the inferior nasal
cavity or to the nasal septum. The lateral rhinotomy has good cosmetic re-
sults and should be utilized for all other tumors

131
Otolaryngolog y Clinical Case Studies Chapter 3 - General Otolaryngology

NoTES • Vestibular stenosis or narrowing- This can be prevented by proper reposi-


tioning and suturing of transfixion and intercaJtilaginous incisions

Endoscopic resection:

Highlights:
• The nasal cavity is decongested with cottonoids soaked with a vasoconstric-
tor decongestant such as oxymetazoline
The tumor is debulked with a microdebrider to the site of origin, which most
commonly involves the maxi llary sinus. A trap should be attached to the
microdebrider to collect the tumor. This can be sent to pathology for review
and to rule out the presence of malignancy
• Resect the middle turbinate for good surveillance exam of the maxillary sinus
• Partial excision of the inferior turbinate as needed. lf the nasolacrimal duct is
transected in the process, then an endoscopic dacryocystorhinostomy should
be completed
• Complete a wide maxillary antrostomy
• Once the site of attachment of the tumor is identified, a large cuff of sur-
rounding mucosa is removed endoscopically
• The bone at the site of origin can be burred to remove microscopic disease to
minimize recurrence rates
• The endoscopic approach has similar recurrence rates as compared to ex-
ternal approaches, approximately 10%

Pitfalls:
• Endoscopic exposure is limited at the anterior and floor of the maxillary
sinus. Therefore, if the preoperative MRI suggests that the tumor involves
these sites, then a transantral approach may be necessary in conjunction
with the endonasal approach for complete resection. Recurrence is most
commonly linked to incomplete resection
• Tumors involving the frontal sinus are sometimes approached both by an
endoscopic and a limited external procedure to insure complete excision ·
• CSF leak- Violation of the skull base with a CSF leak should be repaired
intraoperatively p.~· -·
·- --·

• Injury to the orbit/lamina papyracea - Care should be taken to understand


the anatomy of the lamina papyracea in relation to the attachment of the
middle turbinate and to the maxillary sinus to avoid violating it. However, if a
dehiscence of the lamina papyracea is noted, the operation can proceed as
\
usual while avoiding this area of dehiscence

How would you follow this patient?

• Patients should be started on sinus irrigation to minimize intranasal crusting


• The final pathology results need to be carefully reviewed. If there is any
evidence of squamous cell cancer, then it will be necessary to consult radia-
tion oncology for possible postoperative radiation. There are no randomized
controlled studies available that evaluate the role of postoperative radiation
in situations where malignant cells are found within a resected invet1ed pap-
illoma. In addition, CT with contrast of the neck is important to rule out
regional metastasis to the cervical lymph nodes

132
Chapter 3- General Otolaryngology Otolaryngology Clinical Case Studies

• Given that recurrence is the primary concem following surgical resection of NOTES
inverted papi I lomas. most advocate long-tem1 survei I lance exams

References :

1. Lawson, W, Kaufman. MR, and Biller, HF. Treatment outcomes in the man-
agement of inve11ed pap illoma: an analysis of 160 cases. Laryngoscope.
2003;113:1548-56.
2. Weisman, R. Lateral rhinotomy and media l maxil lectomy. Otolaryngol Clin
North Am. 1995;28: 1145-56.
3. Myers, EN. Operative otolaryngology: head and neck surgery. Phil adelphia:
Saunders; 1997.

133
Otolaryngology Clinical Case Studies Chapter 3- General Otolaryngology

NoTES CASE 3 - DYSPHAGIA - PHARYNX (ZENKER's)


Neelesh Mehendale, MD

A 67 year-old whire male presents with difficulty swallowing.

HPI: state that you would:

• Obtain a detailed medical history beginning with the history of present illness

What additional historical information would you seek? State that you would ask;

• When did the present symptoms begin?


• Was tbe onset sudden or gradual?
• Is there any associated weight loss?
• Is there any pain?
• Oo you have trouble with solids, with liquids, or with both?
Do you regurgitate undigested food ?
• Do you smoke cigarettes or use tobacco?
• Do you drink alcohol?

The patient reports that he first started noticing a "sticking" in his throat
when he swallowed solid food about 4 years ago. The patient reports that
the problem has progressively gotten worse and it is now difficult for him
to eat solids or drink liquids without considerable effort. He states that he
does cough up bits and pieces of undigested food that he had consumed at
an earlier time and his wife complains of his persistent bad breath. He
i-eports that these symptoms have discouraged him from eating well and he
has lost 25 pounds over the past year.

PMH: HTN
PSH: Left inguinal herniorrhaphy - age 28
Allergies: Penicillin - rash
Medications: HCTZ 25 mg po qd
SH: Smoked 2 cigars/week x 30 years - quit 7 years ago
1 glass of red wine/night
Denies illicit drug use
Retired stockbroker
ROS: Patient denies any voice changes. Patient denies any
hematochezia, hemoptysis, or melena. The remainder of the
ROS is normal

What would you look for on PE? State that you would :

• Perform a complete head and neck PE, including direct fiberoptic laryngoscopy

134
Chapter 3 - General Otolaryngology Otolaryngology Clinical Case Studies

PE: NoTES

Temp: 98.8 F. BP: 136186, HR: 76. RR: 1:/.


Ht: 6 '1 ", Wt: 153 lbs.
GA: WDWN male in no obvious distress. Norma/tone of
voice
Head: NCIAT
Ears: EAC:S and TM:~ clear AU
Nose: Septum midline, no obvious masses or lesions
OCIOP: Normal dentition, class 1 occlusion, no palpable or
visible lesions
Neck: No palpable adenopathy or masses, thyroid carti-
lage midline
CN: 11- Xll intake bilaterally
Fib.eroptic scope: Bilateral vocal cords are mobile without masses or
lesions, NP is clea1~ There is slight pooling of se-
cretions in the left pyriform sinus, but no mass· or
lesion is seen

What is your differential diagnosis?

State that the differential diagnosis is for dysphagia


The differential includes Zenker's diverticulum, congenital or acquired ., \ .
I I
stricture, extrinsic compression by neck mass (enlarged thyroid), hypo- I' . ' ;. l
-~
pharyngeal or esophageal carcinoma, Plummer-Vinson syndrome, motil- i.
ity disorders (achalasia, spasm), myasthenia gravis, collagen-vascular
disorders (dennatomyositis, polymyositis), stroke, and trauma
' "'"' ·· 1
'
What diagnostics would you request? State that you would order:

• A contrast radiography study such as a barium swallow or cine esophagram

The hist01y of this pqtient alone raises the suspicion for a Zenker :S diver-
ticulum. The P E does not provide any evidence of mass lesion, CNS lesion,
or trauma.

A Zenker's diverticulum is a protrusion of pharyngeal mucosa between the


oblique fibers ofthe inferior constrictor muscles ofthe pharynx and the trans-
verse fibers of the cricopharyngeus muscle. This space is known as Killian 's
dehiscence. At this point, the diagnosis of a Zenker's diverticulum may be con-
firmed by contrast radiography, including barium swallow and cineesophography.
These tests will demonstrate an outpouching consistent with the diverticulum.
These can range in size from less than 1 em to large pouches that extend into
the chest. Most sacs protrude into the left ~ervical side near the posterior mid-
line and less than 10% protrude into the right side.

135
Otolaryngology Clinical Case Studies Chapter 3- General Otolaryngology

NoTEs Test results:

Barium Swallow - Diverticulum seen just above the level of the upper
esophageal sphincte1: The diverticulum measures 4 cm. No distal esoph-
ageal abnormalities or evidence of mpiration is seen.

Diagnosis:

Zenker~· diverticulum

What are the tr·eatment options and their complications? What would you do
and why?

• State that this is a symptomatic Zenker's diverticulum and you would pro-
ceed to surgical repair using either an open technique or endoscopic tech-
nique
• Zenker's diverticula that are found incidentally and are asymptomatic may
be observed. When the lesions are large enough to be symptomatic, surgery
1s necessary . 1 ,
• Surgical options include any combination ofdiverticulectomy, diverticulopexy,
and cricopharyngeal myotomy. These can be performed via an open or en-
doscopic approach

Although · there are some reports that show no objective benefit of a crico-
pharyngeal myotomy in the treatment of Zenker's diverticulum, there is a trend
toward performing this procedure. 1n fact, in the case of a very small diverticu-
lum, myotomy alone is the preferred operation. Diverticulectomy and
diverticulopexy are both good management options and both have been shown
to have up to 100% success rates. The choice of operation seems to be based
on operator preference, but there is some evidence that diverticulopexy may be
the favored approach in high risk patients. This is because there is a lower
theoretical risk of salivary fistula, mediastinitis, and stricture because the diver-
ticulum is never violated.

Surgical techniques:

Open technique
Highlights and pitfalls:
• General anesthesia
• Perform rigid esophagoscopy with packing of the diverticulum with gauze
• Place NG tube
• Left-sided cervical incision along skin crease halfway between hyoid and
clavicle
• Elevate subplatysmal flaps and retract strap muscles
• Retract pharynx and larynx medially and carotid sheath laterally
• Enter retropharyngeal space and dissect diverticulum from Killian's dehisct>nce
• Perform cricopha1yngeal myotomy
• Clamp neck of sac and excise
• Close sac in layers or with intestinal anastomosis stapler

136
Chapter 3 - General Otolaryngology Otolaryngology Clinical Case Studies

• Close neck incision in layers over Penrose or suction drain NOTES


• Begin nasogastric feeds immediately and begin liquids at 24-48 hours
• Diverticu lopexy is perfom1ed in same manner except the sac is not excised
but is fixed superiorly to prevet1ebral fascia

Endoscopic technique
Highlights and pitfalls:
• General anesthesia
• NG tube not necessary
• Use double-l ipped esophagoscope
• . Place one blade in the esophagus and the other blade in the diverticulum
• The tissue bridge that contains the cricopharyngeus is now between the 2
blades
• Divide tissue bridge using C02 laser, bipolar cautery, or an endoscopic stapler
• Keep patient NPO for 48 hours and then start liquids
• The most feared complication is mediastinitis - can be avoided with meticu-
lous surgical technique
• Other complications include fistu la formation, vocal cord paralysis, and stric-
ture fonnation
• Reoperative procedures have higher incidence of complications than pri-
mary procedures
• Patients with diverticula greater than I 0 em: at significantly higher risk for
complications
• Overall mortality ranges from 0- 3%

How would you follow this patient? State that you would:

• Discharge patient from practice once successfu l oral feeding has been
resumed
• Follow as an outpatient I week later

References:

1. Feeley MA, Righi PD, Weisberger EC, et al. Zenker's diverticulum: analysis
of surgical complications from diverticulectomy and cricopharyngeal myo-
tomy. Laryngoscope. 1999;1 09:858-61.
2. Lacccourreye 0 , Menard M, Cauchois R, et al. Esophageal diverticulum:
diverticulopexy versus diverticulectomy. Laryngoscope. 1994;1 04:889- 92.
3. Narne S, Cutrone C, Chelia B, et al. Endoscopic divetticu lectomy of the
treatment of zenker's diverticulum: results in 102 patients with staple-as-
sisted endoscopy. Ann Otol Rhino! Laryngol. 1999; 108:810-5.
4. Orloff LA. Zenker's and other esophageal diverticula. In: Cummings CW,
Fredrickson JM, Harker LA, Krause CJ, Schuller DE, Richardson MA, edi-
tors. Otolaryngology- head and neck surgery. 3'd ed. St. Louis: Mosby-Year-
book; 1998, 2430-42.
5. Westrin KM, Ergun S, Carlson B. Zenker 's diverticulum-A historical review
and trends in therapy. Acta Otolaryngol. 1996; 116:351-60.

137
Otolaryngology Clinical Case Studies Chapter 3- General Otolaryngology

NoTES CASE 4- DYSPHAGIA


Robert J. Sinard, MD

A /.1 year-old generally healthy female presents with o 1- 2 year history of


increasing problems swallowing.

HPI: state that you would:

• Obtain a detailed medical histOJy beginning with a hist01y of the present illness

What additional historical information would you seek? State that you would ask:

• ls the swallowing problem stable or progressing?


• Has there been weight loss?
• Has the patient changed the types of food she eats because of the problem
or does she continue to eat the same foods, with difficulty?
• What types of food are the hardest to swallow?
• Is there aspiration?
• ls there pain with swallowing or any change in voice?
• Does she regurgitate food swallowed previously?
• Any episodes of pneumonia since the swallowing problem began?

The patient reports that the swallowing problem had been relatively stable
but worsened over the last 6 months or so. She has lost about 5 los. over
this period of time. Bread and grainy foods seem to be the hardest, and
liquids are often needed to wash £hem down. Occasionally she '·gets
choked" when swallowing liqtiids. She has not adjusted her diet signifi-
cantly. There is no pain, and sometimes she gets hoarse. There has been no
regurgitation and no pn~umonia.

PMH: HTN, "mild" diabetes for which she takes no medicines.


She had some atypical chest pains that were evaluated about
2 years ago, but it was felt that her heart was OK. CXR
shows "early" emphysema. She also has arthritis
Allergies: NKDA
Medications : Metoprolol, HCTZ, Vioxx, Xanax
FH: Mother had diabetes
SH: Smoked cigarettes YJ ppd for 20 years but quit 30 years
ago. Has rarely used alcohol
ROS: No other problems except occasional heartburn

What would you look for on PE? State you would:

• Perfom1 a complete head and neck PE including vital signs

138
Chapter 3 - General Otolaryngology Otolaryngology Clinical Case Studies

PE: NOTES

111.al.s.: Temp: 98.6 F, BP: 160/85. HR: 88, Wt: 168 lbs.
GA: WDWN female in no re.~pira!Ory distress with an age-ap-
propriate voice qualir_v
HEENT Ears, nose, OC cleaT: Edentulous but with full upper and
lower plate dentures. OP clear with grossly normal and
symmetric motions of the palate and posterior pharyngeal
wall. Slightly hypoactive gag reflexes Fiberoptic exam shows
no mucosal lesions, grossly normal anatomy of the laryn-
gopharynx, normal vocal cord motion. No neck masses pal-
pable
New·o : No CN defects or gross abnormalities

What is your differential diagnosis'!

• Differential includes cervical esophageal mass, esophageal stricture, diver-


ticulum, achalasia, stlerodenna, neuropathic conditions (diabetes,ALS, MS,
. CVA), cricopharyngeal dysphagia, presbyesophagus/esophageal dysmotility.

What diagnostics would you request?

• Assume that you have all available diagnostic modalities at your disposal. All
of the studies below are acceptable answers, but be prepared to discuss the
techniques, the advantages and limitations of each:
- Random vs. fasting serum glucose with hemoglobin A l C
- Barium esophagram
- Modified barium swallow test
- Flexible endoscopic evaluation of swallowing (with or without sensory
testing)

Test results:

Glucose 145, hemoglobin A1C mildly elevated

• Barium esophagram reveals normal pharyngeal architecture and barium


passes to the esophagus without obstruction. Esophageal walls are
smooth without evidence of ulcer or extrinsic compression except at the
normal aortic impression. Tertiary contractions are present in the mid
esophagus and there is moderate reflux to the mid esophagus with a
hiatal hernia

• Modified barium swallow test reveals mild pooling in the vallecula and
both pyriform sinuses that clears with multiple swallows. There was some
minimal laryngeal penetration but no aspiration. Laryngeal elevation
appeared normal. A barium-coated cookie passed without hesitation

• Flexible endoscopic evaluation of swallovving reveals no pooling of se-


cretions in either pyriform sinus. No laryngeal penetration or aspiration

139
Otolaryngology Clinical Case Studies Chapter 3 - General Otolaryngology

NOTES was noted with either puree or thin liquids. Pharyngeal comracrion
appeared normal and timed appropriately with vocal cord adducLion

Diagnosis:

The diagnosis remains dysphagia, with no specific etiology. The one find-
ing that may point to the most significant problem is the presence of ter-
tim·y contractions of the rnid-esophagus, for which one can give the diag-
nosis of "presbyesophagus. " Though this scenario seems to have an
ungratifying conclusion, it is the most common result of dysphagia evalu-
ation in the elderly. The only treatment modality is swallowing therapy,
and the quality and utility of such therapy depends highly on finding an
appropriate therapist with the skills, knowledge, and enthusiasm to make
a difference.

What are the treatment options and their complications? What would you do
and why?

• Dietary modifications (consistency adjustment through food selections, thick-


eners, etc.)
• Behavioral modifications (small bites, supraglottic swallow, chj.n tucking)
Gastrostomy tube placement with NPO status for extreme cases

140
Chapter 3 -General Otolaryngology Otolaryngology Clinical Case Studies

CASE 5- EPISTAXIS N oTES


M ichelle M arcincuk, MD

A .15 year-old girl presenrs in your office accompanied by her mothe1~ with
bleeding from the left nostril of 2 hours duration.

HPI: state that you would:

• Obtain a detailed medical history beginning with a history of the present


illness

What additional historical information would you seek? State that you would ask:

• How long has the bleeding occurred?


- Is it intem1ittent or continuous?
- Left nostril only or both?
• Is the bleeding out the front only or also down the back of the throat?
• Has she vomited?
· - If so, how much and what does it look like?
• How much blood in total would you estimate came out of the nose?
- A cupfu l?
- A Kleenex full? ·
• Did it start spontaneously or with a sneeze, cough, or trauma?
• What have you done to attempt to control it?
• Has this occurred before?
- How often, how severe, how controlled?
Does she bleed or bruise excessively if she cuts or bumps.herself?
Does she take any aspirin, other medications or herbal medicines?
• Has she had any recent injuries?
• Does she have black stools?
• Has she had difficulty with bleeding (surgeries, extractions, menstrual bleeding)?

Mother reports that child has had intermittent bleeding from the left nostril
in small amounts (stained kleenex) intermittently for three days. No vomit-
ing and normal stools. This has occurred 3- 5 times per month for years
and can be from either nostril or both. Last night the ER physician con-
trolled it with a silver nitrate stick and light ribbon anterior packing but it
began in earnest again this morning. Otherwise, the child is healthy.

PMH: No previous surgery. Occasional cuts and bruises seem to


heal normally
Allergies: NKDA
Medications: Occasional Tylenol
FH: Negative. Mother and father are alive and in seemingly good
health. Maternal grandfather died of a lower GJ hemorrhage
SH: No tobacco, no drugs, no cocaine, no alcohol
ROS: Menses began 3 years ago and are somewhat heavy. Gen-
eral fatigue. No previous history of epistaxis or bleeding
disorders

141
Otolaryngology Clinical Case Studies Chapter 3- General Otolaryngology

NOTES What would you look for on PE? State that you would:

• Perfom1 a complete head and neck exam including vital signs, endoscopic
nasal exam, and the integumentary system in general

PE:

Viw/s: Temp: 98.6 F. BP: 120160, Pulse: 82


GA: 15 year-old girl in NAD holding a face cloth to left nosu·d No
bruises or obvious vascular malformations of skin
HEENT Neck, face and ears - normal
Throat: Normal with headlight and tongue blade except for blood tinged
mucus in OP Multiple punctuate telangiectasia are noted on the
tongue, palate, and gingiva
Nose: Cotton gently reinoved from left nostril. Nasal examination with
headlight and speculum reveals telangiectasias on anterior sep-
tum bilaterally: slow oozing can be seen on the left side

Endoscopic exam with 4 mm - 0 and 30 degree pediatric rigid endoscope


reveals occasional telangiectasias on turbinates and posterior pharyn-
geal wall. Otherwise normal.

What is your differential diagnosis?

• Differential diagnosis should include local factors such as local nasal trauma,
inflammatory diseases of the nasal mucosa, benign and malignant tumors of
the nose, sinuses, and NP. Systemic causes of epistaxis include HHT, coagu-
lation disorders such as hemophilia and.Von Willebrand's disease, HTN, and
arteriosclerotic vascular disease

What diagnostics would you request? Test results:

• CBC - Hemoglobin 9 grarns


- Smear - microcytic hypochromic anemia
- Serum iron and transferrin levels: low serum iron elevated transfer-
rin level
• Reticulocyte count: Elevated
• Platelet count: Normal
• Bleeding time: 5 minutes
• Prothrombin time: Normal
• Activated partial thromboplastin time: Normal
• Plasma fibrinogen : Normal

Diagnosis:

• HHT
• Anemia secondary to chronic blood loss and iron deficiency
• Mild active epistaxis due to HHT

142
Chapter 3 - General Otolaryngology Otolaryngology Clinical Case Studies

HHT (Osler-Weber-Rendu syndrome) is an autosomal dominal1lly rrans- NOTES


miued disease. Pathological~v. mucosal Telangiectasia occur as a result of
a deficiency of muscular and elastic componenrs of the media layer of
mucosal capillaries.

Given this diagnosis, what further testing should be considered?

• MRA or a1teriogram of the bead should be considered as patients with HHT


may also manifest CNS telangiectasias, arteriovenous malfonnations, and
aneurysms .
• Contrasted CT of the chest should be considered to evaluate for potential
pulmonary vascular malfonnations
• Colonoscopy can be performed to assess for possible mucosal telangiecta-
sias of the intestinal mucosa
• Consider evaluation of other family members as this disease is transmitted in
an autosomal dominant fashion, but tends to present during adulthood

Test results:

• MRA - Normal
• Contrast enhanced CT of the chest - Normal
• Colonoscopy - Pending
• FH- Mother has suffered occasional epistaxis. Examination of nasal
mucosa reveals multiple telangiectasia. Death of maternal grandfather
noted in FH

What are the t reatment options and their complications? What would you do
and why?

•- Control the acute epistaxis with careful, limited, left-sided electrocautery


and Iimi ted anterior packing
- Explain to the patient and mother that this is a temporary measure .
- Electrocautery can not be repeated too often, nor perfonned too aggres-
sively or complications such as septal perforation may occur
• Explain available treatment modalities to the patient and her mother
- Schedule pulsed dye laser (585 nm) treatment in the OR or your office
(if the laser is available). This may be done without anesthesia, or with
topical anesthesia. This is preferably done prior to the recurrence of the
bleeding as it is not effective during active bleeding. Explain to mother
that this will have to be repeated at least 3 times to attempt to achieve
long tenn reduction in nose bleeds. No treatments give permanent relief.
Other types of lasers may be used but have added risks such as ND
YAG laser which risks deep destruction and possible septal perforation,
C02 laser which causes mucosal damage, and the argon and KTP la-
sers which also have some risk of mucosal damage
- Septodennoplasty provides 1, 2 or more years of reduction in epistaxis
and can be used if laser treatments fail to give desired results
- Estrogen hom1one treatments have been used topically or systemically
in the past, but are less commonly used since the introduction of newer
laser treatments

143
Otolaryngology Clinical Case Studies Chapter 3 - General Otolaryngology

NOTES • Medical consultation for management of iron deficiency anemia and heavy
menses
• Consider refenal for work-up for potential cerebral and pu lmonary involvement

Potential complications:

• Recun·ent epistaxis
• Fatigue and heaJt failure from anemia
• Soci"al a11d psychological impact of unpredictable recurrent epistaxis

How would you follow this individual? State that you would:

• See her a week after laser treatment in the office


• Review recommendations for home care, including:
- Elevation of head of bed
- Saline and/or saline/glycerin nasal spray to prevent crusting and dryness
- Avoid digital trauma/picking at nasal crusts
Vaporizer use

144
Chapter 3 - General Otolaryngology Otolaryngology Clinical Case Studies

CASE 6- EPIGLOTTIS NOTES


Amy Brenski, MD

27 month WM presents to the p ediatric emergency departmem with 4 hours


offeveJ~ irritability and drooling.

HPI: state that you would ask:


I
• Has there been any noisy breath ing or hoarseness?
• A1.1Y other symptoms of acute upper respiratory or viral illness?
• Cough, runny nose, nausea, vomiting or dianhea?

Mother reports that the child initially had some high-pitched noises with
breathing and tugging at the neck. He was crying and inconsolable. Now
he is drooling and very still. He has been refusing any oral intake for sev-
eral hours. He was not hoarse, but mother states that his cry is different.

PMH: FT/NSVD no complications, no previous hospitalizations


or surgeries
MEDS: Multivitamins, Ibuprofen
Immunizations: Child has not received any of his childhood vaccinations,
parents have refused them due to their other child having
a diagnosis of autism ·
SH: No tobacco exposure, 5 year-old brother with a utism,
Mother works in retail, Father is a CPA, child is in daycare
5 days per week, the child does not use a pacifier or take
a bottle
ROS: Drooling for 3 hours, febrile for 4 hours, no previous epi-
sodes of croup or stridor, no barky cough, no history of
. frequent emesis, diwrhea or constipaTion. No history of
wheezing, chronic cough, asthma or pneumonia. No his-
tory of heart disease

What would you l.ook for on PE?

PE:

1!11.a.Ls.: Temp: 39.2 C, HR: 140, RR: 60 and shallow, 02 sat. 96% on RA
GA: Child in moderate distress, drooling sitting upright, leaning
forward, rapid shallow breathing, flushed cheeks, skin damp,
tracheal tugging
Ears: Cerumen impactions bilaterally
OCIOP: Drooling. (Avoid aggressive attempts to view OP with tongue
depressor in a child in distress.)
Chest: Clear with decreased breath sounds bilaterally, no rhonchi or
wheezing
Extremities: Well-pe1jused, pink, warm, brisk capillary refill

145
Otolaryngology Clinical Case Studies Chapter 3- General Otolaryngology

N OTES What is your differential diagnosis?

• Epiglottitis
• Retropha1y ngeal abscess
• Tracheitis
• Laryngotracheobronchitis
• Airway foreign body

Given the child~· ·rapidly progressing airway obstruction with other find-
ings of acute febrile illness, epiglottitis is most likely. The presence of stri-
dor early on suggests this as well, with the loss of stridor being an omi-
nous sign. Retropharyngeal abscess would likely take longer to develop
and usually has some preceding illness. A foreign body is possible, but less
likely in the absence of a choking episode. Howeve1~ this must be consid-
ered due to the acute nature of the presentation. Also, this child has an
olde1~ possibly developmentally delayed sibling who could have given the
child a foreign body. · Tracheitis or diphtheria are uncommon and usually
involve a more protracted course and usually have .associated stridor.
Laryngotracheobronchitis or croup is generally a less toxic disease pro-
cess and usually presents with a characteristic "barking" cough and· stri-
dor-often biph.asic.

What is your assessment of the patient .and patient's status?

Child is in sev~re respiratory distress. Suspicion for epiglottitis is very high


given clinical scenario outlined above. It important to note that the stridor
present prior to arrival at the ED is now gone. This suggests rapid progres-
sion of airway obstruction and poor ventilation. Despite the essentially nor-
mal oxygen saturation, the child is likely not ventilating well. If a venous
blood gas could be measured, the carbon dioxide would probably be elevated.

What diagnostic tests would you request?

• In this case, none


• In Jess emergent situations a lateral neck x-ray can be used to examine the
appearance of the epiglottis. See below

What are the treatment options? What would you do and why?

• Secure the airway. This child is in danger of experiencing a full respiratory


arrest
• Keeping the child calm is very important
• It is acceptable to attemptorotracheal intubation. However, intubation should
be performed by the most experienced laryngoscopist in the room. This is
usuaily the attending Otolaryngologist or Anesthesiologist. A surgeon should
be prepared to perform an emergency tracheostomy
Ideally, the patient should be transported to the OR suite where a trache-
otomy can be performed most safely if necessary
Attempting intravenous access should be avoided. To that end, keep the
chi:d in the parent or caregiver's arms as long as possible during transport

146
Chapter 3 - General Otolaryngology Otolaryngology Clinical Case Studies

If the child was not in extremis and more stable, what tests might be ordered? NOTES

• Airway films: best perfom1ed with higb-KV technique


- Lateral neck x-ray: This might demonstrate thickened epiglottis (..thumb
print") with air in esophagus (epiglottitis), or widening of retropharyngeal
soft tissues and possible fl uid collection and/or air-fluid levels, or a radio-
opaq ue foreign body at cricopharyngeus or laryngeal inlet
- An A-P film might show subglottic narrowing in the form of a '·steeple
sign" suggesting Jaryngotracheobronchitis or croup
• CXR (portable): Determine any lower airway cause of respiratory distress
and stridor (Radio-opaque foreign body, pneumothorax, pneumonia, air-trap-
ping, atelectasis)
- WBC: Usually elevated in acute epiglottitis with bandemia
- Blood culture·: Often will demonstrate causative organism of airway infec-
tion. Most likely H influenza Type B (HIE), or S. pneumococcus (Cul-
ture of larynx at time oflaryngoscopy will often show same organism)

Outline your plan for obtaining an airway:

• In the OR, with child sitting in his most comfortable position, perform a Mask
induction with a minimally irritating inhalational anesthetic, with all instru-
ments for laryngoscopy or tracheostomy at the ready
- Try to keep the patient spontaneously breathing using the mask to assist
only
• Once asleep, direct laryngoscopy -can be attempted carefully and efficiently
- If it is. possible without too much resistance, pass a small endotracheal
tube (3 .5 or 4.0)
- If a significant amount of resistance is encountered, ~o not make mul-
tiple attempts at intubation, but prepare quickly for tracheostomy
• If possible, continue mask ventilation assistance with 100% oxygen
. -
You have now secured the child's· airway with a 3.5 endotracheal tube. There is
a leak around the tube at 40 em of water pressure. What should you do next?

• A culture of the larynx should be obtained as well as blood cultures after


obtaining IV access and administering an IV fluid bolus ( 10 cc/kg of LR or
0.9 normal saline)
• Once the patient is stabilized, further management of the airway should be
considered

Some advocate immediate tracheostomy to avoid circumferential inj ury to sub-


glottic larynx, which may place the child at risk for subglottic stenosis. This is
one method of managing the patient. Things to consider before performing tra-
cheostomy: availability of pediatric ICU for post-op care, degree of inflamma-
tion and probability of prolonged intubation

It is reasonable to continue endotracheal intubation while medical therapy is


instituted and await response. It is imperative that adequately trained pediatric
intensive care personnel are available for post-op care. If they are not, arrange
for transport to a pediatric tertiary care center. The child must remain deeply
sedated or paralyzed to avoid inadvertent extubation. This holds true for a patient
with a new tracheostomy as well

147
Otolaryngology Clinical Case Studies Chapter 3- General Otolaryngology

NOTES What medical therapy would you 1·ecommend? State that:

• As epiglottitis is usually a manifestation ofHIB, antibiotic choices should be


mad e accordingly. A second generation cephalosporin such as Cefuroxime
(30-50 mg/kg divided tid) is the drug of choice
- In a penicillin allergic patient consider gentamicin or proceed with cepha-
losporin
- In order to minimize inflammation of the larynx an H2-blocker such as
ranitidine or a PPI can be given empirical ly for extra-esophageal reflux
- The use of steroids is controversial. Once IV antibiotics have been ad-
ministered a brief course ofhigh-dose steroids is likely to decrease edema
and inflammation more rapidly (e.g., dexamethasone 0.5-1.0 mg/kg over
12-24 hours). It may reduce the amount of permanent damage to the
subglottis
It is unlikely that pul se-dose steroids will impair the immune system
significantly
- Once a leak develops around the endotracheal tube at 10- 15 em of
water pressure, consider weaning sedation, then extubation
• If the chilc,i fails extubation or never develops a leak around the endotracheal
tube, tracheostomy would be prudent

The child has been extubated and has been transferred to the floor and is
ready to be discharged. They have received their prescription for Cefiiroxime
and a rejerra.f to the health department for vaccinations.

How would you follow this individual? State that you would:

• As long as he is no longer experiencing stridor, discharge to home is reasonable


• One might also give him a prescription for H2 blockers such as Ranifidine or
a PPI such as Lanzoprazol to prevent ongoing inflammation to the larynx
from laryngopharyngeal reflux
• He should be seen in the office in the next few weeks to months
• It might be advisable to evaluate his airway in the near future with broncho-
scopy tQ rule out any laryngeal or subglottic stenosis. Any child with residual
stridor of any kind, hoarseness/voice changes or dyspnea or retractions on
exertion shou ld have the airway evaluated. Since these may not be immedi-
ately apparent, a post-op follow-up appointment or instructions to observe
for these symptoms should be given the parents

In the past 10 years, with widespread use of the HIB vaccine, the incidence of
acute epiglottitis has declined precipitously. Many health care providers have
little experience with this rapidly progressive illness. The key to prevention of
death and other serious sequelae is early recognition ofthe diagnosis and rapid
mobilization of the appropriate services, such as anesthesia and otolaryngology.

With recent controversy about childhood vaccines and autism, more parents are
electing to abstain from the vaccination of their children. This may lead to an
increase in the occurrence of previously dwindling HIB related diseases such
as epiglottitis and meningitis.

148
Chapter 3 - General Otolaryngology Otolaryngology Clinical Case Studies

CASE 7 - OBSTRUCTIVE SLEEP APNEA NoTES


John M. Truelson, MD

A 66 year-old white male presents with a history of previously treated pha-


ryngeal cancer. He complains of malaise and daytime sleepiness. He says
he is depressed.

HPI: state that you would:

• Obtain detailed medical history, beginning with history of present illness

What additional historical information would you seek? State that you would ask:

• What was his previous cancer cell type, staging and treatment?
• How does he obtain nutrition currently?
• Has he experienced recent weight loss?
• Ask him to describe his malaise?
Has he noted any difficulty breathing?

His tumor was a T3 N2a MO SCCA of the left OP, treated 6 years ago with
oropharyngeal resection, which included the lateral wall, part of the pos-
zerior wall, and a neck dissection. His mandible was not resected and the
surgical deject was reconstructed with a pectoralis major myocutaneous
flap. He underwent postoperative radiation. He has been dependent on
PEG feedings since his initial treatment and has Lost 10 pounds over the
last yem: He denies any airway obstruction, and is generally tired all the
time, and says he is being treated for depression. He snores, but feels like
he sleeps adequately.

PMH: HTN, idiopathic polycythemia - treated with peri-


odic phlebotomy
Surgeries: Tonsillectomy, RJH, oropharyngeal surgery - per
HPJ
Allergies: NKDA
Medications: HTN medications, depression meds, NSAIDS for
back pain
FH: NC
SH: Retired, cigarettes 1 ppd x 40 years, quit 1 year
ago. ETOH: one 6 pack/day, quit 6 years ago
ROS: DJD knees and lower back

What would you look for on PE? State that you "wo1,1ld:

• Perform a complete head and neck PE, including vital signs with weight and
height, and a fiber-optic exam

149
Otolaryngology Clinical Case Studies Chapter 3- General Otolaryngology

NOTES PE:

Temp: 99.0 F, BP: 150192. HR: 85, Wt: 230 lbs..


Ht:5 'll"
GA: WDWN NAD, no stridor
Nasal: Normal externallv
Speculum exam: Deviated septum. hypertrophied turbinates
Oral: Adequate dentition, molars removed for radiation.
OP: Lateral wall replaced by p ectoralis flap, no tumor
by inspection, palpation. bimanual exam. Airway
narrowed
Indirect exam: Appears clem; but difficult to see any structures
adequately because of flap and gag
Neck: Lymphedema on the left side and submencal, 120"
mass palpable
Fiber-optic exam: Nasal vault, NP clear. Base of tongue, OP clear of
twn01; unable to see pyriform· well on the left side
because of the flap, but no tumor seen with pha-
ryngeal blowout exam. Larynx appears normal
structurally except for typical post radiation edema,
and vocal cords have normal motion bilaterally

What is your differential diagnosis?

• Differential should include: recurrent tumor, esophageal/pharyngeal stricture


new primary tumor of the esophagus, sleep apnea

Wh;it diagnostics would you request? State that you would request:

• CBC and electrolytes, thyroid function tests


• CT scan
• Cine esophagran1
• Endoscopy
• Sleep study

Test results:

• Electrolytes and CBC: Hct (hematocrit) count 51, TSH, T4 normal


• CT scan head, neck and chest: only distorted anatomy in the left neck
and pharynx, 3 small (< 2 mm) noncalcified opacities in the lungs which
are too small to characterize only, and esophagus is mildly edematous
but M lesions seen
• Endoscopy, including esophagoscopy: negative
• Cine esophagram to evaluate swallowing - poor pharyngeal phase with
mild aspiration
• Polysomnogram - total RDI = 51, lowest oxygen saturation (L02) =
72, PLM = 12

150
Chapter 3 -General Otolaryngology Otolaryngology Clinical Case Studies

Diagnosis: NOTES

The diagnosis is sleep apnea. The paciem has typical history and PE: day-
time sleepiness, depression, high BP, polycythemia (polycythemia is an
unusual finding, but is a real complication of OSAS). He feels like he
sleeps adequately, but is still tired: this is a typical history f or sleep apnea.
Work-up for recurrent tumor is negarive. More importantly. this patient is 6
years out from treatment, so that recurrence is unlikely. He is, of course, at
4risk for a second primary.

What are the treatment options? What would you do and why?

• CPAP titration, reserving any surgical management for CPAP failure


• Consider nasal septal/turbinates surgery
• Management of high BP and polycythemia by internal medicine, although
polycythemia will likely respond to CPAP
• Surgical options include nasal surgery, oral appliances, UPPP or jaw
advancement

Surgical techniques :

Highlights and pitfalls:


1. Nasal surgery:
a. Only use if nasal airway obstr uction is a significant issue physiologically
b. Presence of a deviated septum or hypertrophied turbinates is not suffi-
cient reason for operation, in absence oflimited airflow.
c. Mouth breathing is not sufficient reason for operation. Many patients
mouth breathe habitually and continue to breathe through their mouths
even after nasal surgery has opened the nasal airway sufficiently
2. Jaw thrust device/oral appliance:
a. Often overlooked, but quite effective with mild to moderate OSAS and
well tolerated
b. May rarely reshape the bone so that malocclusion is pem1anent and
requires oral surgery to resolve
3. UPPP:
a. UPPP resolves snoring in a high percentage of patients, but resolves
OSAS only in 50%. Selected patient groups may have higher success
rate
b. Be aware that snoring cessation is not necessarily an indication that
sleep apnea has resolved
4. Advancement surgery:
a. These procedures are reserved for patients fai ting CPAP and UPPP.
Jaw devices may give some pre-surgical indication of success since the
end result is the same: advancement of the tongue base. Tongue ad-
vancement by suture and genioglossus advancement, with or without
GBAT device, all move the tongue forward. Mandibular fracture and
infection of hardware may occur in bony procedures, GBAT especially.
Suture tongue advancement may be done alone, or in addition to, a ge-
nioglossus advancement procedure. Swelling may occur with suture

151
Otolaryngology Clinical Case Studies Chapter 3 - General Otolaryngology

NOTES tongue advancement. Airway problems are infrequent. but precautions


should be taken with close observation
b. Bimaxillary advancement has generally been reserved for use after all
of the above treatments have failed because the procedure has a longer
recovery period than other sleep procedures and may result in malocclu-
sion and malunion or nonunion of osteotomies and malocclusion. How-
ever, it has a high success rate and there is no absolute contraindication
to using it at any point in the treatment protocol, except possible higher
infection rate due to prior radiation

How would you follow this patient?

I. ff CPAP or oral appliances are effective, return for yearly exam if asymp-
tomatic. Follow the patient with possible repeat polysomnogram every few
years. Despite rep011s of good sleep, patients continue to have meaningful
sleep apnea
2. Surgery: repeat polysomnogram 6 and 12 months post treatment. Remember
that Jack of snoring and improved sleep do not necessarily mean the sleep
apnea is resolved. OSAS may be improved, but the patient may need further
treatment

References:

I. Riley RW, Powell NB, Li KK, Troell RJ , Guilleminault C. Surgery and ob-
structive sleep apnea: long-term clinical outcomes. Otolaryngol Head Neck
Surg. 2000; 122(3):415-21.
·2. Ferguson KA. Ono T. Lowe AA. Keenan SP. Fleetham JA. A randomized
crossover study of an oral appliance vs nasal-continuous pos itive airway
pressure in the treatment of mild-moderate obstructive sleep apnea. Chest.
1996; 109(5): 1269-75 .
3. Guilleminault C. Clinical features and evaluation of obstructive sleep apnea.
In: Kryger MH, Roth T, Dement WC, eds. Principles and practice of sleep
medicine. 2nd ed. Philadelphia: Saunders; 1994.
4. . Fairbanks DN. Snoring: surgical vs . .!fonsurgical management. Laryngoscope.
1984;94(9): 1188-92.

152
Chapter 3 - General Otolaryngology Otolaryngology Clinical Case Studies

CASE 8 - MANDIBULAR FRACTURE NoTES


Jesse E. Smith, MD

A 32 year-old male presents to the ER after an assault.

HPJ: state that you would:

• Obtain a detailed medical history beginning with a history of present illness


..
He reports that the incident happened only a jew hours ago. Since that
time he has had severe pain in the area of the }aYI' and has had difficulty
opening and closing his mourh. He also thinks that one of his teeth is
loose. He spit up some blood from his mouth immediate~v after the incident.
but has had little since that time.

PMH: None
Allergies: NKDA
Medications: None
FH: NC
SH: ETOH: 72 oz of beer per day
Tobacco: 1 ppd cigarettes
No JVDA
No history of blood product transfusions
ROS: As above

What additional historical information would you seek? State that you would:

o t..sk if the patient has had braces or other maxillofacial procedures in the past
o Ask if he feels as if his teeth are no longer aligned
o Ask the patient if he has had any history of rheumatic fever or a heart
murmur

The patient states that he has never had braces or any other procedures and
has not been to the dentist in several years. He does feel that his bite is no
longer aligned. He denies any knowledge of murmurs or rheurnatic jeve1:

State that you would:

o Perform an appropriate head and neck PE including vital signs

PE:

J!i1aJ.s.: Temp: 37.2 C, BP: 145/86, Pulse: 96


GA: WD, WN, thin, WM, NAD
HEENT EOMI PERRLA. Both TM & EAC clear

There is no lymphadenopathy noted. The trachea is midline and there is no


thyromegaly. There are 2+ carotid pulses bilaterally. Anterior rhinoscopy
appears clear: CN II-XII appear intact except for anesthesia over~ying the

153
Otolaryngology Clinical Case Studies Chapter 3 - General Otolaryngology

NoTEs distal V3 (mental nerve) distribution. There is some swel!ing and ecchymo-
sis overlying the righr side of his chin
lmraoral exam shows a mobile !!26 and an obvious intraoral laceration in
the same area. There are some cavitations, bur most of rhe other teeth are
in good condition. His bite demonstrates malocclusion with deviation to-
ward rhe left side
The patient has no neck pain with full range of motion of his neck

What is your differential diagnosis?

• Mandib le fracture
• Dentoalveolar fracture
• Dislocated TMJ

What diagnostics would you request? State that you would request:

• Orthopantornogram .(Panorex) A left mildly-displaced subcondylar fracture


with the head of the condyle appearing to be within the joint capsule, and a
right body fracture.

Figure 1

Figure2

154
Chapter 3 - General Otolaryngology Otolaryngology Clinical Case Studies

Diagnosis: NOTES

The diagnosis is a right body fracture through the woth roar of #26 (Fig-
ure 1) and o left subcondylar fracture (Figure 2). This patient will require
an operation for the body fracture. but he does not have an absolute indi-
cation for operative management of the right condylar fracture (there is
minimal displacement, no extracapsular di!>placement, and there are no
foreign bodies). There is a good likelihood that adequate occlusion can be
restored J.l'ith the use of guiding elastics.4 Others would argue that the
subcondylar fracture is best repaired with an open reduction and int,ernal
fixation. This could be accomplished through an external approach or an
endoscopic internal procedure. As jar as the mobile tooth is concerned, it
may be left in position if the periodontal bed is intact and healthy and the
alveolar bone is stable after rigid fixation. The tooth can be supported by
others within the arch bar matrix. These attachments will need to be main-
tained for 2--1 weeks.

What are the treatment options and their complications? What would you do
and why?

• Clindamycin 600 mg NPB on call to the OR for the open fracture


• Ensure that tooth #26 is not mobile enough to become loosened into the
airway. The tooth is slightly mobile, but stable, in its position and does not
currently pose a risk to the airway ·
• Intraoral incision with rigid internal fixation along Charnpy's ideal lines of
osteosynthesis. This is probably the best choice for this patient given the
fracture's location which is somewhat mesial
• Submental skin-fold incision (extraoral) with rigid intemal fixation along
Champy's ideal lines of osteosynthesis. One must be aware of the marginal
mandibular branch ofthe facial nerve with this approach
Modified Risdon incision with rigid internal fixation along Champy's ideal
lines of osteosynthesis

Surgical techniques:

Highlights and pitfalls:


• Consent the patient for the appropriate operation
• Mark the surgical site
• Begin Clindamycin or other appropriate perioperative antibiotics
• Consult anesthesia and use nasotracheal intubation
• Apply arch bars and place the patient into his proper prefracture occlusion.
This can be done by identifying the wear facets on teeth. The loose tooth
should be carefully supported by surrounding teeth and the fracture can be
reduced during the application of the arch bars
• Care must be taken to identify and preserve the mental nerve during expo-
sure and internal rigid fixation of the fracture
• The fracture must be totally reduced before application of the internal fixa-
tion . To prevent mandibular splay, the fracture must be corrected with the
proper width maintained at the angles, and the inferior border plate should be

155
Otolaryngology Clinical Case Studies Chapter 3 - General Otolaryngology

NoTEs slightly over bent. A locking 2.0 mm plating system with bicorticaJ screws and
a monocortical tension band is recommended (See Figure 3), although 2
monocortical miniplates with a tension band, a locking reconstruction plate,
or a 2.0 mm mandibular plate and a tension band can also be employed
• Close the incision in a multilayer fashion after extensive irrigation
• Remove the IMF prior to extubation
• Place guiding elastics on the arch bars once the patient is awake
• Place a jaw support to ensure that the mentalis heals correctly and that chin
and lower lip ptosis do not occur
• Obtain ~post reduction ortbopantomogram (Panorex)
• Maintain a soft diet and arrange frequent follow-up and elastics training
• Watch for signs and symptoms of alcohol withdrawal and encourage the
patient to stop using tobacco products
• Antibiotics may be maintained by an oral route after surgery for 7-1 0 days.
If a mucosal seal around the bone is not present by this time, antibiotics may
need to be continued for a longer period

Figure 3- Proper ORJF of Right Body Fracture

What are the potential complications?

• Malocclusion: most individuals can determine alterations in occlusion less


than 1 mm
• Nonunion, fibrous union, or malunion. Usually a result of infection or im-
proper plate placement
• Infected hardware
• Extrusion of plates and screws
• Scarring
• Neurologic damage
• Loss of dentition
• All transfacial approaches run the risk of injuring the VUthCN

156
Chapter 3 - General Otolaryngology Otolaryngology Clinical Case Studies

Ho'w would you follow this individual? Include immediate post-ope1·ative care NOTES
and long term follow-up:

• Patients can be watched overnight in the hospital or sent home from the
PACU . Patients should be seen within the first 72 hours after the operation
to ensure that:
- The patient has a mucosal seal
- The patient understands all post-operative instructions
The patient can reapply elastics in the correct locations
- The patient has no infection
• During the first 72 hours, watch for signs and symptoms of alcohol with-
drawal and encourage the patient to stop using tobacco products if these are
concerns
• Maintain the patient on a soft diet for 6 weeks
• For the following six weeks after the operation, arrange weekly follow-up.
During these sessions, teach elastics training and ensure that the patient is
not developing trismus or an infection
• Antibiotics may be maintained by an oral route after surgery for 7-1 0 days.
If a mucosal seal around the bone is not present by this time, antibiotics may
need to be continued for a longer period
• If the patient has healed satisfactorily, arch bars may be removed after 6
weeks, or sooner in many cases if elastic training has concluded. Arch bars
should be maintained for 6 weeks if they are acting as the superior tension
band for the ORIF. Arch bar removal can be accomplished in the office
under local anesthesia. However, some patients may need to be taken to the
OR for this procedure
• Patients should be evaluated one more time at 12 weeks post reduction to
ensure that their occlusion is normal and their surgical wounds have healed.
Another post reduction orthopantomogram (Panorex) is optional at this visit

References:

1. Lynham AJ, Hirst JP, Cosson JA, Chapman PJ, McEniery P. Emergency
. department management of maxillofacial trauma. Emerg Med Australas.
2004; 16(1 ): 7- 12.
2. Kempers KG, Quinn PD, Silverstein K. Surgical approaches to mandibular
condylar fractures: a review. J Craniomaxillofac Trauma. 1999;5(4):25- 30.
3. Fonseca RJ. ed. Oral and maxillofacial surgery. Mandible Fractures. lst ed.
Philadelphia: Saunders; 2000.

157
Otolaryngology Clinical Case Studies Chapter 3- General Otolaryngology

NoTES CASE 9 - MAXILLARY FRACTURE


Jesse E. Smith, MD

A 40 year-old female restrained driver presents to the ER after a moder-


ate speed. motor vehicle collision in which she reports a brief loss of
consciousness.

HPI: state that you would:

• Obtain a detailed medical hi story beginning with a history of present illness

She reports that the incident happened only an hour ago. She feels that
her face is swollen and painful esp eciaiZv when she moves. She says that
something feels loose when she tries to close her mouth.

PMH: None
Allergies: Penicillin ·
Medications: Occasional acetaminophen for headaches
FH: NC
SH: ETOH: 1-2 glasses of wine per week
Tobacco: None in 5 years
No IVDA
No histo1y of blood product transjitsions
: ROS: As above

What would you look for on PE? State that you would:

• Perform a complete head and neck PE including vital signs

PE:

Temp: 37.3 C, BP: 150/74, HR: 98, Room air saturation is


97%
· GA: WDWN, WF, NAD
Alert and oriented to person, place, date and event
HEENT There is diffuse swelling and pain of the midface but no
orbital step-offs are noted. The midface and palate are
somewhat mobile. The eyes are quite swollen yet with gentle
lid retraction the extraocular movements appear to be in-
tact. PERRLA, there is subconjunctival hemorrhage on the
right. There is no telecanthus. She is unsure about her
vision as she usually wears glasses and her eyes are swol-
len, but they appear to be grossly intact and she has good
light perception. Both TM's & EAC 's clear.
There is no neck lymphadenopathy noted. The trachea is
midline and there is no thyromegaly There are 2 + carotid
p ulses bilaterally
Anterior rhinoscopy shows a swollen dorsal swface of the
nose and some serosanguineous oozing from the nasa! pas-
sages. CN ///-XII appear intact

158
Chapter 3 - General Otolaryngology Otolaryngology Clinical Case Studies

Intraoral exam shows a mobile hard palate with no inTraoral NOTES


lacerations. There are some old dental caries, bur there
appears to be no new tooth damage. The jaw is nor pain-
fit! and there is no trismus. The patient has a cervical col-
lar in place but complains of no neck pain
The rest of the PE is unremarkable including the ches1,
abdomen and extremities

What is your differential diagnosis?

• Pure Lefort I, II, or III fracture


• Hemilef011 Lefort I, II, or III fracture
• Non-Lefort midface fracture (anterior maxi llary, dentoalveolar, palatal, me-
dial maxillary)
• A combinati~n of any ofthe above including orbital rim fractures and a nasal
fracture

What diagnostics would you request?

• Urine pregnancy test


• A head CT without contrast to evaluate for closed head trauma
• The C-spine should be clinically and radiographically evaluated as soon as
possible
• CT scan without contrast of the face with fine cuts in the axial and coronal
planes. The C-spine must be cleared prior to obtaining coronal films because
of the manner in which the patient must be positioned. The axial images can
be obtained at the same time as the head CT if desired

159
Otolaryngology Clinical Case Studies Chapter 3 - General Otolaryngology

NoTES Test results:

• Jmracranial/y negative. There are some fracture lines no1ed at the me-
dial orbital walls bilaterally and there is blood in the maxillary sinuses
and in the nasal cavities. There is no disruption of the rough outline of
the globe and there are no intra-canal hemorrhages
• The C-spine is cleared radiographically and by PE. The patient is not
inloxicated and the cervical collar is removed

CT of face: On the right a fracture line is seen crossing the frontal sinus,
the medial orbital rims, through .the maxillary sinuses and pterygoid plates.
The maxilla is minimally displaced. This is a hemi-Lefort II. There is no
naso-orbito-ethmoid fracture noted. The orbital floors are intact. On the
left, a fracture line is noted through the medial and lateral buttresses of the
maxilla below the level of the orbit. This is a hemi-Lefort I fracture.

Actions: ·
1. Ensure the airway is stable at all times
2. Ensure that no one places a nasogastric tube in this patient because of
the risk of anterior skull-base damage
3. Obtain an ophthalmology consult because of the risk of optic nerve
trauma. If the patient is unable to participate, a forced duction test can
be done to differentiate between edema and entrapment! The full exam
shows only diffuse periorbital edema. All EOMI

Diagnosis:

The diagnosis is a hemi-LeFort 11 fracture on the right and a hemi-LeFort


I fracture of the left maxilla. The patient will need internal rigid fixation
after reduction and realignment of the fracture. She is not having airway
issues at this time, but to accurately align the impacted fracture, this pa-
tient will need intermaxillary fixation. If no significant ethmoid or skull-
base damage is noted, then a nasotracheal tube can be carefully placed.

160
Chapter 3 - General Otolaryngology Otolaryngology Clinical Case Studies

Howeve1; if there are doubts abouT the ability to safely place This tube, then NoTES
a tracheotomy should be pe1jormed at the time of surge1y Massive f acial
swelling which accompanies facial fractures makes inirial repair quite dif
ficult. Often this edema can be adequa tely reduced within 7-10 days. At
this time another thorough head and ne ck exam should be done lo ensure
thai there is no facial anesthesia.

What are the treatment options and their complications? What would you do
and why?

• Sublabial approach to the lower midface and a transconjunctival (with or


without canthotomy) approach to the orbital rim. This approach is probably
the best suited for the fracture pattem on the right. lt is associated with less
incidence of ectropion and scleral show than other approaches
• Sublabial approach to the lower midface and a subciliary approach to the
orbital rim. This has a higher association with ectropion and can leave visible
scarring
• Sublab.ial approach to the lower midface and a bicoronal or extemal ethmoid
approach to the nasal area and orbital rim. This can be a morbid and unaes-
thetic incision

Highlights and pitfalls:


• Consent the patient for the appropriate operation
• Obtain preoperative photographs
• Mark the surgical site
• Consult anesthesia and discuss intubation techniques
• Apply arch bars and place the patient into her proper prefracture occlusion.
This can be done by identifying the wear facets on teeth
• Care must be taken to identify and preserve the infraorbital nerves during
exposure and internal rigid fixation of the fracture
• The fracture must be totally reduced before application of the internal fixation
• Plate the two lateral buttresses of the maxilla utilizing at least two screw~ on
either side of the fracture line (1.3 vs. 1.5 mm plates). Use smaller plates for
the inferior and medial orbital rims (1.0 vs. 1.3 nun plates) .
• Perform a forced duction ~est intraoperatively after rigid fixation
• Use an elevator to reposition the nasal bones if necessary to their correct
projection and position
• Close the incision in a multilayer fashion after extensive irrigation
• Remove the IMF prior to extubation
• Maintain a soft diet and arrange frequent follow-up

What are other potential complications?

• Malocclusion: Most individuals can determine alterations in occlusion less


than 1 mm
• Nonunion, fibrous union, or malunion. Usually a result of infection or im-
proper plate placement
• Infected hardware
• Extrusion of plates and screws
• Scarring

161
Otolaryngology Clinical Case Studies Chapter 3 - General Otolaryngology

NoTES • Extraocular muscle entrapment


• Neurologic damage
• Tooth root damage from screws
• Scleral show and ectropion

How would you follow this individual?

• The patient may be discharged once she can tolerate a soft mechanical diet
• She may be seen in one week to insure that the incisions are healing well.
Extrusion of plates and screws is usually preceded by infections and is usu-
ally the result of poorly placed hardware. Healing can be assessed by ensur-
ing that the incisions are closing properly. The midface should remain tight
and nonmobile. Occlusion should be checked to ensure that the midface has
not shifted
• Evaluate the right eye for fu ll closure and any evidence of ectropion or scleral
show. Nonabsorbable sutures should be removed if there are any in place. If
scleral show or ectropion become an issue, one can teach the patient mes-
sage techniques for the lower Iid which decreases scar tissue and relaxes
the lower lid tissues, returning the lid posi6on to norn1al and correcting the
deformity

References:

1. Ducic Y, Odland R, Hilger P: Fractures ofthe maxilla. Maxillofacial trauma


monograph. Maisel RH, ed. Alexandria: American Academy of Otolaryngol-
ogy- Head & Neck Surgery Foundation, Inc.; 2001.
2. Manson PN, Hoopes JE, Su CT. Structural pillars of the facial skeleton: an
approach to the management of Le Fort fractures. Plast Reconstr Surg.
1980;66:54.
3. Manson PN, Crawley WA, Yaremchuk MJ, et al. Midface fractures: advan-
tages of immediate extended open reduction and bone grafting. Plast Reconstr
Surg. 1985;76(1 ): 1-10.

162
Chapter 3 - General Otolaryngology Otolaryngology Clinical Case Studies

CASE 10- NASAL FRACTURE NOTES


Robert T. Adelson, MD

A 24 year-old white male is evaluazed in the ED after being srruck in the


face.

HPI: state that you would :


6

• Obtain a detailed medical history beginning with history of present illness

What additional historical information would you seek? State that you would ask:

• What was the mechanism of injury(ies)?


• When did the injury occur?
• What are the patient's symptoms?
• Did he experience loss of consciousness?
• Does he h~ve nasal airway obstruction?
• Has he noted any nasal bleeding or clear flu id coming from his nose?
• Has he had blurred vision, double vision or change of visual acuity?

The·patient reports being struck in the face with a closed fist approxi-
mately six hours prior to his presentation to the ED. He remembers the
entire altercation, continues to have pain only in the nasal area and has
been experiencing intermittent left-sided epistaxis as well as continuous
left-sided nasal airway obstruction. Though the patient ·denies vision
changes, diplopia, and altered facial sensation, his nasal airway obstruc-
tion precludes him from accurately describing his olfactory ability.

PMH: None
PSH: Tonsillectomy and adenoidectomy at age 5 years
Denies any other surgical procedures, specifically denies
any prior nasal surgery
Allergies: NKDA
Medications: Occasional acetaminophen for musculoskeletal complaints ·
SH: Occasional alcohol
No tobacco
Waiter
ROS: Patient denies any pre-existing nasal airway obstruction or
hyposmia. The remainder of his ENT ROS is entirely normal

What would you look for on PE? State that you would:

• Perfom1 a complete head and neck PE, including vital signs and anterior
rhinoscopy using a Fraser-tip suction and a nasal speculum

163
Otolaryngology Clinical Case Studies Chapter 3- General Otolaryngology

NoTES PE:

Viw ls: Temp: 98.0 F, BP: 125175, Pulse: 90. RR: 16


GA: WDWN WM with blood staining The from of his shirt.
AU = B;f/EAC normal, 110 MEE, No hemotympanum. No peTfora-
Tion. A full examination of the facial skeleton, including all pal-
pable buttresses, and bimanual examination of the zygomatic arch
is normal, with the excepTion of a markedly abnormal bony nasal
pyramid. The skin is intact and the ed ematous soft tissues of the
nasal dorsum are now beginning to show the first blush of ecchy-
mosis, which extends into the periorbital region of the left lower
eyelid. There is no subcutaneous facial emphysema. The bony nasal
pyramid is mobile to palpation. Sublabial palpa!ion of the anterior
nasal spine demonstrates 110 fracture in this location. Bimanual
examination of the maxilla is unremarkable
Nose: Vienna speculum and Fraser-tip suction are utilized to clean a large
amount of clot from each nasal cavity. No new epistaxis is ob-
served. The nasal mucosa is edematous and the patient is treated
with intranasal oxymetazoline to allow a speculum exam of the
nasal airway. After thorough decongestion, the caudal nasal sep-
tum appears to deviate to the patients left, while the posterior por-
tion of the septum deviates toward the patient :S right. There is no
bulging, blanching or other deformity of the septum that might
indicate the presence of a hematoma. No dystopia of globe posi-
tion. No chemosis or conjunctival hemorrhage. Visual acuity is
grossly normal. Extra-ocular motion is intact without diplopia
Oral: Normal, no hematoma, no ecchymosis, no trismus, class I occlu-
sion with contact maqe at wear face ts. Salivary glands are palpa-
bly normal. No cervical lymphadenopathy. CN II-XII intact. Anos-
mic upon testing with coffee grounds. No reaction to ammonia

What is your differential diagnosis?

·State that an abbreviated differential is appropriate when the etiology is ob-


vious. The exact fracture pattern may include any of the following:
- Nasal fracture
- Naso-orbito-ethmoid fracture
- LeFort (mid-face) fracture
Frontal sinus fracture

What diagnostics would you request?

Facial photographs- The diagnostic work-up for trauma victims seeks to docu-
ment and clarify the injury and the involved structures. PE identifies a nasal
injury consistent with experiencing an attack by a right-handed assailant. In the
absence of physical findings supportive of additional facial trauma, no diagnos-
tic imaging is required. Should the patient have evidence of concomitant injury,
non-contrast fine-cut CT-scan of the face (axial and coronal views) is the cor-
rect study. Plain film radiography rarely is indicated for facia l trauma, and never

164
Chapter 3 - General Otolaryngology Otolaryngology Clinical Case Studies

is indicated for an isolated nasal injury. Most authors recommend documenta- NOTES
tion of the injury with color photographs as well as obtaining recent, pre-injury
photographs to aid treatment decisions. This patient does not need radiology
studies; however, a standard set of facial photographs are recorded

What are the treatment options and their complications? What would you do
and why?

Bleeding should be controlled. lniti(!l conservative measures should be employed


(oxymetazoline, digital pressure, focal cautery), yet many nasal fractures have
associated mucosal disruption which necessitates intranasal packing. A careful
anterior pack may be placed, using topical and systemic antibiotics to cover,
prophylactically, Staphylococcus aureus as well as the typical sinus patho-
gens. As this patient's epistaxis has cea~ed, no intranasal packing is required.

When should this patient return for repeat evaluation? Why?

• The patient should be instructed to apply ice packs to the nose and follow-up
in 5-7 days for a repeat examination
- By 5 days after injury, most patients have sufficient resolution of nasal
edema to allow discussion of treatment options
- At this follow-up exam, the surgeon can better characterize the extent
of injury, specifically with regard to the presence or absence of a com-
minuted fracture, the location of each nasal bone, and the condition of
the septum

Diagnosis:

Closed nasal fracture, with traumatic nasal septal deviation

What is your next step?

• Given the external nasal deformity and new (and persistent) left-sided nasal
airway obstruction, a closed nasal reduction with possible septoplasty should
be offered to the patient

How should you counsel this patient with regard to treatment options?

• The patient must be counseled that the failure rate for closed nasal reduction
ranges from 1~0%, and fractures that cannot be adequately reduced through
a closed approach may require an open septorhinoplasty

If the fracture is not reduced within the first 3 hours after injury, then a closed
reduction should be undertaken between 5 and 14 days after sustaining a frac-
ture. In the pediatric patient, these fractures tend to heal faster, and therefore
closed nasal reduction must be undertaken within 5-7 days post-injury. Given
the potential for altering a pediatric patient's facial growth centers, conservative
operations are favored over those that involve aggressive osteotomies or carti-
lage resection.

165
Otolaryngology Clinical Case Studies Chapter 3- General Otolaryngology

NOTES Surgical techniques:

Highlights:
• The patient should be consented for a graduated, yet all-encompassing pro-
cedure to be carried out under general anesthesia
• The nasal bones are addressed first, using a Boies elevator or Walsham
forceps. lf the nasal bones and septum have not been returned to the pre-
injwy alignment, the septum must be addressed. Asch forceps can facilitate
manipulation of the nasal septum back into alignment
lnability to re-establish a straight septum requires septoplasty. Release of a
tethering deviated septum from the overlying bones allows a natural position
to be restored. Offending portions of the bony and cartilaginous septum can
be resected, provided that dorsal and caudal cartilage suppo1ts (between 0.8
and 1.25 em wide) are maintained. Many authors asse1t that a 'C'-shaped
septal fracture is a common finding in patients with nasal bones deviated
greater than one half the width ofthe nasal dorsum. Resection of the vertical
and horizontal portions of the fracture will release a tethering force that
otherwise prevents proper reduction
• At this point, failure to achieve a straight nose requires bilateral medial and
lateral osteotomies, fully releasing the bony pyramid and allowing the sur-
geon to determine its final location
• Persistent deformity after completing osteotomies is likely to be the result of
a pre-existing nasal deformity (which may be identified in old photos pro-
vided by the p~tient). The upper laterql cartilages can be released from the
septum to fm1her mobilize the nose
• Continued deformity at this point is an indication-for fracture reduction of the
anterior extension of the perpendicular plate of the ethmoid bone
• The last step in managing continued nasal deformities allows for conserva-
tive rasping of a dorsal hump and placement of camouflaging cartilage
• Bacitracin-coated intranasal splints and a moldabie external nasal cast are
applied when a satisfactory intraoperative result is achieved

Though not the case for this patient, the early correction of severe nasal frac-
tures often requires an open reduction. Failure to correct the bony injury and
persistent nasal deformity are indications to convert the procedure to an open
approach. Wiring or maniplating the bone fragments will help restore contour to
the nasal pyramid. On lay grafts of crushed septal ca1tilage or calvarial bone can
provide projection and establish contour for the nasal dorsum in patients with
severe injuries of the bony pyramid.

Pitfalls:
• Residual external nasal defonnity/nasal airway obstruction: The nasal dor- ~
sum remains mobile for 6-8 weeks after surgery. Patients can be taught to
apply appropriate pressure to effect subtle changes in the appearance of the
nasal dorsum. Injuries of the upper lateral cartilages, unaddressed during
closed reduction, may impede nasal airflow. Persistent deformity or nasal
airway obstruction 6 months after closed reduction should be addressed with
an open septorhinoplasty

166
Chapter 3- General Otolaryngology Otolaryngology Clinical Case Studies

• Septal hematoma: A submucoperichondrial collection of blood can occur as a NOTES


result of the in itial trauma or after nasal surgery. Loss of perichondrial blood
supply, combined with local pressure. can result in cartiIage necrosis as well
as an infection of the septal hematoma. Severe pain after septorhinoplasty
demands that intranasal splints be removed so that a septal hematoma can
be excluded. Prompt incision and drain age will prevent development of a
septal abscess or later "saddle nose" deform ity
• Epistaxis: Th is complication is similar to the septal hematoma in that it may
occur after trauma or subsequent nasal surgery. Conservative measures that
avoid applying significant pressure to the nose (oxymetazoline. icepack, fo-
cal cautery) are the first line of treatment, followed by careful internal pack-
ing with hemostatic material

How would you follow this individual?

• After surgery, the patient is maintained on systemic antibiotics to cover S.


aureus and sinus pathogens (second generation c·ephalosporin or quinolone)
until the splints are removed between post-op days Sand 7. The patient may
begin using nasal saline spray when nasal drainage ceases
• The patient should not participate in activities with a ri sk for nasal injury for
2 months after surgery ·
• Postoperative photographs should be obtained at l week, I month, and 6
months after surgery, to facilitate discussions about revision surgery _
12 months after the·initial operation, a revision open septorhinoplasty may be
offered to patients with persistent complaints regarding the appearance of
the nose and nasal airflow. Discussion should target specific aesthetic and
functional problems as well as the realistic chances for meeting a patient's
expectations

References:

(
1. Bailey BJ, Tan LKS. Fractures of the nasal and frontal sinuses. In: Bailey
BJ, Johnson JT, Healy GB, Pillsbury HC, Jackler RK, Tardy, ME, Calhoun ·
KH, eds. Head and neck surgery-otolaryngology. 3'd ed. Philadelphia:
Lippincott Wil1iams & Wilkins; 2001 , 794-804.
2. Hom DB. Acute nasal fractures. In: Maxillofacial trauma, Maisel, RH, ed.
Alexandria: American Academy of Otolaryngology-Head & Neck Surgery
Foundation, Inc.; 2001, 25-34
3. Staffe l JG. Nasal fracture. ln: Gates G., ed. Current therapy in otolaryngol-
ogy-head and neck surgery. St. Lou is: Mosby-Year Book; 1998, 133-4.
4. Staffel JG. Optimi zing treatment of nasal fractures. Laryngoscope.
2002;112:1709-19.

167
Otolaryngology Clinical Case Studies Chapter 3 - General Otolaryngology

NoTES CASE 11 - ORBITAL FLooR FRACTURE


Jesse E. Smith, MD

A 35 year-old female presems to the ER after a low speed. motor vehicle


collision.

HPI: state that you would:

"
• Obtain a detailed medical hist01y, beginning with a history of present illness

She was a restrained driver who stales that she did not lose consciousness
during or after the collision. Howeve1; she did hit her face and eye on the
steering wheel. She states that her right eye hurts. She wears contact lenses
in both eyes, and reports that her vision is only moderate without her glasses
or contacts.

PMH: None
Allergies: NKDA
Medications: Oral contraceptives
FH: None
SH: Social ETOH only, financial analyst
ROS: As above

What additional historical information would you seek? State that you would ask:

Does she presently have eye pain?


• Does she have double vision?
Is she currently wearing contacts or was she wearing glasses when the
event occurred?
• Has she ever had any ophthalmologic surgeries or procedures?

The patient states that she is having some mild aching eye pain. She denies
double vision in forvvard gaze but you see that the lid is edematous and
covers most of the pupil. She states that she was wearing contact lenses
when the incident occurred. She denies any type of ophthalmic procedure
in the past.

What would you look for on PE? State that you would:

• Perform a complete head and neck PE including vital signs and ophthalmo-
logic exam

168
Chapter 3 - General Otolaryngology Otolaryngology Clinical Case Studies

PE: NoTEs
J!i1.cJ.ls.: Temp: 37.0 C. BP: 145185. Pulse: 96. 0 :: Sar 98%
GA: WD, WN, WF, NAD
HEENT The right periorbital soft tissue has diffuse edema and sur-
rounding ecchymosis. There is a small step-off of the right
inferior orbital rim. No telecanthus or dorsal nasal de -
fects are noted. The lids and lashes appear normal except
for edema. The right eye appears slightly proptotic. Upon
retraction of the swollen lids, the patient can see light.. The
pupil is anisocoric, but her vision appears to be 20190 in
the right eye and 20/80 in the left eye. All extraocular move-
ments are gross~y intact except for limitation of downward
gaze on the right leading to a resultant slight double vi-
sion. There is sorne chemosis of the conjunctiva of the right
eye. Sensation appears grossly intact throughout the face.
There is no contact lens in place

What is your differential diagnosis?

• Orbital floor fracture - Can be referred to as "blow out" fractures if a large


area of the floor is involved and prolapsed inferiorly. "Trap door" fractures
can also exist, but are more common in younger patients with more cartilagi-
nous bony floors ·
• Medial and inferior orbital wall fracture- Because of the thin bone that
composes these two walls, these are common fractures involving the orbit
• Maxillary fracture - LeFort II and ill fractures involve the orb1t by definition
and should always be considered when assessing orbital trauma
• Zygomaticomaxillary fracture ·_ These fractures involve the orbit by defini-
tion and should always be a consideration when assessing orbital trauma
• Orbital Apex Syndrome- Ophthalmoplegia, ptosis, pupillary dilation, and an-
esthesia of the upper eyelid and forehead. The optic canal and SOF. are
located in the apex. Orbital trauma that involves the optic canal usually causes
an injury to the intracanalicular portion of the optic nerve with resultant loss
of vision. Patients with bone fragments impacted into the orbital apex can
present with Orbital Apex Syndrome

What diagnostics would you request? State that you would:

• Ensure that the C-spine is cleared both radiologically and by PE


• Obtain a urine pregnancy test
• Obtain a consult from ophthalmology
• Obtain a fine cut CT scan of the face without contrast of both the axia l and
coronal planes
• Measure the degree of proptosis and visual acuity

169
Otolaryngology Clinical Case Studies Chapter 3- General Otolaryngology

NOTES Test results:

• The CT scan shows on inferior orbiwl wall and rim fracture with marked
edema of the periorbizal tissues. The il?[erior orbital wall and rim defect
is approximately 1.5 x 1. 7 em and extrudes into the righr maxill01:V si-
nus. Some of the periorbital fat has followed the bone into the maxillary
sinus. No other fractures are noted
• The proptosis is a minimal 2 mm. The vision is 20/80 in the right eye and
the same in the Left. The retinal exain is.. normal. The inferior gaze is
slightly limited, but expected for this type of injury. The corneal exam is
normal
• Flexion & extension C-!>pine films are normal. Pregnancy test is negative

Diagnosis:

The diagnosis is a right inferior orbital floor and rim fracture with associ-
ated loss .of orbital volume and a significant floor defect (> 2 cm1) . State
that this patient will need a right open reduction and internal fixation of
the fracture in order to prevent enophthalmos, hypothalamus, diplopia,
and an orbital rim contour deformity

What are the treatment options and the complications? What would you do and
why?

• The transconjunctival approach with or without lateral canthotomy with


cantholysis. Some surgeons think this is a better approach associated with
less ectropion and scleral show
• The subciliary approach with or without lateral canthotomy with cantholysis
• The approach through any preexisting laceration
• The maxillary vestibular approach. This approach would require a wide ex-
posure with a large amount of retraction for exposure. This may place the
infraorbital nerve at risk

Surgicill techniques:

Highlights and pitfalls:


• The operation can either be perfom1ed within 24 hours or at 7-1 0 days post
injury to allow for remission of edema
• Obtain informed consent including the possible use of plates, screws, bone
grafts and alloplastic materials
• Mark the surgical site
• Start perioperative antibiotics
• Take care to not dry or damage the cornea
• Use a transconjunctival or a subciliary incision attempting to avoid a lateral
canthotomy unless lateral exposure is necessary
• Identify the orbital septum, inferior orbital rim, and inferior orbital floor and
dissect the periosteum off of these areas. Dissect posteriorly, laterally, and
medially, until a stable shelf of posterior, inferior orbital floor can be identified

170
Chapter 3 - General Otolaryngology Otolaryngology Clinical Case Studies

• Carefully sweep as much periorbital tissue and orbital fl oor out of the maxil- NOTES
lary sinus as possible and maintain its position with a malleable retractor
• Decide whether alloplastic materials, plates and screws, or bone grafts should
be used to support the defect
• Reapproximate the inferior orbital rim. Reduce and fixate the rim with absorb-
able or titanium plates, putting at least 2 screws into each fracture segment
• Perform a forced duction test
• Reapproximate the periosteum and soft tissues of the access incision
• Place an antimicrobial ointment in the eye
• Check the visual acuity after the operation

What are other potential complications?

• Ectropion - Eversion of the edge of the lower eyelid, especially common


after scarring and heavy retraction
• Cicatricial entropion - inversion of the lash line and lid margin against the
eyeball which can occur with scarring after a transconjunctival incision
Enophthalmos -Recession of the eyeball into the orbit
• Hypothalamus - Inferior displacement of the eyeball into the orbit
• Diplopia- Double vision
• Extrusion of plates and screws - Usually caused by poor placement a·nd
preceded by infection
• Inferior orbital rim contour defonnity - A step-off from improper fracture
alignment or misplaced fracture fragments
• Epiphora- Tearing caused from ·damage or scarring of the tear canalicular
system
Loss of vision
• Ophthalmoplegia- Paralysis of ocular muscles
• Ptosis - Drooping of the upper eyelid from paralysis

How would you follow this individual? State that you would:

• Consider obtaining a post-reduction coronal CT scan without contrast or


plain film radiographs
• Perfom1 a postoperative check to evaluate for severe eye pain and loss of
vision which may herald an orbital hematoma
• Continue cold packs to reduce edema
• Continue 3-5 days of antimicrobial ointment in the eye

Your patient presents 1 week later in your office with good eye closure, but
the sclera is seen below the irjs and the lashes and lower eyelid evert in
such a manner that the conjunctiva is exposed.

What is your next step?

• This is not uncommon after retraction on the lower lid. Most surgeons rec-
ommend massage, hygiene, and tearing products initially, until scarring and
healing are complete. Massage techniques can relax scar tissue and allow
the eyelid to return to its natural position

171
Otolaryngology Clinical Case Studies Chapter 3 - General Otolaryngology

NOTES • If scleral show and conjunctival exposure do not resolve within 8-12 months,
a tarsal strip, palatal graft, and/or a lower eyel id fu ll thickness skin graft
cou ld be attempted

Follow-up:

• The patient should be seen 24 hours after the OR, at 5-7 days post-op as an
outpatient and regu larly (every few months) thereafter until healing is complete

References:

I. Bumstine MA. Clinical recommendations for repair of orbital facial frac-


tures. Curr Opin Ophthalmol. 2003; 14( 15):236-40.
2. Anderson PJ, Poole MD. Orbital floor fractures in young chi ldren. J
Craniomaxillofac Surg. 1995;23(3): 151-4.

172
Chapter 3 - General Otolaryngology Otolaryngology Clinical Case Studies

CASE 12 - PENETRATING NECK TRAUMA NOTES


Michelle Marcincuk, MD

A 2j year-old man is brought into the ER with a stab wound to the right
side of the neck.

HPI: state that you would:

• Obtain a detailed medical history beginning with a history of the present


illness

What additional historical information would you seek? State that you would ask:

• What were the circumstances of the injury?


• Was the patient drinking alcohol or taking drugs?
• Was he unconscious?
• Is he complaining of dizziness, weakness, numbness, double vision, trouble
swallowing or breathing, or hoarseness?
• Has he noted any bleeding from the nose, mouth or ear?

The patient reports that he has none of the above. He was just going to the
grocery store, when he was assaulted and stabbed in the neck.

PMH: Healthy accountant previously in good health Exercises regu-


larly. Takes no medications, no aspirin
FH: Negative
SH: Rarely drinks alcohol, and does not use drugs. Nonsmoke1:
ROS: Negative

What would you look for on PE? State that you would:

• Perfonn a complete head and neck exam including a neurologic exam

PE:

Normal pulse, BP and temperature.


Respirations 30/min.
0 2 saturation: 95% on 3L of oxygen
GA: 25 year-old man in NAD, responsive and coopera-
tive. No stridor or sterto1: His voice appears normal
HEENT Ears, nose and mouth: normal
Throat: Flexible fiberoptic laryngoscopy reveals.a normal
larynx and vocal cord function with small amount
of blood pooled in the pyriform sinuses
Neck: Stab wound 2 em in width located 1 fingerbreadth
below the cricoid, just to the right of the midline.
Subcutaneous emphysema is present as well as a
hematoma 3 em in diameter on the right lower neck.
Slight ooze of red blood from the wound
Neurologic: Normal

173
Otolaryngology Clinical Case Studies Chapter 3 - General Otolaryngology

NOTES What is your differential diagnosis?

• Injuries associated with a stab wound to zone I oftbe neck include possible
penetration ofthe esophagus or trachea, and possible injury to the right lCA,
internal jugular vein, or other major blood vessels

What diagnostics would you request? State that:

This patient is hemodynamical ly stable, but symptomatic from his wound (a l-


though the hematoma is nonexpanding, he has subcutaneous emphysema).
Therefore, a CXR, aJteriogram, and Gastrografin swallow study are mandatory
for this zone I injwy. State that you would request:
• CXR
• Carotid arteriogram
• Gastrografin swallow

The following studies are optional depending on the suspicion of the examiner
for other associated injuries, or to help plan treatment interventions:
• CT scan
. us
• Hemoglobin

Test results:

Pneumomediastinum is noted on CXR (See Figure 1). Otherwise, it shows


no widened mediastinum, apical capping, pleural effusion, deviated tra-
chea, or pneumothorax. The aortic knob is not obscured.

The arteriogram shows an injury to the right common carotid artery. The
aorta, brachiocephalic, vertebral and subclavian arteries are normal.
A Gastrografin swallow shows extravasation of contrast from the esopha-
gus (See Figure 2) .

The CT scan reveals subcutaneous air dissecting _in the tissue planes of the
right neck and a 3 em hematoma lateral to the right common carotid ar-
tery. No evidence of laryngeal or tracheal infwy is evident.
US: normal duplex Doppler studies
Hemogloblin: 13.5 grams

Diagnosis:

The diagnosis is penetrating stab wound to the zone 1 of the right neck
with injury to the esophagus and right common -carotid artery.

What are the treatment options and their complications? What would you do
and why?

• The vascular injury demands that the neck be explored surgically and the
injuries repaired

174
Chapter 3 - General Otolaryngology Otolaryngology Clinical Case Studies

What is the surgical route for the neck exploration? NOTES

• A direct approach through a horizontal incision over the wound is recommended

Sm·gical techniques:

Highlights and pitfalls:


• 2 units ofbloodare typed and cross matched and made available
• Endotracheal intubation is performed in the OR with a tracheostomy set
available
• A horizontal incision centered on the wound is made and the flaps elevated
above and below
• The JCA and internal jugular vein are approached above and below the site
of injury for control. Repair depends on the pa1ticular type of injury, but may
include primary repair, repair with a vein or PTFE (Gore-Tex) patch graft, or
repair with a bypass graft if the artery is otherwise irreparable
• The esophagus is repaired and the adjacent trachea explored. A nasogastric
tube is placed
The wound is drained and closed

What are the potential complications?

• Unrecognized injury to vessels and nerves with postoperative bleeding or


neurological deficits. For example, the vagus nerve, recurrent laryngeal nerve,
accessory nerve, sympathetic chain, brachial plexus, inferior thyroid artery
or vein, transverse cervical mtery or vein, vertebral artery, thyrocervical
trunk, suprascapular artery or vein, and anterior or external jugular veins are
all at risk of injury
• Wound infection

How would you follow this individual'!

• Broad spectrum antibiotics-to cover skin and aerodigestive tract flora should
be given prior to surgery and continued postoperatively for 5 days. For ex-
ample: clindamycin, ampicillin/sulbactam, or cefuroxime plus metronidazole
• Drain and NG tube removal at the discretion of the surgeon. Nothing to eat
or drink by mouth for 5 days
• Intensive care for 1 or 2 days to monitor neurologic status

Outcome:

S~xhours after surgery, the patient is found to have left-sided arm and leg "
weakness.

What is your next step?

• PE: The patient is hemodynamically stable and otherwise unchanged, with


no evidence of recurrence of neck hematoma. The Jackson Pratt drains
appear to be functioning and unclogged

175
Otolaryngology Clinical Case Studies Chapter 3- General Otolaryngology

NOTES • Imaging: options include an MRl of the head to look for evidence of a stroke
versus a repeat carotid angiogram to evaluate for thrombus of right carotid
at the site ofthe vascu lar repair

Test results:

• An MRI of the head with contrast is obtained, showing evolution of a


cerebral infarction of the right hemisphere
• Carotid arteriogram: Complete obstruction of flow at the site of surgi-
cal repair

What are your treatment options?

• Emergently retum to the OR to explore the right neck and site of carotid
repair. Obtain proximal and distal control of the vessel, open the arterial
repair, remove the thrombus, and repair the site. Consider using a shunt to
bypass arterial flow around the site of repair
• Initiate anticoagulation protocol .in the ICU, monitoring for resolution versus
progression of neurologic deficits, as well as for complications of bleeding
from anticoagulation given the patient's postoperative and post-traumatic state

Due to the early postoperative nature of the complication, reexpoloration


of the carotid artery is done. A clot at the site of repair is found and re-
moved. The artery is repaired and the patient is returned to the ICU The ·.
neurologic deficits resolve and no further complications are encountered.

Figure 1- CXR showing pneumomediastinum

176
Chapter 3 -General Otolaryngology Otolaryngology Clinical Case Studies

Figure 2- Gastrografin swallow study showing NOTES


extravasation of contrast from the esophagus

Follow-up:

Discharge once neurologically stable. Neurology consult advised


• Follow 7-1 0 days post-op for suture removal
• Arrange for appropriate rehabilitation for any post operative neurological
deficits
• Follow until neurological deficits are resolved or determined to be pem1anent

References:

1. Kim MK, Buckman R, Szeremeta W. Penetrating neck trauma in children: an


urban hospital's experience. Otolaryngol Head Neck Surg. 2000; 123:439--43.
2. Stanley RB, Armstrong WB, Fettem1an BL, Shino ML. Management of
extemal penetrating injuries into the hypopharyngeal-cervical esophageal
funnel. J Trauma. 1997;42:675-9.
3. Biffl WL, Moore EE, Rehse DH, eta!. Selective management of penetrating
neck trauma based on cervical level of injury. Am J Surg. 1997;174:678-82.
4. Eddy VA.ls routine arteriography mandatory for penetrating injuries to zone
I of the neck? J Trauma. 2000;48:208-14.
5. Sekharan J, Dennis JW, Veldenz HC, et al. Continued experience with physi-
cal examination alone for evaluation and management of penetrating zone 2
neck injuries: resu lts of 145 cases. J Vase Surg. 2000;32:483-9.
6. Demetriades D,Asensio J, Velmabos G, et al. Complex problems in penetrat-
ing neck trauma. Surg. Cl in North Am. 1996;76(4):661 - 83.
7. Klyachkin M, Rohm iller M, Charash W, et al. Penetrating injuries of the
neck: selective management evolving. Am Surg. 1997;63(2): 189- 94.
177
Otolaryngology Clinical Case Studies Chapter 3 - General Otolaryngology

NoTEs CAsE 13 - BLuNT NEcK TRAuMA


Jesse E. Smith, MD

A 45 year-old male presents to the ER after an aggravated assault in which


he was hit in the neck. You are called emergently to evaluate him.

HPI: st:He that you would:

•• Obtain a detailed medical history, beginning with a history of present iI Iness

He is a homeless man that was attacked in the street


• He can hardly speak from hoarseness, and is having increased stridor
and uses accessory muscles for respiration
• He denies previous operations or p revious trauma to his neck
• He has not s[flr up any blood, but has coughed up a small amount of
blood tinged sputum

PMH: None
Allergies: NKDA
Medications: None
FH: None
SH: Tobacco 1 ppd; ETOH 1, 6-pack of beer/day; No I VDA
ROS: As above

What additional historical information would you seek? State that you would:

• Ask if the patient has known pulmonary disease such as COPD or reactive
airway disease
• Ask if he feels short of breath

The patient states that he has never been d iagnosed or treated for pulmo-
nary ailments. He does feel short of breath.

• Perform a complete head and neck PE including vital signs and ophthalmo-
logic exam

PE:

l11.a.Ls.: Temp: 37.0 C, BP: 165/95, Pulse: 110


0 1 Sat: 92% on 2 L nasal cannula
GA: WDWN, WM, with difficulty breathing

The patient has an ecchymotic and swollen area around the right side of
the neck near the thyroid cartilage. This area is tender to palpation. There
is some crepitus in the soft tissues around the midline of the neck but there
are no external lacerations. The OC and OP are cleaT: He has bilateral 2+
carotid pulses.

178
Chapter 3 - General Otolaryngology Otolaryngology Clinical Case Studies

What is your differential diagnosis? NOTES

• Thyroid ca1tilage fracture. This can be appreciated first by palpation and


then confim1ed by CT scan
• Glottic mucosal laceration. This can be confim1ed by laryngoscopy
• Separation of the thyroid cani !age or trachea. Again, this diagnosis can be
made by palpation and confirmed with aCT scan, although the oatient would
likely be in much greater distress and there would be large amounts of air
spreading through the soft tissues of the neck
• Arytenoid dislocation. This can sometimes be visualized by aCT scan, but
laryngoscopy will al low better visualization and may show fixation of the
cricoarytenoid joint. Manipulation of the ca1ti lage into position is seldom
successfu l
• Glottic hematoma. This can be seen by laryngoscopy and aCT scan. The
mucosa ofthe glottis will be edematous and ecchymotic

What diagnostics would you request? -State that you would:

• Arrange for direct laryngoscopy and rigid esophagoscopy. First, the airway
must be secured. An awake local tracheotomy is a reliable and conservative
technique for securing an·airway in a patient with laryngeal injury. Endotra-
cheal intubation can be difficult, cause additional damage to the larynx, and
obscure and impair successive examinations and repair
• Ensure that the C-s"pine is evaluated both radiologically and by PE. If this has
not been evaluated prior to surgery, then maintain in-line stabilization during
the procedure
• Obtain a portable CXR

Test results:

Evaluation after an awake local tracheotomy reveals a right mucosal lac-


eration in the false vocal cord with associated edema and erythema. By
palpation, a step-off _is noted within the right thyroid cartilage framework.
The · laryngeal cartilage is stable and the anterior commissure is intact.
Esophagoscopy is normal. Oblique & flexion -extension C-spine plain film
are also normal.

Diagnosis:

The diagnosis is a minimally displaced but stable right thyroid cartilage


fracture with an associated right mucosal false vocal cord laceration.

What would you do next?

• Surgical repair

179
Otolaryngology Clinical Case Studies Chapter 3 - General Otolaryngology

NOTES What are the surgical ro utes to the thyroid cartilage and laryngeal framework?

• Horizontal skin crease incision at the level of the cricothyroid membrane


• Midline thyrotomy
• Approach through any preexisting laceration

Surgical techniques:

Highlights and pitfalls:


• As discussed previously, an awake local tracheotomy with direct laryngos-
copy and rigid esophagoscopy have already been performed
• You will already have obtained informed consent for repair, including the
possible use of plates and screws
• Mark the surgical site .
• Start perioperative antibiotics
• Open reduction and internal fixation of the right thyroid cartilage fracture
with either absorbable miniplates or titanium miniplates. Repair of the mu-
cosal laceration
• Leave the patient's tracheotomy tube in place until the edema has resolved
and the patient can tolerate capping. Consider leaving the cuff up for 7-10
days
• Consider placing a NG feeding tube under direct visualization for nutritional
support during healing

What are the potential complications?

· • Hoarseness or dysphonia. With laryngeal injuries, hoarseness and dysphonia


are not uncommon. Scarring, cricoarytenoid joint fixation, and adhesions can
occur, all limiting the return of the patient's prior voice
• Aphonia. An uncommon complication, but always a concern with laryngeal
framework injuries and scarring
Aspiration. Closure of the glottis is a key portion of normal deglutination. If
the tracheotomy tube is too large, if pain limits the patient, or. if scarring
occurs limiting medialization of the TVC, swallowing can be affected
• Extrusion of plates and screws. With continual movement ofthe larynx and
the soft nature of cartilage, this is a remote possibility

How would you follow this individual? State that you would:

• Consider obtaining a postreduction CT scan without contrast to evaluate


proper alignment of the laryngeal cartilages after reduction and fixation
• Speech therapy should be notified and a plan should be developed to insure
that the patient has every opportunity to regain full function of his larynx
• Leave the tracheotomy tube in place until glottic edema is decreased and the
patient can tolerate swallowing and capping for more than 3 days. Serial
fiberoptic laryngoscopy exams can provide usefu l information about the reso-
lution of edema, the amount of scan·ing, the movement of the TVC, and the
glottic aperture

180
Chapter 3 - General Otolaryngology Otolaryngology Clinical Case Studies

o If the patient"s airway is stable, and he is unable to tolerate capping in the NoTES
hospital. the patient may go home after 3-5 hospital days as long as he and
his caretakers understand how to properly manage his tracheotomy tube .
The patient should be discharged home with a PPl

The patie711 returns ro your office 1 week after discharge. He was dis-
charged home with a suction machine, saline bullets. a PPI. and the proper
supplies and directions for tracheotomy care. He cleans his inner cannula
every .:/ hours as instructed and caps his tube during the day. Howeve1~
during the lasr few days it has become more difficult ro breath with the .tube
capped and he has noted some blood-tinged tracheal secretions and in-
creasing dysphonia.

What would you do and why?

• A complete examination of the patient reveals a loosely tied tracheotomy


tube in place with some blood-tinged secretions. The tracheotomy tube is
removed. The tube is a non-cuffed, fenestrated tracheotomy tube with a
fenestrated inner cannula. The tracheal mucqsa is slightly erythematous, and
there is a small tuft of granular tissue extending from around the superior
surface of the surgical site. Fiberoptic laryngoscopy examination reveals a
healing mucosal laceration and bilateral TVC movement. There is still some
diffuse glottic edema noted but the glottic aperture seems to have improved
smce pnor exams
• The physician should reassess the tracheotomy tube
- Is the tube too large? .
- Is the fenestration causing granulation tissue because the tube is too
loosely held in place?
- Should the patient have a smaller, non-fenestrated·tube inserted?
Shou ld the granulation tissue be removed?
• If the granulation tissue is ~asy to remove it can be removed in the clinic, and
bleeding controlled with either chemical or heat cautery
- A smaller non-fenestrated tracheotomy tube can be placed to reduce the
likelihood of granulation tissue formation

How would you then follow this individual?

• Tracheotomy tube capping may continue, and if the patient continues to do


well, be may be decannulated within 1 week
• Repeat flexible fiberoptic laryngeal exams over the next year to assess heal-
ing and scar tissue fonnation
• Continue PPI unti~ patient is decannulated, the tracheotomy has healed, and
swallowing is nom1al ·

References:

I. Park SS. Blunt trauma to the face & neck: initial management. Compr Ther.
1997 23( II):730-5.
2. Sweeney TA, Marx JA. Blunt neck injury. Emerg Med Clin North Am.
1993; II (I ):71-9.

181
Otolaryngology Clinical Case Studies Chapter 3 - General Otolaryngology

NOTES CASE 14 - SIALOLITHIASIS


Robert T. Adelson, MD

A 42 year-old female presents wirh a 3-day history of pain and swelling in


the righr submandibular region.

HPJ: state that you would:

illness
~ .
Obtain a detailed medical history beginning with a history of the present

What additional historical information would you seek? State that you would ask:

• Are there any palliative or provocative features for these symptoms? Does
pain and swelling increase around meals? '.
• Has the patient ex.perienced submandibular area pain and/or swelling at any
time in the past?
Has she noted any associated fever or weight foss?
• Has she experienced pain or difficulty with swallowing?
• Has she been in close contact with cats or person with TB?
• Does she have any risk factors for HIV infection?

The patient describes a 3-day history of right submandibular area pain


and swelling exacerbated by eating and relieved to a small degree by anti-
inflammatory medication, but without returning to her pre-morbid asymp-
tomatic state. The patient has never before experienced this combination
of symptoms. She specifically denies fevers, trismus, xerostomia, dysph-
agia and weight loss, but does :admit to odynophagia when the swelling is
maximal. She does not recall any exposure to cats, or individuals known to
have TB. The patien·t denies any history of trauma or behaviors that might
expose her to infection with HIV

PMH: Allergic rhinitis, controlled with antihistamines and intra-


nasal corticosteroids ·
No history of gout
PSH: Laparoscopic cholecystectomy (1997)
Extraction of wisdom teeth (1989)
Allergies: NKDA
Medications: Desloratadine, budesonide
FH: Hashimoto s thyroiditis (mother); NIDDM (paternal grand-
mother)
SH: Tobacco 12 ppd: ETOH, 3- 4 drinks/week, business
consultant
ROS: As above in HPJ. In addition: no otalgia, no xerostomia,
no night sweats, no dentalgia, no skin changes

What would you look for on PE? State that you would:

• Perform a complete head and neck PE including vital signs

182
Chapter 3 - General Otolaryngology Otolaryngology Clinical Case Studies

NoTES
1!i1.ctb_: Temp: 100.5 F. HR: 75, BP: 135170. RR: 22
GA: WDWN, WF, NAD
AU: TMIEAC normal. no MEE, No perforation
Nose: Straight septum, mucosal pal/01: no secrelions, mild infe-
rior turbinate hypertrophy (bilat), no lesions
OPIOC: Tonsils <1+, no oropharyngeal erythema, but mild
cobblestoning of the orophm~vngeal mucosa. The remain-
der of !he OC is normal: teeth are in good repail: no pal-
pable lesions. The tongue is mobile and the FOM is soft.
Bimanual examination of the right submandibular gland
demonstrates an enlarged gland, firmer than normal and
distinctZv painful to palpation. The floor of mouth is soft.
and palpation of the duct of the submandibular gland lo-
calizes a firm object medial to the right first molm: Cloudy
\ saliva is expressed from the papilla of the right Wharton s
duct during bimanual examination. The papilla of Wharton 5·
,·' duct is normal bilaterally, as are the intraoral openings of
both Stensen s ducts

The parotid glands and left submandibular glands are palpably normal.
The right submandibular area is swollen; no skin changes are noted.
Shorty cervical lymphadenopathy is palpable in right levels II and Ill. No
other cervical lymphadenopathy. CN II- XII intact.

. What is your differential diagnosis?

.- In a patient"with new onset of right submandibular area swelling and symp- ·


toms consistent with salivary colic, the most likely diagnosis would be sialolithi-
asis with or without concomitant sialadenitis
Further work-up with various imaging modalities and exploration with
sialoendoscopy can distinguish between this diagnosis and a number of
similar appearing conditions. These conditions include neoplastic disease,
sialadenitis resulting from ductal stricture or·ductal polyps, odontogenic
infection, viral infection (mumps, HIV, coxsackie virus), granulomatous
disease (mycobacterial, actinomycosis, cat-scratch disease, toxoplasmo-
sis), sialadenosis, Mikulicz's disease, Sjogren's syndrome, and vascular
malformations

·! What diagnostics would you request?

• Plain films of the submandibular gland. Radiographic studies are the highest
yield investigation in a case of suspected sialolithiasis. Many authors believe
imaging should be reserved for those patients with non-palpable stones in the
face of a history and PE otherwise consistent with sialolithiasis. Familiarity
with different imaging options will allow the physician to choose an appropri-
ate study (or combination of studies)
- Plain film radiography (intraoral, AP open-mouth view and submental
occlusal view) is valuable since almost 90% of submandibular gland stones
are radiopaque, and 75-85% are located within Wharton's duct. Plain

183
Otolaryngology Clinical Case Studies Chapter 3- General Otolaryngology

NOTES film studies can establish the size, location and number of stones, thereby
determining treatment options available to the patient

Test results:

In this case, plain films conjirm the presence of a solitary elliptical 12 mm


stone located at the posterior Y 3 of Wharton s duct, extending posteriorly
to the region of the first molar

If plain films fail to demonstrate a sialolith, what other modalities can be used to
study a patient without a palpable stone?

• Imaging with CT scans or sialography. Fine cut CT scans with and without
IV contrast should identify all stones and provide additional infOJmation re-
garding the degr~e of glandular inflammation, the surrounding neck anatomy,
ruling-out unusual conditions for this case, such as ranula, neoplasm, and
vascular malformation. It is important to obtain one set of images without
contrast, as contrast can preclude visualization of a stone. Sialography is
contraindicated in the presence of acute infection, and may be both inappro-
priate and superfluous in cases where a stone has already been located by
plain film radiography. Sialograms are helpful in describing the salivary drainage
system, detecting I 00% of stones, characterizing strictures, depicting extrin-
sic or intrinsic glandular compression by tumors, and illustrating the general
inflammatory state of the salivary gland
• US is useful in identifying and localizing sialoliths, and may be the study of
choice in a patient with acute infection who cannot undergo sialography.
However, limited experience·with US diagnosis of acute salivary gland disease
combined with the superior diagnostic capabilities and nearly universal avail-
ability ofCT, has practically eliminated US from the work-up of sialolithiasis
• Sialoendoscopy is useful in identifying radiolucent stones as well as differen-
tiating ductaJ. lesions (polyps, mucous plugs) from stones; however, in the
vast majority of cases, standard imaging techniques will accurately guide
treatment decisions

Diagnosis:

Right submandibular sialolithiasis with acute sialadenitis. The stone is lo-


cated in the posterior portion of Wharton~- duct, near the hilum.

What are the treatment options and their complications? What would you do
and why?

• A 12 mm submandibular stone will not pass spontali'eously, and therefore


conservative management with sialogogues, hydration, g·land massage and
local heat is not an appropriate treatment choice
• Antistaphylococcal antibiotics should be initiated and the surgical options of
sialithectomy versus sialadenectomy should be addressed with the patient

184
Chapter 3 - General Otolaryngology Otolaryngology Clinical Case Studies

How does one decide between sialolithectomy and sialoadenectomy in the NOTES .
management of Wharton's duct stones?

Palpable stones within Wharton ·s duct should be removed through a transoral


approach, under local anesthesia, preserving the submandibu lar gland. In large
studies, this technique has been successfu l with both distal stones ( 100%) as
well as the more proximally located hilar stones (92%). SialadF' 1ectomy may be
required to treat proximally located stones, those patients who fail a conserva-
tive, gland-sparing intervention, or patients with recurrent episodes of sialolithi-
asis. Stones located behind the posterior edge of the mylohyoid muscle present
significant challenges for transoral sialoithectomy. Dissection ofWharton 's duct
distal to the posterior edge of mylohyoid is technically difficult, places the lingual
nerve at greater risk. of injury, and typically requires a general anesthetic to
allow for adequate exposure, stone removal, and repair of the duct. Though the
transoral route may allow a functional gland to be preserved, many surgeons
prefer sialoaderiectomy for such proximally positioned stones

Are any other surgical options available?

• Sialoendoscopic techniques have introduced minimally invasive techniques


to this surgical problem
• Mechanical extraction of stones and balloon dilation of Wharton's duct (with
or without papillotomy) is a conservative initial procedure that can avoid the
rare complication oflingual nerve injury associated with transoral sialolithectomy
• Pulsed dye laser lithotripsy under sialoendoscopic guidance has been suc-
cessful in treating hilar stones from 5-17 mm in size and in experienced
hands may provide an effective alternative to transoral sialolithectomy

The above techniques are limited more by availability of technology than by


success of the procedures, and likely represent viable and equally successful
approaches to a stone impacted within Wharton's duct.

Can the lithotripsy techniques that have been successful for kidney stones be
emplored in management of salivary gland stones?

• ESWL has been applied to the treatment of sialolithiasis since I989


- Stones are disintegrated into fragments less than 2 mm in diameter, which
should be cleared from the duct with the assistance of sialogogues and
hydration

How can you determine if a patient is a good candidate for ESWL management
of a salivary duct stone?
.
• A functional salivary gland is required for washout offragments, and this can
be assessed by glandular swelling in response to a sialogogue (i.e., a positive
"gum test")

ESWL is more successful in parotid disease than for submandibular stones and
for stones< I 0 mm in diameter. A high failure rate for ESWL in management of
intraductal submandibular stones has led some authors to consider this location
a contraindication to ESWL.

185
Otolaryngology Clinical Case Studies Chapter 3 - General Otolaryngology

N OTES What are your· recommendations for treatment?

• A palpable stone anterior to the third mandibu lar molar is wel l positioned for
a transoral sialolithectomy
• The intraductal location is a contraindication to ESWL, and the availabi lity of
equipment precludes sialoendoscopic techniques

The patient elects to undergo a traditional transoral sialolithectomy.

Discuss your surgical techniques:

The following procedure is performed in the otolaryngology office after obtain-


ing appropriate infom1ed consent

Highlights and pitfalls. Availability of an operating microscope or loupes can


facilitate cannulation of the duct and any ductal repair that is required
• Antistaphylococcal antibiotics are given, and the patient rinses her mouth
with Peridex
• Topical anesthetic is applied to the dried mucosa of the FOM and a regional
·lingual nerve block.is achieved
o A lacrimal probe is used to cannulate the right Wharton's duct and the stone
is palpated in the expected location between the first and second molars
• Direct dissection over the tip of the lacrimal probe, into the duct, will expose
the stone
• The stone is removed and the incision may be left either open or sutured to
the surrounding mucosa with absorbable stitches
• The gland is vigorously massaged and irrigated to remove stagnant secre-
tions and microliths remaining in the ducts
• (Alternatively, the ostium of the right Wharton's duct is incised with a# 11
scalpel or with one blade of an iris scissor. Dissection is carried posteriorly in
a continuous fashion until the stone is exposed·and removed. Parenchymal
incisions as well as ductal incisions are sutured to the mucosa, developing a
new drainage pathway)

What are the potential complications?

• Stricture: Strongly associated with sialadenitis and sialolithiasis, strictures


may also result from ductal instrumentation. Strictures can lead to further
obstruction and episodes of acute sialadenitis. Sialoendoscopy allows identi-
fication of ductal strictures and balloon catheter di lation of these lesions with
excellent results
• Acute suppurative sialadeniti~ : Patients with retained stones or inflammatory
strictures may suffer recurrent obstruction and infection ofthe submandibu-
lar gland
• Recurrence of sialo lithiasis: Gland-preserving techniques are associated with
an 18% risk of recurrence of sialolithiasis. The inciting condition is unknown,
and is not resolved by simple sialolithectomy. The ductal injury may serve as
a nidus for lithogenesis
• Lingual nerve injury: This nerve injury anesthetizes the ipsilateral tongue and
OC and can be devastating to some patients, yet it is a seemingly avoidable

186
Chapter 3 -General Otolaryngology Otolaryngology Clinical Case Studies

complication ofboth sialadenectomy and sialol itbectomy. The lingual nerve NOTES
is at put at significant risk during transoral sialolithectomy if dissection ex-
tends posterior to the first molar, toward the posterior edge of the mylohyoid.
Most authors recommend sialadenectomy for stones in this location, as lin-
gual nerve injury is a much more remote risk during this approach
• Sialadenectomy complications: Patients who either fail stone removal by gland-
preserving measures, or those with stones not amenable to simp le
sialolithectomy, should be aware of the risks of sialadenectomy. Although a
rare injury, the lingual nerve and, to a lesser degree, the hypoglossal nerve.
are both at risk during excision of the submandibular gland. Transient (<30%)
and permanent (0-8%) dysfunction of the marginal mandibular nerve is a
risk with serious functional and aesthetic consequences for the patient. These
complications are more likely in patients with significant inflammation or
scarring that hinders proper identification of structures

How would you follow this individual? State that:

• - Precautions are made to decrease the risk of surgical site infection and speed
resolution ofher sialoadenitis
• The patient is maintained on antistaphylococcal antibiotics, increased hydra-
tion, sialogogues, and Peridex mouthwash for 7 days after surgery
• The patient should be re-examined on POD 7 for persistence of infection or
submandibular swelling that requires additional work-up
• A submandibular neoplasm, synchronous stone, or granulomatous disease
may be the underlying cau.se for refractory symptoms

The patient may continue to use anti-histamines for treatment of his aller-
gic rhinitis, as no medications have been specifically associated with
sialolithiasis.

References:

I. Gayner SM, Kane WJ, McCaffrey TV. Infections of the salivary. glands. In:
Cummings CW, Fredrickson JM, Harker LA, Krause CJ, Schuller DE,
Richardson MA. eds. Otolaryngology-head and neck surgery. 3'd ed. St.
Louis: Mosby-Yearbook; 1998, 1234-46.
2. Kim RH, StJimling AM, Grosch T, Feider DE, Veranth JJ. Nonoperative
removal of sialoliths and sialodochoplasty of salivary duct strictures. Arch
Otolaryngol Head Neck Surg. 1996; 122:974-6.
3. Rice DH. Nonneoplastic diseases ofthe salivary glands. In: Bailey BJ, Johnson
JT, Healy GB, Pillsbury HC, Jaclder RK, Tardy, ME, Calhoun KH, eds.
_Head and neck surgery-otolaryngology. 3med. Philadelphia: Lippincott Wil-
liams & Wilkins; 2001, 453-62.
4. Williams MF. Sialolithiasis. Otolaryngol Clin North Am. 1999;32:819-34.
5. Zenk J, Constantinidis J, Al-Kadah B, lro H. Transoral removal of subman-
dibular stones. Arch Otoaryngol Head Neck Surg. 200 I ;127:432-6.

187
Otolaryngology Clinical Case Studies Chapter 3- General Otolaryngology

NoTES Surgical techniques

• The traumatic laceration itself often provides adequate access for wound
exploration and duct repair
• Pre- and peri-operative antibiotics should broadly cover oral flora, with par-
ticular attention to organisms typically implicated in parotitis (Staph aureus).
- Suggested: Clindamycin (900 mg IV q8h for 24 hours) or Unasyn (3 gm
IV q6h for 24 hours), with conversion thereafter to the oral route
(Ciindamycin or Augmentin, as detailed be low) for 7 days
• Devitalized tissue shou ld be removed and the wound thoroughly irrigated
• The cornerstone of this procedure is conect identification of the parotid duct
injury. This goal can be achieved by intraoral cannulation of the parotid duct
with a 20 gauge silastic catheter followed by injection of a small bolus of
saline or methylene blue dye

Highlights and pitfalls:


• Methylene blue should be used in small quantities and with caution, as the
· dye can stain sunounding tissues and make further exploration more difficult
• The ductal ~njury is repaired (as detailed below)
• Layered closure of the wound, with an active suction, closed system drain
placed near the salivary gland injury. Meticulous closure of the facial skin
• A pressure dressing applied to the parotid gland will encourage salivary flow
in the typical direction
Pre- and post-operative antibiotics to cover Staph aureus and typical oral flora

How does the recommended method of repair differ for the various regions of
the parotid gland drainage system?

• Injuries occur in 1 of 3 general regions of the ductal system. Each is ame-


mible to a particurar type of repair
• Parotid gland or proximal duct injury is best treated by suture repair of the
parotid capsule or duct
• Interruption of the duct as it courses across the masseter muscle is most
commonly repaired with microsurgical techniques allowing direct end-to-end
repair of the duct over the silastic catheter. The proximal and distal ends of
the duct are located and debrided. The duct is mobilized as much as is pos-
sible, to allow for a tension-free repair. Most authors agree that 9-0 or 10-0
monofilament (nylon or proline) sutures in an intem1pted or running fashion
will provide an adequate closure. The silastic catheter allows salivary flow
through the recently traumatized duct and helps the surgeon avoid inadvert-
ent placement of stitches into the back wall of the duct. Stents can be sewn
to th"! buccal mucosa, as they should remain in place for 7-14 days
• Sectioning of the distal portion of the duct, as it courses around the anterior
border of the masseter and pierces the buccinator muscle, results in a distal
remnant which may not be amenable to primary repair. Stensen 's duct may
be re-implanted directly into the buccal mucosa in a more posterior location,
and stented for 7-14 days, as described above

190
Chapter 3 - General Otolaryngology Otolaryngology Clinical Case Studies

A segmental loss of Stensen's duct prohibits primary end-to-end repair. There NOTES
are 4 proposed solutions to this difficult surgical problem. More proximally lo-
cated losses can be treated with ligation of the duct, followed by a painful period
of parotid swelling and later. by atrophy of the gland. A total parotidectomy will
eliminate difficulties in management of severe duct injuries; however, this ap-
proach introduces those risks of facial nerve injury, cosmetic deformity and
gustatory sweating associated with total parotidectomy. Attempts can be made
to re-implant the duct in the OC, provided enough of a remnant is available to
the surgeon. Finally, interposition vein grafting has been successfully employed
in the repair of segmental duct loss, though this procedure requires harvesting of
a vein as well as a total of 2 microsurgical anastomoses.

I • What are the potential complications of parotid duct injury and operative repair?

• State that a sialocele is one of the more common post-operative/post-trau-


matic complications. This collection of saliva in the soft tissues of the face
may result from unrecognized or poorly repaired parotid parenchyma or duct
mJunes
• How can you diagnose a sialocele? Aspiration of fluid and laboratory assay
finding amylase levels > l 0,000 units/L will confirm the diagnosis
• What is the management of a sialocele? Repeated aspiration, pressure dress-
ings, eliminating PO diet, and antisialagogue medications are the first step in
medical management of a post-traumatic sialocele. Recently, authors are
reporting successful medical management of sialoceles with Botox injec-
tions into the offending lesion. Surgical treatment of a sialocele may involve
questionably effective procedures, such as a tympanic neurectomy, or a more
successful total parotidectomy, which is made increasingly risky by the sur-
rounding bed of saliva and inflammatory tissue. Conservative measures are
strongly favored in the treatment of sialoceles ·
• A more unusual complication is a salivary fistula. The external drainage of
saliva through the skin can result from INAD duct repair or from rupture of
a longstanding sialocele
What is the treatment for salivary fistulas? Fistulas that present early in the
post-operative course should be re-explored with revision of a faulty duct
repair and/or identification of an initially unnoticed ductal or parenchymal
...
injury. The options for management of a salivary fistula are similar to those
described for management of a sialocele, with glandular sources having a
better prognosis than ductal sources, and conservative measures favored
over surgery

How would you foll~w this individual? State~that:

The patient should be evaluated in the office setting between POD 5 and 7 to
allow timely suture removal and assessment for early complications of duct
repair (sialocele or wound infection). The parotid duct stent is removed be-
tween POD 7 and 14, as this decision is guided by the PE findings. The
absence of undue parotid swelling and early surgical complications in con-
junction with visual confirmation of salivary flow through the repaired duct
should prompt removal of the stent

191
Otolaryngology Clinical Case Studies Chapter 3 - General Otolaryngology

NOTES • Post-operative antibiotics should be continued for I week (amoxicillin 500mg


po bid or Clindamycin 300 mg po qid); however, there is no clear consensus
on this issue
• The patient is followed closely in the immediate post-operative period for
development of clinically apparent complications, such as wound infection,
sialocele. or salivary fistula . Patient education regarding the signs and symp-
toms of infection is the best sentinel for early identification of surgical com-
plications. After the initial 2 office visits, the uncomplicated patient could be
seen in 6 months to assess the final aesthetic and functional result of this
facial trauma
• Attention is paid to appropriate rehabilitation of the facial wound. Dermabra-
sion and other methods of scar revision are tailored to the patient's needs

References:

I. Haller JR. Trauma. In: Cummings CW, Fredrickson JM, Harker LA, Krause
CJ, Schuller DE, Richardson MA, eds. Otolaryngology-head and neck sur-
gery. yct ed. St. Louis: Mosby-Yearbook; 1998, 1247-54.
2. Gibson FB. Management of Soft-Tissue Trauma. In: Bailey BJ, Johnson JT,
Healy GB, Pillsbury HC, Jackler RK., Tardy ME, Calhoun KH, eds. Head
and neck surgery-otolaryngology. 3'ct ed. Philadelphia: Lippincott Williams &
Wilkins; 2001, 756-804. ·
3. Haller JR. Trauma to the salivary glands. Otolaryngo l Clin North _Am.
1999;32:907-18.
4. Lewkowicz AA, Hasson 0, Nahlieli 0. Traumatic injuries to the parotid
gland and duct J Oral Maxillofac Surg. 2002;60:676-80.
5. Heymans 0, Nelisen X, MedotM, Fissette. Microsurgical repair of Stensen 's
duct using an interposition vein graft. J Reconstr Microsurg. 1999; 15: 105-8.

192
Chapter 3 -General Otolaryngology Otolaryngology Clinical Case Studies

CASE 16- Luowic'sA.NGINA WITH TRACHEOTOMY NoTES


Larry L. Myers, .MD

You are consulted to see a .:/3 year-old man who is in the ER. He is drool-
ing, has inspiratory strid01~ has a muffled voice, and appears fi'ightened.
He has a rapid RR and has fevel:

HPI: state that you would:

• Obtain a history of present illness


• Ask about the acuity of illness? (hours, days, or weeks)
• Ask how long has he had fever?
• Ask how long has he had difficulty breathing?
• Ask about a recent history of dental disease

Subject reports increasing shortness of breath and subjective fever for


last 3 days.

Allergies: NKDA
Medications: A ugmentin (1 day)
FH: Negative
SH: ( - ) Tobacco, ( - ) ETOH, painter
PMH: NC
PSH: Abscessed tooth pulled 1 week ago. Patient did not fill
post-procedure Rx until 1 day ago
ROS: NC
What would you look for on PE? State that you would:

• Perform a complete head and neck examination including vital signs

PE: ·

1:11@.: Temp: 39 C, BP: 120184, HR: 110, RR: 24, 0 1 sat 93%
GA: Ill-appearing man, sitting on edge of bed, scared facies,
difficulty breathing
HEENT Swollen FOM with tongue pushed to roof of mouth.
Brawny, swollen, erythematous submental region
' Fiberoptic
'

.I laryngoscopy: Nearly obstructed upper air¥vay at level of base of tongue,


cannot fully visualize epiglottis

What diagnostics would you request: State that you would request:

• Laboratory studies: CBC, BUN, Cr, blood culture

193
Otolaryngology Clinical Case Studies Chapter 3 - General Otolaryngology

NOTES Test results:

White blood cell count J.l, 000. BUN 26. Creatinine 1. Blood culture
pending.

What is your differential diagnosis?

• To generate a broad differential diagnosis for most conditions of the head


and neck, the use of a mnemonic may be helpful. For this specific case, the
mnemonic generates the following possibilities

Mnemonic: K.IJTTENS

K (congenital) Un likely, as this is an acute process


I (infectious) Acute process: Ludwig's angina, bacterial (aerobic/
anaerobic), TB, atypical TB, neck abscess
I (inflammatory) Angioedema, allergic reaction
T (tumor) Benign, malignant, metastatic
T (trauma) Foreign body
E (endocrine) Thyroid mass/malignancy
N (neurogenic) Unlikely
S (systemic) Autoimmune disorders, amyloidosis, sarcoidosis

Diagnosis:

The diagnosis is Ludw_ig :S angina. Ludwig s angina is a bacterial cellulitis


involving the submandibular and sublingual spaces. 1t often occurs fol-
lowing an infection of the roots of the teeth, such as a tooth abscess or
after a mouth injury. This condition can precipi!ate acute airway compro-
mise requiring an emergent tracheotomy.

What are the treatment options and their complications? What would you do
and why?

• In this case, an emergency tracheotomy is performed for airway protection


• After an airway has been established, incise and drain submandibular and
sublingual spaces and culture fluid for aerobic, anaerobic, TB, and fungal
organisms relate to Ludwig's
• Administer empirical IV antibiotics to treat an infection arising from oral
flora. The most common organisms are alpha-hemolytic streptococcus and
staphylococcus. High dose penicillins or Clindamycin are appropriate antibi-
otic dioices •

Surgical techniques:

• Tracheotomy
Highlights & pitfalls:
- Inject inferior midline neck soft tissue with local anesthetic
Horizontal incision in midline of neck between cricoid and sternal notch
- Divide or resect subcutaneous fat to level of strap muscles

194
Chapter 3 - General Otolaryngology Otolaryngology Clinical Case Studies

- Divide strap muscles in midline N OTES


- Elevate cricoid with cricoid hook
- Divide thyroid gland anterior to trachea
- Inferiorly-based Bjork flap or tracheal ring window of2nd or J'd tracheal ring
- Dissection too far lateral in subcutaneous fat will cause bleeding from
anterior jugular veins
- Not dividing in midline may cause damage to contents of carotid sheath
and/or inabi Iity to identify trachea
• Incision and drainage of neck/submental abscess
Highlights:
- Choose a natural skin crease inferior to course of marginal mandibu lar
branch of facial nerve
- Incise through the subplatysma to the deep neck spaces
- Culture fluid, if any
- Break up loculations
- Use passive drains
- Close skin loosely to allow egress of infected fluid
Pitfalls:
- An incision too superior on the neck puts the marginal mandibular nerve
at risk
Failure to break up loculations will not allow adequate drainage of deep
neck abscesses

Potential complications:

• Ludwig's angina: Mediastinal extension, pharyngomaxillary space extension,


osteomyelitis, airway obstruction
• Tracheotomy: Bleeding, subcutaneous emphysema, postoperative scar for-
mation in lower neck, anoxic cerebral injury or death if airway not estab-
lished in a timely fashion

How would you follow this individual~ State that:

• The patient can be decannulated if he tolerates tracheotomy tube capping


for greater than 6 hours
- This establishes that the upper airway is patent and stable
- Use culture results and antibiotic sensitivities to guide fmal antibiotic choice
• Most causative organisms are anaerobic
- Use intravenous Clindamycin, 3'd generation cephalosporin, or a fluoro-
quinolone
• Obtain a dentistry consultation to fully evaluate dentition
Dental abscesses or infected teeth are the usual causes of Ludwig's
angma

195
Otolaryngology Clinical Case Studies Chapter 3 - General Otolaryngology

NOTES CASE 17 - NECROTIZING FASCIITIS OF THE NECK


Barbara A. Schultz, MD

A 19 year-old male presented to the ER ·with a I -day history of mild swell-


ing and erythema in the anterior neck extending from the sub mental area
to the cricoid area. Th is was very painful and associated with a fever of
102 F

HPI: state that you would:

• Obtain a full medical history

What additional historical information would you seel<? State that you would ask:

• 1s there pain anywhere else?


• Is there drainage from the wound?
• ls there a history of dental infections?
Is there a history of any immunocompromise such as diabetes, AIDS, steroid
use, IV drug use, organ transplant?

The mass and pain came up rapidly and is very tender to touch and is also
associated with tenderness when moving the neck. He complains of a loose
lower right molar, which has been bothersome for 3 months. ·

PMH: DNI on insulin shots bid with recent elevation of fasting


blood glucose to 250
Allergies: NKDA
Medications: Insulin, Motrin forpain
FH: Negative
SH: Smokes ~ ppd and drinks 2 beers/day
ROS: Noncontributory

What would you do next?

Perform a complete PE including vital signs

PE:

Yi1.als_: Temp: 102.2 F, BP: 125185, Pulse: 120


GA: Very ill appearing male but without airway distress or
stridor 9

HEEFT: TMs clear, nose clear, mouth and OP reveal edema and
erythema around a palpably loose 2nd right mandibular
molar, which is tender to palpation with a dental mirror
and poor dental hygiene in general. The tonsils and floor
of mouth are unremarkable. Indirect laryngoscopy is clea1~
There is erythenia measuring 5x7 em around a small area
of purulent drainage just superior to the hyoid bone. There
is no firm underlying mass but there is marked crepitus of
the neck on palpation

196
Chapter 3 - General Otolaryngology Otolaryngology Clinical Case Studies

Chest Clear to auscultation, heart RRR no murmurs NOTES


Extremities Normal
New-a Normal

What is your differential diagnosis so far?

• Infection related to the poor dentition


• Infected neck cyst
• Deep neck abscess without palpable mass effect yet
• Necrotizing fasciitis with palpable crepitus

After completing your full Hx and PE, what would you do next?

• Culture (aerobic, anaerobic TB and fungal) and gram stain the purulent ma-
terial draining from the neck to differentiate clostridia vs. strep, Staph and
other organisms mentioned below
• Mark the area of erythema of the neck and date and time this marking to
watch for advancement
• Draw labs including CBC, SMA-6, creatinine, ESR, blood cultures, UA
• Start broad-spectrum antibiotics
• Order a stat CT scan ofthe neck without and with contrast and include the
upper chest to look for gas in the soft tissues tracking down into the chest or
any evidence of an abscess in the chest

Test results:

• WBC is 17,000, glucose is 320


• Cultures are pending
• UA shows 2+ glucose and large ketones
• The CT scan did not show a frank abscess but there are multiple areas
of gas within the different fascial compartments of the neck and edema
in the submental area
• There is no extension into the chest by chest CT (discuss the value of CT
vs. MRI)

Diagnosis:

Necrotizing fasciitis of the anterior neck of odontogenic origin compli-


cated by underlying diabetes. This is a very virulent, rapidly progres-
sive subcutaneous and fascial space infection.

What would you do next and why?

Take the patient to the OR for a neck exploration, drainage and debridement
of devascularized tissues and fascia.

Surgical techniques:

Highlights and pitfalls:


• Open the fascial planes well above and below the areas in question
• Remove the infected tooth and ask oral surgery to evaluate the teeth

197
Otolaryngology Clinical Case Studies Chapter 3- General Otolaryngology

NOTES • Irrigate with copious amounts of irrigant and pack the wound open. Plan to
re-explore the following day in the OR especial ly if there is any devitalized
tissue and continue to remove necrotic tissue
• Watch for airway involvement and any evidence that the infection is cours-
ing inferiorly through the "danger space" into the chest

What are the risk factors for a deep neck space infection?

• Poor dentition
• Neck trauma
• Traumatic intubation
• Traumatic fore ign body ingestion
• IV drug use
• Idiopathic 60%

Discuss the likely bacterial findings and treatment:

• Mixed anaerobic/aerobic infection


Organisms of odontogenic origin such as strep pyogenes, strep viridans,
peptostreptococcus, staph aureus, haemophilus influenza, clostridia (gas gan-
grene), enterobacteriaceae

Discuss his medical management, especially IV antibiotics:

• IV antibiotics to cover gram-positive organisms including anaerobes and nar-


row the coverage based on culture results
• Drug choices:
- Primary:
Clindamycin (Cleocin) 900 mg IV q 8 hrs
• Penicillin G I0 mu IV q 6 hrs
• If polymicrobial, consider adding:
- Gentamicin 80 mg IV q 8 hrs
- Ceftazidi'me (Fortaz) I gm IV q 12 hrs
- Imipenem (Primaxin) 500 mg IV q 6 hrs
- Meropenem (Merrem) I gm IV q 8 hrs
- Alternatives:
• Ampicillin/sulbactam (Unasyn) 3 gt11 IV q 6 hrs
• Vancomycin 1 gm IV q 12 lm plus Metronidazole 500 mg IV q 6 hrs
• Ceftriaxone (Rocephin) 1-2 gm IV q 24 hrplus Metronidazole (Flagyl)
500 mg IV q 6 hrs
• Others as dictated by gram stain
• The diabetes needs tighter control with a sliding dose of regular insulin ini- •
tially then Humulin NPH and Regular as needed
• Internal medicine consult for improved diabetes control then diabetes educa-
tion for long term compliance with diet, exercise and medications. Further
education about infection control and prevention with special emphasis on
dental and foot care
• IV fluids to maintain good renal output and prevent renal failure in the face
of an overwhelming infection
• 65% develop extension into the chest, so consider a thoracic surgery consu It
early on
198
Chapter 3- General Otolaryngology Otolaryngology Clinical Case Studies

• Other services such as oral surgery, internal medicine, infectious disease and NOTES
nutritional support are invaluable

What long-term potential problems exist?

• Dental issues
• Diabetes control and education
• Watch for involvement of the great vessels with either thrombosis or carotid
blowout
• Nerve loss due to infection and or debridements
• Airway compromise requiring tracheotomy

What is Lemierre's syndrome?

The criteria for Lemierre's syndrome are:


• Thrombosis of the internal jugular vein
• Secondary to an oropharyngeal infection
• Sepsis .
• Metastatic septicemia with lesions in the lungs, liver or musculoskeletal system

What are the boundaries of the "Danger space"?

Superior- Skull base


• Inferior - Diaphragm
• Anterior- Alar fascia
Posterior - Prevertebral fascia
• Lateral - Transverse process of the vertebra

lt is-through the "Danger space" that an infection which originates in the neck
may track down into the chest. Thoracic surgery consultation is appropriate
early in the course since a thoracotomy may be indicated if the infection does
track down into the chest based on repeat CT scans.

References:

1. Hamza NS, Farrel J, Strauss M, Bonomo RA. Deep fascial space infection
of the neck: a continuing challenge. South Med J. 2003;96(9):928- 32.
2. Moore BA, Dekle C, Werkhaven J. Bilateral Lemierres syndrome: a case
report and literature review Ear Nose Throat J. 2002;81 :234-52.
3. CallenderTA. Necrotizing fasciitis ofthe head and neck. BCM Grand Rounds
,-· Archives, Department of Otorhinolaryngology and Communicative Sciences.
Dec 3 1, 1992.
4. Dierks EJ, MeyerhoffWL, Schultz BA, Finn R. Fulminant infections of od-
ontogenic origin. Laryngoscope. 1987;97 :271-4.
5. Fairbanks DN. Pocket guide to antimicrobial therapy in otolaryngology- head
and neck surgery, 12th ed. Alexandria: American Academy of Otolaryngol-
ogy-Head & Neck Surgery Foundation, Inc.; 2005, 41.

199
Otolaryngology Clinical Case Studies Chapter 3 - General Otolaryngology

NOTES

200
Chapter 4 - Laryngology Otolaryngology Clinical Case Studies

CHAPTER41CASE 1 - UNILATERAL VOCAL FOLD PARALYSIS NoTEs


Daniel Ratcliff, MD

A 62 year-old male presents vvith chronic hoarseness of 8 weeks· duration.

HPI: state that you would:

Obtain a detailed medical history beginning with a history of present illness

What additional historical information would you seek?

Is the voice change intennittent or constant?


Is there any associated dysphasia, odynophagia. cough or dyspnea?
Does the patient have a history of vocal overuse or abuse?
Is there a history of tobacco or alcohol abuse?
Is the hoarseness associated with breathiness or shortness of breath?
•- Are the symptoms worse under emotional stress?
• Has there been an episode of pneumonia?

PMH: COR artery disease, HTN


PSH: Right foot amputation (trauma)
Allergies: NKDA
Medications: Lisinopril, Metoprolol
FH: COR artery disease, diabetes, lung cancer
SH: 60 pack-year tobacco history, 2-3 oz ETOH per day, retired
boilermaker
ROS: Negative (including cough, URI symptoms, post-nasal drip
and GER symptoms, hemoptysis and weight loss)
)
The patient reports the hoarseness is constant in nature without any asso-
ciated dysphagia, odynophagia cough or dyspnea. He denies any signifi-
cant vocal abuse but he does smoke and drink heavily. He has not had any
) episodes of pneumonia. Interestingly, he has -noticed that he ccm only get a
few words out before he feels short of breath This "breathiness" has sig-
nificantly limited his ability to participate in his weekly coffee socials.
)

) What do you look for on PE? State that you would:

~)
Perfonn a thorough head & neck exam including vital signs

_,i PE:

Temp: 98.7 F. BP: 142196, Pulse: 66, RR: 12


Yit.als..:
GA: Obese white male in NAD
Ears. EAC and TM are normal
Nose: No abnormality noted
OPIOC: No mucosal lesions or masses seen
Neck: Trachea midline. No palpable masses. 2+ carotid pulses on either
side

201
Otolaryngology Clinical Case Studies Chapter 4- Laryngology

NoTES FDL: NP clear of masses or lesions. Base of tongue and epiglottis WNL.
The right true-vocal fold is mobile. The left !rue vocal fold is
paramedian in position and non-mobile. There is no visible glot-
tic mass

What is your differential diagnosis?

Non-laryngeal malignancy - Most common identified cause


• Trauma/thyroidectomy- Also includes the trauma of prolonged intubation
Neurologic - Central intracranial process or peripheral nerve process
Idiopathic- Ultimately the most common diagnosis

Diagnosis:

State that the diagnosis is leji true vocal fold paralysis.

Further assessment and discussion:

State that because of the high risk of extra-laryngeal malignancy, it is vital


that the patient undergo radiologic evaluation from the brainstem to the
upper chest. This may include the following:
CXR .
Abnormal lesion in the left lung apex
CT-scan of the brainstem/neck/chest
Normal brainstem
Normal neck
Lesion within left lung apex highly suspicious for malignancy
Mention that in other settings, MRJ and/or laryngeal EMG may prove help-
. ful to establish the exact etiology of true vocal fold paralysis
Pulmonary/thoracic surgery consultation
• Consider medialization of the left true vocal fold for symptomatic relief

What are the treatment options and their complications? What would you do
and why?· ·

Gelfoam injection - Temporary (6-8 weeks' duration)


Fat/collagen injection- Permanent in some cases, but generally resorbs in
varying amounts
• Teflon injection- No longer utilized due to granuloma formation
• Type 1 thyroplasty - Silastic block
Adduction arytenopexy
Laryngeal reinnervation- Nerve muscle pedicle or ansa-hypoglossai-RLN
anastomosis

202
Chapter 4 - Laryngology Otolaryngology Clinical Case Studies

References: NOTES

I. Benninger MS, Crumley RL Ford CN, eta!. Evaluation and treatment of


the unilateral paralyzed voca l fo ld. Oto laryngol Head Neck Surg.
1994; II I :497-508.
2. Zeitels SM. New procedures for paralytic dysphonia. Otolaryngol Clin North
Am. 2000;33:841 - 53 .
3. Rental E, Rontal M, Silvem1an B, eta!. The cl inical differentiation between
vocal cord paralysis and vocal cord fixation using electromyography. Laryn-
goscope. 1993; 103:133-7.
4. Harries ML. Unilateral vocal fo ld paralysis: a review of the current meth-
ods of surgical rehabilitation. J Laryngol Otol. J 996; 1I 0: 111-6.
5. Wang RC, Miller RH. Hoarseness and vocal cord paralysis. Head Neck
Surg Otolaryngol. Byron Bailey, ed. 2001.

203
Otolaryngology Clinical Case Studies Chapter 4- Laryngology

NoTEs CASE 2 - VocAL CoRD NoDULE


Michael Yium, MD

A 28 year-oldfemale reports a 4 month hist01y ofhoarseness and "laryngitis. "

HPI: state that you would:

Obtain a detailed medical history beginning with a history of the present


illness

What additional historical information would you seek? What are some key
elements specific to a voice history?

Identify the problem as the patient sees it


• Get a developmental history of the problem: chronology, duration, onset,
previous episodes, variability, associated signs or symptoms
PMH including medications
• Smoking history

The patient is a 28 year-old bartender at a local dance club with a 5 pack/


year tobacco history who reports gradual onset of chronic hoarseness.
The dance dub in which the patient works is loud and requires that she
speak loudzv with her customers. She -reports that for the past 4 months her
voice is ''rough " or "husky" and that it sounds like she has laryngitis all
of the time. She denies throat pain, but states that her voice is worse if she
is working long hours during the week. Othe·r than brief episodes of hoarse-
ness associated with URis the patient reports no previous problems with
her voice. She tried to quit smoking for a month but this had no effect on
her voice. She takes Motrin occasionally, ·but no other medications. She
drinks 2 cups of coffee per day. She denies a history of dysphagia,
odynophagia, or weight loss.

PMH: Nasal fracture 10190 (soccer injury)


PSI-I: None
Allergies: NKDA
Medications: Occasional Motrin, OCPs
ROS: Occasional morning cough, otherwise noncontributory
FH: Mother hlo DM, Father h/o HTN
SH: !!.? ppd cigarettes for 5 years, 1-2 alcoholic beverages per
day, 3-4 alcoholic beverages on the weekends

What would you look for on PE? State that you would:

Perform aPE
Start with vital signs and then move on to a head and neck exam

204
Chapter 4 - Laryngology Otolaryngology Clinica l Case Studies

PE: NoTES

~ .. Temp: 99.0 F. HR: 75. BP: 130180, RR: 10


GA: Well developed thin female in no apparent distress
Ears: TM intact and mobile bilaterally
Nose: Clear anteriorly bilaterally
OC!OP: 2+ symmetric tonsils, FOM soft. BOT soft, no lesions
noted
Ne ck: No lymphadenopathy. normal thyroid
Salivary glands: No masses
Voice: Breathy. frequent pitch breaks, inappropriately loud
for the environment
Neuro: CN 11-XJJ are intact, the patient is alert and appro-
priate
.•. Indirect laryngoscopy: The patient has a sensitive gag reflex, but you can
) tell that both vocal cords are mobile, although the
entire length of the vocal folds is not visible to this
exam

What diagnostics would you request? State that you would:

• Request fiberoptic laryngoscopy- Fiberoptic laryngoscopy: A thorough vi-


sual exam of the glottis is mandatory in the evaluation of the hoarse patient.
If indirect laryngoscopy is insufficient, then often a fiberoptic exam can
provide a better look at the tme vocal fo lds

During fiberoptic laryngoscopy this patient is noted to have small, bilat-


eral, symmetric true vocal fold .masses located ·at the j unction of the ante-
rior and middle thirds of the true vocal fold. They are whitish in appear-
ance. Both vocal cords move normally. There is an "hourglass·· configura-
,_) tion to the glottis during phonation. The larynx is sensate throughout. Epi-
glottis, BOT. false cords and pyriform sinuses appear normal.
)
) What is your differential diagnosis? State that the differential diagnosis includes:
)
• Vocal cord nodules
• Polypoid corditis
Laryngeal polyps
Malignancy
Adult laryngeal papilloma
Granulomas
• lntracordal cyst
) Laryngopharyngeal reflux
)
Diagnosis:
·-·i
State that the diagnosis is bilateral true vocal fold nodules.

205

I )
Otolaryngology Clinical Case Studies Chapter 4- Laryngology

NOTES What diagnostics would you request next?

State that you would request videostroboscopy:


Video stroboscopy: This will help document the patient's current exam
and will allow comparison in the future. Videostroboscopy in this pa-
tient reveals normal symmetry, but with reduced glottic closure due to
an hourglass shaped gap. The mucosal wave is intact and the masses
on the true cords appear to be a focal point of the wave, creating the
"hourglass" appearance
State that 5 important elements of stroboscopy include:
Symmetry of movement
Periodicity of vibration
Glottal closure
Amplitude of vocal folds
- Mucosal wave

Is there a need to perform MSL, acoustic analysis, or CT scan? State that none
of these tests are indicated at this time. !?r t~e follow~ng reasons:

• MSL - Is not indicated at this time. Should these lesions persist despite a
full 6 months of treatment, or should the appearance of the lesions be-
come more suspicious for malignancy or hemorrhagic polyp, then MSL is
a consideration
Laryngeal EMG- Is useful in detennining the etiology ofTVC immobility,
but is not useful in the diagnosis or management ofTVC nodules
Acoustic analysis -Provides a large amount of voice data, and while it is
useful for research purposes, there is little clinical benefit for the average
patient
CT scan of the neck with contrast- The diagnosis ofTVC nodules is made
by history, PE, and laryngoscopy. CT is not warranted at this time

What are the treatment options and their complications? What would you do
and why?

State that the mainstay of treatment of vocal nodules is speech therapy.


With a compliant patient the vast majority of nodules will resolve
An initial trial of 2-3 months of speech therapy is prudent. The goal of
speech therapy is to improve vocal hygiene and educate the patient regard-
ing vocal abuse/misuse
Immediate surgical resection is not indicated
State that you would treat an~ coexisting GERD/LPR in the following way:
Dietary modifications • .
1. Reduce/eliminate caffeine and alcohol
2. Improve hydration
- PPI. E.g., rabeprazole 20 mg po qd

A late recommendation may be to consider a career change since her work


environment is a continuing risk factor for voice abuse
State that few patients will fail to respond to speech therapy. Surgical re-
moval of nodules is rarely indicated prim· to appropriate speech therapy

206
Chapter 4 - Laryngology Otolaryngology Clinical Case Studies

If nodules do not respond to honest, dedicated therapy, then it puts the NoTEs
diagnosis of nodules into question, and microsurgical examination andre-
moval may be indicated

After 6 months of speech therapy and P P1 treatment, the patient~· hoarse-


ness persists, although her voice has mildly improved. She still has some
roughness and breathiness to her voice. Her fiberoptic exam reveals that
the nodules have decreased in size but are still present. She is interested in
surgical removal.

What would you do next?

State that nodules are removed under MSL using microsurgical instruryJents
• Care is taken to resect only the tiny amount of mucosa required to remove
the nodules and not disturb the underlying lamina propria

How wo~ld you follow this individual?

State that postoperative voice rest and reflux prophylaxis are important
mea.sures to promote scar-free healing
Speech therapy is continued postoperatively to assist with vocal hygiene
') Re-evaluation in the clinic with fiberoptic laryngoscopy is recommended 4-
6 weeks postoperatively
Further follow-up visits will be contingent on the patient's progress

References:
.l
·I
1. Gould WJ, SataloffRT, Spiegal JR. Voice surgery. St. Louis: Mosby-Year-
i
book; 1993.
) 2. Buckmire RA, Rosen CA. Vocal polyps and nodules. eMedicine 2003.
'I

/
I www.emedicine.com/ent/topic3 52.htm
~
;

--t

207
Otolaryngology Clinical Case Studies Chapter 4- Laryngology

No:rEs CASE 3 - SPASMODIC DYsPHONIA


Daniel Ratcliff, MD

A 32 year-old female presenls to your clinic complaining of 5 months of an


abnormal voice.

HPJ: state that you would:

Obtain a detailed medical history beginning with a history of present illness

What additional historical information would you seek? State that you would ask:

Is the voice change inte1mittent or constant?


• 1s there any associated dysphagia, odynophagia or dyspnea?
Does the patient have a history of vocal overuse or abuse?
Is there a history of tobacco or alcohol abuse? Does alcohol affect the
quality of the voice?
Is there a strained component to the voice?
• Does singing or laughing ameliorate the symptoms?
• Are the symptoms worse under emotional stress?

The patient reports that her voice has felt strained for the last 5 months-.
She denies any dysphasia, odynophagia or dyspnea. There is no signifi-
cant history of vocal abuse or misuse and she does not_use any alcohol o·r
tobacco. Singing and laughter produce a normal voice while emotional
stress often makes her voice sound even more strained.

PMH: Chronic low back pain, fibrornyalgia


PSH: Open reduction internal fixation of tibia fracture (motor ve-
hicle collision) 6 years ago
Allergies: Darvon~rash ·
Medications: Oral contraceptives, vitamins
FH: COR artery disease, diabetes
SH: D_enies use of tobacco, alcohol, JV-drugs
ROS: Negative (including cough, URI symptoms, post-nasal drip
and GER symptoms)

What would you lool< for on PE? State that you would:

Perform a thorough head and neck exam including vital signs, and a CN exam

PE:

1:11.aLs.: Temp: 98.6 F, BP: 122180, Pulse: 82, RR: 12


GA: Healthy alert WF in NAD. Her voice has a strain-
strangle pattern
HEENT Normal
CN exam: No abnormality noted

208
Chapter 4 - Laryngology Otolaryngology Clinical Case Studies

What diagnostics would you request? NOTES

Because of the patient's strained and strangled sounding voice, she should
undergo a voice evaluation that may include:
Fiberoptic laryngoscopy
Laryngeal stroboscopy
Laryngeal EMG

Test results:

Fiberoptic laryngoscopy
- Abnormal, involuntary co-contraction of the vocalis muscle group
during phonation. Exam appears normal at rest
Laryngeal stroboscopy
Mucosal wave maintained during singing
Laryngeal EMG
- Normal synchronous CAP are noted
Inappropriate activity of the vocalis complex during phonation

Laryngeal EMG may prove helpful to establish the neuromuscular status


. of the larynx, but this mode of evaluation may not be universally avail-
able. EMG information shduld be evaluated for the presence of spontane-
ous compound action potentials, wave form morphology, and recruitment.
The presence of spontaneous activity suggests that the nerve is intact, and
intact morphology of the compound action potentials waveform suggests
. synchronous activity. ·

What is your differential .diagnosis?

·• Adductor spasmodic dysphonia


Psychogenic dysphonia
Essential tremor
• Parkinson's disease
Presby larynges
• Myoclonus
Stuttering

Diagnosis:

Adductor spasmodic dysphonia

What are the treatment options and their complications? What would you do
and why?

Botox injection - Temporary and each injection is associated with a period


of breathiness and dysphonia, which may last for a week or 2 prior to the
start of symptomatic relief. Typically, this form of therapy must be repeated
every few months. Needle placement for injection can be guided through
the use oflaryngeal EMG

209
Otolaryngology Clinical Case Studies Chapter 4 - Laryngology

NOTES Selective adductor denervation - Can result in aberrant reinnervation of the


adductor muscles with simi lar or worse voice dysfunction:
Recurrent laryngeal nerve section is considered by many to represent
an obsolete fonn of therapy for spasmodic dysphonia
A modification of this procedure has been described that involves se-
lective section of nerve branches to the thyroarytenoid muscle. Resu Its
from this fonn of therapy remain prelimina1y
Thyroplasty (lshiki Type Il) has been suggested for patients who expe~ience
the abductor fon11 of spasmodic dysphonia. This procedure would provide
little relieffor this patient, who is experiencing adductor spasmodic dysphonia

Treatment:

Injections are made into I or both vocal cords in I sitting depending on practitioner
preference. Localization of needle into the thyroarytenoid muscle is by either
direct visualization (flexible laryngoscope) or by EMG guidance.

How would you follow this individual?

Initially the effects ofBotox injection are noticed between 4-10 days after
injection
The benefit of injections generally lasts about I0 weeks, but the response
can be highly variable from patient to patient

References:

I. Blitzer A, Brin MF, Stewart CF. Botulinum toxin management of spasmodic


dysphonia (laryngeal dystonia): a 12-year experience in more than 900 pa-
tients. Laryngoscope. 1998; l 08:1435--41.
2. Bielamowicz S, SquireS, Ludlow CL. Assessment ofposterior cricoarytenoid
botulinum toxin injections in patients with abductor spasmodic dysphonia.
Ann Otol Rhino! Laryngol. 2001;110:406-12.
3. lsshiki N, Haji T, Yamamoto Y, et al. Thyroplasty for adductor spasmodic
dysphonia: further experiences. Laryngoscope. 2001; 111:61 5-21.
4. Dejonckere PH. Perceptual and laboratory assessment of dysphonia.
Otolaryngol Clin North Am. 2000;33(4):731-50.
5. Blitzer A, Brin MF. Evaluation and management of hyperfunctional disor-
ders. In: Cummings CW, Fredrickson JM, Harker LA, Krause CJ, Schuller
DE, Richardson MA, eds. Otolaryngology- head and neck surgery. 3'd ed.
St. Louis: Mosby-Yearbook; 1998.

210
Chapter 4 - Laryngology Otolaryngology Clinical Case Studies

CASE 4- LARYNGOTRACHEAL TRAUMA NoTES


Alan D. Murray, MD

A 12 year-old girl sustained a fall and complains of neck pain.

HPI: state that you would ask:

How did the child fall?


What part of the body did the child strike?
When did her neck pain start?
Where is her neck pain located and what movements precipitate it or make
it worse?
Does she have any breathing difficulties, shortness of breath, or noisy
breathing?
How is her voice quality?
• Has the child had any swallowing difficulties?
Are any of the symptoms worsening?

Both the parents and the child report that she was swinging on the monkey
bars at school when she slipped and hit the front of her neck on the bars.
Her neck pain started immediately. The pain is not worsening, but she has
point tenderness over the front of hei- neck, especially over her thyroid
cartilage. Moving the soft ·tissues of her neck up and down exacerbates
her pain. She doesn i complain of breathing difficulties, but seems to be
mildly short of breath. She is hoarse, and mom states that her hoarseness
is not normal. She denies pain with swallowing.

PMH: Normal delivery, FT No history of intubation at birth.


SurgHx: None
Hasp: None
)
I
FH: No significant contributing FH. specifically no anesthesia
') problems
Allergies: NKDA·
Medications: .None
. I
..1
SH: Lives with parents, older brother
ROS: No other problem

j What would you look for on PE? State that you would:

Perform a complete head and neck exam including vital signs


} PE:

11t.CJ.is.: Temp: 98.6 F. BP: 120170, Pulse: 100, RR: 30


GA: She has biphasic stridor and appears to be working hard
to breathe. She sounds hoarse, but she does not complain
of subjective airway distress
HEENT She has point tenderness over her thyroid notch. There is
no erythema of the anterior neck, but crepitus is noted dur-
ing palpation. Crepitus stops at the clavicle and does not
extend up into the face
211
Otolaryngology Clinical Case Studies Chapter 4- Laryngology

NOTES What diagnostics would you request? State that you would:

• Determine patient's oxygen saturation


Consider a flexible fiberoptic examination
Plan to schedule the child for a direct laryngoscopy and bronchoscopy un-
der general anesthesia. If a flexible laryngoscopy was not perfonned ini-
tially, this could be done prior to induction of anesthesia at the time of the
operative procedure. The child is not in extremis, but has an airway injury
that needs to be evaluated and there needs to be airway control
High kilovolt plain fi lm of the neck may be useful to help in obtaining
more infmmation on the nmTowing present in the subglottic larynx or
mass effect from hematoma
CT scan of the larynx would be helpful to evaluate the cartilaginous
structure of the airway, but CT scan can be dangerous if the airway- is
tenuous
An evaluation of the C-spine is needed and is ideally perfonned using flex-
ion/extension films. ACT scan of the C-spine is considered more accurate
than cervical x-rays. Radiographic studies are dependent on the patient's
condition; if there is respiratory difficulty, the studies may need.to be de-
ferred. If appropriate C-spine studies cannot be obtained, and patient should
managed as if she had a C-spine .injury

Test results:

• 02 saturation: 97% on room air


CT scan: Normal appearing supraglottic and glottic larynx. No masses
or lesions nOled. Narrowing of subglottic larynx is noted. No other
airway abnormalities are seen. Narrowing determined to be 75%. Be-
cause cartilage is less well defined in small children, cartilaginous
fractures may be difficult to visualize. Consequently, some experts ad-
vise against relying on CT for diagnosis
Flexible fiberoptic exam demonstrates decreased mobility of both vo-
cal cords with poor abduction. (A flexible fiberoptic examination . may
be somewhat controversial, particularly if child is in some extremis.
This particular child is older, has good saturations, so the risk is low.
Another concern is the fact that the child has crepitus, a result of some
airway injury, and coughing and struggling with a flexible exam could
further compromise the airway by increasing the amount of subcuta-
neous emphysema.) The child is stable and breathing comfortably.
Flexible laryngoscopy demonstrated reduced vocal cord mobility bi-
laterally, so the suspicion for ainvay separation is increased

What is your differential diagnosis?

Airway fracture
Airway foreign body
• Subglottic edema
Subglottic stenosis (congenital or acquired)
Respiratory papillomatosis

212
Chapter 4 - Laryngology Otolaryngology Cl inical Case Studies

Subglottic/tracheal neoplasms NoTES


Tracheal anomalies (stenosis, malacia. compression)

What would you recommend next and why?

Intra-operative evaluation of the airway followed, if necessary, by open or


closed operative repair and possible tracheotomy

How would you perform the procedure?

Explain to the parents the need for laryngoscopy and bronchoscopy under
anesthesia

Have instruments available in OR for emergent tracheotomy. If there is any


question abour worsening of the air.-vay, a tracheotomy under local is per-
formed, but on children it can be difficult because children have dijjiculty
cooperating.

Explain to the parents that there is a very high likelihood that a tracheotomy
wilt be needed for ajrway control along with further surgery to repair an
mrway InJUry
Talk with the anesthesiologist prior to the procedure and explain what you
'plan to do
Do not attempt endotracheal intubation unless it is a life or death emergency
• If cervical x-rays were nQ.t performed, the patient needs to be treated as
though she had a cervical injury when performing laryngoscopy
Remember that an esophageal injury may be associated with her airway
injury. Plan for rigid esophagoscopy at the time of the procedure

The options for airway control include:


• Placement of a bronchoscope
Intubation
Tracheotomy
0 0

Placement of a bronchoscope through the edem·atous subglottic area would result


in control of the airway, but only a small diameter bronchoscope could be used.

Intubation through the subglottic narrowing would be dangerous since one would
not be able to visualize the tube passing distal to the point of separation. Be-
cause the tube is curved, an endotracheal tube can pass out of the area of
separation and create a false passage.
9

A.t racheotomy would result in.the most stable airway.

How would you proceed?

Perform a tracheotomy under local anesthesia


The child should be kept spontaneously breathing - No paralytic agents
should be used
Use topical anesthetics on larynx and trachea prior to insertion of the bron-
choscope to minimize stimulation and movement

213
Otolaryngology Clinical Case Studies Chapter 4- Laryngology

NOTES When the chi ld is adequately anesthetized, insert the laryngoscope to ex-
pose the larynx and insert the bronchoscope
If the narrowing is too narrow, may need to decrease the scope size or use
a telescope only
Measure the airway diameter with an endotracheal tube and check for a
leak to detem1ine the extent of the nmTowing

What would you do next?

Results of airway evaluation: Initial lm:vngoscopic evaluation demonstrates


subglottic narrowing. There is concern about airway separation because
of the crepitus noted on exam.

Tracheotomy is completed.

Current literature recommends repair of the a irway trauma within 24 hours of


injury ifthe patient's overall condition permits since success is greater in imme-
diate as compared to delayed repair.

State that you would explore this child's neck for an airway fracture.

If an airway separation is identified, primary closure of the separation with


interrupted absorbable sutures and outer stay sutures (to help further anchor the
closure) should be primary treatment. If a laryngeal fracture is present (thyroid
aJa, for example), reapproximate the fractures via suture, plating (absorbable)
or wiring (adult patients). If a laryngeal comminuted fracture exists, stent place-
ment with stent removal in 2-4 weeks is usually recommended. Keep the tra-
cheotomy in place until the stent is removed and allow for the airway to "settle
down" over 3-4 weeks prior to de_cannulation.

Describe potential postoperative complications:

• Granulation tissue
• . Airway stenosis
Vocal cord paralysis

Potential postoperative complications include granulation tissue development


and stenosis at site of repair. Granulation tissue may develop at the anastomotic
site or where the mucosa was denuded. Use of the C02 laser may aid in re-
moval of the granulation tissue. If stenosis persists after repair, then dilation of
the airway, laser treatment of the scar with dilation, and/or laryngotracheal re-
constr Jction may be needed once the scar has matured.

Patients with airway separation are at risk for vocal cord paralysis due to dam-
age to the recurrent nerves. lf there is unilateral vocal cord paralysis, wait for
compensation ofthe contralateral vocal cord. If permanent, bilateral vocal cord
para 'ysis accompanies the injury, arytenoidectomy or laser cordotomy may be
needed to create an adequate airway.

214
Chapter 4 - Laryngology Otolaryngology Clinical Case Studies

How would you follow this individual? State that you would: N oTES

Keep the tracheotomy in place until the ai1way is stable and edema has
resolved
• Perform serial bronchoscopies to evaluate the airway over time. When the
airway appears stable, consider decannulation. If there is any concern that
decannulation may not be successful, consider a sleep study with the tra-
cheotomy tube capped
Admit the child to the hospital after decannulation. Keep extra tracheotomy
tubes of smaller sizes in the room. Use continuous pulse oximetry to assess
air exchange during the hospital stay
lf the vocal cords are inj ured, decannulate only if cord function returns or
procedures to improve the airway have been performed and the patient is
able to tolerate capping of the airway

References :

1. Butler AP, Wood BP, O' Rourke AK et al. Acute external laryngeal trauma:
experience with 112 patients. Ann Otol Rhino) Laryngol. 2005;1 14: 361-8.
2. Schaefer SD, Close LG. Acute management of laryngeal trauma. Ann Otol
Rhino! Laryngol. 1989;98:98-1 04.
... Schaefer SD. The treatment of acute external laryngeal injuries. "State of
~.

the art." Arch Otolaryngo l Head Neck Surg. 1991 ; 11 7:35- 9.


i
4. Willging JP, Myer CM. Blunt pediatric laryngeal trauma. In: The pediatric
airway. Philadelphia: Lippincott Williams & Wilkins; 1995, 181- 94.
)-
)

)
)
)
.,

,.)

·I
/
'

)
'

)
~

;:.
I
I
'

!
"

·'

_, :
2 15

-)
Otolaryngology Clinical Case Studies Chapter 4 - Laryngology

NoTES CASE 5- AcQUIRED SuBGLOTTIC STENOSIS


Larry L. Myers, MD

A 50 year-old woman with a 1 week hist01y of upper respirat01y tract infec-


tion presents with increasing wheezing and shortness of breath on exertion.

HPJ: state that you would:

Obtain a detailed medical history beginning with a history of present illness


• You wou ld specifically ask how long she has been short of breath
• Ask about stridor. Is it inspiratory, expiratory or biphasic
• Ask about any associated voice changes
• Ask if there is any productive or non productive cough
• Ask if there is any fever
• Ask if there is a history of prolonged intubation

Subject reports similar; worsening episodes that occur every year for the
last 3 years. Symptoms improve somewhat after each upper respiratory
tract infection resolves.

Allergies: NKDA
Medications : PPI
PSH: Severe automobile crash 4 years ago. Multiple intubation
attempts at scene of crash were unsuccessful. Patient under-
went emergent tracheotomy at sce.ne. Patient 5:ustained dif-
fuse axonal injury from a closed head inj ury. She was
decannulated after 1 week
PMH: Laryngopharyngeal reflux
FH: Negative
SH: ( - ) Tobacco, ( -) ETOH. real estate agent
ROS: NC

What would you look for on PE? State you would:

• Perform a complete head and neck examination including vital signs and
examination of the larynx

PE:

Temp: 36 C, BP: 134/ 70, HR: 90, RR: 24, 0,


·sat95% -
GA: Nfildly labored breathing using accessory
muscles of respiration. Not cyanotic
HEENT Vertical scar midline neck
Auscultation: Expiratory wheezing at mid to lower neck
Fiberoptic·
laryngoscopy: Soft tissue below glottis, vocal folds bilaterally
mobile

216
Chapter 4- Laryngology Otolaryngology Clinical Case Studies

Figure 1 NoTES

What diagnostics would you request?

To generate a broad differential diagnosis, the use of a mnemonic may be


helpful .
Mnemonic: KITTTENS
K (congenital) . coogential subglottic stenosis, subglottic vascular mal-
formation
I (infectious) bacterial (aerobic/anaerobic), viral, TB, atypical TB,
protozoa
I (inflammatory) subglottic stenosis (acquired), tracheitis
T (tumor) benign (chondroma, fibroma), malignant (carcinoma),
metastatic
T (trauma) foreign body
E (endocrine) thyroid malignancy
) N (neurogenic) unlikely
S (systemic) Wegener's granulomatosis, sarcoidosis, autoimmune
disorders
)

After establishing that patient does not have an acute airway, then pro-
ceed with work-up to fully evaluate lower airway.

ACT scan of the neck should be obtained next. This will demonstrate the
region of the stenosis and determine if there is any extrinsic compression of
the trachea
A lateral neck x-ray may be used if aCT scanner is unavailable

217

,/ '
Otolaryngology Clinical Case Studies Chapter 4- Laryngology

NOTES Test results:

Laboratory studies are all WNL.


CT scan of the neck demonstrates an area of narrowing within the tra-
chea. This area of stenosis begins at the level of first tracheal ring and
appears to extend in a circumferential fashion for a length of approxi-
mately 2 em. At its narrowest point, the stenosis measures 5 mm.

What are your top 3 differential diagnoses?

• Acquired subglottic stenosis


Neoplasm
Wegener's granulomatosis

Diagnosis:

The diagnosis is acquired subglottic stenosis. State that the acquired sub-
glottic stenosis in this case is secondary to history of emergent, high-placed
tracheotomy. In the case of this patient, her condition is exacerbated by
upper respiratory tract infections and by laryngopharyngeal reflux as a
result of soft tissue edema superimposed upon her fixed stenosis.

What are the treatment options and their complications? What would you do
and why?

Initial treatment for symptomatic subglottic stenosis is airway protection,


supplemental oxygen, steroid administration, and, if available, hdiox (he-
lium-oxygen mixture intended to decrease viscosity of inspired gas in order
to ease br~athing)
Elective surgical treatment is individualized and is indicated when a symp-
tomatic patient does not respond to supportive therapy
Treatment may also include tracheotomy tube placement
Tracheal dilation is condemned because it may qenude mucosa and worsen
stenosis

Discuss the surgical options for subglottic stenosis:

Endoscopic: Endoscopic radial laser incisions


Open: Hyoid arch transposition, tracheal resection, cricoid excision with
thyrotracheal anastomosis
Highlights (tracheal resection':
Anterior neck approach avoiding injury to recurrent laryngeal nerves
Resect diseased subglottic segment of trachea
Tracheal mobilization and laryngeal release techniques required for gaps
greater than 3 em
Primary .anastomosis with monofilament suture
Pitfalls:
Recurrent laryngeal nerve injury
Anastomotic leak

218
Chapter 4 - Laryngology Otolaryngology Clinical Case Studies

Potential complications of surgery: NOTES

Chronic subglottic stenosis


Recurrent laryngeal nerve injury

How would you follow this individual?

The patient is followed by monthly flexib le laryngoscopies to assess sub-


glottic region
Post-operative care must include treatment of laryngopharyngeal reflux
Treatment consists of:
PPI therapy
Diet modification
Avoidance of foods/habits known to reduce tone of lower esoph-
ageal sph incter (acidic and spicy foods, tobacco, alcohol)
Weight reduction

219
Otolaryngology Clinical Case Studies Chapter 4 - Laryngology

NoTES

220
Chapter 5 - Pediatric Otolaryngology Clinical Case Studies

CHAPTER 5/CASE 1 - STRIDOR NOTES


Romaine F. Johnson, MD

A 3 month-old male with persiszent noisy breathing.

HPI: state that you would:

Obtain a more detailed history, beginning with chief complaint

What additional information would you ask? State that you would ask:

Describe the stridor. Inspiratory, expiratory, or both?


Obtain birth history. Was the child born immature?
What factors affect the symptoms? Position, feeding, activity?
• ·Have there 'been episodes of cyanosis or extreme distress?
Any other medical problems?
Any problems with reflux?
Any feeding difficulties, sign of aspiration?
Is the chi ld gaining weight?
How well does the child sleep?

The family noted noisy breathing shortly after they brought the baby home.
He is especially noisy when sleeping or lying flat. He also has increased
noisy breathing when he eats. He has not had any episodes of cyanosis or
). apneas. He is gaining weight but does occasionally have difficulty feed-
ing. He does rejlux his baby formula often but the parents do not think he
aspirates or chokes on his food.

. PMH!birth: FT without any complications or prolonged hospitalizations


PSH: Circumcision
All: NKDA, NKFA
~)
Medication: None
·FHISH: Lives at home with family. No pets. No smoke exposure
.,
ROS: Negative
:J
What would you look for on PE?

State that you would perfonn a complete head and neck examination in-

cluding flexible laryngoscopy
)

). PE:
)
.Y.i1fJJ.s_: Temp: 98.6 F. RR: 20, Wt: 5 kg, Ht: 60 em
GA: Well developed male with some mild inspiratory stridor at rest
Ears: EAC and TM clear
Nose: Septum midline, mucosa pink, and no lesions
OCIOP: No lesions
Neck: Supple, no masses, lymphadenopathy, or thyromegaZv
NPL: NP clear, larynx shows foreshortened aryepig lottie folds

221

)
Otolaryngology Clinical Case Studies Chapter 5- Pediatric

NcnEs with some arytenoid prolapse. The mytenoids also appear


erythematous and edematous. TVC are mobile bilaterally. You do
nore that the stridor becomes more pronounced with the child 's
agitation during the examination

What diagnostic test would you order?

May consider plain films of the neck to assess the airway or CXR

What is your differential diagnosis?

• Laryngomalacia - This history and PE is most consistent with laryngomalacia.


Laryngomalacia is the most common laryngeal disorder seen in infants. It is
diagnosed by history and PE, especially flexible laryngoscopy. Lmyngos-
copy can show a "tubular" or "infantile" larynx. The stridor is produced
because supraglottic structures (e.g., the arytenoids) prolapse into the air-
way on inspiration. The exact etiology oflaryngomalacia is unknown but is
thought to be secondary to immaturity ofthe cartilaginous framework of the
larynx .
GERD- Reflux exacerbates laryngomalacia in many instances and may be
the primary offender in some. This patient exhibits some signs of reflux;
specifically, the posterior glottic area showed edema and erythema
Secondary airway lesion (e.g., tracheomalacia)- A secondary ai·rway lesion
may be present in a small proportion of patients with laryngomalacia and
should therefore be kept in the differential

What diagnostics would you order? ..

No further tests are needed if the case represents a classic presentation of


laryngomalacia that is mild in nature
Direct laryngoscopy and bronchoscopy should be done if surgery is being
considered and to detect a secondary lesion

What would you do next?

Order medical therapy and clinical observation for this mild case of
laryngomalacia. Medical therapy would consist of a trial of anti-reflux medi-
cation (e.g., PPI)
• Surgery would be indicated if the symptomatology was more severe; e.g.,
failure to thrive

Surgical technique:

Supraglottoplasty is the most commonly used techn ique to treat


laryngomalacia. The technique involves releasing the AE fo ld, which tends
to be short. This approach helps to prevent the epiglottis from prolapsing
into the airway. Additionally, many practitioners also remove the cuneiform
carti lage. This maneuver helps to reduce arytenoid prolapse. The surgery is
done via suspension laryngoscopy. Bronchoscopy is done first to rule out a
secondary airway lesion. The releasing of the AE folds and removal of the

222
Chapter 5 - Pediatric Otolaryngology Clinical Case Studies

cuneif01m cmtilage can be accomplished with microlaryngeal scissors or NOTES


the C02 laser. Patients are observed overnight in the hospital, typically in
an JCU or step-down unit. Some surgeons leave the patient intubated over-
night, although many do not
Temporary tracheotomy has been discussed in the past, but it is almost
never used today

What are the potential complications?

• Thankfully complications are rare and usual ly minor. Temporary airway


swelling resulting in upper airway obstruction is possible~ Intraoperative
steroids are employed to prevent this occurrence
Aspiration can result as well. This complication is also rare and if it occurs
is usually temporary. Severe cases of permanent aspiration are very rare
Persistent laryngomalacia is perhaps the most common complication, re-
sulting in the need for further management. In these cases it is important to
perform flexible laryngoscopy and determine which laryngeal structures
are still prolapsing into the airway and then address them surgically. Also, a
secondary lesion must be considered

How would you follow this individual?

Follow-up examination in several weeks for medical management and 1-


week for surgical management. If successful, the patient should have sig-
')
I
nificant improvement in symptoms. Flexible laryngoscopy is done to evalu-
ate the larynx; however, the clinical history is the most important aspect of
follow-up management

) References:
)
1. Prescott CA. The current status of corrective surgery for laryngomalacia.
Am J Otolaryngol. 1991 ;2(2):230-5 .
2: Holinger LD. Etiology of stridor in the neonate, infant and child. Ann Otol
) Rhino) Lal)'ngol. 1980;89:397-400.
•.
)

,,)

..))

223

)
Otolaryngology Clinical Case Studies Chapter 5 - Pediatric

NoTES CASE 2- SuBGLOTTIC STENOSIS

Alan D. Murray, MD

A 2 momh-old child is referred for evaluation of noisy breathing.

HPI: state that you would ask:

Has the noisy breathing been recent or lifelong?


Is the noisy breathing worsening?
ls it worse when awake or asleep?
• Are there times or activities during which the noisy breathing is worse?
Does the child have difficu lty eating?
• Is there a histmy of intubation? .
What type of noise occurs (high pitched noise, inspiratory or expiratory)?
~ ·Any recent colds?
• Any apneic spells or cyanosis?

The parents report that the noisy breathing has developed after being dis-
charged from the hospital and has worsened since. The noise occurs both
awake and asleep, but is somewhat better when asleep. The noise does
worsen when crying, and it appears to be with both inspiration and expi-
ration. The child has no eating difficulties, but over the past several days,
the child :S appetite has decreased. There have been no apneic spells, nor
any cyanotic spells. The child does not have a cold.

PMH: Born 32 weeks premature and intubated at birth for respiratory


distress. The child remained intubated for 2 weeks, and failed
extubation once. The child was successfully extubated after 4
weeks. The child was discharged after 5 weeks. The child has had
no other health problems and no operations

The child had stridor after the first attempt at extubation and it lasted
about 6 hours before getting reintubated.

Allergies: NKDA
Medications: None
FH: Negative
SH: Lives with parents, one older sibling, and no other health
problems
ROS: No additional information

What would you look for on PE?

Perfom1 a comp lete head and neck exam, including vital signs

224
Chapter 5 - Pediatric Otolaryngology Clinical Case Studies

NOTES

Vitals: Temp: 98.6 F. BP: 100/60, Pulse: 100, RR: 28


GA: Child with biphasic strid01~ but in no distress, no retrac-
tions, strong cry, no hoarseness
HEEN T Normal ENT external exam
Additional
information
with exam: Does the child have any lesions on the body (i. e.. heman-
giomas) ?

What diagnostics would you request?

High kilovolt plain film of the neck may be useful to help in obtaining more
infonnation on the narrowing present in the subglottic larynx and may also
hint towards a hemangioma
Check oxygen saturation
Flexible fiberoptic examination. (Flexible exam in office may be somewhat
controversial, particularly if child is in some extremis. Exam would be per-
fanned only to assess the· glottis and subglottis. Some pediatric otolaryn-
gologists would not perform a flexible exam. in the office)

Test results:

02 saturation 97% on room air


Flexible fiberoptic exam done in office: mobile vocal cords bilaterally,
no laryngomalacia

What is your differential diagnosis? Differential diagnosis should include:

• Subglottic stenosis (congenital or acquired)


Subglottic hemangioma or other neoplasms
• Subglottic cysts
• Tracheal anomalies (stenosis, malacia, compression)

What would you do next? State that you would:

} Plan to schedule the child for a direct laryngoscopy and bronchoscopy un-
)
der general anesthesia. If a flexible laryngoscopy was not performed ini-
tially, it could be done prior to induction of anesthesia at this time. The
)
immediate problem is to detennine if the child is having worsening respira-
') tory difficulty and needs to be evaluated now' or if she is stable and the
evaluation can be scheduled in the near future. Since the child is not in
extremis, the procedure does not have to be done emergently, but will need
to be done soon (in the next week).

How would you perform the procedure?

Would schedule the child for a possible overnight stay and possible ICU
admission

225

)
Otolaryngology Clinical Case Studies Chapter 5- Pediatric

NoTEs Explain to the parents that intubation or even tracheostomy may be needed
post-operatively if swelling occurs
Talk with the anesthesiologist prior to the procedure and explain what you
plan to do
Keep the child spontaneously breathing- No paralytic agents shou ld be
used
Apply topical anesthetics to the bronchal mucosa prior to inset1ion of the
bronchoscope
When the child is adequately anesthetized, insert the laryngoscope to ex-
pose the larynx and then insert the bronchoscope
lf the narrowing is too severe, may need to decrease the scope size or use
a telescope only
Insert endotracheal tubes of increasing size until there is no leak to provide
an objective estimate of actual airway diameter

Test results:

From direct laryngoscopy and bronchoscopy:


Normal appearing supraglottic and glottic larynx. No masses or le-
sions noted. Noted narrowing of subglottic larynx, no m_asses obstruct-
ing. No other airway abnormalities. Narrowing determined to be 70%

What are the treatment options and their complications? What would you do
and why? ·

The child is ·doing well and has no swelling, but is still asleep in OR and
has been found to have 70% stenosis. Parents gave a history of worsening
noisy breathing, so the options at the present include:

Tracheotomy- The benefit of a tracheotomy is a controlled airway. Risks


include bleeding and pneumothorax perioperatively and, long term, trache-
otomy plugging with airway obstruction. Laryngotracheal reconstruction
could be planned for later
Other optioJ:.lS such as dilation would only be temporary and may make
things worse - not a recommended option. Laser procedures also may
produce more scarring, but if it is a thin web, there may be some consider-
ation of a reasonable argument for quadrant laser resection with dilation.
Controversial option is the use of Mitomycin C applied to the treated area
• Immediate laryngotracheal reconstruction. Candidates for surgery include
children with subglottic pathology. Preferably the children are over 2 years
of age, and Cotton's study describes a higher success rate if they are over
4 years old with a grade 3 or better stenosis. Good candidates should not
have any respiratory disease and reflux must be under control. The decision
as to whether to undergo " single stage" reconstruction versus 2-stage re-
construction is determined by several factors:
Single stage reconstruction requires post-op intubation for 7-14 days,
depending on whether anterior grafting or anterior posterior grafting is
performed (7 for ante1ior only and 14 for anterior/posterior). Grafting
material is typically costal cartilage, but auricular cartilage can be used

226
Chapter 5 - Pediatric Otolaryngology Clinical Case Studies

in Grade I and II stenosis. The usual recommendation is to extubate NOTES


when air leak develops. The child should be observed in the ICU for at
least 24 hours after extubation and be kept in the hospital for 2-3 days
afterwards
A 2 stage reconstruction means keeping the tracheotomy in place until
the stent is removed. Typically recommended in patients who have a
more severe stenosis (Grade III or IV). The stent is left in place 2-4
weeks
A partial cricotracheal resection is recommended for patients with severe
Grade Ill and IV stenosis. With this patient's findings, it would not be a
first option for treatment
Watchful waiting may work temporarily, but if the child continues to worsen,
or obtains an URI that exacerbates the stenosis, then an emergent situation
may occur later

Surgical technique:

Highlights and pitfalls:


• Use a vertical incision in young children
Many children require subcutaneous fat removal to expose the anterior
strap muscles
The strap muscles are divided in the midline through the midline rap~e
The thyroid isthmus is typically identified: its superior portion is at or near
the cricoid cartilage
The isthmus is divided to allow for exposure of the tracheal surface
Stay sutures are placed laterally, at about tracheal rings 2-4
A vertical incision is made in the trachea, and an age-appropriate trache-
otomy tube is placed
The tracheotomy tube is secured with tracheotomy twill ties and not su·-
tured to the neck (the neonatal and pediatric tracheostomy tubes do not
have an inner cannula, and unsutured allows for easy removal, particularly
in an emerg~nt situation) .
The stay sutures are labeled left and right and are used if the tube falls out.
The first tracheotomy tube change is on day 5
Criteria for anterior cricoid split would include children with isolated sub-
glottic pathology, no oxygen requirement, no reactive airway disease or
GER. Depending on the size of the child, the recommended length of intu-
bation is 7 days if over 2500 grams, and 14 days if under 2500 grams in
weight. The success rate is dependent on the severity of narrowing, but in
general is about 70% in extubati_on and not needing a tracheotomy. Compli-
cations of an anterior cricoid split are similar to tracheotomy, with the addi-
tional problem that failed extubation after split may require reintubation and
possible tracheotomy

227
Otolaryngology Clinical Case Studies Chapter 5 - Pediatric

NOTES How would you follow this individual?

After tracheotomy:
The child would need tracheotomy care in the hospital and at home. At least
two caregivers need to have training in care of the tracheotomy and in
CPR. Depending on the severity of the child 's health status, there may be a
need for home health nursing
Schedule periodic office visits to check for wound breakdown
Serial bronchoscopies should be performed on a 6-1 2 month basis to evalu-
ate granuloma fom1ation or changes to the airway caliber
After cricoid split: Plan for extubation in the ICU if the child has an appro-
priate air leak around endotracheal tube (air leak less than 25 em H20).
Prescribe steroids 24 hours before and after extubation. Re-evaluate the
airway via direct laryngoscopy and bronchoscopy after 7 days, and if the
airway is patent, one can either plan for extubation in OR or return to the
ICU for extubation when off sedating medications. Plan for the child to stay
in ICU at least 24 hours after extubation to evaluate for airway edema and
need for reintubation. PO feeds can begin 24 hours after extubation if the
child is doing well. lf stridor returns after a course of steroids, additional
steroid should be given followed by a slower taper. If stridor continues
repeat direct laryngoscopy and bronchoscopy. If restenosis occurs, the child
may need tracheotomy placement or a repeat cricoid split with anter1or
graft placement
·Plan for follow-up direct laryngoscopy and bronchoscopy about 3-4 weeks
after extubation to reassess airway
Unless symptomatic, there is no need for further follow-up

References:

1. Cotton RT, Myers CM III, Bratcher GO, et al. Anterior cricoid spiit, 1977-
1987. Evolution of a technique. Arch Otolaryngo l Head Neck Surg.
1988;114:1300-2.
2. Gustafson LM, Hartley BE, Liu JH, et al. Single stage laryngotracheal re-
construction in children: a review of 200· cases. Otolaryngol Head Neck
Surg. 2000;127:289-92.
3. Hartley BE, Gustafson LM, Liu JH, et al. Duration of stenting in single-
stage laryngotracheal reconstruction with anterior costal cartilage grafts.
Ann Otol Rhinal Laryngol. 2001 ;110:413-6.
4. Bolinger LD. Diagnostic endoscopy of the pediatric airway. Laryngoscope.
1989;99:346-8.
ii. Silva AB, Lusk RP, Muptz HR. Update of the use of auricular cartilage in
·laryngotracheal reconstruction. Ann Otol Rhino! Laryngol. 2000; 109:343-7.

228
Chapter 5- Pediatric Otolaryngology Clinical Case Studies

CASE 3 - FoREIGN BoDY IN THE AIRWAY NoTES


Alan D. Murray, MD

A 2 year-old with recent onset of cough.

IIPI: state that you would ask:

• How long has the cough been present?


ls there an associated viral infection?
Are other members of the family ill?
ls there a history of respiratory difficulty in the past? Asthma history?
• Is there a hist01y of choking?
Has the child had any cyanotic/apneic episodes recently?
Has the child had a noisy breathing history or any breathing difficulty?

The parents report that the child had been well until today, when he had
what sounded like "coughing or choking" after running around the house.
Since that point 2 hours ago, his cough seems to have gotten beller but still
persists. No one else in the house is sick. The child has had colds in the
past, but has never been diagnosed with asthma. He has not had any cy-
anotic/apneic spells and does not have noisy breathing episodes. He was
breathing somewhat ''hard" initially but is getting better. He has no runny
nose or feveJ:

Questions to ask about the child's PMH include:

History of intubation?
• History of any previous airway problems?
• History of previous surg~ries and hospitalizations?
History of asthma or other respiratory diseases?
Was the child choking on something or eating at the time the coughing
began?
• Are there other children at home?
-r
PMH: The child was a normal term infant with no perinatal prob-
lems. The child has never had surgery and never been hos-
.'
pitalized. The parents state that the child was not eating at
the time. The child has two older siblings
)
Allergies: NKDA
Medications: None ..
FH: Negative
SH: Lives at home ·with parents
ROS: No additional information

What would you look for in aPE?

State that you would perform a complete head and neck exam on the child
with vital signs

229
Otolaryngology Clinical Case Studies Chapter 5 - Pediatric

NoTES PE:

Vitals: Temp: 99.0 F, BP: 100160, Pulse: JJO, RJ?: 20


GA: Child resting in moms lap with no work of breathing. No
retractions and no hoarseness
HEENT Normal ENT exam. No nasal congestion

What would you do further in the PE?

Perfonn a heart and lung exam


Would you do a flexible fiberoptic exam?

Heart auscultation is normal. Chest auscultation demonstrates decreased


breath sounds over right lung fields and expiratmy wheezing. Leji lung
jields were cleaT~ Flexible fiberoptic exam _would not produce any useful
additional information mid would not be recommended

What diagnostics would you recommend at this point?

CXR: Would do a PA and lateral exam (discuss the need for a preoperative
CXR and what you would see on the CXR Also can discuss other types of
radiographic studies such as lateral decubitus films and inspiratory-expira-
tory films, with their benefits)
Check the oxygen saturation

Discuss at this point that radiographic studies are an adjunct to the history and
physical findings in this case and not the sole decision-making tooL

Test results:

The CXR findings demonstrate no abnormalities and no foreign body


identified
Inspiratory/expiratory films would demonstrate air trapping with lack
of deflation of the right lung field if the foreign body is partially ob-
structing the bronchus, but it still may be inconqlusive. With the history
presented, an additional CXR would not add to determining the diagnosis
Child's oxygen saturation is 94% on room air

What is your diagnosis at this point?

Airway foreign body in the right bronchus.

The history of the unwitnessed choking episode together with the PE find-
ings of decreased breath sounds•on the right side lead to. the diagnosis,
even with a normal CXR. Other diagnoses such as asthma or URI would be
less likely. Asthma is characterized by bilateral, not unilateral wheezing.
An URI typically does not have such an abrupt onset and the child does
not exhibit any other signs/symptoms of an URI.

What is the next plan in treatment?

Schedule for direct laryngoscopy and bronchoscopy with foreign body removal
under general anesthesia

230
Chapter 5 - Pediatric Otolaryngology Clinical Case Studies

If the child is in distress, then the surgery should be performed as soon as NOTES
possible. Generally, the child should be NPO for appropriate period of time
(generally 6-8 hours for solids and 2 hours for clear liquids) prior to induction of
anesthesia. This child is not in respiratory distress, but is exhibiting a decrease in
oxygen desaturation at rest. Consequently, the procedure should be perfonned
once the child has been NPO an appropriate period of time. The child will need
to continue to be monitored until surgery, in the event the child's respiratory
status worsens.

How would you plan for the pr.ocedure?

Need to obtain consent from the family and to discuss the risks, including
bleeding, infection, injury to the airway with resultant pneumothorax neces-
sitating intubation, ai1way swelling requiring intubation, and tooth, gum or lip
illJUry
Obtain instrumentation for the procedure. Need age .approp1iate laryngo-
scopes and bronchoscopes. Optical forceps with telescope on the table,
with additional non-optical forceps readily available. Equipment needs to be
in working order and the availability of adequate equipment needs to be
verified before the child comes to the OR. A second bronchoscope needs to
be available and ready to use quickly. It is preferable to have one size
bronchoscope smaller and larger available
Discuss the surgical plan with the anesthesiologist involved with the case.
He/she needs to be on "the same page"

Surgical technique:

Highlights and pitfalls:


The child should be kept spontaneously b:reathing throughout the procedure.
NO paralytic agents should be used
With the child spontaneously breathing and after lidocaine has been applied to
) the larynx, an age-appropriate laryngoscope is placed to expose the larynx
• An age-appropriate bronchoscope is inserted through the larynx into .the
trachea, and the child is then ventilated through the bronchoscope. The
bronchoscope is passed to the carina and the obstructed bronchus is identi- .
fied. The opposite bronchus is examined first. If there are secretions sur-
rounding the foreign body, they are suctioned
The tip of the bronchoscope is placed immediately above the foreign body.
'
,I
Optical forceps are placed through the bronchoscope and, with the bron-
) choscope in position above the foreign body, the forceps are placed around
) the object. The bronchoscope is advanced towards the foreign body, so that
' the forceps and bronchoscope are "one unif" The object,' forceps and bron-
choscope are removed as a unit
) If wedged distally in airway secondary to edema, the procedure should be
·'
I stopped, the child intubated, given steroids for 48-72 hours, and returned to
OR for second attempt at removal
If unable to remove through larynx, perform a tracheotomy and remove
through the tracheostoma
If foreign body is dropped in the airway and cannot be regrasped easily and
quickly, and if the child is developing respiratory distress, the foreign body

231

)
Otolaryngology Clinical Case Studies Chapter 5- Pediatric

NoTES should be pushed back into same bronchus. Ifforeign body is placed back in
opposite bronchus, the opposite lung could be only functioning lung and
would produce respiratory failure
After the foreign body is out, the bronchoscope is reinserted to reevaluate
the airway

The child underlfvent successful removal of the foreign body in the right
1nain bronchus. The foreign body was a small toy part.

What would you do next?

If you are able to suction all pmulent secretions from the airway, antibiotics
are unnecessary
Steroids should be prescribed to reduce postoperative edema
Monitor the child postoperatively at least 2-4 hours. If the child is doing
well and meets post-anesthesia criteria for discharge, he/she can be dis-
charged home

What are the potential complications?

Bleeding from granulation


Most bleeding stops, although minimal bleeding can persist in the recov-
ery room. If significant bleeding persists, may need return to OR, but
this is very rare
Airway edema from too big a bronchoscope or a prolonged procedure or
from excessive manipulation. If edema is severe, may peed intubation
Airway traiuna from the foreign body embedded or from instrumentation
Inability to remove the foreign body (foreign body wedged in place distally
or unable to remove through larynx)
Dropping foreign body in airway upon removal
Pneumothorax/pneumomediastinum
May occur if there was traumatic removal of the foreign body or from
injury with the bronchoscope or a sharp in~txument

Follow-up:

If child does well after the procedure, follow-up with primary care 1-2
weeks postoperatively is appropriate
Need to consider incomplete foreign body removal if the child continues to
have persistent atelect-asis or pneumonia

References:
~

l . Murray AD and Wainer DL. Methods in instrumentation for removal of


airway foreign bodies . Operat Tech Otolaryngol Head Neck Surg. 2002; 13:
2-5.
2. Reilly J, Thompson J, MacArthur C, et al. Pediatric aerodigestive foreign
body injuries are complications related to timeliness of diagnosis. Laryngo-
scope. 1997;107: 17- 20.
3. Silva AB, Muntz HR, Clary R. Utility of conventional radiography in the
diagnosis and management of pediatric airway foreign bodies. Ann Otol
Rhino! Lmyngol.l998; 107:834-8.

232
Chapter 5- Pediatric Otolaryngology Clinical Case Studies

CASE 4- CRANIOFACIAL ANOMALY NoTEs


Brett A. Miles, DDS, MD
John E. McClay, MD

You are called to evaluate a 2 day-old infant in the nursery with noisy
breath ing.

What is your first concern in a patient like this?

Is the infant clinically stable?


Is the infant in any distress?

How is that determined?

How does the child look?


Is he tachypnic? Is he hypotensive? Is he tachycardic?
Is he retracting? Suprasternal, subcostal, intracostal?
Does he have nasal flaring?
Is he desaturating?
Has he had a blood gas? What is his pH, C02, 02?
• Has the infant required CPAP or endotracheal intubation?

At present, the infant is stable without significant retractions. He has not


been intubated. They have not done a blood gas but his oxygen satura-
tions have been in the high 90's. He has had some occasional events of
obstruction and desaturation which improved with positioning.

Now what do you do?

After concluding that the infant is stable and without significant upper air-
way obstruction, inform the pediatrician that you will evaluate the infant
that day when time permits, but to please call you immediately if his condi-
tion deteriorated.

When you arrived at the bedside to examine the infant, you would continue
to formulate your hist01y of present illness by asldng such questions as:

Respiratory:

What does the noisy breathing sound like?


!t is snorting or snoring?
It is wheezing or stridor?
When does it occur?
It is intermittent?
Is it positional?
Is it heard on inspiration, expiration, or both (biphasic)?
Does eating make it wor.se?
Has the child stopped breathing?
Has the child had cyanosis?

233
Otolaryngology Clinical Case Studies Chapter 5 - Pediatric

NoTES What is the character of the child's voice?


Is the cry strong?
Does the infant have any reflux-type symptoms. such as spitting up?
Is the noise related to eating?
Does the child cough when be eats?
Have the symptoms been present since bi11h or have they developed since
the child began feeding?
Also, has the child had any nasal secretions?

Birth history:

Were there any problems with the child's birth?


What is his gestational age?
Any perinatal infections?
Any meconium aspiration?
Were their any maternal infections or illnesses prenatally or at birth?
Has he been intubated- Was he on a ventilator?
Is he on any medications?
Does he have any medical problems )mown? Cardiac problems?
Has he had any surgeries?
Any FH of similar symptoms or problems, including nasal airway obstruc-
tion, congenital abnormalities or chromosomal aberrations
How old are the parents? It is important to ask about maternal and paternal
age, as the risk of many chromosomal abnormalities, and congenital syn-
dromes is directly related to parental age

The infant was delivered vaginally; labor was not induced and was un-
eventful. He was 39 weeks and weighed approximately 3~ kilos. There
was no significant prenatal history and these symptoms have been ob-
served intermittently over the last 2 days. The child has not stopped breath-
ing or become cyanotic, but seems to have more difficulty with snoring
type symptoms while on his back. He improves on his stomqch. He has not
had significant nasal secretions and has not been on any merfications for
that. The child has not been intubated and is not currently on nasal CPAP
There is no nas,al airway obstruction. There is no significant FH of nasal
symptoms or congenital anomalies. The parents are in their mid 20 s.

State that you would do a complete PE at this point. What would you look for on
the PE? State that you would:

Listen for the nature of the sound - Strido:r;, sterter


Look for obstructive signs like nasal flaring or suprasternal, mtercostal, or
subcostal retractions
Assess his vital signs: HR, RR, BP, and temperature
Do a complete PE - Craniofacial structure, external ears, nose and eyes.
Perform anterior rhinoscopy by looking into nose with otoscope. Look into
OC. Look at mandible for retrognathia. Auscultate lungs, neck and chest.
Ascultate nose bilaterally to detem1ine airflow and any asymmetry in air-
flow between right and left nares

234
Chapter 5 - Pediatric Otolaryngology Clinical Case Studies

PE: NoTES ·

Vitals: Temp: 36.7 C, Pulse: 115, BP: 95/54, RR: 26, Pulse oxime-
ter 98% oxygen saturation on room air
GA: Breathing comfortably now, mild suprasternal retractions,
no audible stridor while prone. Some mild stertor is noted
Craniofacial: Ears seem slightly low set, there are no prec.,.ricular pits or
tags, there may be some retrognathia and mild hypertelorism,
external nose normal
EACITM: Slightly floppy ear canals. Quick glimpse of tympanic mem-
branes show the right drum looks a little dull
Nose: No congestion, no drainage, no masses intranasally
OCIOP: The mouth appears of normal shape and size. Intraoral
examination reveals a significant u-shaped cleft palate, no
significant tonsillar tissue
Neck: Full range ofmotion, no masses or adenopathy
Chest: Mild biphasic stridor. Lungs clear to auscultation
C/V RRR, no murmurs
Extremities: Normal

There is no clear evidence of airway obstruction or respiratory distress while


prone. However, despite his comfortable appearance, you observe mild su-
prasternal retractions which become much more marked when he lays su-
pine. His oxygen saturation remains 98% and his vital signs are unchanged.
With the infant supine, you continue your exam but quickly notice worsen-
ing subcostal retractions and development of stridorous breathing.

What do you do now because of his change in respiratory status?

·You promptly respond by turning him prone and observing prompt resolu-
tion of his symptoms. You may also try to pull the jaw forward to see if it
improves his breathing.

235
Otolaryngology Clinical Case Studies Chapter 5- Pediatric

NOTES What is your diagnosis?

At this point the retrognathia and cleft palate should make one suspicious
for the Pierre-Robin sequence.

What is Pierre-Robin sequence and why is it called a sequence?

It is a condition defined by a series of anomalies which arise as a conse-


quence of a single malformation. The inciting event is likely mandibular
hypoplasia around the ninth week of gestation, which then displaces the
tongue posteriorly and precludes normal fusing ofthe posterior palate:
- Micrognathia is present in 92% of cases, and is defined as retraction of
the mandibular dental arch 10-12 mm posterior to the maxillary arch
Glossoptosis is present in 70-80% of cases
Cleft palate is usually u-shaped and seen without a cleft lip

What is the etiology of airway obstruction in these patients?

Multifactorial with both anatomic and neuromuscular components


Sher described a classification scheme based on findings during flexible .
fiberoptic nasopharyngoscopy:
- Type 1: Obstruction is due to posterior movement of the tongue con-
tacting the posterior pharyngeal wall (true glossoptosis- 59%)
Type 2: The tongue moves posteriorly but the palate beco!?es sand-
wiched between the tongue and velum, causing nasopharyngeal ob-
struction (21%)
- Type 3: The lateral pharyngeal walls move medially, opposing one an-
other (10%)
Type 4: Pharynx constricts in a sphincteric manner (10%)

Who was ·Pierre Robin?

Pierre Robin, a French dental surgeon, published the first report of an infant born
with cleft palate, micrognathia and glossoptosis in 1926. However, in 1891,
Lannelongue and Menard were actually the first to describe the triad of fmdings in
two infants. Until1974, a child born with this triad was said to have Pierre-Robin
syndrome. Thereafter, the term syndrome was reserved for those embryologic
errors in a which a single etiology caused the simultaneous presence of multiple
anomalies. The term sequence, thus, has come to describe a series of anomalies
that resulted from a cascade of events initiated by a single malformation.

Are there other craniofacial syndromes that have these abnormalities?

Other common disorders associated with or that have the Pierre Robin sequence:
Stickler syndrome
Velocardiofacial syndrome
Treacher-Collins syndrome (mandibulofacial dysostosis)
Nager syndrome
Spondyloepiphyseal dysplasia congenital- these patients have a small chest,
cervical instability that can cause compression of the brainstem and/or the
C-spine and, often have tracheobronchomalacia

236
Chapter 5 - Pediatric Otolaryngology Clinical Case Studies

Is the Pierre-Robin sequence itself associated with other abnormalities? NoTES

Other otolaryngologic manifestations include:


Otitis media in greater than 80% of patients
Auricular anomalies in 75% of patients, while aural atresia is not commonly
seen
CHL in approximately 67% of patients
Studies of the temporal bones of affected patients have revealed a wide
range of anatomic anomalies, ranging from abnormal stapes footplate to
aplasia of the lateral semicircular canals and an enlarged vestibular aque-
duct, and nearly universal poor pneumatization of the mastoid cavity·
Laryngomalacia and GERD

System manifestations are common with some reporting an incidence of over


80%. Ocular anomalies are reported in 10-30% of patients. The following lesions
occur in decreasing order of frequency: hypermetropia, myopia, astigmatism, cor-
neal sclerosis, and nasolacrimal duct stenosis.

Cardiovascular findings such as benign murmurs, pulmonary stenosis, patent


ductus arteriosus, patent foramen· ovale, atrial septal defect, and pulmonary
HTN have all been documented. Their prevalence varies in the literature from
5-58%.

Anomalies involving the musculoskeletal system are the most frequent systemic
anomalies (noted in 70- 80% of cases). They include syndactyly, dysplastic pha-
langes, and polydactyly, to name a few. In the lower extremities, foot anomalies
(clubfeet, metatarsus adductus), femoral malformations (coxa varus or valgus,
sh01t femur), hip anomalies (flexure contractures, congenital dislocation), anoma-
lies of the knee (genu valgus, synchondrosis), and tibial abnormalities have been
reported. Vertebral column deformities include scoliosis, kyphosis, lordosis, ver-
tebral dysplasia, sacral agenesis, and coccygeal sinus.

The -inGidence of CNS manifestation is around 50% and includes language de-
lay, seizure disorders, and hydrocephalus.

How do you determine if the ears are low set?

Drawing an imaginary line from the lateral canthus around the side ofthe skull
marks where the helix should begin. Normally, the upper 20-30% of the ear is
above this line, but if the top of the helix is below this imaginary line then a
craruofacial abnormality should be of concern.

How does one classify mandibular deformities?

Prizansky's classification scheme for mandibular deformities:


Type I: Mandibular ramus and- condyle are present but hypoplastic
Type Ila: Involves the condyle
Type lib: Absence of a satisfactory articulation
Type III: Complete absence of the vertical ramus

237
Otolaryngology Clinical Case Studies Chapter 5- Pediatric

NOTES What other abnormalities are seen in children with craniofacial syndromes and
what are a few syndromes with that feature?

Hvpertelorism - Craniofacial dysostosis, mainly Apert and Crouzon's syn-


drome (these two are very similar except that A pen has the added finding
of syndactyly), Noonan syndrome
Down-slanting eyes- Trisomy 21, mandibulofacial dysostosis (Treacher-
Collins syndrome)
Facial asymmeny/hemifacial microsomia- Goldenhar syndrome
Mid-face hvpoplasia- Mandibulofacial dysostosis, Apert and Crouzon's
syndromes
Cleft palate- Velocardiofacial syndrome, Pierre-Robin sequence, Stickler
syndrome, mandibulofacial dysostosis
CHL- All syndromes listed here

How does ·one define and measure hypertelorism?

Hypertelorism is a widening of the space between the eyes and is not uncommon
in craniofacial syndromes. Inter-orbital distance is correctly measured clinically
as the distance between the right and left dacryon points, which are the bony
anterior lacrimal crests. Normal values in adults are 25 rnm for a female and 28
I1ll11 for a male. Normal values in the pediatric population are presented below:

Age Distance
Infant 15 rnrn
1 18.5 rnm
2 20.5 rnm
3 21.0 mm
5 22.0 rnrn
7 23.0 tmn
10 25.0 rnrn
12 26.0mm

. What would be the next step in evaluating this child?

The first concern will be the status of the airway. The infant should have a
flexible nasolaryngoscopy exam to evaluate for other causes of airway obstruc-
tion. The retrognathia and micrognathia are usually the cause of the infant's
airway obstruction and may respond to conservative measures such as prone
positioning or nasopharyngeal airway.

How do you decide if !he child is safe to go home?

In the past, many experts advocated that many of these infants would over-
come their airway obstruction by age 6 months with even minimal mandibular
growth; however, this teaching is no longer commonly advocated. A thorough
history of obstruction events, apneas, desaturations and retractions should be
obtained in all positions. Just because children are fine in the prone position does
not mean they are if they roll over. Furthetmore, there is an increased incidence
ofSIDS in the prone positioning from suffocation, so it is not recommended.

238
Chapter 5 - Pediatric Otolaryngology Clinical Case Studies

Some advocate obtaining polysomnography studies on all of these patients prior · NoTES
to their discharge home and especially if any surgical intervention is perfonned.
For example, Arlen and Christian reported on 11 patients with severe Pierre-
Robin sequence who underwent MDO to correct severe airway obstruction.
Sleep studies were obtained on 7 of these patients and were nmmal soon after
surgery. Some recommend obtaining sleep studie~ on most all syndromic chil-
dren, while advocating watchful waiting for those with Stickler syndrome and
non-syndromic Pierre-Robin sequence, because syndromic children are much
more likely to also have a central component to their sleep disturbances.

What if the child's oxygenation seems to fall to the 70s every time the child is
turned supine?

The child should have surgical intervention, either a tracheotomy or MDO, before
being discharged.

Are there any other surgeries that have been attempted in the past?

In the past, attempts were made to control the infant's symptoms via a variety
of tongue-to-lip adhesion methods. However, these methods have been largely
abandoned due to low success rate and significai)t morbidities including dehis-
cence, scarring and feeding diffic~lties.

How do you decide whether to pla~e a tracheotomy or perform MDO?

It starts with a discussion with the parents. Tracheotomy is the gold standard for
~irway management. However, it may take up to 2 Y2 years for the child to attain
enough mandibular growth to allow decannulation. MDO is at the forefront of
treatment for newborns with micrognathia or retrognathia and severe respira-
tory obstruction, most commonly associated with Pierre-Robin sequence,
Treacher-Collins syndrome, Nager syndrome, and velocardiofacial syndrome.
Mendal, et al. reported an 88% success rate with internal MDO without tra-
cheostomy in infants with Pierre-Robin sequence and severe obstruction. He
also reported an 82% success rate in older micrognathic children with OSA.
Early MDO has been successful in avoiding tracheotomy in many of these
children. Denny and Kalantarian were able to avoid tracheotomy in 5 newborns
(age range, 6-26 days) diagnosed with Pierre-Robin sequence and severe tongue
base obstruction by performing early MDO. They reported an 67.5% increase
in mean effective airway space. In patients who have a tracheotomy preopera-
tively, decannulation protocol involves endoscopic evaluation, direct laryngos-
~copy, and bronchoscopy perfonned about 1 month after distractor removal. In
the end, we as physicians and surgeons outline the choices to improve the child's
airway and help the parents make the choice based on the risks and benefits of
all therapies involved.

How does distraction of the mandible work?

MDO allows for anterior displacement of the tongue with increases in mandibu-
lar projection. The intimate relationships of the genioglossus and geniohyoid
with the anterior mandible are likely the most important factors in relieving

239

.·l
Otolaryngology Clinical Case Studies Chapter 5 - Pediatric

NOTES tOngue base and supraglottic obstruction in micrognathic patients. In a recent


review of their 15 years of expetience, Fritz, et al. found that endoscopic exami-
nation was the most important determinant of successful MDO candidates.
Those patients most amenable to surgical improvement displayed severe tongue
base obstruction of the hypopharynx and supraglottic compression secondary to
retrodisplacement of the mandible. Micrognathic supraglottic compression is a
fixed obstruction seen both on inspiration and expiration, in contrast to dynamic
inspiratory supraglottic obstruction seen with laryngomalacia. An important tool
in endoscopic evaluation is the jaw thmst maneuver. This effectively mimics the
airway changes induced by MDO and is an accurate predictor of the potential
success of surgical intervention. The effects of this maneuver can be docu-
mented by videotape and reviewed.

Do all children with mandibular retrognathia and micrognathia do the same?

No, tracheotomized patients with complex congenital syndromes had a success


rate of only 17% with inability to decannulate as the main indicator of failure.
The most common cause for failed decannulation was neurologic hypoxia.

How is the procedure performed?

The procedure is performed with nasotracheal intubation and general anesthe-


sia through a 3 em external incision in_infants or an intraoral incision in older
children. Cortical osteotomies are performed either in front of or behind the
angle of the mandible with preservation of the inferior alveolar nerve. Pins are
then placed 5-8 mm from the osteotomy at the inferior border of the mandible.
The distractor is applied to the. body of the mandible. The distractor may be
external or internal. The procedure is performed bilaterally, and the mandibular
body is lengthened, approximately 0.5-1.5 mm per day. Average distraction for
an infant is 15 mm. End points of distraction are most accurately based o·n
resolution of tongue base and supraglottic obstruction on endoscopic evaluation.

Should a~y· other specialists be involved?

Genetics consults would be indicated and chromosome evaluation should be


undertaken in all patients suspected of having a congenital syndrome. A search
for abnormalities of other organ systems such as the genitourinary system, gas-
trointestinal tract, and musculoskeletal system should be undertaken because
these organ systems are involved to varying degrees in a multitude of congenital
syndromes and their discovery would aid in the diagnosis and treatment plan.

Any other tests you, as an otolaryngologist, would recommend?

Otoacoustic emissions or a screening ABR should be performed early as a


baseline. Many of these children have will have a component of hearing loss,
either conductive or sensorineural. Studies have shown that cochlear implants,
when indicated, can be successfui in these children, but these children may
require careful mapping and higher levels of electrical stimulation of the implant
compared to n01mal child implantees.

240
Chapter 5- Pediatric Otolaryngology Clinical Case Studies

Should cleft palates be repaired in all affected patients? NoTES

A cleft palate in certain syndromic children, especially those with craniofacial


synostosis, has been used to reduce the need for a tracheotomy because it helps
alleviate nasal airway obstruction in children with midface retrusion. Sculerati,
et al. reported performing palatal split procedures on Apert, Pfeiffer, and Crouzon
syndrome children to alleviate airway obstruction. They found that in these
children whose cleft palate was repaired before tracheotomy, postoperative
obstruction often required long term tracheotomy. This was in contrast to patients,
often with Pierre-Robin sequence, who had transient postoperative edema
requiring short-te1m tracheotomy. At the time of surgical correction of the palate,
a dissolvable stitch should be placed in the tongue to help pull it forward in case
of further obstruction following the surgical procedure.

Ultimately, care of these children requires a multidisciplinary approach. The


team should include a pediatrician, otolmyngologist, plastic surgeon, speech thera-
pist, and additional specialists as dictated by the individual chi ld's needs.

Below are listed common craniofacial syndromes associated with otber associ-
ated abnormalities. Also listed in the table are the markers that are noted for
each syndrome.

Primary Features Causative


Syndrome (not in 100%) Gene/Trait/Marker
Sitckler Syndrome Marfanoid habitus, severe AD, Collagen II and
myopia, arthritis, mandibular IX
hypoplasia (similar to
nonsyndromic Pierre-
Robin)

Noonan syndrome Short, ptosis, down-slanting Clin 12q


palpebral fissures,
hyperteioriism, low-set ears,
low posterior hairline,
pulmonary valve stenosis,
learning disabilities

Treacber-Collins Midface dysostosis, upper Treacle


syndrome airway narrowing due to
small NP, malformations of
auricle (including microtia)
& CHL from malformed
ossicles, malar hypoplasia
with flat supraorbital ridges,
cleft palateNPI (30%)

Van der Woode syndrome Cleft lip+/- palate, lip pits lp43

24 1
Otolaryngology Clinical Case Studies Chapter 5 - Pediatric

NOTES Waardenburg syndrome Increased inner canthal PAX6


distance, heterochromia,
white forelock, SNHL, cleft
palate, hypoplastic alae nasi
and broad nasal bridge

Velocardiofacial syndrome Velopharyngeal deticiency At least 90% of


from an overt, submucous patients have a deletion
or short palate or hypotonia; of the long arm of
have varying degrees of chromosome 22
cardiac anomalies: (22q 11)
medialized carotid arteries
in nasopharynx; facial
features include an
elongated face with nanow
palpebral fissures, small or
misshaped ears and small
round mouth; about 10% of
patients have the DiGeorge
sequence

Branchial syndrome Branchial cyst, hearing loss, Unknown


ear malformations and
kidney abnormalities

Crouzon syndrome CraniosynostosisProptosi.s, FGFR2, FGFR3


rnidface hypoplasia,
mandibular prognathism,
beaked nose,+/- CHL from
EAC atresia/stenosis

Smith-Lemli-Opitz MR, two-three toe AR


syndrome syndactyl, genital
abnormalities, cleft palate,
polydactyl

Alport syndrome Sensorineural hearing loss Xq21.3-q22


and renal disease

References:

1. Sculerati N, Gottlieb MD, Zimbler MS, Chibbaro PD & McCarthy JG. Air-
way management in children with major craniofacial anomalies. Laryngo-
scope. 1998;108:1806- 12.
2. Moore MH. Upper airway obstruction in the syndromal craniosynostoses.
Br J Plast Surg. 1993;46:355-62.
3. Arlen D & Christian A. New technique for airway correction in neonates
with severe Pierre-Robin sequence. J Pediatr 2005;147:97-101.
4. Tomaski, eta!. Airway obstruction in the Pierre-Robin sequence, Laryngo-
scope. 1995;105:111-4.

242
Chapter 5- Pediatric Otolaryngology Clinical Case Studies

CASE 5- LYMPHATIC MALFORMATIONILYMPHA!'lGIOM.A NoTES


Brett A. Miles, DDS, l\1D
John E. McClay, MD

You are consulted to evaluate a 2 year-old male for left-sided facial swell-
ing anterior and inferior to the le.fi ear lobe and overlying the angle of the
mandible.

HPI: state that you would:

Obtain a more detailed medical history

What additional historical information would you seek? State that you would ask:

How long has the swelling been present?


Was the swelling present at birth?
Does the swelling change in size?
And if it does change in size, what makes it change, or is it random?
Does the area increase in size when the child is sick or with increased
activity?
Is there a his~ory of trauma to the area?
Is the lesion present bilaterally?
Is the child having pain associated with the swelling?
Has the area ever seemed red or infected?
Does the skin overlying the area ever change color or appear bluish?
Has the child ever taken any antibiotics for the swelling?
Has there ever been swelling in the neck related to the lesion?
Has the child ever experienced respiratory difficulty with the lesion?
Doe-s the child have other swellings or lesions elsewhere?

The mother states that the child was delivered vaginally without incident.
There is no known FH of congenital birth defects or syndromes. The lesion
was not present at birth and was noticed approximately 2 months previ-
ously. There iS no hist01y of trauma to the area. The lesion has slowly been
increasing in size. The child complains intermittently of mild pain, approxi-
mately once per week over the last few weeks. The area has never become
red or inflamed and the child has never taken antibiotics for the lesion.
The child has never experienced respiratory difjiculty from the lesion and
the lesion has not involved the neck previously. The lesion did seem to
increase in size slightly during a recent URJ, but does not increase in size
with activity/exercise. The lesion is isolated to the left side and the parents
state they have never noticed swellings or lesions elsewhere on the child.

PMH: No known medical problems


PSH: No prior surgical procedures
Allergies: NKDA
Meds: None
FH: Negative
SH: Negative

243
Otolaryngology Clinical Case Studies Chapter 5 - Pediatric

NOTES What do you look for on PE?

S tate that you would perform a complete head, neck, and PE including vital
signs and nasopharyngoscopy

PE:

Vitals: Temp: 98.6 F. BP: 70145, Pulse: 85, RR: 18


GA: Child appears calm, no evidence of distress
HEENT 2 x 3 em soft, compressible mass over(ving the left mandibu-
lar angle. The mass is not sign?ficantly tendet; non-fluctu-
ating, and without evidence of overlying skin changes. No
erythema or inflammation qssociated with the lesion is noted.
The mass is not freely mobile
CN: 111-Xll normal. Left CN VII House/Braclanann scale 1/6
Eyes: Normal exam. Reactive pupils, EOM normal
Ears: No external abnormalities noted. AD - normal TM intact.
AS - normal T1vf intact, no effusions noted
OC/OPI
nasal cavities: No masses, lesions, or other abnormalities. No palatal clefts
or defects noted. Stenson 's!Wharton 's ducts. normal, clear
salivmy flow noted bilaterally
Neck: Supple with no masses, nodes or bn1its noted on _e.;tamination.

The remainder ofthe PE is WNL and no other masses or lesions are identified.
. .

What is your differential diagnosis?

Chronic infection/recurrent sialadenitis:


Sialolithiasis
Salivary duct stenosis
Chronic bacterial infections (i.e., staphylococcus & H. influenzae)
Benign lymphoepitheliallesion
Cat scratch disease
TB
Non-tuberculous mycobacterium
Actinomycosis

Partotid cysts/tumors
Benign:
Parotid cyst
Pleomorphic adenoma
Wru.thin's tumor

Malignant:
Mucoepidermoid carcinoma
Adenoid cystic carcinoma
t\Cinic cell carcinoma

244
Chapter 5 - Pediatric Otolaryngology Clinical Case Studies

Congenital/developmental: NOTES
Venolymphatic malfonnation
Lymphatic malformation (previous cystic hygroma lymphangioma)
Venous malformation
Hemangiomic malfonnation
First branchial cleft cyst

Miscellaneous tumors:
Lipoma/fibroma
Lymphoma
Sarcoma

What are the most likely lesions?

• Congenital lesions because of young age including:


Lymphangioma
First branchial cleft cyst
Hemangioma
Maybe chronic infection like cat scratch or non-tuberculous mycobactetiurn
Maybe parotid tumor, benign or malignant

What do you do next? State that you would:

Perform fiberoptic nasopharyngoscopy/laryngoscopy


Obtain CT neck with contrast

Test results:

Fiberoptic nasopharyngoscopyllaryngoscopy examination is WNL. No


nasophmyngeal or hypophmyngeal masses/lesions noted. Normal glot-
tis/pyriform sinuses
CT scan of the neck shows a left-sided 2.5 x 3.3 em multiloculated
cystic mass overlying the left paroti~ gland with fluid levels. Few s.epta
,.' noted within the mass. No evidence offat plane obliteration or inflam-
., mation surrounding th~ mass. Minimal enhancement of septations with
/

contrast noted. No cervical masses or lymphadenopathy noted on the


examination
Consider MRI evaluation of the mass for additional information or
confirmation

What is your diagnosis?

Lymphatic malfonnation- This diagnosis is generally based on radiological


criteria. The cystic nature of the lesion makes the possibility of a benign/
malignant tumor less likely in this patient. Additionally, there is absence of
fat plane obliteration or inflammation which argues against a malignant or
infectious lesion. Vascular lesions/malformations generally enhance with
contrasted tomography. Branchial cleft cysts are usually unilocular. If pre-
viously infected, there could be some loculations. However, based on the
history, there have been no previous infections. The fact that the lesion

245
Otolaryngology Clinical Case Studies Chapter 5 - Pediatric

NoTES appears as a benign, multi-loculated, cystic lesion leads you toward the
diagnosis oflymphangioma. A branchial cleft cyst could still be in the differ-
ential. A MRI is often obtained in large and mixed microcytic and macro-
cytic lymphatic malformations, but unnecessary in this particular case

What is a lymphatic malformation?

Rare congenital abnormal growth of lymphatic vessels which is typically


diagnosed in the pediatric population

How do they develop?

While originally thought to be a neoplastic process, current theory centers


around abnormal development to regional lymphatic tissue. Three main theo-
ries have been proposed for the formation oflymphatic malfmmations. The
first theory is that primordial lymphatic sacs fail to connect with the venous
system, resulting in abnmmal drainage in regional lymphatics. The second
theory is that deposition oflymphatic tissue into abnormal areas during em-
bryogenesis results in regions which fail to join the normal lymphatic sys-
tem. The third theory is that there is abnormal budding of the lymphatic
system with failure to fuse with main lymphatic channels. Currently,· the
exact etiology oflymphatic malformations remains unlrnown.

What are the classifications/types of lymphatic malformations and. how does


one differentiate between the lesions?

Binary classification system adopted by the International Society for the Study
ofVascular Anomalies (ISSVA) in 1996-:

I. Vascular tumors:

Hemangioma:
Presentation:
Generally present at bilih or within a few weeks after. Red or bluish
compressible mass if some skin component. Can be totally isolated in
the parotid gland or other organ and have no skin component. May be
pulsatile or exhibit a bruit on examination. Lesion is "fast flow" on color
flow Doppler examination
Clinical course:
Grows rapidly in size during the first year of life and slowly involutes
over the next 2-10 years. Steroids in the first year help decrease the
growth while it is in the proliferative phase

Heman!!ioendothelioma:
Presentation:
Rare pediatric vascular tumor intermediate between hemangioma and ·
angiosarcoma. Typically a solitary lesion of the proxi:r;nal extremities
and trunk; cervicofacial lesions rare
Clinical course:
Lesions are treated with multimodal therapy with 12-24% mortality.
High incidence of recurrence

246
Chapter 5- Pediatric Otolaryngology Clinical Case Studies

Malignant vascular tumors: NOTES


Angiosarcoma
Hemangiopericytoma

II. Vascular malformations:

Capillary malformation:
Presentation:
Transient macular stains seen in Caucasian infants around nares, nape
of neck, etc. Called Nevus flammeus neonatorum, stork bite, salmon
patch, angel's kiss. May be permanent capillary malformation such as
in "port wine stain" which is associated with a unilateral trigeminal
dermatome
Clinical course:
Benign transient lesions with the exception of port wine stains which
are treated with either flashlamp pulse dye, KTP or copper bromide
laser, depending on severity

Venous malformation:
Presentation:
Soft compressible mass. May appear dark blue or purple due to vascu-
lar components contained within the lesion. Lesion is "slow flow" on
color flow Doppler examination
Clinical course:
Generally not present at birth. Generally does not fluctuate in size, but
usually enlarges over time. Often asymptomatic when patient younger,
and pain develops as children get older. Most likely will need some
intervention

Lymphatic malformation:
Macrocystic- Contain large thick-walled cysts (>2 em radiologically/
histologically) with mild infiltration of surrounding tissues
Microcystic - More extensive infiltration of soft tissues in neck with
smaller cystic structures (<2 em radiologically/histologically), often dif-
ficult to surgically resect
Presentation:
Soft, smooth non-tender mass which is compressible and may be trans-
illuminated. Fluctuation in size is common with URJ and may increase
in size over time
Clinical course:
Usually cosmetic effects only. Occasionally may compress the airway
or become infected, requiring more urgent intervention. Can regress
after infection .

Arteriovenous malformation:
Contains both lymphatic and venous components and may present with
features consistent with either lesion. Typically a "fast flow" lesion on
flow Doppler examination

247
Otolaryngology Clinical Case Studies Chapter 5 - Pediatric

NoTES What do you want to do at this point?

State that there are different types of treatment you would discuss with the
patient's parents, including the risks and benefits of each option for their
child. Always remember, it is their child. All we do is to provide them with
knowledge so they can make an informed decision for their child.

Options:

1. Surgical excision:

Indications for surgery:


Recurrent pain/infection (usually following URI)
Aural/ocular involvement (rare)
Aesthetically disturbing appearance

Surgical technique:

Highlights and pitfalls:


Complete excision seems to be the most reliable way to control lymphatic
malformations
Complete excision of the lesion using techniques of oro-facial resection,
functional neck dissection and/or superficial/total parotidectomy
In this case a modified face lift incision with removal of the mass can be
entertained. A partial superficial parotidectomy may be performed if the
parotid gland is involved with the lesion
Surgical excision oflymphatic malformation may be quite extensive, espe-
cially in oro-fac~al/cervicallesions
Relationship to adjacent structures can lead to significant complications
Partial excisions lead to poor outcomes and increase the rate of complications
Staged surgical excision may be indicated for unusually large or complex
lesions
Exposes patient to the risks and morbidity of surgical procedure
Complication rate for all head and neck lymphangiomas does decrease with
well locali~ed lesions of the parotid, submaxillary, and posterior cervical
regiOns
Patients with extensive or bilateral lesions involving the tongue, FOM, and
pharynx tend to require more procedures, have higher recurrence and com-
plication rates, and have more symptomatic disease
Supra-hyoid vs. infra-hyoid: There is generally an increase in recurrence
rate, morbidity, and complication rate for supra-hyoid lesions relative to infra-
hyoid lesions

Pearls:
If large cervicofacial and tongue or pharyngeal lesions exist, the central
component of the lesion (i.e., tongue or pharyngeal) should be operated on
first. If the neck lesions are resected first, the lymphatics of the tongue and
pharynx are blocked and these lesions swell up. Since the lymphatics are
already compromised, the swelling takes a long time to decrease

248
Chapter 5 - Pediatric Otolaryngology Clinical Case Studies

Similarly, when counseling the parents prior to surgery, let them know the NoTES
edema created from the surgery may take up to 6 months to resolve

2. Sclerotherapy:

Highlights and pitfalls:


Non-invasive technique. The cyst contents are extracted and the sclerosing
agent is infiltrated. Often requires more than I treatment, even for a single
cyst. Technique usually requires general anesthesia
Avoids risk of surgical procedures for complicated lesions
OK-432 (PicibanilChugai Pharmaceutical Co., Tokyo, Japan):
Lyophilized mixture oflow virulence streptococcus pyogenes
Incubated with benzylpenicillin (PCN allergy-contraindicated)
0.05% risk of anaphylaxis
86-96°!o effective for most macrocystic lesions with few cysts
Not effective for microscopic lesions
Only available in clinical trials (2005)
Low grade fever and localized inflammation common

Injection ofOK-432 produces an inflammatory reaction leading to the de-


struction of the epithelial lining with subsequent sclerosis and cicatricial
contraction of the lesion. May increase lymphatic permeability of cystic
vascular walls, promoting lymph flow and cyst shrinkage. Minimal necrosis
is observed.

Ethanol:
Limited use in highly vascular lesions due to systemic absorption
Significant necrosis and damage to surrounding normal tissue
Increased rate of scar formation

Various results reported in the literature and not utilized by many practitio-
ners. Difficult to control spread of necrosis due to leakage into tissue. Can
be ~elpful in venous malformations, as well as lymphatic malformations.

.,' Bleomycin:
Limit total dose to 450 mg as the risk of pulmonary fibrosis increases
significantly at doses greater than 500 mg
Dose of 50 mg is used at each session
Greater than 90% response rate for cystic and cavernous types
Not rarely

Induces inflammatory response which induces fibrosis of the lesion ~and


prevents accummulation oflymphatic fluid. Diffusion ofbleomycin is ham-
pered if a significant vascular component exists within the lesion.

Others:
Corticosteroids/Tetracycline - Generally less effective than other
sclerosants. Less compelling evidence available
- Fibrin glue - Recent reports state good results with intralesional injec-
tion of fibrin sealant; may be inj ected to aid in surgical removal oflesion

249
Otolaryngology Clinical Case Studies Chapter 5 - Pediatric

NoTES Pearls:
tvlinor adverse effects such as swelling, pain, and fever are common
Major anaphylactic reactions reported
Sounding necrosis with ethanol
Pulmonary fibrosis with Bleomycin

3. Observe the lesion:

Highlights and pitfalls:


Small asymptomatic lesions may be observed, thereby avoiding surgery
Lesions may regress spontaneously
Lesions may expand and require treatment at a later presentation
Lesions may become infected, complicating surgical excision

The parents decide to watch the lesion for now. They will follow-up with
you when they have problems.

References:

1. Riechelmann H, Muehlfay G, Keck T, et al. Total, Subtotal, and Partial


Surgical Removal of Cervicofacial Lymphangiomas. Arch o ·tolaryngol Head
Neck Surg. 1999; 125:643-8 .
2. Naidu S, McCalla M. Lymphatic malformations of the head and neck il)
adults: a case report and review of the literature. Ann Otol Rhinal Laryngol.
2004;113:218-22.
3. Kennedy T, Whitaker M, Pellitteri P, eta!. Cystic hygroma/lympangioma: a
rational approach to management. Laryngoscope. 2001; 111:1929- 37.
4. Bloom D, Perkins J, rvt;anning S. Management oflymphatic malformations.
Curr Opin Otolaryngol Head Neck Surg. 2004; 12:500-4.

250
Chapter 5- Pediatric Otolaryngology Clinical Case Studies

CAsE 6- TRACHEOESOPHAGEAL FISTULA N oTES


John E. McClay, MD

Patient is a 2 month-old male wirh choking episodes, intermittent noisy


breathing and mild failure to thrive, currentl_v in the ER.

What is your first concern in a patient like this? State that you would ask:

Is the patient stable?


Is the patient in distress?

How is that determined? State that you would ask:

How does the child look clinically?


Is he tachypneic? Is he hypotensive? Is be tachycardic?
Is he retracting? Suprastemal, subcostal, or intracostal?
Does be have nasal flaring?
Is he desaturating?
Has he had a blood gas test? What is his pH, C02, 02?
Why is he in the ER now?

The infant is stable for now, resting comfortably in his mothers arms. He
started choking uncontrollably an hour ago, stopped breathing and turned
blue. The parents called 911 and he was transported to your facility by
ambulance. He now is not tachypneic nor tachycardic. His last BP reading
10 minutes· ago was stable. He is on a pulse oximeter reading between
95% and 98%. He has no nasal flaring or retractions. A blood gas was
not done.
)
HPJ: state that you would ask:

Feeding:
When did the parents first notice the choking?
Is the choking related to eating?
Is the stridor related to eating?
Has the child had pneumonia or respiratory illness?
Is the child breast or bottle-fed?
Has the child tried other fommlas, ifbottle-fed?
; Is the child spitting up?
Does the child latch on and suck well?
Does the child have~any nasopharyngeal reflux?
l

Does the child drool?

Respiratory:
What does his breathing sound like?
Is it snorting or snoring?
Is it wheezing or stridor?
When does it occur?
Is it intermittent?

251
Otolaryngology Clinical Case Studies Chapter 5 - Pediatric

NoTES Is it positional?
Is it heard on inspiration, expiration, or both (biphasic )? •
Does eating make it worse?
Has the child stopped breathing?
Has the child had cyanosis?
What is the character of the child's voice?
Is the c1y strong?

What additional history would you seek from the parents or caregiver?

PMH:
Were there any problems with the child's birth?
What is his gestational age?
Any perinatal infections?
~ . Any meconium aspiration?
Has he been intubated- Was he on a ventilator?
Is he on any medications?
Does he have any known medical problems? Cardiac problems?
Has he had any surgeries?

His parents first noticed the choking episode with eating. He often chokes
with feeding, but they are not sure it is every time he eats. He is breast fed,
can latch on well and is gaining weight. He dqes spit up, especially during
and after eating, and occasionally while in his bou_ncy seat. It is not pro-.
jectile vomiting. His breathing worsens when eating, both becoming louder
and faster. His breathing sounds like wheezing. He has had one bout of
pneumonia, 2 weeks ago, _a nd was discharged last week from the hospital.
TiVhile in the hospital, he was intubated for 2 days because of trouble breath-
ing. His noisy breathing dor;s not worsen with position. He is quiet some-
times, especialZv sitting up in car seat. His cry is good and strong. He was
born vaginally at 38 weeks gestation and not intubated. He was intubated
perinatally. There were no perinatal infections or meconium aspiration.
His pediatrician put him on Zantac, which has helped the spitting up. He
has no other illnesses, including cardiac disorders; he has had no surgeries.

What would you look for on the PE? State that you would:

Listen for the nature of the sound- Stridor stertor


Look for obstructive signs like nasal flaring or suprasternal, intercostal, or
subcostal retractions
Assess his vital signs: HR, RR, BP, and temperature
If unstable, move directly to the OR for direct laryngoscopy and bronchos-
copy and possible other treatment based on findings
If stable, complete PE - Craniofacial structure, external ears, nose and
eyes. Perform anterior rhinoscopy by looking into nose with otoscope. Look
into OC. Evaluate mandible for retrognathia. Auscultate lungs, neck and
chest ·

252
Chapter 5- Pediatric Otolaryngology Clinical Case Studies

PE: NoTEs
Temp: 36.7 C. HR: 115, BP: 95154, RR: 26, Pulse oxime-
ter 98% oxygen saturation on room air
GA: Breathing comfortably now, no retractions, no audible stridor
Craniofacial: Ears in normal position, no retrognathia, no hypertelerism,
external nose normal
EAC!TM· Relatively normal for a 2 month-old with slightly floppy
ear canals
Nose: No congestion, no drainage intranasally
OCIOP: No cleft palate, no high arched palate, no significant ton-
sillar tissue
Neck: Full range of motion, no masses or adenopathy, mild stridor
auscultated
Chest: Mild biphasic stridor. Lungs clear to auscultation
CIV RRR, no murmurs
Extremities: Normal

i What is your differential diagnosis?


.;

!
Subglottic stenosis
\
Tracheomalacia
LTC
Neurological delay with uncoordinated swallow and aspiration
TEF
GER
Vascular compression (double aortic arch, aberrant pulmonary artery)
.-) External compression on trachea and/or esophagus by intrathoracic lesion
Other congenital airway lesion
.j

' l What would you do next?


l
) There are really 2 options. If a child is stable in the ER, one may want to.
consider getting more information by performing a flexible nasopbaryngo-
..,
_./
scopy and laryngoscopy. Since the child bad a significant airway event at
home, regardless of what the flexible laryngoscopy reveals, be will need a
formal direct laryngoscopy and bronchoscopy in the OR. Bronchoscopy is
the gold standard to determine the pathological anatomical abnormality in
l the airway. If that is normal, one would then order other tests to determine
'
I
the physiology of the dysfunction, starting with a barium swallow with fluo-
roscopy. That test will give information on swallowing function and GER. If
this child were not seen emergently, this test could be performed before the
direct laryngoscopy and bronchoscopy and in that setting would reveal un-
known information on the trachea, such as tracbeomalacia

The child is taken to the OR and direct laryngoscopy and bronchoscopy


is performed. The supraglottic, glottic and subglottic larynx appears
normal, except for some mild erythema of the petiole of the epiglottis
and some edema and erythema of the posterior glottis.

253
Otolaryngology Clinical Case Studies Chapter 5 - Pediatric

NoTES I The trachea looks like this. What is your diagnosis?

Diagnosis:

TEF

What would you do next?

State that you keep the child NPO and contact the pediatric surgery service to
repair the abnormality

What is the etiology of TEF?

Two theories are proposed. The first theory, proposed by EI Smith in 1957,
states that an incomplete separation of the respiratory and digestive division of
the primitive foregut occurs during the third to fourth week of gestation (embryo
size 3 mm). According to Smith, normally a ventral diverticulum (the future
larynx and trachea) distal to the pharyngeal pouches develops from the primi-
tive foregut by midline migration of the lateral foregut ·grooves. Aberrant growth
of the grooves causes EA, while failure of grooves to fuse causes TEF. Kluth
felt these abnormalities result from defective fetal development such as insuffi-
cient circulation and hypoxemia that occurs after complete tracheal and esoph-
ageal differentiation. ,

What are the types of TEF?

TEF is often associated with EA. The 5 types of TEF have been described.
They include: (1) EA with a distal TEF (86%), (2) Isolated EA with no fistula
(8%), isolated TEF with no atresia (so-called H type) (4%), EA with proximal
and distal fistulas (1 %) and EA with a proximal fistula (1 %). This child's pre-
sentation would be consistent with an H-type TEF. Children with the most

254
Chapter 5- Pediatric Otolaryngology Clinical Case Studies

common type of abnonnality, EA with distal TEF regurgitate food at birth since NOTES
they have no connection to the stomach from the esophagus. See diagram below:

'1.. \.
;E~ \

IJ
~
.5. 7.7%
~ iSOLATED
EA

-~~~
tl ·
f4.J
l. '
t; :,
·""'-

t~\ o.s%
-'tj
.,
~~·\ o.7%
\Y~r
£A WIT;; PRO:<IMAL . EA WITH DOUBLE
. ·: ::.F I T~f
-- ~-

v
.JL-

~
Figw-e 67-1 from Pediatric Otolaryngology, 4th ed. Bluestone CD, Stool SE,
)
Alper CM, Arjmand EM, et al, eds. 4th ed. Philadelphia: Saunders; 1282,
) Copyright ©2003 Elsevier Inc. Used with pe1mission.
)
Are there associated abnormalities with TEF?
)
) Overall, it is estimated that other anomalies occur in roughly 50% of cases. The
) VACTERL syndrome can occur in part or total with TEF. Low birth weight and
) Down's syndrome occur with increasing frequency. GER (occurring in 40-
70%) and tracheomalacia (occurring in various degrees in most all patients) can
) be seen with TEF. Occasionally LTC can be seen in a child with TEF. If a LTC
) is present, a TEF is seen 20% of the time
)
' Additional problems include abdominal distention, airway difficulty, p'neumonia
)
and atelectasis. When infants with TEF strain, cough or cry, air can enter the
stomach through the fistula and the stomach and small intestine can become
dilated, elevating the diaphragm and causing difficulty breathing. The reflux of
food and gastric secretions may also occur up the esophagus and tlu·ough the
fistula into the tracheobronchial tree, causing pneumonia and atelectasis

255

)
Otolaryngology Clinical Case Studies Chapter 5- Pediatric

NOTES Are there any prenatal signs a child may have a TEF?

The nonspecific signs of polyhydramnios on ultrasonography performed during


pregnancy suggest possible gastrointestinal obstruction, seen more with EA and
distal TEF than an H-type TEF. Additionally, with EA. a dilated pouch and
absence ofliquid filled loops ofbowel could be seen.

How do the pediatric surgeons repair this abnormality?

For an H-type fistula tract, the cervical approach is used because the tract is
higher in the neck. For a TEF with EA, a lateral thoracotomy approach would
be used. The tract is ligated and often divided. Some repmis indicate using
cautery or sclerosis agents to scar the tract and obstruct it.

What are the highlights and pitfalls of surgery?

Pre-operative:
The diagnosis with an H-type fistula is hard to make. The radiologist per-
forming _the esophagram should know to specifically look for an H-type
fistula. Placing the child in the prone position may help th~ contrast flow
forward into the trachea, improving the likelihood the tract is seen. Bron-
choscopy can yield a definitive diagnosis as well. The H-type fistula does
not normally look like the endoscopic photograph of Figure 1. It .is fre-
quently a very small red spot on the posterior trachea just below the cricoid.
If the red spot is seen, it can be cannulated with a ureteral stent and then an
esophagoscopy can be performed to confirm the stent is in the esophagus

Intra-operative:
The definitive repair can be performed if the ureteral stent is seen in the
esophagus. It is retrieved so that both ends are hanging out of the mouth.
The patient is positioned on his back with his head turned to the left so an
incision can be made on the right side ofthe neck just above and parallel to
the clavicle. A right-sided approach will avoid the thoracic duct. One~ the
incision is made, the tracheo-esophageal groove is dissected with care to
avoid the recunent laryngeai nerve. The level of the fistula is easier to
identify with the help of the anesthesiologist tugging on the two ends of the
ureteral stent which is looping through the fistula. Once the fistula is dis-
sected, it is divided and closed with absorbable suture. The tract is usually
divided close to the esophagus to avoid narrowing the airway. A strap muscle,
such as the omohyoid, can be placed between the two suture lines to help
decrease the risk of recurrenc~

Post-operative:
H-type fistulas have less of a chance of having significant tracheomalacia
than the EA/TEF type of abnormality

The pediatric surgeon repairs the fistula, but the child has mild airway
stridor/wheezing with no desaturations or cyanosis. You evaluate the child
and notice mild retractions suprasternally and mild wheezing.

256
Chapter 5 - Pediatric Otolaryngology Clinical Case Studies

What would you do? NoTES

State that you would perform a direct laryngoscopy and bronchoscopy

BronchoscopJ' reveals moderate tracheomalacia of roughly 50-60% ob-


struccion around the site of the TEF

What do you do now?

If the child's airway is stable as defined by absence of cyanosis, apneas, signifi-


cant stridor, wheezing, or retractions, the child can be observed. A sleep study to
ensure a stable airway and/or a pulse oximeter at night might be helpful to make
sure his oxygen saturation maintains at an approp1iate level. Tracheomalacia
usually improves over time on its own. Tracheornalacia associated with TEF
often takes longer to improve than isolated tracbeornalacia.

References:

I. Spitz L. Esophageal Atresia: Past Present and Future. J Pediatr Surg.


1996;31(1): 19-25.
2. Bolinger LD, Lusk RP, Green CG, eds. Pediatric laryngology and broncho-
esophagology. Philadelphia: Lippincott Williams & Wilkins; 1997, 197-202.
3. Morrow S, Nakayana D. Congenital malformations of the esophagus. In:
Bluestone CD, Stool SE, Alper CM, Arjmand EM, eds. Pediatric otolaryn-
gology. Philadelphia: Saunders;2003, 1280-8.

Speciai thanks to Steve Megison, MD, Associate Professor of Pediatric


Surgery at.University ofTexas Southwestern Medical School and Children's
Hospital of Da]las, for his help on identifying the pearls and pitfalls of
surgery.

257
Otolaryngology Clinical Case Studies Chapter 5 - Pediatric

NOTES CASE 7- TRACHEOMALACIA


Romaine F. Johnson, MD

A 2 month-old female with stridor for approximately .J weeks.

HPJ: state that you would:

Obtain a more detailed birth, medical, and respiratory history

What additional information would you seek? State that you would ask:

Describe the stridor. Inspiratory, expiratmy, or both?


Obtain birth history. Was the child born immature?
What factors affect th~ symptoms? Position, feeding, activity?
Have there been episodes of cyanosis or extreme distress?
Any other medical problems?
Any problems w1th reflux?

The episodes of stridor started about 1 week after birth. The ·child was
born vaginally and there were no complications. Initially, she did well but
started having noisy breathing with eating, making feeding somewhat dif-
ficult. Her symptoms have gotten worse over the last several weeks, in-
cluding increased difficulty. with feeding, increased stridor when lying su-
pine and 1 episode of extreme distress with cyanosis resulting in an ER
visit. She is otherwise healthy.

PMH: Born at FT. vaginal delivery. One ER visit in the last week
for an episode of cyanosis. Discharge after negative
work-up with fo{low-up with you
ALlergies: · NKDA
Medication: None
FH: Noncontributory
SH: Lives at ho71Je with parents. Has 2 older siblings
ROS: Positive for reflux with feeds. Otherwise negative

What do you look for on PE?

State that you would complete a head and neck examination including vital
signs and flexible laryngoscopy

PE:

Ki1.ciLl.: Temp: 98.6 F. RR: 20, Wt: 4.2 kg, Ht: 55 em


GA: Well developed female with some mild expiratory stridor at
rest
Ears: EAC and TM clear
Nose: Septum midline, mucosa pink, and no lesions
OCIOP: No lesions
Neck: Supple, No masses, lymphadenopathy, or thyromegaly

258
Chapter 5 - Pediatric Otolaryngology Clinical Case Studies

NPL: NP clear, larynx normal in appearance, TVC mobile bilater- NoTES


ally. You do note that the expiratory stridor becomes more
pronounced with the childs agitation during the examination

What diagnostics would you request'!

Direct laryngoscopy and rigid bronchoscopy


Other tests depend on findings:
Consider CXR, CT scan (chest), and UGI

Test results

Direct la~yngoscopy and rigid bronchoscopy showed mid-tracheal com-


pression resulting in approximately 50% obstruction. The obstruction
was pulsatile in nature.

What is your differential diagnosis?

Type 1 tracheomalacia - The above history is suggestive of tracheomalacia.


Tracheomalacia results from immaturity of cartilaginous framework of the
trachea resulting in stridor that is typically expiratory. This contrasts with
laryngomalacia that is generally inspiratory. Tracheomalacia can also be
secondary to external compression of the trachea from an intrathoracic
cause such as a vascular ring or compression. Tracheomalacia (Type 3)
can also result from prolonged intubation or tracheitis but this feature is not
present in this case .
Type 2 tracheomalacia- Tracheomalacia due to external compression from
a thoracic lesion
H-type tracheocutaneous fistula- Although rare, H-type tracheocutaneous
J fistula can cause symptomatic tracheomalacia in lieu of other symptoms.
) Other types ofTEF also commonly cause tracheomalacia; additional symp-
toms related to breathing and aspiration point to the diagnosis
)
Tracheal ·stenosis - Congenital tracheal stenosis (i.e., complete tracheal
) rings) can present with stridor early.in life as well. This disorder should be
~ kept in the differential UJ!til ruled out by endoscopy
GER- Reflux can cause, as well as exacerbate, airway symptoms in children

) Diagnosis:

Type 2 tracheomalacia

"
j
I
What treatments would you begin?
)
The vast majority of cases of tracheomalacia resolve on their own within 2
years oflife. Mild cases should be followed. Empiric treatment ofGI reflux
should be considered. Cases that should be treated are more severe, espe-
cially those involving "reflex" apneas
Cases due to external compression (Type 2, as in this case) can be treated
by addressing the cause. In this case, aortopexy could be recommended.
The efficacy of aortopexy is somewhat controversial, but there is enough

259
)
Otolaryngology Clinical Case Studies Chapter 5 - Pediatric

NoTES empiric evidence of its efficacy to recommend it in a chi ld who is very


symptomatic and vascular compression is the culprit
BiPAP can also be effective especially for nighttime symptoms
Intraluminal stents have been used to treat tracheomalacia with limited suc-
cess. If the malacia is due to a short segment of diseased trachea, then
stenting is reasonable. Long segment tracheomalacia would not be as ame-
nable to stenting. One can even consider tracheal resection in this scenario
Tracheotomy can be used for severe cases of long segment tracheomalacia.
It can be combined with BiPAP to help treat the tracheomalacia

Surgical management of tracheomalacia:

Surgery is indicated for only severe cases. The surgery is problem-specific


The etiology of secondary tracheomalacia should be addressed; e.g., per-
forming an aortopexy
Short segment tracheomalacia (2-4 rings of trachea) can be treated with
intraluminal stents (e.g., Dumont)
• Tracheal resection can be considered for severe cases after other more
conservative techniques have been tried
Long segment tracheomalacia can be treated with BiPAP. If the patient
cannot tolerate nasal positive pressure, then tracheotomy with the addition
of BiPAP can be instituted. Often, tracheotomy alone can decrease inspira-
tory pressure gradients enough
.
to alleviate tracheomalacia without
.
BiPAP

Surgical techniques:

Highlights and pitfalls:


• Intraluminal stents can be employed for short-term (days to weeks) and
long-term (months to years) management of short segment tracheal dis-
ease. Stents are generally placed under direct vision, although certain types
can be placed fluoroscopically. In general, an appropriate stent is of the
right diameter, shape, and length. Most stents are designed to be placed in
the mainstem tracheal, although there are stents designed for the mainstem
bronchi and even Y-shaped stents to place at the carina. Complications are
related to airway obstruction, mucus plugging of the stent, stent migration,
and granulation tissue formation. Palmaz stents, used in the past, are made
of metal mesh. These stents were notorious for granulation tissue formation
and difficulty in removing after intermediate to long-term placement. Cur-
rently used stents are usuaJly of silicon
Tracheal resection with end-to-end anastomosis can be used for more se-
vere cases of tracheomalacia. In general, these cases will come from Type
~

2 and 3 etiologies. Conventional wisdom is that roughly 2 em of trachea can


be safe ly removed and repaired with primary anastomosis. Longer seg-
ments need other techniques such a pericardia) patch, slide tracheoplasty,
etc. Complications from tracheal resection are related to stenosis at the
new anastomosis or dehiscence of the suture line. Both problems are un-
common but when they occur can result in long-tem1 problems
Tracheostomy is also usually used for more severe cases and is often com-
bined with positive pressure support (e.g., BiPAP). Pitfalls of tracheostomy

260
Chapter 5 - Pediatric Otolaryngology Clinical Case Studies

related specifically to tracheomalacia include granulation tissue fonnation NOTES


at the distal tip of the tracheostomy tube. This can occur because the malacic
segment can constantly rub against the tracheostomy tube

How would you follow this patient?

Primary cases of tracheomalacia that are mild can be fo llowed clinically


with office visits and periodic bronchoscopy to assess the degree of
tracheomalacia. In this case, aortopexy would be strongly considered due
to the patient's episode of reflex apnea. The post-op management from the
otolaryngological perspective would be the same-office fo llow-up and pe-
riodic bronchoscopy to access the degree of tracheomalacia

References:

1. Messineo A, Filler RM. Tracheomalacia. Semin Pediatr Surg. 1994;3( 4):


253-8.
2. Carden KA, et al. Tracheomalacia and tracheobronchomalacia in children
and adults: An in depth review. Chest. 2005; 127(3):984-1005.

261
Otolaryngology Clinical Case Studies Chapter 5 - Pediatric

NOTES CASE 8- JUVENILE pAPILLOMA


Michelle Marcincuk, MD

A 29 month-old girl is brought lO your offke by her mother for persistent


hoarseness of over one years duration.

HPI: state that you would:

Obtain a detailed medical history beginning with a history of the present


illness

What additional historical information would you seel{? State that you would:

Ask questions that will suggest a differential diagnosis


Detem1ine whether the problem is due to a functional versus an anatomic
issue, infectious versus noninfectious etiology, and whether it is a static or
dynamic problem
Is the hoarseness continuous or intermittent?
Is there anything that brings on the hoarseness?
Is there a cough, shortness of breath, or vomiting?
Is it a "muffled" or '_'raspy" voice?
• Is she feeding well and gaining weight?
• Have there been any episodes of respiratory distress, cyanosis, or apnea?·
History of intubation?

Mother reports that the_hoarseness is contin"uous with no apparent causes.


No associated cough. She has had noisy breathing attributed to asthma or
croup by her pediatrician. The hoarseness and noisy breathing have be-
come progressively worse with no report of acute distress or cyanosis. She
is feeding well and has gained weight appropriately. ·

PlvJH: The child was the product of a normal term pregnancy and
delivery. No history of intubation
Allergies: NKDA
Medications: Occasional inhaler use for ''asthma"
FH: Negative
SH: Negative
ROS: Negative

What would you look for on PE? State that you would:

Perform a complete head and neck exam including vital signs and flexible
fiberoptic laryngoscopy
·• Due to the age of the child, a mirror exam is not likely to be tolerated

262
Chapter 5 - Pediatric Otolaryngology Clinical Case Studies

PE: NoTEs
](j_wj_s_: Normal
GA: 29 month-old child in NA D. "'Raspy., but not
'"breathy., voice. Mild ro moderate stridor on both
inspiration and expiration. Worse on inspiration
Ears: Normal
Nose and throat: Normal
Neck: Normal

Flexible fiberoptic laryngeal exam: irregulm~ pink, exophytic, peduncu-


lated and sessile masses on the false vocal cords, epiglottis and aryepig-
lot!ic folds. Similar appearing masses also present on the inferior swface
of the TVC. See Chest: clem~ Abdomen: normal (Figure 1).

·what diagnostics would you request? State that you would:

• Get a CXR to evaluate for pulmonary involvement


May consider ordering a CT or MRl of the neck or chest if the nature or
extent of the masses is unclear. The choice of imaging modality will depend
on the suspected nature of the lesion. For example, MRI may be used for
suspected vascular anomalies, whereas CT may be useful for other soft
tissue tumors

Test results:

CXR normal

What is your differential diagnosis?

) • Differential diagnosis should include juvenile papillomata of the larynx


Other pediatric laryngeal mass lesions which can cause stridor include he-
mangiomas, foreign bodies, and laryngeal cysts or granulomas

..j Diagnosis:

Based on the characteristic appearance of the lesions, the diagnosis is


juvenile papillomatosis of the larynx.
)
What would you do next?

Discuss the diagnosis, prognosis, and your recommendations with the


patient's mother
Emphasize that this is a chronic disease that may last several years, but
usually disappears around or before puberty
There is no cure, but current treatments may control the disease
Multiple surgeries for debridement are often necessary averaging 4.4
per year but varying greatly in number from one child to another
Removal may involve microdebridement or laser resection. Earlier surgi-
cal control may prevent the need for tracheotomy for airway obstruction

263
Otolaryngology Clinical Case Studies Chapter 5 - Pediatric

NcnEs The use of intralesional injection of cidofovir, an acyclic nucleoside


phosphonate with antiviral activity, is presented as an investigational
treatment option
Interferon, with its potential complications and unpredictable response,
is discussed but is not chosen as a therapeutic modality at this time
The Recurrent Respiratory Papilloma Task Force Practice Guidelines
for Management of Children with RRP does not find a role for routine
imaging with CT or MRJ in the initial diagnosis of papillomatosis. How-
ever, these imaging modalities may be usefu l in following pulmonary
extension of the disease in affected individuals

What are the treatment options and their complications? What would you do
and why?

Options for surgical removal include cold knife removal, excision with a
microdebrider, or laser excision
Cold knife excision allows specimens to be obtained for biopsy with minimal
damage to excised tissue
Excision with a microdebrider is a useful technique, but must be used with
care around the TVC to prevent unintentional mucosal damage that can
lead to scarring. When using the microdebrider, the surgery must be paused
periodically to achieve hemostasis with cotton pledgets. The laser has the
advantage of achieving concurrent hemostasis during the excision. How-
ever, it carries a small risk of aiJ"Way fire and must be used carefully to
avoid laser damage to other areas of the airway ·
• The C02 laser is chosen as the treatment of choice using spontaneous
respiration and intralesional cidofovjr
The anesthetic options include:
General anesthesia using a small, " laser-safe" endotracheal tube to
control the risk of fire and explosion. This presents. some limitation of
working space and visualization within the larynx
Jet ventilation. This option, according to some laryngologists, may be
associated with seeqing of papilloma virus lower in the airway
- ·Apneic anesthesi"a affords limited working time with the laser during
apneic pauses in ventilation
Spontaneous respiration ane~thesia. This technique requires very close
monitoring of blood oxygen saturation and vital signs and could require
emergency intubation

Surgical technique:

Highlights:
Thorough and carefi.1l consultation with the anesthesiologist pre-op is critical
The C02 laser is prepared and the power, spot size and pulse duration
adjusted. The test application is performed on an appropriate target
Anesthesia is induced
The suspension laryngoscopy is performed with the appropriate laryngoscope
The C02 laser is used to remove the papillomas. A biopsy with cupped
forceps is obtained at the first surgery for definitive pathologic diagnosis
and documentation

264
Chapter 5 - Pediatric Otolaryngology Clinical Case Studies

To avoid laryngeal web fonnation, the aggressiveness of the papilloma re- NOTES
moval is limited, particularly where raw opposing surfaces may lead to web
fonnation. Papilloma may have to be left behind on one opposing surface to
be removed during a subsequent procedure. This is especially important at
the anterior commissure, posterior commissure, and interarytenoid area
Intralesional injection of cidofovir may be used. A dose of steroids may be
given intraoperatively to decrease postoperative swelling

Adjuvant therapy may be indicated for individuals with severe recurrent papillo-
matosis. Severe disease is usually defined as requiring surgery more frequently
than 4 times a year for 2 years, distal extralaryngeal spread, or rapid regrowth
of the papillomas causing airway compromise. Adjuvant therapies may include
alpha-interferon, photodynamic therapy, indole-3-carbinol, or antiviral medications.

Pitfalls:
Laryngeal web formation
• Seeding of virus and papilloma growth in the lower airway
Airway obstruction or need for a tracheostomy
Rapid papilloma recurrence requiring multiple procedures
Intraoperative fire or explosion from laser use in the airway

How would you follow this individual? State that you would:

Schedule office visits regularly starting about once per month


• Re-operate as needed to control disease

Figures :

Figure 1: Left- papillomas of the glottic surface of the epiglottis and false vocal
cords
Right- papillomas of the aryepiglottic folds, false vocal cords, and
extending from the inferior surface of the TVC

265
Otolaryngology Clinical Case Studies Chapter 5 - Pediatric

NOTES References:

I. Silverman DA, Pitman MJ. Current diagnostic and management trends for
recurrent respiratory papillomatosis. Curr Opin Otolaryngol Head Neck Surg.
2004; 12(6):532- 7.
2. Silverberg MJ, Thorsen P, Lindeberg H, Ahdieh-Grant L, Shah KV. Clinical
course of recurrent respiratory papillomatosis in Danish children. Arch
Otolaryngol Head Neck Surg. 2004;130:7 11-6.
3. Patel N, Rowe M, Tunkel D. Treatment of recurrent respiratory papilloma-
tosis in chi ldren with the microdebrider. Ann Otol Rhino! Laryngol.
2003;1 12(1):7- 10.
4. Pasquale K, Wiatrak B, Woolley A, Lewis L. Microdebrider versus C02
laser removal of recurrent respiratory papillomas prospective analysis. Laryn-
goscope. 2003; 113(1 ): 139-43.

266
Chapter 5 - Pediatric Otolaryngology Clinical Case Studies

CASE 9-CHOANALATRESIA NoTEs


Brett A. Miles, DDS, MD
Orval Brown, MD

You are consulted to evaluate a 1 day-old female for respiratory distress


characterized by vigorous respiratory efforts. chest retractions and
cyanosis.

HPI: state that you would:

Obtain a more detailed medical history beginning with the birth history
What are the details of the bit1h history?
Is the stridor present only when the airway is obstructed or is the stridor
constant?
What is the character of the respiratory effort?
• Is the cyanosis cyclical or constant?
Is the cyanosis relieved by crying?
Is the cyanosis exacerbated during feeding?
What is the birth weight of the infant?

What additional historical information would you seek? State that you would ask:

Is there a history of congenital birth defects in the family?


Has the mother had a history of thyroid disease or recent treatment with
thyroid medications (i.e., methimazole, carbimazole)?

The mother states that the child was delivered vaginally without incident.
Prenatal testing was normal and did not reveal any suspected abnormali-
ties during the pregnancy. The birth weight of the infant was 6.5 lbs. The
mother denies any medical problems, including thyroid disease. There is
no known FH of congenital birth defects or. syndromes. The cyanosis ap-
pears to worsen during feedings, and when she is resting quietly (no cry-
ing), the infant appears to have obstructed breathing with chest wall re-
tractions and nasal flaring. The cyanosis appears to cycle between epi-
sodes of worsening cyanosis relieved by vigorous crying.

PMH: No known medical problems


PSH: No prior surgical procedures
Allergies: NKDA
Meds: None
FH: Negative
SH: Negative

What do you lool{ for on PE?

·, State that you would perform a complete head, neck, and PE including vital
signs and nasopharyngoscopy

267
Otolaryngology Clinical Case Studies Chapter 5 - Pediatric

NOTES PE:

Yiw.ls_: Temp: 98.6 F, BP: 55132, HR: 155, RR: 48, Wt: 6.5 lbs
Oxygen saturation 80% on room air
GA: Neonate with increased respiratory effort, some mild chest
retractions. Slightly cyanotic
Neuro: Responds appropriately to tactile and verbal stimuli. Re-
~ponds to painful stimulus with vigorous crying.
HEENT No cranial congenital anomalies or microcephaly noted
Eyes: Normal exam. Reactive pupils
Ears: No external abnormalities noted. AD - normal TlYJ intact.
AS - normal TM intact, no effusions noted
Nose: Moderate nasal flaring on inspiration: mucoid discharge
in nares bilaterally, otherwise WNL
OCIOP!
nas.al cavities: No masses, lesions, or other abnormalities, No palatal clefts
or defects noted
Nec k: Supple with no nodes or bruits
Chest: Slight retractions with inspiratory effort
Abdomen: Soft WNL
Extremities: Normal, no evidence of syndactyly, polydactyly
Genital: Normal female

What is your differential diagnosis?

Septal trauma during birth (partial or complete nasal obstruction)


Choana! atresia
Pyriform aperture stenosis
Congenital subglottic stenosis
Tracheoesophageal fistula
Congenital mass
Hemangioma
Glioma
Encephalocele
Hematoma

What do you do next? State that you would:

Stabil ize the infant with an oral airway/appliance


Ensure appropriate feeding (i.e., McGovern Nipple or NG tube)
Attempt to pass 6-8Fr NG tube
Suction nasal airway, apply oxymetazoline nasal spray.
P!::tce wisps of cotton in front of the nares and watch for the presence or
absence of nasal air movement
Alternatively, occlude mouth and place glass slide in front of nares, observe
for fogging
Endoscopic oronasal examination
Obtain maxillofacial CT scans in axial and coronal planes (suction nasal
airway, oxymetazoline spray prior to scanning)
Obtain cardiology consult, genetics and nutrition consults
Monitor for OSA with oximet1y, ECG, ABG as needed

268
Chapter 5 - Pediatric Otolaryngology Clinical Case Studies

Test results: NoTES

Endoscopic examination reveals bilateral narrowing of the nasal cavity


with thickening of the vome1~ and membranous obstn1ction of the choanae
bilaterallv.

CT scan confirms the findings of the endoscopic exam with bilateral nar-
rowing of the nasal cavity at the level of the choanae, thickening of the
vomer and deformation of the lateral pterygoid plates bilaterally. Approxi-
mately 60% osseous/40% membranous involvement of the choanae noted
on the scan.

Genetics consult is negative for congenital syndromes.

Cardiology consult negative for congenital cardiac defects.

Diagnosis?

Bilateral choana/ atresia, with approximately 60% osseous/40% membra-


nous involvement of the choanae.

What treatment would you begin?

• Surgical therapy: .
Transnasal endoscopiC repair is the most commonly used surgical mo-
dality although other methods are utilized such as laser ablation, curet-
tage, puncture techniques, transseptal, and transpalatal repair. Endo-
scopic transnasal surgery offers advantages in that it is less invasive,
offers less post-operative morbidity, and has fewer effects o~ oral in-
take· or speech in the post-operative period. Transpalatal approach of-
) fers superior access for difficult cases, especially with increasing thick-
ness of the osseous portion of the atretic plate. Direct visualization of
the operative field is another advantage of the transpalatal approach.
Proponents of the transpalatal appruach feel that a lower rate of re-
;! stenosis is achieved as opposed to the transnasal approach, although
this assertion remains controversial. The major disadvantage of the
transpalatal approach is disruption of the palatal suture which may re-
sult in growth disturbances or occlusal discrepancies
) • Surgical goals:
Removal of obstructing portion of posterior septum/vomer
Removal of superigr-lateral nasal wall and lateral pterygoid plate if
necessary
Preserve mucosal flaps to line lumen; may be beneficial, but remains
controversial

269
Otolaryngology Clinical Case Studies Chapter 5 - Pediatric

NOTES Surgical technique:

Highlights:
I. Place incisions at the osseocartilaginous junction at the level of the
maxillary crest. Proceed with elevation and mobilization of the muco-
periosteum of the posterior y, of the bony septum. Occasionally, inci-
sions on the contralateral side may be necessary for instrumentation
and visualization
2. Minimize dissection superiorly as this is unnecessary and may lead to
increased granulation tissue post-operatively. The middle turbinate may
be utilized as a guide for the superior limit of the dissection
3. Remove the maxillary crest on the involved side sequentially from ante-
rior to posterior. In younger patients, back-biting forceps, down-biting
forceps, or bone rongeurs are sufficient; drilling will be required in more
mature patients
4. Remove the posterior 1;3 of the vomer and the lower quarter of the
perpendicular plate in order to enlarge the choana! opening medially
5. Remove portions of the lateral pterygoid plate to enlarge the choana]
opening laterally
6. In bilateral cases, removal of the anterior wall, intersinus septa, and
floor of the sphenoid sinus, with ligation of the sphenopalatine artery,
may be necessary to achieve a common choana! opening

Pitfalls:
1. Granulation tissue - commonly recurs in the post-operative period re-
quiring revision surgery or laser ablation. May be minimized by limiting
surgical dissection. The role of post-operative steroids and mitomycin-
C in the prevention of post-operative granulation Ussue remains unclear
2. Septal perforation may persist, especially in bilateral cases. Repair with
traditional/endoscopic techniques
3. Hemorrhage- Can be significant due to the blood supply of the region.
Intraoperative cautery and ligation of the sphenopalatine artery may be
required
· 4. Infection - Rare, most cases can be treated successfully with post-
operative oral antibiotics
5. CSF leak- Rare, managed with traditional endoscopic sinus surgery
techniques
6. Avoid transpalatal surgery in growing child due to occlusal/growth
considerations

~ostoperative therapy:

Although controversial, consider post-operative stenting (i.e., endotracheal


tubes/Foley catheter) 4- 6 weeks
• If stents are utilized, maintain the stents with frequent suction and a saline
irrigation
Antibiotic therapy should be administered to reduce risk of purulent rhinorrhea
Anti-reflux therapy should be administered to reduce the incidence of granu-
lations and stenosis

270
Chapter 5 - Pediatric Otolaryngology Clinical Case Studies

How would you follow this patient? NoTES

The patient should be exam ined every 2-3 weeks (subsequent to nasal
stent removal if indicated) with flexib le endoscopic examination and dilation
procedures as needed
If cicatricial banding or severe re-stenosis occurs, revision surgery may be
required

References:

1. Samadi DS, Shah UK, Handler SD. Choana! atresia: a twenty-year review
of medical co-mor bidities and surg ical outcomes. Laryngoscope.
2003; I 13(2):254-8.
2. Brown OE, Pownell P, Manning SC. Choana] atresia: a new anatomic clas-
sification and cli nica l managemen t applications. Laryngoscope.
1996; 106(1 ):97- 101.
3. Park AH, Brockenbrough J, Stankiewicz J. Endoscopic versus traditional
approaches to choana! atres ia. Otolaryngol Clin NorthAm. 2000;33 :77- 90.
4. Cedin ~C, Peixoto Rocha JF Jr., Depperrnann MB, Moraes Manzano PA,
Murao M, Shimuta AS . Transnasal endoscopic surgery of choana! atresia
without the use of stents. Laryngoscope. 2002(4); 112:750- 2.

27 1
Otolaryngology Clinical Case Studies Chapter 5 - Pediatric

NOTES

272
Chapter 6- Rhinology Otolaryngology Clinical Case Studies

CHAPTER 6/CASE 1- RECURRENT SINUSITIS NoTEs


Amber Luong, MD, PhD

A 5 year-old white male presents with a ]-week history offoul-smel!ing


nasal purulem drainage and a nocturnal cough.

HPI: state that you would:

• Obtain a more detailed birth, medical and rhinologic history beginning with
the chief complaint

What additional historical information would you seek? State that you would ask:

• Any history of prior episode-s of symptoms?


• Prior treatment regimens and their outcomes
• Any history of fevers, chills or evidence of systemic involvement?
• Any history of nasal airway obstruction?
• Any reGent trauma or epistaxis?
Any history of foreign body in the nasal cavity?
Any allergy symptoms?
• Any smoke exposure?
• Daycare attendance?
• Describe the cough. Productive?

Initially, the parents noted the cough, which is non-productive. The puru-
lent nasal discharge was noted soon afterwards. There ·is no history of any
fevers or chills. Howeve1~ mom believes that he has appeared less ener-
. getic. No trauma or epistaxis. The child dem·~s placing anything in his
nose. He has a history of recurrent purulent nasal drainage and cough.
Last year, he was treated on 4 separate occasions with antibiotics that
would stop the purulent drainage and cough for several weeks. He suffers
from seasonal allergy symptoms including itchy eyes, rhinitis, nasal air-
way obstruction and sneezing. His parents do ·not smoke and he is ·not
regularly exposed to second-hand smoke.

PMH: FTwithout any birth complications, no known medical prob-


lems; no history of hospitalizations or ER visits. History of
recurrent sinusitis. Had been treated for pneumonia at age 3
PSH: Tonsillectomy and adenoidectomy at age 4 for snoring and
nasal breathing, bilateral grommet tubes placed at 8
months-old
Allergies: NKDA
Medications: Amoxicillin and clavulanate, Flonase
FH: Father with allergies. Mother with no known medical
issues
SH: Lives with both parents and a younger sister
The patient just started kindergarten about 1 month ago.
Prior to kindergarten, he attended daycare
ROS: No vision changes, GI complaints, changes in bowel hab-
its, urinary problems, asthma symptoms

273
Otolaryngology Clinical Case Studies Chapter 6 - Rhinology

NoTES What do you look for on PE?

• State that you would perfonn a complete head, neck, and rhinology PE in-
cluding vital signs and nasal endoscopy

PE:

.l!'i1a.f.s_: Temp: 98.6 F. BP: 105152, HR: 90, RR: 18, Wt: 53 !bs,
Ht: 46 in
GA: Shy male in NA D and othe1-v.,ise appearing healthy
Head: Normal cephalic, no asymmetry
Ears: AU - normal, anterior focus of myringosclerosis, normal
mobility, no MEE, no masses
AS - same as AD
Eyes: No proptosis/telecanthus, no restricted gaze. no diplopia,
normal exam
Nasal cavity: Right cavity with edematous mucosa, congested, turbinate
hypertrophy. Left cavity with nasal polyps extending from
maxillary antrostomy and ethmoid .cells. Edematous mucosa
and turbinate hypertrophy. Purulent rhinorrhea noted from
both cavities
Nasal: After decongestant applied, a nasal polyp was noted on
the right endoscopy extending from the posterior ethmoid
cells
OC: No masses, lesions, or other abnormalities, no obvious post-
nasal drainage
Neck: Supple with no nodes or bruits
Neuro: CN normal

What diagnostics would. you request?

• Sweat test and/or genetic testing for CF


• Allergy consl].ltation
• Labs- CBC with differential, immunoglobulin panel
• CT without contrast of the sir.uses

What is your differential diagnosis?

• Recurrent rhinosinusitis with nasal polyposis - The above history is consis-


tent with temporal and symptom criteria for recurrent rhinosinusitis as de-
fined and supported by the American Academy of Pediatrics, AAOHNS .
Recurrent is identified as 3 episodes of acute bacterial sinusitis in 6 months
9

or more than 4 episodes in I year separated by symptom-free periods. Acute


bacterial sinusitis is diagnosed by clinical symptoms such as nasal discharge,
postnasal drainage or cough that persist for longer than 10 days or URJ
symptoms associated with severe symptoms such as elevated temperature
late in the course of an infection. Recurrent rhinosinusitis in children is often
a consequence of attendance at day care, allergic and nonallergic rhinitis,
CF, an immunodeficiency disorder, ciliary dyskinesia or an anatomical defect.
Therefore, the diagnosis of recurrent rhinosinusitis, especially with nasal pol-
yps, in a child should prompt an evaluation for the predisposing factor

274
Chapter 6 - Rhinology Otolaryngology Clinical Case Studies

• CF is often diagnosed before rhinosinusitis develops and usually before the NOTES
age of2. However, some patienrs with mild clinical fom1s ofCF can present
with pansinusitis. All ch ildren with nasal polyps should be screened for CF.
The incidence of pansinusitis in CF patients is I00%. Approximately 6-50%
of CF patients suffer from nasal polyps. CF can be diagnosed by a Gibson-
Cooke sweat chloride test, a quantitative pilocarpine iontophoresis sweat
test or genetic testing
• Immunodeficiency - There are about 70 primary immune deficiency dis-
eases. The hallmark of these diseases is increased susceptibility to infections
and hence they are a cause of recurrent infections. The type of immunoglo-
bul in deficiency influences the type of recurrent infections. Recurrent sinus
infections are more commonly associated with immunoglobulin G4 deficiency
as compared to deficiencies in other types of immunoglobulins
·• Dysfunction or abnormality of ciliary structure as in Kartagener syndrome -
Kartagener syndrome is an autosomal recessive disease defined by situs
inversus, chronic sinusitis and bronchiectasis.lt is also refeJTed to as primary
ciliary dyskinesia. Poor mucociliary clearance results in chronic upper and
lower airway disease. Nasal polyps are present in approximately 30% patients
Allergic rhinitis- Allergic rhinitis is commonly associated with and often
identified as a risk factor for recurrent sinusitis. Studies on nasal airflow and
mucociliary clearance show impaired function more commonly in allergic
patients than non-allergic patients
• Antibiotic-resistant bacteria accummulation - Bacteriology studies in pa-
tients with recurrent sinusitis have shown a higher incidence of gram-nega-
tive rods and beta-lactamase-producing strains of Haemophilus influenzae
and Branhamella (Moraxella) catarrhalis. Therefore, culture of sinus con-
tents and culture-directed antibiotics may reduce the incidence of recurrent
sinusitis. Antibiotic-resistant bacteria are more conm1only associated with CRS
• GERD can present with atypical symptoms including cough and
hyperphlegmia, which overlap with rhinosinusitis symptoms. In addition, GERD
can be an initiating and contributing factor in rhinosinusitis

Results:

• Sweat test showed a sodium concentration of 83 mmol/L. Sodium con-


centration of greater than 60 mmol/L is diagnostic of CF in addition,
CF can be diagnosed by DNA-based genetic test. The test is based on
screening the most common mutations associated with CF in the CF
transmembrane regulator gene
• Allergy evaluation was negative for atopic disease
• The CBC was WNL. There was no elevation of white blood cells. There
was also no evidence of pancytopenia. The immunoglobulin levels were
all WNL for this child
• CT scans in CF patients will always show paranasal sinus disease, which
does not correlate with clinical symptoms. A CT scan is not necessary
for the diagnosis of rhinosinusitis. Howeve1~ in a patient who will likely
require surgical intervention, a CT scan provides anatomical informa-
tion. The CT scan revealed complete opacification of bilateral maxillary
and ethmoid sinuses. No evidence of bony erosion or abnormal anatomy

Diagnosis?

Recurrent sinusitis with nasal polyposis as a manifestation of CF

275
Otolaryngology Clinical Case Studies Chapter 6- Rhinology

NOTES What treatment would you begin?

• Antibiotics are the mainstay treatment for the episode of acute rhinosinusitis.
In this patient with CF and history of recurrent rhinosinusitis, antibiotic-resis-
tant bacterial strains or infections with Pseudomonas aeruginosa are not
uncommon. Purulence from the maxillary cavity should be cultured. The
patient should be empirically placed on high-d0se Augmentin.lf no response
is noted in 48-72 hours, Augmentin should be changed to a fluoroquinolone
such as Levaquin to cover Pseudomonas, unless culture results can better
dictate the choice of antibiotic. A I 0-14 day course is normally recommended;
however, the data is poor regarding the optimal duration of therapy. Some
follow the clinical symptoms and continue antibiotics for at least 1 week
after the resolution of sinus symptoms. In a patient who has been treated
with several courses of antibiotics to treat acute rhinosinusitis, consider ob-
taining sinus cultures. This information may be important if the patient fails
empiric antibiotic therapy
• Controversy remains around the role and timing of sinus surgery in this pa-
tient population. FESS in patients with CF and nasal polyposis has been shown
to improve quality oflife and facilitate application of topical antibiotics and
steroids. Other studies have shown no statistical improvement in pulmonary
function or reduced need for systemic or topical steroids. Some studies have
shown a decreased recurrence of acute sinusitis after FESS. With recurrent
acute rhinosinusitis without decrease in recurrence rate with antibiotics and
1isk factors for developing CRS, there are several indications for FESS in
this patient. The CT scan will be important for anatomical information dur~
ing surgery. The CT scan information can also be utilized for image-guided
. sinus surgery. Prior to surgery and after resolution of t~e acute bacterial
sinus infection, a course of systemic steroids and intranasal topical steroids
may control the n~sal polyps and intranasal mucosal inflammation
• Contact pediatrician or make referral for management and further work-up
of newly diagnosed CF. CF is an autosomal recessive disorder that is char-
acterized by dysfunction of epithelialized surfaces. This is a result of muta-
tions in the CFTR gene that is a chloride transporter. This chloride channel
protein is expressed in airways, the gastrointe.stinal tract, the genitourinary
tract and sweat glands. Con·sequently, patients can present with complica-
tions in any or all.ofthese systems

Surgical management of nasal polyposis:

• The goal of surgical management is to remove nasal polyps, to improve ap-


plication of topical antibiotics and steroids and to restore normal ventilation
or mucociliary clearance. In a recent study in CRS, levels of cytokines impli-
cated in perpetuating the inflammatory cycle and antigen burden withiJlothe
nasal cavity were reduced after FESS. Although surgery is not designed as a·
cure for this complex disease, a majority of patients who undergo FESS have
improvements in sinus symptoms
• Post-op care:
- Consists of routine clinic visits with nasal endoscopic exams to evaluate
for synechia and state of paranasal mucosa
- Typically, the medical therapy regimen that was initiated pre-operatively
is continued after surgery. Changes in medical regimen are made as
needed and are dictated by symptoms

276
Chapter 6- Rhinology Otolaryngology Clinical Case Studies

- Controversy exists as to the post-operative care after FESS. Some ad- NOTES
vocate aggressive debridement whi le others recommend no further ma-
nipulation to minimize post-surgical inflammation. In a 5 year-old child,
aggressive sinus debridement will be limited practically
- Although no prospective studies exist on post-operative antibiotics, many
prescribe a short 7- 10 day course of antibiotics
- In addition, systemic steroids that are tapered over 3 weeks are often
advocated to minimize post-surgical inflammation. Again, no controlled
studies have evaluated this therapy

S urgical techniques:

• FESS

Highlights:
• CT sinus sq.ns in the coronal plane must be available in the OR. If an
i·mage-guided navigation system is to be used in surgery, then these CT im-
ages must be obtained through the appropriate protocol. Image-guided sys-
tems can be helpful in cases where the anatomy is distorted such as in revi-
sion sinus surgery, extensive nasal polyps and tumors ·
• Inject local anesthesia to assist with hemostasis- Using 1% lidocaine with
1:100,000 epinephrine, the following sites are injected: greater palatine fora-
men, anterior and superior aspect of middle turbinate, insertion site of unci-
nate process and anterior face of ethmoid bulla
• Place Afrin-soaked pledgets in each nasal cavity for decongestion
• Consider septoplasty if a deviated septum prevents adequate visualization
and access of the sinuses
• Medialization of middle turbinate for access ofparanasal sinuses. Some sur-
geons will suture the middle turbinates to the septum
• Proceed with the uncinectomy
Maxillary antrostoniy- The natural ostium i~ probed to identify its location.
The antrostomy is performed, not only to provide ventilation, but also to serve
as a landmark for the orbit
• Anterior ethmoidectomy- The ethmoid bulla is the largest anterior ethmoid
cell. lt is entered at an inferior and medial site to minimize risk to orbit. The
lateral wall of ethmoid bulla is the lamina papyracea
• Identify basal lamella- This structure represents the vertical attachment of
the middle turbinate and separates the anterior ethmoid cells from the poste-
rior ethmoid cells. Remove basal lamella without disrupting horizontal basal
lamella, which exposes branches of internal maxillary artery and can cause
unnecessary bleeding
• Posterior ethmoidectomy- The skull base is normally widely exposed after
a posterior ethrroidectomy. Identify any sphenoethmoidal cell (Onodi cell),
which can extend superior and lateral to the sphenoid sinus. This cell can
house a dehiscent optic nerve. After the skull base is identified within the
sphenoid sinus, a posterior to anterior dissection along the skull base is per-
formed to complete the total ethmoidectomy. Avoid dissection medial to the
middle turbinate where the cribriform plate can lie inferior to the roof of the
ethmoid sinus
• Sphenoid sinusotomy as needed - Identify sphenoid ostium either lateral to
middle turbinate after the inferior portion of superior turbinate has been re-
moved or medial to the middle turbinate at the level of the maxillary ostium. Be

277
Otolaryngology Clinical Case Studies Chapter 6- Rhinology

NOTES aware of the location of the carotid artery, optic nerve and skull base once the
sphenoid sinus is entered. In this case, a sphenoid sinusotomy is not necessary
• Frontal sinusotomy as needed - At 5 years of age, the frontal sinus is not
completely developed. However, if a frontal sinusotomy is necessary, open
frontal recess carefully since this is the site of greatest scarring. It is bordered
anteriorly by the posterior wall of the agger nasi cell, medially by the anterior
insertion site of the middle turbinate and laterally by the lamina papyracea

Pitfalls:
• Com pi ications are typically categorized as either major or minor. Major com-
plications are rare and are reported as less than I%. These include blindness
and intracranial violation. The minor complications such as synechia are also
uncommon at S-8%
• Orbital complications including optic nerve injury, orbital hematoma, and vio-
lation oflamina papyracea
- Violation oflamina papyracea with herniation of orbital fat does not ne-
cessitate cessation of surgery. The site of dehiscence need only be
avoided. To avoid damage, always use the microdebrider tangential to
the lamina papyracea
- Orbital hematomas need to be addressed aggressively to reduce intraor-
bital pressure and control bleeding. Removal of any nasal packing and a
lateral canthotomy and cantholysis can reduce intraorbital pressures.
Elevated intraorbital pressure longer than 60-90 minutes can result in
permanent blindness
• Intracranial violation with CSF leak
- The fovea ethmoidal is and cribriform plate, which is commonly inferior
to the roof of the ethmoid sinus, is prone to damage. One should avoid
superior dissection medial to the insertion ofthe middle turbinate
- The medial skull base has much thinner bone than the lateral counterpart
- Any intraoperative CSF leak should be repaired immediately

How would you follow this patient:

• Patients with CF are prone to recurrent rhinosinusitis, which often progresses


to CRS. Aggressive medical management is important; however, many pa-
tients require revision surgery. Such medical therapy includes regular clinic
visits for surveillance endoscopy and possible debridement. Progression of
clinical symptoms may necessitate a repeat CT S.~~,;.,];!1.e.,.J~~-'t:r~qce_ of
nasal polyps may require revision FESS
• As mentioned above, CF is a multisystem disease. Therefore, a pediatrician
fam iliar with managing CF needs to follow the patient for development of
other organ involvements •

References:

I. Bikhazi NB. Contempora1y management of nasal polyps. Otolaryngol Clin


North Am. 2004;37:327-37.
2. Tandon R & Derkay C. ContemporaJy management of rhinosinusitis and
cystic fibrosis. Curr Opin Otola1y ngo l Head Neck Surg. 2003; II (I ):4 1-4.

278
Chapter 6- Rhinology Otolaryngology Clinical Case Studies

CASE 2 - CHRONIC SINUSITIS NOTES


Amber Luong, MD, PhD

A 36 year-old WF presents with a chief complaint of right greater than left


nasal airway obstruction thai has worsened over !he last 6 monlhs.. She
reports persistent post-nasal drainage that is worse when she lies down to
sleep. She finds herself always clearing her throat and struggles wilh a
chronic cough. Frontal headaches are a weekly occurrence and most no-
table when she feels f acial pressure.

HPI: state that you would:

• Obtain a more detailed rhino logic and medical history beginning with the
chief complaint

What additional historical information would you seek? State that you would ask:

• How long has she suffered from this constellation of symptoms?


• Has there been a change in her environment in the last 6 months?
• Does she suffer from nasal drainage and, if so, describe the type of drainage
• Does she have a productive cough?
• Any associated neurologic complaints in addition to the headaches?
• Is her sense of smell altered?
• Any history of asthma, aspirin sensitivity, allergies?
Does she smoke or is she exposed to smoke? .
• What current medications has she tried to control her symptoms?
• Has she experienced any trau.ma or surgery to her nose?

She has suffered from these symptoms for a couple of years, but these
symptoms have been most notable over the last 6 months after moving to
Texas. She has 2 indoor cats. In addition to the post-nasal drainage, _she
describes intermittent purulent discharge on blowing her nose. The pus
resolves with a· l 0-day course of antibiotics, which are p rescribed by her
primary care doctor. Her last course of antibiotics was several months
ago. Her cough is non-productive. She denies smoking but is exposed to
her husband's second-hand smoke. Her headaches are throbbing and ex-
tend across her forehead and behind her eyes. She denies any prodromal
symptoms. No other neurologic complaints associated with her headaches.
She suffers from asthma and thinks she has seasonal allergies, but has
never been formally evaluated for allergies. She has no problems with
aspirin. She uses inhalers to control her asthma. When her facial pressure
is severe, she takes over-the-counter "sinus medicines, " which relieve her
nasal congestion. She has a nasal steroid spray at home, which she takes
occasionally. She has never had a broken nose or surgery on her nose.
Howeve1~ she has heard about sinus surgery and is very interested in it.

PMH: Asthma with 2 ER visits for exacerbation of asthma symp-


toms. Last ER visit was 5 years ago. No hospitalizmions

279
Otolaryngology Clinical Case Studies Chapter 6 - Rhinology

NoTES PSH: No prior history of nasal or sinus surgery


Allergies: PCN
lvledications: Pseudoephedrine to help with nasal ain-vay obstruction on
occasion, albuterol inhaler. Advair. occasional Nasonex
and Sudafed
FH: Farher is treated for asthma and her mother suffers from
perennial allergies
SH: Administrative secretary. Married. 2 children. No HIV
risk factors. No history of tobacco use. Social drinker
She is exposed to second-hand smoke from her husband
ROS: Negative

What do you look for on PE?

• State that you would perform a complete head, neck, neurology and rhinol-
ogy PE including vital signs, otoscopy, fiberoptic laryngoscopy, and nasal
endoscopy with and without decongestant

PE:

Temp: 98.6 F. BP: 111/75, Pulse: 74, RR: 18,


Wt: ·132 lbs.
GA: Caucasian female in NAD and otherwise appear-
ing healthy; her speech is hyponasal
Face: Mild pain on palpation over maxillary and frontal
sinuses
Eyes: Pupils equal, round, reactive to light, no conjunc-
tivitis
Ears: AD - normal clear TM. normal mobility, no middle
ear effusion
AS - same as AD
Nose: Normal external exam without grossly visible mass,
aligned nasal dorsum, no external deformity
Nasal endoscopy: Septum deviated to the right with inferior spur,
edematous mucosa, turbinate hypertrophy. De-
creased mucosal edema with decongestant appli-
cation. Bilateral pus noted lateral to middle turbi-
nate. No polyps. NP clears with patent eustachian
tube
OC: No masses or lesions. Post-nasal mucous drain-
age, cobblestoning of OP
Neck: Supple with no nodes or bruits
Fiberoptic
laryngoscopy: NP and OP clem~ bilateral true vocal cord mobile,
no erythema or edema suggestive of laryngopha-
ryngeal reflux, no masses/ lesions
New-a: CN normal

280
Chapter 6 - Rhinology Otolaryngology Clinical Case Studies

What diagnostics would you request? NOTES

• Labs- CBC with differential


• Perform allergy evaluation with RAST or skin test
• CT of sinuses without contrast in the coronal orientation -This is obtained
while the patient is on maximal medical therapy

Test results:

• CBC is normal without a shiji


• Allergy testing indicates elevated IgE levels to various antigens includ-
ing animal dander and various grasses
• CT scan reveals a deviated septum to the right and complete opacifica-
tion of bilateral maxillary sinuses with obstruction of the osteomeatal
complex. There is opacification of many of the ethmoid cells. Anterior
ethmoid disease is obstructing the frontal recesses with mucosal thick-
ening of the frontal sinuses. Sphenoid sinuses are clear

What is your differential diagnosis?

• The constellation of symptoms and physical fmdings characterize rhinosinusitis.


In 1996, a Rhinosinusitis Task Force, estab lished by the AAO-HNS, identi-
fied major and minor symptoms that are associated with rhinosinusitis. A
diagnosis of rhinosinusitis included 2 or more major factors or J major and 2
minor factors. The major factors are facial pain/pressure, nasal obstruction,
nasal drainage, hyposmia/anosmia, findings af.purulence 6n nasal cavity exam
and fever. The minor factors include headache, fever, halitosis, fatigue, den-
tal pain, cough and ear pressure/otalgia. In 2002, the above criteria were
revised for CRS and included the need to demonstrate on PE evidence of
inflammation. Rhinosinusitis is clinically divided into 5 categories based on
duration of symptoms: acute, subacute, chronic, recurrent and acute exacer-
bations of chronic disease. Acute is defined as less than 4 weeks, subacute
has symptoms persisting for 4- 12 weeks and CRS involves symptoms for
longer than 12 weeks. Based on the duration of this patient's symptoms, she
presents with CRS
• GERD can present with atypical symptoms includ ing cough and
hyperphlegmia, which overlap with rhinosinusitis symptoms. In addition, GERD
can be an initiating factor for rhinosinusitis

Diagnosis?

CRS

What treatment would you begin?

• CRS is a broad term describing·an inflammatory condition that can result


from a number of diverse etiologies. CRS resu lts from interplay among envi-
ronmental factors, host immune system, and anatomical configurations within
the nasal cavity and paranasal sinuses. Despite the complex interaction of

281
Otolaryngology Clinical Case Studies Chapter 6- Rhinology

NoTES the above factors, CRS is generally well managed by medical therapy. Treat-
ment of the underlying etiology is addressed if known. Oftentimes, CRS
develops from a combination of different factors. Therefore, the treatment
ofCRS centers on controll ing the inflammatory reaction within the nasal and
paranasal cavities. The goals of medical management are to reduce symp-
toms and to improve quality of life, not to normalize CT scans
- Antibiotics- The role of antibiotics in CRS is controversial. Although
CRS is primarily recognized as an inflammatory disease, bacteria are
thought to perpen1ate the cycle. Therefore, antibiotics are still commonly
prescribed in patients who have not been recently treated. Beta-lactamase
resistant Staphylococcus aureus, Streptococcal pneumoniae, gram-
negative rods such as Pseudomonas aeruginosa and anaerobes are
associated with CRS. First-line antibiotics include amoxicillin with
clavulanate, cefuroxime or clarithromycin. A 21 -day course is normally
recommended; however, the data is poor regarding the optimal duration
of therapy
- Nasal steroid spray- Topical steroids are effective in controlling local
nasal mucosal inflammation. The patient is currently on Nasonex, but is
· not compliant with daily usage. The need for a topical corticosteroid
needs to be emphasized to the patient. Nasal steroid sprays are pre-
scribed as 1 spray in each nostril daily. This can be increased to twice a
day as needed .
- Sinus irrigation - Normal saline irrigation solution or nasal saline spray is
recommendl':<d for mechanical cleansing of the sinuses. This is particu-
larly important in patients in whom visible rhinorrhea is noted on PE.
Using a bulb syringe or Water Pik nasal irrigation attachment, 0.9%
saline solution is directed toward the inner margin of the eye within the
nasal cavity. The solution is then allowed to passively drain from the
nose and mouth. This irrigation should be performed daily
Oral decongestants- Symptomatic treatment of nasal congestion can
be relieved with oral or topical decongestants. Since long-term topical
decongestants can be addictive and cause rebound·congestion, oral de-
congestants are more often ~ecommended if not contraindicated by the
presence.of coronary artery disease or uncontrolled HTN
• Managing allergies
- Provocation of allergies resuIts not only in allergy symptoms but can also
cause radiographic changes consistent with rhinosinusitis, suggesting a
link between allergies and CRS. In this patient with CRS and allergies,
control of allergy symptoms will be important in the management of her
CRS
- Antihistamine - Serves to manage allergy symptoms, which can per-
petuate nasal and paranasal inflammation
- Leukotriene inhibitor- Leukotrienes have pro-inflammatory capabilities
and promote eosinophilic inflammation in the airway and sinuses, char-
acteristic of allergies and asthma. Total serum eosinophil level, which
correlates with symptoms, was significantly reduced in clinical trials with
leukotriene inhibitors
- Smoke can often iiTitate the nasal mucosa. Therefore, recommenda-
tions to reduce her exposure to tobacco smoke should be given

282
Chapter 6- Rhinology Otolaryngology Clinical Case Studies

- Consider immunotherapy - Immunotherapy induces tolerance to antigens NoTES


by ultimately causing a reduction in production ofT-cell cytokines such as
IL-5 and IL-13 that are actively involved in the inflammation associated
with allergies. No specific studies have evaluated the role of immuno-
therapy in managing CRS symptoms in patients without allergies. There-
fore, its use should be limited to patients with CRS and allergies

Surgical management of CRS:

• FESS - CRS recalcitrant to medical therapy is a relative indication for a


FESS. If nasal endoscopy is inconclusive or surgery is contemplated after
maximization of medical therapy, then proceed with CT scan. The informa-
tion on the CT scan can often complement the nasal endoscopy findings.
However, it has been shown that CT scan findings correlate poorly with
sinus symptoms. The Lund-Mackay staging system can be applied to the CT
scan findings to provide an objective measure for the disease process. Patients
with persistent symptoms despite med ical therapy and anatomical anomalies
that can cause nasal obstruction and/or opacified sinuses with osteomeatal
complex obstruction, may benefit from FESS
• The goal of surgical management is to restore normal ventilation or mucociliary
clearance. In addition, FESS, in recent studies, may reduce the levels of
cytokines perpetuating the inflan1matory cycle and decrease the antigen bur-
den within the nasal cavity. Although surgery is not designed as a cure for
this complex disease, 80-90% of patients who undergo FESS report im-
provement in sinus symptoms
• Post-op care:
- Consists of routine clinic visits with nasal endoscopic exams to evaluate
for synechia and state of paranasal mucosa
- Typically, the medical therapy regimen that was initiated pre-operatively
is continued after surgery. Changes in medical regimen are made as
needed and as dictated by symptoms
- Controversy exists as to the post-operative care after FESS. Some ad-
vocate aggressive debridement while others recommend no further ma-
nipulation to minimize post-surgical inflammation
- Although no prospective studies exist on post-operative antibiotics, some
prescribe a short 7- 10 day course of antibiotics, especially if pus is noted
during surgery
- ln addition, systemic steroids that are tapered over 3 weeks are often
advocated to minimize post-surgical inflammation. Again, no controlled
studies have evaluated this therapy

Surgical techniques:

• FESS

Highlights:
• CT sinus scans in the corona l plane must be available in the OR. If an
image-guided navigation system is to be used in surgery, then these CT
images must be obtained through the appropriate protocol. Image-guided

283
Otolaryngology Clinical Case Studies Chapter 6 -Rhinology

NOTES systems can be helpful in cases where the anatomy is distorted such as in
revision sinus surgery, extensive nasal polyps and tumors
• Inject local anesthesia to assist with hemostasis- Using 1% lidocaine with
I: I00,000 epinephrine, the fo llowing sites are injected: greater palatine fora-
men, anterior and superior aspect of middle turbinate, insertion site of uncinate
process and anterior face of ethmoid bulla
• Place Afrin-soaked pledgets in each nasal cavity for decongestion
• Consider septoplasty if a deviated septum prevents adequate visualization of,
and access to, the sinuses
• Medial ization of middle turb inate for access of paranasal s inuses. Some sur-
geons will suture the middle turbinates to the septum
• Proceed with the uncinectomy
• Maxillary antrostomy- The natural ostium is probed to identify its location.
The antrostomy is performed not only to provide ventilation but also to serve
as a landmark for the orbit
• Anterior ethmoidectomy- The ethmoid bulla is the largest anterior ethmoid
cell. It is entered at an inferior and medial site to minimize risk to orbit. The
lateral wall of ethmoid bulla is the lamina papyracea
• Identify basal lamella -This structure· represents the vertical attachment of
the middle turbinate and separates the anterior ethmoid cells from the poste-
rior ethmoid cells. Remove basal lamella without disrupting horizontal basal
lamella, which would expose branches of internal maxillary artery and could
cause unnecessary bleeding
• Posterior eth~oidectomy- 'J:he skull base is normally widely exposed after
a posterior ethmoidectomy. Identify any sphenoethmoidal cell (Onodi cell),
which can extend superior and lateral to the sphenoid sinus. This cell can
hous~ a dehiscent optic nerve. After the skull base is identified, a posterior to
anterior dissection along the skull base is performed to complete the total
ethmoidectomy. Avoid dissection medial to the middle turbinate ':"here the
cribrifom1 plate can lie inferior to the roof of the ethmoid sinus
• Sphenoid sinusotomy as needed - Identify sphenoid ostium either lateral to
middle turbinate after the inferior portion of superior turbinate has been re-
. moved or medial to the middle turbinate at the level ofthe maxillary ostium. Be
aware of the location of the carotid artery, optic nerve and skull base once the
sphenoid sinus is entered. In this case, a sphenoid sinusotomy is not necessary
• Frontal sinusotomy as needed -Open frontal recess carefully since this is
the site of greatest scarring. It is bordered anteriorly by the posterior wall of
the agger nasi cell, medially by the anterior insertion site of the middle turbi-
nate and laterally by the lamina papyracea

Pitfalls:
• Complications are typically categorized as either major or minor. Maj or com-
plications are rare and are reported as less than 1%. These include blindness
and intracranial violation. The minor complications such as synechia are also
uncommon at 5-8 %
• Orbital complications including optic nerve injury, orbital hematoma, and vio-
lation oflamina papyracea
- Violation of lamina papyracea with hemiation of orbital fat does not ne-
cessitate cessation of surgery. The site of deh iscence need only be

284
Chapter 6- Rhinology Otolaryngology Clinical Case Studies

avoided. To avoid damage. always use the microdebrider tangential to NOTES


the lamina papyracea
- Orbital hematomas need to be addressed aggressively to reduce intraor-
bital pressure and control bleeding. Removal of any nasal packing and a
.lateral canthotomy and cantholysis can reduce intraorbital pressures.
Elevated intraorbital pressure longer than 60-90 min·1tes can result in
permanent blindness
• Intracranial violation with CSF leak
- The fovea ethmoidal is and cribriform plate, which is commonly inferior
to the roof of the ethmoid sinus, are prone to damage. One should avoid
superior dissection medial to the insertion of the middle turbinate
- The medial skull base has much thinner bone than the lateral counterpart
- Any intraoperative CSF leak shou ld be repaired intraoperatively

How would you follow this patient:

• It should be emphasized that CRS and allergies are often life-long ailments
for which a cure is not available. Rather, medical and surgical management
is geared toward minimizing symptoms and improving quality oflife. This will
require regular clinical evaluations and often various changes in the medical
regimen based on the spectrum of symptoms. Surgery is utilized when signs
and symptoms become recalcitrant to medical therapy
• With evidence of allergies, this patient should be refen-ed and followed regu-
larly by an allergist
• Regular clinic visits with evaluation of symptoms and nasal mucosa will dic-
tate changes in medical management. Acute exacerbation of symptoms is
treated with short-qurst systemic steroids. If purulent drainage is noted dur-
ing exacerbation, a 7-14 day course of antibiotics is often prescribed
• Again, if signs and symptoms worsen despite changes in and maximization
of medical therapy, then a repeat CT scan of the sinuses and possible revi-
sion surgery may be necessary

References:

1. Kaliner MA, Osguthorpe JD, Fireman P, et al. Sinusitis: bench to bedside. J


Allergy Clin Immunol. 1997;99:8829-47.
2. Pelikan Z & Pelikan-Filipek M. Role of nasal allergy in chronic maxillary
sinusitis - diagnostic value of nasal challenge with allergen. J Allergy Clin
Immunol. 1990;86:484- 9 J.
3. Benninger MS, Ferguson BJ & Hadley JA. Adult chronic rhinosinusitis: defi-
9

nitions, diagnosis, epidemiology, and pathophysiology. Otolaryngol Head Neck


Surg. 2003;129(3 Suppl):Sl- 32.

285
Otolaryngology Clinical Case Studies Chapter 6 - Rhinology

NoTEs CASE 3 - SPHENOID SINUSITis


Daniel Ratcliff, MD

A 36 year-old male presents to your clinic 'rvith a 3-week history of headache.

HPI: state that you would:

o Obtain a detailed medical hist01y beginning with a history of the present illness

What additional historical information would you seek? State that you would ask:

• What is the exact location of the headache?


• Are the symptoms constant or intermittent; if intermittent, are there triggers?
• Are there any associated nasal symptoms such as congestion, rhinorrhea or
change in sense of smell?
• Does the patient have any visual complaints?
• Are there any aural symptoms such as hearing loss, tinnitus, vertigo or otorrhea?
• Does the patient have a history of any numbness or weakness of the head
and/or neck?
• Is there any neck stiffness, fever or photophobia?

The patient reports that the headaches are constant in nature and usually
retro-orbital in location. He denies any vision changes, otorrh~a. tinnitus,
hearing loss or vertigo. However, he reports intermittent right-sided nasal
discharge (purulent) with fevers of up to 102 degrees. He states that occa-
sionally he feels numb over his right cheek. He· denies any neck stiffness,
fever or photophobia.

PMH: The patient denies other medical problems


PSH: Left knee arthroscopy 2 years prior
Allergies: Penicillin
Medications: None
FH: Negative
SH: Denies use of tobacco, alcohol, IV-drugs
ROS: Negative

What would you look for on PE? State that you would:

• Perform a thorough head & neck exam including vital signs, a neurologic
v exam and nasal endoscopy

PE:

J:1J.n1s..:. Temp: 101 F, BP: 130/90, Pulse: 110, RR: 18


GA : Ill-appearing (toxicJ. male in NAD
HEENT Normal except for scanty mucopurulence in the right
nare. There is no stiffness of the neck and no evi-
dence of photo phobia

286
Chapter 6 - Rhinology Otolaryngology Clinical Case Studies

New·o: Speech is fluent: the patient is alert and oriented. NOTES


The moror and sensory exam is within normal lim-
its. The CN exam is unremarkable with exception of
hypoesthesia in the right V2 distribution
Nasal endoscopy: Mucopurulence surrounding the right middle and
superior turbinates

What is your differential diagnosis?

• Sinusitis (bacterial, fungal)


• Mucopyocele of the frontal, ethmoid or sphenoid sinuses
• Anterior skull base/pituitary mass
• Migraine headache
• Trigeminal neuralgia
• Brain abscess, septic thrombophlebitis

What would you do next?

• Because of the patient's toxic appearance, tachycardia and V2 hypoesthesia,


you would arrange for the patient to be admitted for evaluation

What diagnostics would you request?

• CT-scan of the brain/sinuses with contrast


Results:
- Homogenous, isolated opacification of the right sphenoid sinus with-
out expansion or bony erosion
• Culture with gram stain of nasal purulence
Results:
- Gram positive cocci (cultures pending)
• CBC with differential
Results:
- White blood cell count is 15,000 with a left shift

Diagnosis:

Acute sphenoid sinusitis

What are the medical options for therapy?

• Broad-spectrum antibiotics.ln this penicillin allergic patient consider a broad


spectrum fluoroquinolone, possibly an oral fluoroquinolone such as levofloxacin
(because of the equivalent bioavailability to IV levofloxacin). The selected
antibiotic should be adjusted as culture results return
• Nasal toilet (mucolytics, decongestants, aggressive saline lavages)
• Consider repeating the CT-scan in 36-72 hours if the patient fails to show
signs of improvement or worsens

After 48 hours, the patient continues to complain of retro-orbital head-


ache with fevers over 102 degrees F Repeat _CT scan demonstrates contin-
ued sphenoid opacification.

287
Otolaryngology Clinical Case Studies Chapter 6- Rhinology

NOTES What are the surgical options of therapy?

• In most patients, sphenoid sinus decompression shou ld be performed via an


endoscopic approach
• However, transnasal approaches are still a viable method including the
transseptal approach (via either a columellar or sublabial incision), the
transethmoidal approach, the transorbita l approach and the transpalatal
approach

Surgical technique:

Highlights and pitfalls:


• An endoscopic approach is typically the least invasive method and often can
provide for the most physiologic method of drainage/decompression via the
natural ostium
• The natural ostium of the sphenoid sinus can be localized using Bugler's
technique:
- The superior meatus is entered through the basal lamella
- The anterior-inferior aspect of the superior turbinate is identified along
with the lamina papyracea posteriorly and the skull base superiorly
- The natural ostium of the sphenoid sinustypically lies within this "box,"
and is often identified by gently removing the inferior portion of the supe-
rior turbinate
- Care must be taken with this technique to avoid an iatrogenic CSF leak
due to aggressive manipulation of the superior turbinate
•· In all cases, extreme care should be taken when working in the superior or
lateral portions of the sphenoid sinus as these areas carry the greatest risk of
neurovascular injury

What are the potential complications of sphenoid sinusitis?

• Orbital cellulitis
Orbital abscess
• Cavernous sinus thrombiti s
• Subdural/epidural abscess
• Bacterial meningitis
• Blindness
• Sepsis
• Hypopituitarism
• Cerebral infarction

How would you follow this individual?

• Initially, the rhinorrhea, fever and white blood cell count should be followed
for improvement
Cultures should be checked and antibiotics adjusted accordingly
• Consideration should be given to a repeat CT scan at 6 weeks in patients with
clinical evidence of improvement to assure resolution of sphenoid opac ification

288
Chapter 6- Rhinology Otolaryngology Clinical Case Studies

References: NOTES

I. LewD, Southwick FS, et al. Sphenoid sinusitis: a review of30 cases. N Engl
J Med 1983 ;309: 1149-54.
2. Elwany S., Yacout YM, Talaat M, et al. Surgical anatomy of the sphenoid
sinus. J Laryngol Otoll983;97:227-41.
3. Ghorayeb BY Sphenoidotomy. Head Neck 1987;9:244- 249.
4. Levine H. The sphenoid si nus, the neglected sinus. Arch Otolaryngol
1978;1 04:585-7.
5. Hnatuk LA, Macdonald RE, Papsin BC. Isolated sphenoid sinusitis: the Toronto
Hospital for Sick Children experience and review of the literature. J
Otolaryngol. 1994;23:36-41.
6. RatcliffDJ, Marple BF. Sphenoid sinus disease. In: Bailey BJ, Johnson JT,
Healy GB, Pillsbury HC, Ja~kler RK, Tardy ME, Calhoun KH, eds. Head
and neck surgery-otolaryngology. 3rd ed. Philadelphia: Lippincott Williams &
Wilkins; 2001,794-804.

289
Otolaryngology Clinical Case Studies Chapter 6 - Rhinology

N(HES CASE 4 - ACUTE FRONTAL SINUSITJS


Amber Luong, MD, PhD

A 40 year-old male reports a 2-week history of nasal congestion. and


intermittent frontal pressure/headaches.

HPI: state that you would:

• Obtain a detailed medical history beginning w~th a history ofthe present illness

What additional historical information would you seek? State that you would ask:

• Does the patient have a history of sinus disease or polyps?


• Is his pain worse on the right or left or nonspecific?
• Does he have fever, chills or malaise?
• Does he report rhino1Thea and, if so, is it purulent?
Are there any vision changes?
• Does he have any history of sinus surgery?
• Is there any history of head and neck trauma?
• Does he have any changes in his sense of smell?
• Has the patient received any previous treatment for his condition?

40 year-old male with a several year history of intermittent ''colds" (4- 5


infections over the last 3 years, last infection was about 9 months ago).
These infections are usually handled with 1-2 week course of antibiotics
given by his primary care physician. He now notes a 2~ week history of
nasal congestion, right frontal headaches, rhinorrhea, intermittent fevers,
and hyposmia. He has been treated by his family practice physician with a
7-day course of Zithromax, but has noted no improvement. In fact, the
patient feels worse now than a week ago. His headache has become quite
severe. He has no history of head trauma.

PMH: Allergic rhinitis, GERD. HTN, DM controlled with oral


hypoglycemics
PSH: None
Allergies: NKDA
Medications: Motrin 400mg po tid, ZithromtLY, Allegra, Flonase,
rabeprazole, meiformin, captopril
FH: Negative
SH: Occasional ETOM. nonsmoker
ROS: Noncontributory

What would you look for on PE? State that you would:

• Perform a PE
• Start with vital signs and then move on to a head and neck exam

290
Chapter 6 -Rhinology Otolaryngology Clinical Case Studies

PE: NoTES

l11.nls_: Temp: 100 F. Pulse: 88, BP: 145/90, RR: 12


GA: Well developed male in 110 apparent distress
Ears: TM intact and mobile bilaterally
Nose: Nasal passages clear anteriorly on the !ef•_ By endoscopic
exam, there is edema of inferior and middle turbinate on
the right. Erythematous mucosa and heavy mucopurulent
secretions are noted on the right side and landmarks are
not well visualized due to heavy secretions
OCIOP: 1+tonsils bilaterally, no erythema, no masses noted. FO/v!
and tongue soft
Neck: No LAD, normal thyroid
Face: Patient is lender to palpation over the right supraorbital
region. There is mild edema to the right upper eyelid No
proptosis
Salivary: No parotid masses
Neuro: EOMI, CN II-XII intact, alert and appropriate

What is your differential diagnosis?

• State that the differential should include simple acute frontal sinusitis, com-
plicated acute frontal sinusiti s, frontal sinus mucocele, al lergic fungal sinusi- ·
tis, migra_ine headache, and paranasal sinus neoplasm .

What diagnostic tests would you request? State that you would ask for:

Labs- Including CBC, Chern l 0


Imaging studies - CT scan of the sinuses with contrast
• Consultations: internal medicine, ophthalmology

Test results:

• Labs CBC: · WBC: 20.1, Hgb 16.1, Hct 39, Pit 300 Differential reveals
85% neutrophils, 6% bands. Chem 10 is WNL except for glucose.
Glucose: Serum glucose 230
• Imaging studies: CT scan of the sinuses and orbits reveals complete
opacification of right frontal sinus with mucosal thickening of right
ethmoid and maxillary sinuses. There are inflammatory changes to the
right preseptal orbital tissues. Consultation with the ophthalmologist
reveals no visual acuity changes. No1:mal intraoeular pressure. No re-
striction. Periorbital edema

Consultation with internal medicine assists with hyperglycemia control. Glucose


is now controlled in the 70-110 range.

Diagnosis:

The diagnosis is acute frontal sinusitis with periorbital cellulitis.

291
Otolaryngology Clinical Case Studies Chapter 6 - Rhinology

NOTES What are some complications of acute frontal sinusitis?

• Neurologic: Subdural abscess, epidural abscess, meningitis, and brain abscess


• Orbital: Orbital cellulites or subperiosteal abscess
• Osteomyelitis of frontal bone

What are the treatment options and their complications? What would you do
next and why? (Inpatient vs. outpatient work-up?)

• State that in this diabetic patient, with evidence of complicated acute frontal
sinusitis, inpatient work-up is warranted
• This patient should be admitted for inpatient work-up and treatment for sev-
eral reasons:
- Serial ophthalmologic exams
- IV antibiotics
Control ofhyperglycemia
- Monitoring for neurologic complications
- Possible surgical intervention

What are your medical options?·

• Given no bacterial isolate has been obtained, would initiate broad coverage
with IV antibiotics. IV antibiotics: cefuroxime 1.5 g IV q 8h, Flag~ I 500 g IV
q 8h
• Nasal irrigation with NS qid
• Afrin decongestion bid x 3 days
• Guaifenesin 1200 mg po bid+
• Analgesia with oral hydrocodone and possibly IV morphine

In acute sinusitis, the most likely bacterial organisms are str\!ptococcus pneu-
monia, haemophilus influenzae, moraxella catarrhal is; anaerobes are rarely found
in acute sinusitis.

The microbiology of chroJ!ic sinusitis involves anaerob_es more often than in


acute sinusitis. In addition to the organisms found in acute sinusitis (streptococ-
cus pneumonia, haemophilus influenzae, and moraxella catarrhalis), organisms
commonly found in chronic sinusitis include: alpha streptococci, staphylococcus
aureus, pseudomonas aeruginosa, and anaerobes.

In this diabetic patient who has failed a trial of outpatient antibiotics and
who is also demonstrating signs and symptoms of complicated acute si-
nusitis, p second line drug that is active against b-lactamase producing
strains is indicated Examples include:
1. Amoxicillinlclavulanate
2. Cefuroxime
3. Levojloxin

The -patient is admitted and started on IV antibiotics and nasal hygiene. The
patient remains afebrile, and his WBC has decreased to 16. On hospital day
3 the patient reports increasing severity of right frontal and periorbital pres-
sure despite IV morphine. Op~thalmologic exam reveals increased supraor-
bital edema, mild chemosis. No double vision. No visual acuity changes.
292
Chapter 6 - Rhinology Otolaryngology Clinical Case Studies

What would you do next? State that you would: NOTES

• Reimage with CT of the sinuses/orbits with contrast


• CT reveals complete opacification of right frontal sinus and complete opaci-
fication of anterior ethmoid sinuses
• There art inflammatory changes in the right medial orbit
• No intracranial lesions are noted

Surgical intervention is now indicated to drain the fron tal and ethmoid
sinuses and obtain cultures. The patient has not responded to IV antibiot-
ics alone.

What are your surgical options? Endoscopic vs. open approach:

• State that 3 options include:


I. Frontal sinus trephination
2. External frontoethmoidectomy
3. Endoscopic frontal sinusotomy

• Open/external considerations:
I. Frontal sinus trephination- Frontal sinus anatomy must be amenable to
trephination .
2. External frontoethmoidectomy is the mainstay of therapy for compli-
cated frontal sinusitis. _This approach has been associated with delayed
stenosis of frontal recess due to medialization of orbital contents

• Endoscopic considerations:
I. Skilled endoscopic sinus surgeon
2. Hemostasis is critical and visualization may be difficult in the face of
acute inflammation
3. Lack of visualization or poor decompression of abscess necessitates
drainage via an external approach

Surgical technique:

Highlights and pitfalls:


• Frontal sinus trephination:
- Note extent and anatomy of frontal sinuses on CT scan
- Area of entry is about 1 em lateral to midline along a horizontal line
drawn across the superior orbital rims. Should be consistent with patient's
frontal sinus anatomy
18 g needle with angiocatheter is inserted
- Confirn1 placement by aspiration of air and presence of inigation fluid in
nose. Obtain pus for cultures
- Secure catheter for inlgation
• External frontoethmoidectomy:
- Incision begins at inferior margin of medial brow, then curves dow·n to-
ward medial canthus, and then along sidewall of nasal dorsum
- Injection of 1% lidocaine with I: l 00,000 epi prior to incision enhances
hemostasis

293
Otolaryngology Clinical Case Studies Chapter 6- Rhinology

NOTES - Incision is carried down through periosteum


- Supraorbital neurovascular bundles are preserved. Often. angular artery
requires Iigation or bipolar cautery
- Dissection should proceed in subperiosteal plane to protect orbital con-
tents, lacrimal sac, medial canthus, and trochlea
Anterior ethmoidal artery is ligated
Ethmoid is entered through lacrimal fossa with osteotome
- Ken·ison rongeur used to widen etlunoidectomy. Lamina papyracea is taken
down. Ethmoidectomy is complete to allow widening of frontal recess
- Superior dissection does not proceed above frontoethmo idal suture line,
which corresponds to level of anterior cranial fossa
- Obtain cultures when pus is encountered
• Endoscopic frontal sinusotomy:
- Decongest with Afrin-soaked cottonoid. Inject lateral nasal wal l with
I% lidocaine with I: 100.000 epinephrine for hemostasis
- Remove superior 2/3s of uncinate for exposure of bulla ethmoidalis and
terminal recess, agar nasi cells
- Palpate frontal recess using frontal sinus probe. Use mucosal sparing
technique whenever possible using through-cutting instruments
- Chain Kerrison rongeur is useful for removing roof of terminal recess or
agar nasi cells in order to widen frontal sinusotomy
- Obtain cultures when pus is encountered

Potential complications:

• Intracranial injury- Epidural/subdural hematoma, brain injury requires im-


mediate CT imaging and neurosurgical consultation
• CSF leak - Neurosurgical consultation for lumbar drain may be amenable to
endoscopic closure
• Orbital injuries - Ocular pain, proptosis, and vision Joss indicate retrobulbar
hematoma. Hematoma requires immediate ophthalmologic consultation, man-
nitol, lateral canthotomy with subsequent exploration for bleeding source.
High dose steroids are controversial .
• ·Diplopia - May result from injury to medial rectus muscle. This requires
ophthalmologic consultation. Strabismus surgery may be required
• Frontal recess stenosis- External frontoethmoidectomy may result in steno-
sis of frontal recess due to medialization of orbital soft tissue following exter-
nal ethmoidectomy. May require revision endoscopic or external surgery in
future

How would you follow this individual? State th'at you would:

• Continue frontal sinus lavage for 3-5 days until symptoms improve, at which
time catheter can be removed. Final IV antibiotic therapy should be directed
at bacterial species isolated from cultures
• Once clinical improvement is noted, i.e., improved pain, improved edema,
then IV antibiotics can be switched to an oral antibiotic and continued to
complete at least a 2-week course. Nasal irrigations should be continued
long term and are paramount to minimize scar formation

294
Chapter 6- Rhinology Otolaryngology Clinical Case Studies

• Reexamination in the clinic for endoscopic debridement is perfom1ed I week NOTES


postop. In addition, the patient is restarted on his allergic rhinitis medications
and consideration should be given to allergy testing/immunotherapy to assist
with mucosal sinus disease as a long tem1 adjunct to his nasal irrigations

References:

1. In: Bai ley BJ, Johnson JT, Healy GB, Pillsbury HC, Jaclder RK, Tardy ME,
Calhoun KH, eds. Head and neck surgery-otolaryngology. 2nd ed. Philadel-
phia: Lippincott Williams & Wilkins; 1998, 44 1- 55.
2. McLaughlin RB, et al. ClinicalJy relevant frontal sinus anatomy and physiol-
ogy. Otolaryngol Clin North Am. 200 I ;34( 1): l- 22.
3. Klossek JM, Fontanel JP. Fontal Sinus Irrigation: Indications, Results, and
Complications. Oto laryngol Clin NorthAm. 200 I;34(1 ):91 - 100.
4. Bailey BJ, Calhoun K.H, Coffey AR, Neely .TG, eds. Atlas ofhead and neck
surgery-otolaryngology. Philadelphia: Lippincott Williams & Wilkins; 1996,
868- 9.
5. Kuhn FA, Javer AR. primary endoscopic management of the frontal sinus.
Otolaryngol Clin North Am. 2001;34(1):59-75.

295
Otolaryngology Clinical Case Studies Chapter 6 - Rhinology

NOTES CASE 5- CEREBROSPINAL FLUID RHINORRHEA


Neelesh Mehendale, MD

This 36 year-old African-American male complains of water coming out of


the leji side of his nose.

HPJ: state that you would:

• Obtain a detailed medical history beginning with the history of present illness

What additional historical information would you seek'? State that you would ask:

• When did this problem sta1t?


• Does anything make it better or worse?
• Has he been involved in any trauma recently or had any nasal surgery?
• Does he have allergies?
Has he had any photophobia, stiff neck, or headaches?
• . Has he had any fever?

The patient states that he was in his usual state of health until 2 weeks ago
when he was a restrained driver in a motor vehicle accident. He briefly
lost consciousness at the scene of the accident and was taken to a nearby
community hospital. He states that they performed some x-rays and a CT
scan of his head and informed him that there were no fractures or evi-
dence of brain injury. He was admitted for observation and discharged
home the next day in stable condition. He recovered at home and went
back to work 3 days later. The patient did well until yesterday when he
started noticing jluid dripping from his left nostril when he was leaning
over his desk. The patieiu denies any prior nasal surgery. He also denies
any photophobia, stiff neck, headache, or feva

PMH: None
PSH: None
Allergies: Sulfa drugs - rash
Medications: Daily vitamin
SH: Non-drinke1: non-smoker, no illicit drug use, chemical
engineer
ROS: ROS is '!egative except for mild pain in the left side of his
face, which has decreased since the accident

What would you look for on PE? State that you would:

• Perform a complete head and neck PE including nasal endoscopy exam

296
Chapter 6 - Rhinology Otolaryngology Clinical Case Studies

PE: NoTEs
J!.i.lgjs_: Temp: 98.6 F, BP: 132178, HR: 78, RR: J.l
GA: Well-developed, well-nourished male in no acute
distress, normal tone and quality of voice
Head: Normocephalic, there is slight bil~'eral infraorbital
ecchymosis, but no palpable stepojjs or bony
abnormalities
Ears: EAC's and TM's clear AU, no MEE seen
Nose: Anterior rhinoscopy clear except pooling of secre-
tions in leji nasal sill, right nare clear
OCIOP: Normal dentition, Class I occlusion, no obvious
lesions or masses
Neck: No paip able masses or adenopathy, thyroid carti-
lage is midline
CN: II-XII intact
Rigid rhinoscopy: Right nasal cavity without lesions or masses, left
nasal cavity without any masses, no mucosal le-
sions seen, fluid seen running ~own from middle
turbinate

When the patient leans forward and performs a Valsalva maneuve1~ there
is a profuse amount of clear drainage from the left nostril.

What is your differential diagnosis?

The differentia l diagnosis would include CSF rhinorrhea, allergic rhinitis, vaso-
motor rhinitis, and viral infection. CSF rhinorrhea can be caused by trauma,
mass effect, or idiopathic reasons.

What diagnostics would you request? State that you would:

State that y~u would request a beta-2 transferrin. analysis of his rhinorrhea
fluid
• If laboratory facil ities are not available for this test, then one should look at
glucose levels ofthe fluid or perform the " halo" test

Test results:

Laboratory data:
Sample is positive for beta-2 transferrin

CSF rhinorrhea is at the top of the differential diagnosis and correct diag-
nosis requires: (1) determining the fluid is from the nose, (2) determining
the fluid is CSF, and (3) localizing the fistula. The PE has determined that
the fluid is emanating ji·om the nasal cavity. The next step would be to
confirm that the fluid is indeed CSF. Sending the fluid for immunoelectro-
phoretic identification for beta-2 transferrin is the most specific confirma-
tory test available. Glucose levels in the collected fluid can also indicate

297
Otolaryngology Clinical Case Studies Chapter 6- Rhinology

NOTES CSF If the level is > 30 mgldl then the fluid is most likely CSF If no /abo- ·
raLOry testing is available, the classic bedside test is the "halo test. ., In this
test, a clear fluid area will surround a blood stain when CSF mixed with
blood is absorbed into paper, wound dressings. or bed linens.

'\Yhat is your next step?

• State that you wou ld obtain an axial and coronal CT scan of the anterior skull
base and paranasal sinuses with I mm cuts

Test results:

Fine-cut CT scans show a 5 x5 mm bony defect in the area of the left. eth-
moid roof There is no evidence of encephalocele or meningocele.

Numerous techniques have been employed to image the skull base for localiza-
tion of a CSF fistula, including intraoperative fluorescein dye injection, radionu-
clide cisternogram, and pluridirectional tomography. Fluorescein dye injected
irito the subarachnoid space can be seen as ye llow-green fluid emanating from
the site of the CSF leak. Although fluorescein dye is specifically labeled that it is
not for use in the subarachnoid space, it is sometimes used in this manner in
some facilities. Complications from intrathecal fluorescein include lower ex-
tremity weakness, numbness, generalized seizures, transverse myelitis and CN
deficits. Radionuclide studies involve injecting radioisotopes intrathecally and
testing for their appearance on nasa'l pledgets. This test, however, has a high
rate of false-positives because of absorption through the bloodstream and redis-
tribution to nasal mucosa. Nonionic contrast CT scan, where nonionic contrast
is injected into the subarachnoid space and visualized on CT, is effective oniy
wh.en there is an active leak. More recently, MRI scans have sh9wn promise in
localizing CSF fistula. The MR cisternogram is a thin-cut, very heavily T2-
weighted image with TR's in the 5000-6000 range. It is sometimes referred to
as fast spin echo imaging. The advantage ofMRl is that it is non-invasive, does
not involve radiation exposure, and has multiplanar images.

In the case above where the PE and history have indicated that this is a leak
from the nasal cavity, a high-resolution CT scan (1 mm cuts) may be ad-
equate for localization of the leak. The CT scan should include direct coro-
nal as well as a;r:ial views of the paranasal sinuses and anterior skull base.

Diagnosis:

Post-traumatic CSF rhinorrhea from a defect in the left. ethmoid roof

What are the treatment options and their complications? What would you do
and why?

• Attempt to manage this patient non-operatively with bed rest, head elevation
and stool softener

298
Chapter 6- Rhinology Otolaryngology Clinical Case Studies

The majority of traumatic CSF fistulas heal without surgical intervention. The NoTES
risk of deve loping meningitis in the first 3 weeks after trauma has been rep011ed
to be approximate ly 3-11%. Prophylactic antibiotics have not been shown to be
effective in the prevention of meningitis and are not recommended in posttrau-
matic patients. Conservative management includin g bed rest with head eleva-
tion and stool softeners should be instituted.

Outcome (including complications):

The patienl is admitted to the hospital and placed on bed rest with heqd of
bed elevated. Two days late1: the patient still has CSF rhinorrhea.

What would you do next? State that you would:

• Attempt lumbar drainage for 3-5 days .

A lumbar drain is placed and the patient is drained 150 cc!day for 5 days.
At the end of this period, the patient continues to have profuse rhinorrhea.

What is your next option? State that you would:

Proceed with surgical correction of the defect via an endoscopic transnasal


approach

At this point, the patient has failed conservative therapy and non-opera-
tive management and the next step would be- surgical repair. of the defect.
Historically, intracranial approaches were used to repair skull base de-
fects, but more recently, extracranial approaches have proven to be more
effective and safe!'. The major requirement for the extracranial approach
is the preoperative localization of the defect. For closure of the ethmoid
roof defect in this patient, the endoscopic extracranial approach is the
best approach. This surgery qm be performed through an external inci-
sion or intranasally, although the intranasal approach is the preferred
method.

Surgical technique:

Highlights:
• General anesthesia
• Topically decongest and vasoconstrict nasal cavity
-• Perform endoscopic ethmoidectomies and middle meatus antrostomies as
needed to access site of defect and maintain anatomical landmarks
• Use the endoscope and sinus instruments to visualize the defect
• Elevate the mucosa around edges of the defect
• Seal the defect with either rotational mucosal (mucoperichondrial or muco-
periosteal) flaps, free temporalis fascia, or irradiated cadaveric dermal ma-
trix (A iloDenn) using an overlay technique
• Place absorbable and hemostatic packing material such as Surgicel or Gel foam
snugly against the graft

299
Otolaryngology Clinical Case Studies Chapter 6- Rhinology

NoTES • A watertight seal is paramount so fibrosis can produce a permanent seal


• Postoperative instructions should include the use of laxatives, avoidance of
straining, and bed rest for 3-5 days

Pitfalls:
• Complications include stroke, blindness, meningitis, and recurrence of the
leak
• The recurrence rate is from 0-24%

How would you follow this patient? State that you would:

• See the patient back in the office I week postoperatively


• Instruct the patient to notify you immediately ifleak recurs or if he develops
a severe headache, stiff neck, fever or disorientation
• Avoid instrumenting the nose aggressively at this point because fibrosis is
still occurring
• Allow the packing material to resorb on its own
• See the patient again 3 weeks postoperatively and use a nasal endoscope to
visualize and confirm the site of repair
• Schedule a final postoperative visit 3 months after the repair to ensure that
the repair has not failed. If there is no evidence of a recurrent leak at this
time, the patient can return on an "as needed" basis

References:

1: Eljamel MS. Antibiotic prophylaxis in the management ofCSF fistula. Surg.


Neurol. 1998;50(4):387.
2. Nachtigal D, Frenkiel S, Yoskovitch A, et a!. Endoscopic repair of cere-
·brospinal fl.uid rhinorrhea: is it the treatment of choice? J Otolaryngol.
1998;27:258-62.
3. Nuss DW, Constantino PD. Diagnosis and management of cerebrospinal
fluid leaks. In: Lucente FE, ed. Highlights of the instructional courses of the
American Academy of Otolaryngology-Head and Neck Surgery. St. Louis:
Mosby-Yearbook; 1995.
4. Zapalac JS, Marple BF, Schwade ND. Skull base cerebrospinal fluid fistulas:
a comprehensive diagnostic algorithm. Otolaryngol Head Neck Surg.
2002; 126:669- 76.

300
Chapter 6 - Rhinology Otolaryngology Clinical Case Studies

CASE 6- MAXILLARY SQUAMOUS CELL CARCINOMA NOTES


Lance Oxford, MD

A 68 year-old white male presents with left facial pain for 3 months and
loose upper dentition for 3 ·weeks.

HPI: state that you would:

• Obtain a detailed med ical hist01y beginning with a history of the present illness

What additional historical information would you seek? State that you would ask:

• What is the character of his pain?


• Has he noticed vision changes, diplopia, epiphora?
• Has he had a prev ious episode, a history of sinusitis or facial trauma?
• Has he had a previous history of cancer; FH of cancer, prior XRT, tobacco or
occupational exposure to carcinogens?
• Has he had facial numbness or change in occlusion?

The patient complains of gradually increasing leji facial pain that is con-
stant. moderately relieved with acetaminophen and NSA IDS, and wars-_
ened with chewing. The pain is dull and does not radiate. He S·tates that
two of his upper teeth on the left have gradually become loose. He has
noticed left-sided nasal congestion associated with occasiomil blood tinged
secretions during the past two months. He reports numbness of his upper
leji teeth and Left midface. The patient denies any vision changes or diplo-
pia. He has no previous history of sinusitis, facial trauma, XRT, or FH of
cance1~ He has a history of smoking and has no known exposure to in -
haled carcinogens.

PMH: HTN controlled with metoprolol


Allergies: NKDA
Medications: Metoprolol 50' mg po bid
FH: Negative
SH: 50 pack years, 12 beers/week, no HIV risk factors, retired
mailman
ROS: Negative

Wbat would you look for on PE? State that you would:
9

• Perfom1 a complete head and neck exam including vital signs and nasal
endoscopy

301
Otolaryngology Clinical Case Studies Chapter 6- Rhinology

NOTES PE:

l!i1.als_: Temp: 97.6 F. BP: 128170, Pulse: 82. RR: 18


GA: WJ'vf in NAD
HEENT PERRLA. EOMI. conjunctiva clem~ left epiphora
EAC clear. Tl\!fs intact
Nose: Septum deviated to right, right nare dear, left nare with
ulcerative mass of medial nasal wall involving inferior and
middle meatus
OP: Ulcerative mass of anterior left hard palate, with loose #10
and 11. poor dentition. uvula midline, tonsils 1+
Neck: No LAD, supple
New·o: Decreased sensation in disn·iburion of left infra-orbital nerve
Endoscopy: Ulcerative mass of medial nasal cavity, normaL superior
meatus, sphenoethmoid recess, and NP

What diagnostics would you request? State that you would:

• Biopsy the mass


Order axial and coronal CT with 3 mm cuts of sinuses and neck CT with
contrast

Test results:

Biopsy positive for SCCA


• 4.5 em enhancing mass involving the medial, inferior and superior walls
of the maxillary sinus and subcutaneous tiss!"es of left cheek. No en-
larged cervical lymph nodes or intraorbital abnormalities

What is your differential diagnosis?

• The scenario is most consistent with a neoplasm of the left maxillary sinus.
Infectious etiologies are less likely. Invasive fungal sinus~tis can produce
necrosis of the medial nasal wall and neurological deficits, but patients pre-
senting with invasive fungal sinusitis are typically immunocornpromised and
present more. acutely. Inflammatory disorders such as Wegener's granulo-
matosis and pseudotumor are less common causes of sinonasal or orbital
symptoms. Benign conditions such as a sinus mucocele may result in orbital
symptoms but are not associated with an ulcerative mass

Diagnosis:

. T3NOMO SCCA of the left maxillary sinus

A primary neoplasm of the maxillary sinus is.the most likely diagnosis given the
rarity of metastatic spread. A malignancy ofthe hard palate with superior extension
could also be considered in the differential diagnosis. SCCA accounts for the
majority of malignancies of the maxillary sinus. The neoplasm more commonly
affects males and typically presents in the sixth and seventh decades. Reported
risk factors include a history of inverted papi_lloma and exposure to thorotrast
contrast and nickel. SCCA typical ly presents at an advanced stage, with more

302
Chapter 6- Rhinology Otolaryngology Clinical Case Studies

than 80% of tumors stage III or IV. Lymph node or distant metastases are rare NOTES
on presentation. Less common malignancies of the maxillary sinus include
adenocarcinoma, adenoid cystic carcinoma, lymphoma, melanoma, and sinonasal
undifferentiated carcinoma.

What are the treatment options and their complications? WJ:---t would you do
and why?

• The standard treatment of sinonasal SCC is surgical resection and postop-


erative radiation therapy. Recent studies have reported favorable responses
with multimodality therapy with preoperative chemotherapy with 5-FU and
cisplatin, followed by resection and XRT. Nibu, et al. (2002) reported an
overall 5 year survival of 86% in 33 patients with maxillary sinus SCCA.
Even with skull base involvement, a 60% survival was achieved
• Baseline labs to include CBC, Chern 10, PT, PIT, HIV, and hepatitis serologies
• EKG
• CXR (CT chest if patient had any cervical node enlargement)
• Oral surgery evaluation to assess need for extractions to decrease risk of
post-XRT osteoradionecrosis
• Prosthodontist evaluation to create a palatal prosthesis
• Ophthalmology evaluation

Surgical technique:

Highlights and pitfalls:


• Consent for panendoscopy, maxi llectomy, possible orbital exenteration, pos-
sible calvarial bone graft, local flap, and free flaps
• Weber-Ferguson incision combines lateral rhinotomy incision with upper lip
split
Given involvement of orbital floor bone, a total maxillectomy is indicated
• After m~'<.illectomy, send periorbita for frozen section. Traditionally, iftumor
involves the periorb ita or other intraorbital structures, then orbital exentera-
tion is indicated. Recent articles report similar local control rates with orbital
preservation in selected cases of infiltration of the periorbita
• If sufficient maxillary dentition is present, then a temporary prosthesis is
placed
• Alternatively, the defect may be reconstructed with a free flap. A rectus
abdominis musculocutaneous flap provides sufficient bulk to fill the maxillary
defect and an orbital exenteration cavity. For patients with prior abdominal
surgery that might compromise a rectus ~ap, a free latissimus dorsi flap may
be used. Patients desiring dental implants may have reconstruction with a11>
iliac crest or scapular flap to provide free bone for the implants
• With orbital preservation and resection of the orbital floor, the floor should be
reconstructed to prevent enophthalmos. Options include a pedicle vascular-
ized calvarial bone graft pedicled on a superficial temporoparietal fascial flap
or free calvarial bone grafts fixed with titanium mini-plates and covered with
a temporoparietal fascial flap
• Tracheotomy if any concern of airway obstruction secondary to flap
reconstruction

303
Otolaryngology Clinical Case Studie's Chapter 6- Rhinology

NoTES Complications:

• If the globe is preserved, the patient is at risk for ectropion and enophthalmos
• Vision loss may result from postoperative XRT
• Flap loss
• Velopharyngeal insufficiency secondary to palatal resection

How would you follow this individual'?

• Frequent follow-up postoperatively to debride the maxi1\ary cavity if a pros-


thesis is used. The patient should continue to follow-up with the prosthodon-
tist for any necessary modifications of the prosthesis
• Screening exams every 4- 6 weeks during the first year to monitor for recur-
rence. Exams every 2 months in the second year, and every 3 months during
the third year. If the patient has no evidence of disease 3 years postopera-
tively, then follow-up may be extended to a biannual or annual schedule
• A baseline CT or MR1 scan may be obtained at 2-3 months after XRT.
Alternatively, a scan may be obtained on the basis of any signs or symptoms
of concern to evaluate for a possible recurrence
• If the patient develops any cervical lymphadenopathy, then a FNA and CT
should be performed. Ifthere are any features suggestive of regional recur-
rence, then a neck dissection is indicated

References:

1. Dulgueorov P, Jacobsen MS, Alia! AS, et al. Nasal and paranasal sinus car-
cinoma: are we making progress? A series of 220 patients and systematic
review. Cancer. 2001;92:30 12-29.
2. Myers LL, Nussenbaum B, Bradford CR et al. ·Paranasal sinus malignancies:
an 18 year single institution experience. Laryngoscope. 2002; 112: 1964-9.
3. Tiwari R, Hardillo JA, Mehta D, et al. Squamous cell carcinoma of the max-
illary sinus. Head Neck. 2000;22: 164-9.
4. Nibu K, Sugasawa M, Asai M, et al. Results of multimodality therapy for .
squamous cell carcinoma of maxillary sinus. Cancer. 2002i94: 1476-82.
5. Imola MJ, Schramm VL. Orbital preservation in surgical management of
sinonasal malignancy. Laryngoscope. 2002; 112:1357- 65 .

304
Chapter 6- Rhinology Otolaryngology Clinical Case Studies

CASE 7- ETHMOID ADENOCARCINOMA NOTES


Lance Oxford, MD

A 72 year-old white male presents with blood tinged rhinorrhea and fron-
tal headaches for 3 months.

HPJ: state that you would:

• Obtain a detailed medical history beginn ing with a history of the present
illness

What additional historical information would you seek? State that you would ask:

• What is the character of his pain?


• Has he had headaches, rhinorrhea, hyposmia, nasal obstruction, epistaxis,
vision changes, epiphora?
Has he had previous episodes, a history of sinusitis or facial trauma?
• Does he have a previous history of cancer, FH of cancer, prior XRT, tobacco
or occupational exposure to carcinogens?
• Does he have facial numbness or a change in occlusion?

The patient complains of gradually increasing, constant frontal headaches


with minimal relief with NSAIDs and acetaminophen. He reports intermit-
tent blood tinged rhinorrhea on the right and decreased smell during the
past 3 months. He has no history of previous sinusitis and denies any
ocular or other associated symptoms.

PMH: GERD managed with Pepcid


Allergies: NKDA
Medications: Pepcid 20 mg po bid, ASA 81 mg po qd
FH: Negative
SH: 40 pack years, 6 beers/week, no HIV risk factors, retired
truck driver
ROS: Negative

What would you look for on PE? State that you would:

• Perform a complete head and neck exam including vital signs and nasal
endoscopy

PE:

Ei1als_: Temp: 98.3 F, BP: 125170, Pulse: 72, RR: 16


GA: White male in NAD
HEENT PERRLA, decreased lateral and medial gaze OD, conjunc-
tiva clem~ vision 20130, OU, EAC clem: TJvfs intact
Nose: Left nare clea1~ blood tinged rhinorrhea from right middle
meatal region

305
Otolaryngology Clinical Case Studies Chapter 6 - Rhinology

NOTES OP: Tonsils I +, uvula midline, no palpable masses


Neck: No LAD, supple
Neuro: CN 1-12 intact, decreased smell
Endoscopy: Small amount of blood tinged rhinorrhea of right middle
meatus and mass of right superior nasal cavity

What diagnostics would you request?

• Axial and coronal contrast CT with 3 mm cuts to evaluate the paranasal


sinuses, orbit, and brain
• Endoscopic biopsy of right ethmoid mass in OR

Test results:

• 4. 0 em enhancing mass involving right anterior and posterior ethmoid


sinuses and nasal cavity with erosion of the lamina papyracea, fovea
ethmoidales, and cribriform plate. No intracranial abnormalities
• Adenocarcinoma demonstrating the presence of goblet cells ·

What is your differential diagnosis?

• The scenario is most consistent with a neoplasm of the right ethmoid sinuses

The most common paranasal sinus malignancy-is SCCA. Other potential malig-
nancies include adenocarcinoma, olfactory neuroblastoma (esthesioneuroblas-
toma), sinonasal undifferentiated carcinoma, and adenoid cystic carcinoma.
Malignancies such as melano~a, lymphoma, and hemangiopericytomas are rare
etiologies. Infectious causes such as invasive fungal sinusitis, and granuloma-
tous diseases such as Wegener's granulomatosis may also be considered in an
extended differential diagnosis.

Diagnosis:

Intestinal type adenocarcinoma of the right ethmoid sinuses

Adenocarcinomas account for 17-90% of ethmoid mal ignancies depending on


geographic location and occupational risk factors. Chronic exposure to wood or
leather dust has been cited as a risk factor for the development of sinonasal
adenocarcinoma in numerous studies. Woodworkers in England have greater
than a I 000 fold increased incidence of ethmoid adenocarcinoma.

Sinus adenocarcinomas may develop from either minor salivary gland tissue or
the epithelium.llistologically, adenocarcinomas are classified as low grade, high
grade, or intestinal type. Intestinal or colonic type adenocarcinoma is consid-
ered a high grade neoplasm.

306
Chapter 6- Rhinology Otolaryngology Clinical Case Studies

What are the treatment options and their complications? What would you do NOTES
and why?

• Ethmoid adenocarcinoma typically requires a craniofacial approach for re-


section. Postoperative radiation therapy is recommended for positive mar-
gins, dural or cribriform plate invasion, and high grade lesions adjacent to the
cribriform ·plate

Wax, et al. reviewed 8 cases of adenocarcinoma of the ethmoid sinuses treated


with craniofacial resection with follow-up ranging from 9 months to 5 years.
Four patients received postoperative radiation therapy. Seven patients have no
evidence of disease, including 5 patients with greater than 3 years follow-up.
Only I patient had positive margin secondary to brain involvement and died of
local recurrence at 8 months. A series of 11 5 patients treated with craniofacial
resection reported major complications in 35% and operative mortality in 3.5%.
The 5 year disease specific survival was 58%.

What would you do next?

• Baseline labs to include CBC, Chern 10, PT, PIT, HIV, and hepatitis serologies
• EKG
. cr neck to evaluate for cervical lymphadenopathy
• CXR (CT chest if patient had any cervical node enlargement)
• Neurosurgical consultation to coordinate resection
• Ophthalmology evaluation

Surgical technique:

Highlights and pitfalls:


• Consent for panendoscopy, craniofacial resection, possible orbital exentera-
tion, local flap, and free flaps
• Transfacial approach is traditionally accomplished with a lateral rhinotomy.
incision to allow exposure of the nasal cavity, ethmoid sinuses, and medial
orbital wall
• The medial canthal ligaments may be tagged and detached from the lacrimal
crest in the subperiosteal plane. After periosteal elevation, a curved osteotome
is used to make low lateral osteotomies through the frontal process of the
maxilla. The lateral nasal wall may then be outfractured and retracted medially
• Bicoronal incision with elevation of an anteriorly based pericranial flap
• Neurosurgery removes a frontal bone flap
- The fronta: lobe is then elevated from the cranial base. The olfactory
nerves are severed at the cribriform plate. The dura may be elevated as
needed to expose the planum sphenoidale, fovea ethmoidales, orbital
roofs, and the base of the anterior clinoid processes
- After intracranial and transfacial exposure, tumor is resected en bloc
- Send fro?en sections of dura and periorbita as indicated
- Management of potential orbital invasion as described in Rhinology Case 6
- The anterior cran ial fossa defect is reconstructed with the anterior based
pericranial flap

307
Otolaryngology Clinical Case Studies Chapter 6- Rhinology

NOTES - With larger lesions requiring extensive skull base, maxillary, or orbital
resections then a free flap is utilized. The prefened flap is a rectus
myocutaneous abdominis flap for large defects
- During closure of the facial incision, the medial canthal tendon is resus-
pended to the medial orbit
- CT head in AM of POD l to evaluate for intracranial hemorrhage,
pneumocephalus or cerebrovascular ischemia

Complications:

• Stroke
• CSF leak- Postoperatively, the head of the bed is elevated and stool soften-
ers are prescribed to decrease intracranial pressure. If a CSF leak occurs, it
is initially managed with a lumbar drain. Persistent leaks may require re-
exploration
• Pneumocephalus- May be managed conservatively if no mass effect. Sig-
nificant pneumocephalus requires re-expJoration and closure of the commu-
nication between the paranasal space and the intracranial cavity
• . Vision Joss may result from postoperative XRT
• Flap loss

How would you follow this individual?

Screening exams every 4-6 weeks during the first year to monitor for recur-
rence. Exams every 2 months in the second year, and every 3 months during
the third year. If the patient has no evidence of disease 3 years postopera-
tively, then follow-up may be extended to a biannual or annual schedule
• A baseline CT or MRI scan may be obtained at 2-3 months after XRT.
Alternatively, a scan may b~ obtained on the basis of any signs or symptoms
of concern to evaluate for a possible recunence
• If the patient develops any cervical lymphadenopathy, then a FNA and CT
should be performed. If there are any features suggestive of regional recur-
rence, then a neck dissection is indicated

References:

J. Roux FX, Brasnu D, Menard M, et al. Adenocarcinoma of the ethmoid si-


nuses. Acta Neurochir (Wien) 1989;98: 129-34.
2. Nunez F, Suarez C, Alvarez I, et al. Sino-nasal adenocarcinoma: epidemio-
logical and clinico-pathological study of34 cases. J Otolaryngol. 1993;22:86-
90. •
3. Wax MK, Yun JK, Wetmore SJ, et al. Adenocarcinoma of the ethmoid sinus.
HeadNeck.1995;17:303- 11.
4. Shah JP, Kraus DH, Bilsky MH, Gutin PH, Harrison LH, Strong EW. Cran-
iofacial resection for malignant tumors involving the skul l base. Arch
Otolaryngol Head Neck Surg. 1997; 123:1312-7.

308
Chapter 6- Rhinology Otolaryngology Clinical Case Studies

CASE 8- WEGNER'S GRANULOMATOSIS NoTES


C. Spencer Cochran, MD

A 47 year-old male presents with a chief complaint of progressive bilateral


nasal airway obstruction and occasional epistaxis.

HJ>I: state that you would:

• Obtain a detailed medical history beginning with a history of the present


illness

What additional historical information would you seek? State that you would ask:

• How long have the symptoms been present?


• Has he experienced nasal pain, hearing loss, or difficulty breathing?
Does he have a history of allergic rhinitis?
• Has he experienced any constitutional symptoms such as weight loss, mal-
aise or intermittent fever?

The patient reports a several month history of increasing intermittent


epistaxis, which is self-limited cznd not initiated by any events. He reports
general malaise, and a 20 pou~1d weight loss. He has been treated by his
primary care physician for "sinus infections " with multiple antibiotic regi-
inens, yet he has continued to have symptoms. He has a chronic cough. He
denies seasonal allergies or abuse of any drugs

PMH: HTN. chronic sinusitis but no prior sinus or septal


surgeries
Allergies: · NKDA
j\;fedications: Atenolol
FH: Negative
SH; Denies tobacco, alcohol, or drug· use
ROS: · HEENT + ocular pain, decreased hearing
cv No chest pain or hx of myocardial infarction
PULM: Chronic cough with occasional hemoptysis, wheezing and
shortness of breath
Gl: No hematemesis or hematochezia
Renal: No history of renal calculi
Neuro: + headache, no CVA
.
What would you look for on PE? State that you would:

• Perform a complete head and neck exam including vital signs and nasal
endoscopy

309
Otolaryngology Clinical Case Studies Chapter 6 - Rhinology

NOTES PE:

YlliiLs.: Temp: 37.0 C, Pulse: 80, RR: 18. BP: 137185


GA: Thin white male in no apparent distress
HEENT: Normocephaliclatraumatic PERRLA. Extraocular move-
menTs intact. Vision is gross~y intact. Nasal exam reveals
midline septum and hyperemic, friable mucosa throughout.
Rigid nasal endoscopy identifies diffuse nasal crusting and
a 2 em posterior septal perforation. TiV!s are intact but with
bilateral serous effusions. OP clear without masses.
+ mucosal ulceration posterior-pharyngeal wall
NECK: Supple, no palpable lymphadenopathy
NEURO: CN 11-XJI intact

What is your differential diagnosis?

• Wegner's granulomatosis, sarcoidosis, cocaine-induced midline destructive


lesions, neoplasm, infections including fungal or bacterial chronic sinusitis

What diagnostics would you request? State that you would:

• Order.CBC, ESR, C-ANCA, UA


• CT scan of sinus
• Nasal biopsy

Test ~esults:

• CBC with differential count - Anemia of chronic disease


• ESR - 92 mml hr
• C-ANCA positive
• UA - 3+ hematuria
• CT - Erosion of posterior septum, medial maxillary sinus wall and tur~
binate erosion
• Biopsy - Necrosis with serpiginous borders and focal microabscesses,
granulomatous inflammation, and fibroid necrosis affecting the walls
of medium sized vessels

Diagnosis:

The diagnosis is Wegner s granulomatosis.

State that Wegener's granulomatosis is a multisystem disease that is classically


characterized by necrotizing granulomas of the upper and lower respiratory
tract, disseminated vasculitis, and glomerulonephritis. While the etiology ofWG
remains unknown, evidence supports an autoimmune cause. Clinical manifesta-
tions and organ involvement of Wegner's granulomatosis vary widely, and the
disease primarily affects the upper and lower respiratory tracts and kidneys.

31 0
Chapter 6 - Rhinology Otolaryngology Clinical Case Studies

Involvement lim ited to the head and neck occurs in 45% of cases. Sinonasal NoTES
structures are the most commonly affected areas of the upper aerodigestive
tract producing such symptoms as nasal obstruction from mucosal edema, rhin-
orrhea. ulcerations and crusting, and epistaxis. Otologic manifestations occur in
approximately 25-40% of patients. Symptoms are related to either primary in-
volvement of the middle ear and mastoid mucosa or secondarily from eusta-
chian tube dysfunction arising from the associated nasal disease. CHL can oc-
cur secondary to MEE or from scarring of the TM. Sensmineural hearing loss
(bilateral and profound) is less common, but thought to arise from vasculitis of
the coch lea or from granulomas of the auditory nerve. Laryngotracheal involve-
ment ranges from hoarseness due to mucosal inflammation to airway obstruc-
tion arising from subglottic stenosis.

What are the treatment options and their complications? What would you do
and why'!

• Treatment :
- Start high dose corticosteroids- Prednisone I mg/kg/day
- immediate referral to a rheumatologist/immunologist for immunosup-
pressive therapy that would potentially include cyc lophosphamide or
methotrexate
- TMP-SMX
• Appropriate consultations with rheumatology, pulmonology, nephrology
• Symptomatic treatmenrof sinonasal, otologic, and laryngic symptoms:
- myringotomy and tympanostomy tube placement for symptomatic MEE
- Nasal irrigations and debrideinents
- Laryngoscopy (direct or fiberoptic) to evaluate for laryngeal involve-
ment with potential interventions ranging from intralesional steroid injec-
tions for mil.d lesions to tracheostomy for airway compromise

How would you follow this pa_tient?

• Treatment and fo llow-up should be.a multi-disciplinary approach. Because


of a SO% relapse rate, patients should be seen frequently in the outpatient
setting and focus should be on disease involvement of pertinent organ sys-
tems and adverse side-effects of therapeutic agents
• Patients with otologk manifestations should have serial audiograms and
tympanograms to assess the stability of their hearing
• The course of the disease may be followed with serial titers of C-ANCA,
but these levels do not always correlate with the response to treatment

References:
"
·1.. Leavitt RY, Fauci AS, Bloch DA, et al: The American College of Rheumatol-
ogy 1990 criteria for the classification of Wegener's granulomatosis. Arthri-
tis RJ1eum. 1990;33(8): 11 01-7.
2. Takagi D, Nakamaru Y, Maguchi S, Furuta Y, Fukuda. Otologic manifesta-
tions of Wegener's granulomatosis. Laryngoscope. 2002; 112(9): 1684-90.
3. Langford CA, Talar-Williams C, Barron KS . A staged approach to the treat-
ment ofWegener's granulomatosis: induction of remission with glucocOJ1i-
coids and daily cyclophosphamide switching to methotrexate for remission
maintenance. A11hritis Rl1eum. 1999;42( 12):2666-73.

311
Otolaryngology Clinical Case Studies Chapter 6 - Rhinology

NOTES CASE 9- ALLERGIC FUNGAL RHINOSINUSJTIS


Amber Luong, MD, PhD

A 2 3 year-old white maLe presents with a chief complaint of /eft-sided nasal


airway obstruction for 2 years. He has occasionally noticed a mass
extending out from the inside of his left nasal cavil):

HPI: state that you would:

• Obtain a more detailed rhinologic and medical history beginning with the
chief complaint

What additional historical information would you seek? State that you would ask:

• What makes the mass larger? Smaller?


• Describe the mass?
• Does he have recurrent epistaxis?
• Does he note nasal drainage? If so, describe the consistency
• Can he appreciate smells?
• Any face pain or pressure?
• Does he suffer from allergies? Asthma? Aspirin sensitivity?
• Does he experience Yisual disturbances?
• Any headaches or neurological problems?
ls he immunocompromised? Treated with chemotherapy for leukemia or
lymphoma? Suffering from CF? Any other immunocompromising illnesses?
• · Any unintentional weight loss? Night sweats?

The nasal mass was visible about 2 weeks ago. It receded in size after he
was given a course of prednisone .for an asthma exacerbation. He de-
scribed the mass as like a green grape. He denies any recurrent epista.;'(is.
He denies any watery rhinorrhea, but reports extremely thick greenish
mucous when he blows his nose. He· is unable to smell or breathe through
his nose when the mass is large, but since the last course of steroids, he
has been able ·to breathe better. He has no face pain or pressure. He has
had asthma since the age of 14 and is treated with an inhaler, occasional
nebulizer treatments at horne and courses of prednisone during exacerba-
tion of symptoms. He sziffers from occasional allergy symptoms, primarily
in the spring. No problems with aspirin. No visual disturbances are noted
and he denies regular headaches or other neurological problems. He de-
nies any significant PMH that would make him immunocompromised. No
unintentional weight loss or night sweats.

PMH: No known medical problems, no history of hospitalizations


or ER visits
PSH: No prior history of nasal or sinus surgery
Allergies: NKDA
Medications: Pseudoephedrine to help with his nasal congestion. Advail;
Albuterol and Atrovent nebulize/~ Afbutero! inhaler; occa-
sional Sudafed for nasal congestion

312
Chapter 6- Rhinology Otolaryngology Clintcal Case Studies

FH: Father is treated f or asthma. No hisloT~v of cancers NoTEs


SH: Graduate s!udenl. Single. No HJV risk fac wrs. No history
of construclion or glass work. No history of tobacco use.
Soc ial drinker
ROS: Negative

What do you look for on PE?

• State that you would perfo1m a comp lete head, neck, neurology, and rhinol-
ogy PE including vital signs and otoscopy and nasal endoscopy with and
without decongestant

PE:

l!i1als.: Temp: 98.6 F, BP: JJ0/70. HR: 68. RR: 16, Wt: 185 lb.
GA: Young Caucasian male in NAD and otherwise appearing
healthy; he has a hyponasal voice
Head: Normal cephalic, no asymmetry
Ears: AD - anterior focus of 1nyringosclerosis but otherwise nor-
mal TM, normal mobility, no MEE, no masses
AS - same as AD
Eyes: No proptosis/telecanthus, no restricted gaze, no diplopia,
normal exam
Nasal cavity: Right cavity with edematous mucosa, congested, septal spur
with septal deviation to the right, turbinate hypertrophy.
Left cavity with nasal polyps. edematous mucosa, turbi-
nate hypertrophy
Nasal
endoscopy: After decongestant applied, a nasal polyp stalk traced ·back
to ma;dl/ary ostium, thick mucin noted from infundibulum
OC: No masses, lesions, or other abnormalities. no obvious post-
nasal drainage
Neck: Supple with no nodes or bruits .
Neuro: CN normal

What diagnostics would you request?

• Labs - CBC with differential


• Obtain total lgE serum level
• Consider RAST or allergic skin testing to evaluate allergies. Fungal antigens
should include Aspergillus, ·sipolaris and Alternaria. Additional antigens
may need to be tested because fungal antigen panel is unique to a geographic
reg10n
• Obtain CT scan without contrast of the paranasal sinuses in the axial and
coronal planes

::S13
Otolaryngology Clinical Case Studies Chapter 6- Rhinology

NOTES Test results:

• CBC is within normal range. The differential shows an elevation of


eosinophils over the normal range. Total eosinophil coum is 100,000
cells/m/ (1 0%)
• Total serum lgE level is 1954 mg!dl. Normal values are usually less
than 50 mgldL
• The RAST evaluation showed elevated JgE to various f ungi, ragweed
and cockroach
• CT scan shows complete opacification of the left maxif!ary and ethmoid
sinuses. Characteristic CT f indings consistent with AFRS mucin are noted,
including heterogeneous areas of signal inlensity. This is thought to
reflect the accumulation of manganese, iron and calcium salt precipita-
tion within the mucin. In addition, e.tpansion of the maxillary sinus is
noted

What is your differential diagnosis?

• CRS with nasal polyps- This constellation of symptoms and physical find-
ings characterize rhinosinusitis. Based on the duration of this patient's symp-
toms for at least 2 years, he suffers from chronic sinusitis. On anterior rhi-
noscopy, edematous cystic lesions are identified consistent with nasal polyps ·
• AFRS- A clinically distinct form of CRS, AFRS was described about 25
years ago. It is characterized by an eosinophilic mucin containing fungal
antigens and by a systemic fungal allergy. The diagnostic criteria established
by Bent and Kuhn is the most widely recognized and consists of 5 findings:
lgE-mediated hypersensitivity, nasal polyps, characteristic CT scan findings, ·
positive fungal stain or culture from mucin, and eosinophilic mucin with fun-
gal elements without evidence of i.nvasive process. The diagnosis of AFRS
cannot be confirmed without histologic analysis ofthe eosinophilic mucin
• Invasive fungal rhinosinusitis- With asthma that occasionally requires sys-
temic steroid, this patient has a small risk for being immunocompromised.
History and PE usually differentiates non- invasive and invasiv~ fungal si-
nusitis. Without a history of a fever, not actively taking system ic steroids, no
other risk of being immunocompromised, and a rhinoscopic exam showing
no invasive features, invasive fungal rhinosinusitis is low on the differential
diagnosis
• JNA - Needs to be considered in any teenage or young adult male with a
nasal mass. Patients usually present with a history of a nasal mass associ-
ated with frequent epistaxis. On exam, a JNA can usually be differentiated
from nasal polyps. The mass usually originates near the posterior middle
turbinate rather than from the middle meatus. As the mass enlarges, it ob-
structs the nasal cavity and NP and causes distortion ofsun·ounding anatomy

Diagnosis:

AFRS. Confin110tion of the diagnosis cannot be completed until after sur-


gery where mucin is collected and evaluated for the presence of fungi.

314
Chapter 6 - Rhinology Otolaryngology Clinical Case Studies

What treatment would you begin? NOTES

• Based on the current understanding of the pathophysiology of AFRS. wh ich


is an atopic cycle perpetuated by exposure to fungal antigen and the result-
ant inflammation, a comprehensive management of the disease includes re-
duction of antigen via surgical removal of fungal mucin and control of inflam-
mation. Therefore, the initial treatment entails FESS to remove fungal mu-
cin. Pre-operative treatment includes:
- One week preoperatively, the patient is started on systemic Prednisone
(0.5 mg/kg, about 40 mg per day) to reduce nasal polyps and inflamma-
tion. The patient should be consented for its use; risks of mood changes,
weight/appetite gains, blood sugar elevation, osteoporosis, aseptic femo-
ral head necrosis, and peptic ulceration are discussed. To protect against
stomach ulceration, a PPI or H2 blocker is concomitantly prescribed.
Steroids _need to be prescribed carefully in some, such as those with
poorly controlled diabetes or HTN, psychiatric disorders, or AIDS
- A nasal steroid spray is started with the systemic steroids. The dose is
one spray in each nostril twice a day
- In this patient who suffers from asthma, consider using a leukotriene in-
hibitor. The use ofleukotriene inhibitors has reduced dependence on other
asthma medications such as acute-acting bronchodilators. On the other
hand, there is less data on leukotriene inhibitors and allergies
• The goals of FESS are complete extirpation of fungal mucin, permanent
ventilation and drainage of affected sinuses, and post-operative access of
disease sinuses. A specimen ofthe collected fungal mucin should be sentto
pathology for confirmation of AFRS. In addition, mucin should be sent for
fungal culture. Fungal stains are more frequently positive than fungal cul-
tures. However, both should be sent. In addition, the nasal polyps are sent to
pathology to confirm diagnosis and to rule out any concomitant malignancy
- Histology on this patient's nasal masses is consistent with nasal polyps:
edematous tissue characterized by pseudostratified ciliated columnar
epithelium, thickened epithelial basement membrane and significant num-
ber of eosinophils
The histology of allergic mucin in this case is characterized by typical
Charcot-Leyden crystals, numerous eosinophils, and scattered fungal
hyphae
• Systemic steroids that were initiated pre-operatively are continued post-op-
eratively. The dose and duration of systemic steroids is not standardized.
Many wean steroids completely over a month. Others wean steroids to a
low dose and maintain this dose for a year or longer. Finally, some physicians
wi ll wean the steroids to a low dose and then ultimately wearf the dose to
'

zero over several months. There are no randomized controlled studies ad-
.
dressing this issue of systemic steroid dosage and duration
• A nasal steroid spray is continued after FESS. Topical steroid is a mainstay
of daily medical therapy to control nasal inflammation
• Nasal saline inigations are started immediately post-operative and continued
indefinitely ·
• Initiate immunotherapy after FESS
- The role of immunotherapy in the management of AFRS is still contro-
versial. Within the first 3 years, use of immunotherapy has shown a

31 5
Otolaryngology Clinical Case Studies Chapter 6- Rhinology

NOTES significant reduction in systemic steroid dependence and rate of re-op-


eration. However, long-tem1 studies revealed that if the disease is con-
trolled by any means for the first 3-5 years. the rate of recurrence is
similar with or without the use of immunotherapy
• Initiate general allergy evaluation. More than 67% of patients with AFRS
rep011 a history of allergic rhinitis
• Consider topical anti-fungals. The use of topical anti-fungals is still prelimi-
nary and data supporting their efficacy is inconclusive. Several studies have
shown no statistically significant clinical improvement with topical anti-fungals.
ln studies showing improvement with topical anti-fungals, they have been
delivered as si nus irrigations. Therefore, many argue that the benefit from
the anti-funga l stems primarily from the mechanics of the sinus irrigation.
This area is under active investigation

Surgical techniques for FESS:

Highlights:
• CT sinus in the coronal plane must be available in the OR. If an image-
guided navigation system is to be used in surgery, then these CT images must
be obtained through the appropriate protocol. Image-guided systems can be
helpful in cases where the anatomy is distorted such as in revision sinus .
surgery, extensive nasal polyps and tumors
• Inject local anesthesia to assist with hemostasis- Using 1% Lidocaine with .
1:100,000 epinephrine, the fol lowing sites are injected: greater palatine fora-
men, anterior and superior aspect of middle turbinate, insertion site of unci-
nate process and anterior face of ethmoid bulla
• Place Afrin-soaked pledgets in each nasal cavity for decongestion.
• Consider septoplasty if a deviated septum prevents adequate visualization
and access of the sinuses
• Medialization of middle-turbinate for access ofparanasal sinuses. Some sur-
geons will ~uture the middle turbinates to the.septum
• Proceed with the uncinectomy
• Maxillary antrostomy- The natural ostium is probed to identify its location.
The antrostomy is performed not only to provide ventilation-but also to serve
as a landmark for the orbit
• Anterior ethmoidectomy - The ethmoid bulla is the largest anterior ethmoid
cell. It is entered at an inferior and medial site to minimize risk to orbit. The
lateral wall of ethmoid bulla is the lamina papyracea
• Identify basal lamella- This structure represents the vertical attachment of
the middle turbinate and separates the anterior ethmoid cells from the poste-
rior ethmoid cells. Remove basallamella.without disrupting horizontal basal
lamella, which exposes branches of internal-maxillary artery and can cause
unnecessary bleeding
• Posterior ethmoidectomy- The sku ll base is nom1ally widely exposed after
a posterior ethmoidectomy. Identify any sphenoethmoidal cell (Onodi cell),
which can extend supe~ior and lateral to the sphenoid sinus. This cell can
house a dehiscent optic nerve. After the skull base is identified within the
sphenoid sinus, a posterior to anterior dissection along the skull base is per-
formed to comp lete the total ethmoidectomy. Avoid dissection medial to the

316
Chapter 6 - Rhinology Otolaryngology Clinical Case Studies

middle turbinate where the cribrifonn plate can lie inferior to the roof of the NoTES
ethmoid sinus
• Sphenoid sinusotomy as needed- Identify sphenoid ostium either lateral to
middle turbinate after the inferior portion of superior turbinate has been re-
moved or medial to the middle turbinate at the level of the maxillary ostium.
Be aware of the location of the carotid artery, optic nerve and skull base
once the sphenoid sinus is entered. In this case, a sphenoid sinusotomy is not
necessary
• Frontal sinusotomy as needed- Open frontal recess carefully since this is
the site of greatest scarring. It is bordered anteriorly by the posterior wall of
the agger nasi cell, medial ly by the anterior insertion site of the middle turbi-
nate and laterally by the lamina papyracea. Again, in this case, a frontal
sinusotomy is not needed

Pitfalls:
• Complications are typically categorized as either major or minor. Major com-
plications are rare and are reported as less than l %. These include blindness
·and intracranial violation. The minor complications such as synechia are also
uncommon at 5-8 % .
• Orbital complications including optic nerve injury, orbital hematoma, and vio-
lation of lamina papyracea
- Violation of lamina papyracea with herniation of orbital fat does not ne-
cessitate cessation of surgery. The site of dehiscence need only be
avoided. To avoid damage, always use the microdebrider tangential to
the lamina papyracea
- Orbital hematomas need to be addressed aggressively to reduce intraor-
bital pressure and control bleeding. Removal of any nasal packing and a
lateral canthotomy and cantholysis can reduce intraorbital pressures.
Elevated intraorbital pressure longer than 60-90 minutes can result in
pem1anent blindness
• Intracranial violation with CSF leak
- The fovea ethmoidalis and cribriform plate, which is commonly inferior
to the roof of the ethmoid sinus, is prone to damage. One should avoid
superior dissection medial to the insertion ofthe middle turbinate
- The medial skull base has much thinner bone than the lateral counterpart
- Any intraoperative CSF leak should be repaired intraoperatively

How would you follow this patient:

• Meticulous endoscopic rhinology exams and possible debridement are impor-


tant for post-operative surveillance for recurrence. For the first month, weekly
clinic exams with debridement of nasal crust and residual mucin is performed.
After the first month, clinical surveillance exams are scheduled monthly for the
first year. Then patients should be seen at some regular interval, usually every
2 months, for at least l disease-free year. Recurrence is not uncommon and
many patients undergo more than one surgical procedure in a lifetime
• Some follow total serum lgE levels as a measure of treatment response. If
serum levels rise more than or equal to I 0% of the pre-operative levels and
symptoms increase concurrently, this may indicate disease recurrence

317
Otolaryngology Clinical Case Studies Chapter 6- Rhinology

NoTES • Systemic steroid therapy remains one of the most prevalent means of man-·
aging this disease and minimizing recurrence. However, the dose and dura-
tion remains a controversial topic
- Some taper Prednisone to a low dose such as 20 mg/kg. A nasal mucosa
staging system described by Kupferberg can be used to monitor re-
sponse. If stage 0 is maintained at this dose for several months, then the
dosage is lowered and again regular rhir.oscopy exams are performed.
This cycle is repeated until steroids are tapered to 0. Topical nasal ste-
roid spray is typically increased to 3-4 times a day as the systemic ste-
roid is reduced for maintenance therapy
- Others taper the steroids to a minimum dosage of 5 mg every other day,
which is continued for several years
- Finally, immunotherapy has been shown to minimize steroid usage. Ste-
roids can be tapered post-operatively to 0, while the patient is initiating
immunotherapy. Systemic steroids in this protocol are reserved for short-
burst therapy when recurrence of disease is suspected
• If the disease does not respond to steroids, revision FESS may be necessary
• Suspected recurrence can be evaluated by CT scans

References:

I. Manning SC, Holman M. Further evidence for allergic pathophysiology in


allergic fungal sinusitis. Laryngoscope. 1998; 108( 10): 1485- 9.
2. Mabry RL, Marple BF, Folker RJ, Mabry CS. Immunotherapy for allergic
fungal sinusitis: three years' experience. Otolaryngol Head Neck Surg.
1998; I 19:648-51.
3. Schubert MS, Goetz DW. Evaulation and treatment of allergic fungal sinusi-
tis, II: treatment and follow-up. J Allergy Clin Immunol. 1998; 1"02:395-404.
4. Marple BF. Allergic fungal rhinosinusitis: current theories and management
strategies. Laryngoscope. 200 I; I I 1: 1006-19.
5. Kupferberg SB, Bent JP, Kuhn FA: Prognosis for allergic fungal sinusitis. J
Otolaryngol Head Neck Sur~. 1996, 117:35-41.
6. Kuhn FA, Swain R. Allergic fungal sinusitis: diagnosis and treatment. Curr
Opin Otolaryngol Head Neck Surg. 2003 ; 11: 1-5.

318
Chapter 6- Rhinology Otolaryngology Clinical Case Studies

CASE I0- INVASIVE FUNGAL RHINOSINUSJTIS NOTES


Amber Luong, MD, PhD

A 33 year-old Hispanic female presents with a chief complaint of a hole


wiihin her pala!e.

HPI: state that you would:

• Obtain a more detailed medical history beginning with the ch ief complaint

What additional historical information would you seek? State that you would ask:

• How did she get the hole in her palate?


• Is it stable in size? Getting larger or smaller?
• How long has it been there?
• ls it painful?
• Any recent trauma?
• Unintentional weight loss?
• Night sweats?
• Any nasal complaints?
• Any neurological complaints such as headaches, vision changes or weakness?
CutTent medications, specifically immunosuppressants?
• ls she diabetic, immunocompromised, suffering from serious illnesses?

She noted the defect about 1- month ago: It was initially a small mass that
became an ulcer. It grew in size within the first week ofits qevelopment to
its current size of 1 em. Since then, it has remained this size. It is not pain-
ful. She denies any difficulty eating or drinking associated with this le-
sion. She reports that about 1 month prior to developing this mass, she had
a severe URI. She had nasal airway obstruction and significant nasal drain-
age. Within 2 weeks, she developed anosmia and has not had any ability to
smell since then. She denies any recent trauma or unintentional weight
loss. Denies any night sweats. She has had a chronic mild to moderate
frontal headache, which started when she lost her sense of smell. She has
had no changes in her vision. She reports numbness over left cheek for
Last 2 weeks.

Pji,;JH: History of SLE. She was diagnosed at age 19 yrs. She de-
veloped lupus nephritis and then renal failure. She ulti-
mately undenvent renal transplant 2 years prior to presen-
tation. Since her transplant, she has been on immunosup-
p. ·essant medications. Also. she developed diabetes second-
ary to steroid use. She also suffers from arthritis. also sec-
ondary to the SLE
PSH: Right renal transplant
Allergies: NKDA
i\1/edications: Prednisone 10 mg dai~v. Tacrolimus 6 mg BID, Azathio-
prine, multivitamin. Epogen and Avandia
FH: Mother had systemic lupus erythematosus

319
Otolaryngology Clinical Case Studies Chapter 6 - Rhinology

NoTES SH: Not H'orking. Single. Has received blood transfusions. No ·


history of tobacco or alcohol use
ROS: Complains of muscle and joim stiffness/soreness. Has a
chronic problem with diarrhea

What do you look for on PE?

• State that you would perform a complete head, neck, rhinologic and neuro-
logic PE including vital signs and rhinoscopy

PE:

1!11aJ..s.: Temp: 98.6 F. BP: 140185, Pulse: 86, RR: 18, Wt: 155 lbs,
Ht: 5 '1''
GA: Hispanic female in NAD, she is stout with a moon face and
buffalo hump consistent with long-term prednisone use. She
has hypernasal speech
Head/face: Symmen-ic, no masses/lesion, rounded face, no obvious rash
or lesions
Ears: Bilateral TJl.ds clear with no MEE, normal exam
Eyes: Normal intercanthus distance, PERRL, EOMI, normal exam
Nasal cavity: Crusting within and throughout both nasal cavities, large
septal perforation, dark discoloration of the left middle
turbinate, no active rhinorrhea
Rhinoscopy: Nasal crusting extends to the posterior cavity, normal mu-
cosa of the NP, dark discoloration of left middle turbinate
extends to the posterior aspect
OC: A 1 em thru-and-thru ulcerative lesion on the posterior left-
side of hard palate, no erythema of surrounding mucosa,
no mucosal discoloration, no other lesions or masses
Neck: Supple with no nodes or bruits, thick neck with excess adi-
pose tissue
New-o: CN normal e.-tcept for insensate left aspect of palate. Nor-
mal gait. No obvious weaknesses

What is your differential diagnosis?

• Invasive fungal rhinosinusitis - Invasive fungal sinusitis can present as an


acute fulminant or a chronic variant. The acute invasive fungal rhinosinusitis
typically presents in an immunocompromised, i11-appearing patient. Common
presenting symptoms include a fever with a rapid onset of nasal congestion
or drainage. The disease rapidly progresses with intracranial or orbital inva.:
sion proceeding within hours to days. The diagnosis of acute invasive fungal
rhinosinusitis is suspected on rhinoscopy and confirmed with a biopsy of
nasal mucosa. Treatment is started immediately, if suspected. The chronic
variant progresses at a slower rate, over days to months. Aspergillus
fumigatus is the primary fungus responsible for the chroniG variant. Fever is
not a presenting symptom. Nasal symptoms such as anosmia, presence of
nasal mass, and nasal drainage are typical. On rhinoscopy, evidence of an
invasive process such as septal perforation or a fistula is often present

320
Chapter 6 - Rhinology Otolaryngology Clinical Case Studies

• Natural ki ller!T-celllymphoma - Once termed lethal midline granulomatous NoTES


disease, a highly destructive angiocentric fmm ofT-cell lymphoma wil l present
with soft tissue destruction and nasal mass
• Cocaine abuse- Long-term use of cocaine can cause sinonasal ulcerative
lesions. This is an uncommon complication of cocaine use
• Granulomatous disease- Wegener's and sarcoidosis can both present with
sinonasal ulcers. As systemic diseases, both typically involve other organ
systems. Wegener's granulomatosis is a vasculitis that commonly affects
the resp iratory tracts and kidneys. Sarcoidos is is characterized by
noncaseating granulomatous lesions, which can be found in any pa11 of the
body. The etiology of sarcoidosis is unknown but the pathophysiology may
stem from a hyperactive, aberrant immune system
• Noma- Also known as gangrenous stomatitis, noma is an acute, fulminating,
gangrenous OC lesion that affects severely malnourished individuals. Fuso-
spirochetal organisms are the most common bacteria cultured from these
destructive lesions
• Cleft palate- This congenital deformity has an incidence of 1:1000 live births.
It often presents in association with a cleft lip. This defect requires surgical
repair, which is normally performed in childhood

What diagnostics do you request?

• Obtain a nasal ml,lcosa biopsy and send for anaerobic, aerobic and fungal
cultures in addition to pathology
• Obtain labs: CBC with differential, chemistry panel, ACE level, C-ANCA
• Obtain an expedited CT with and without contrast of the paranasal sinuses in
the axial and coronal planes ·
• Obtain an expedited MRl with gadolinium offace/orbit/head

Test results:

• Nasal mucosa biopsy .showed angioinvasive fungal hyphae, sinonasal


mucosa and bone with necrosis, and evidence _of chronic inflammation.
Silver stain noted filamentous fungi with branching septate hyphae.
Viral and bacterial cultures were negative
• CBC and chemistry panel were all WNL except creatinine at 1.8. ACE
level was WNL. C-ANCA was negative for Wegener :S·
• CT sinus revealed severe soft tissue edema of the nasal mucosa, prima-
rily on the left. Sinus mucoperiosteal thickening is noted within the left
maxillary, anterior and posterior ethmoid and frontal sinuses. An obvi-
ous dehiscence of the left hard palate is noted, creating an oronasal
fistula. The right nasal mucosa has mild nasal mucosa edema without
right-sided paranasal mucoperiosteal thickening
• MRI confirmed mucoperiosteal thickening. In addirion, there was ab-
normal enhancing soft tissue extending into the left pterygomaxillary
fissure with loss of cortical margin along the posterior maxillary antrum

Diagnosis?

Invasive fungal rhinosinusitis, chronic variant

321
Otolaryngology Clinical Case Studies Chapter 6 - Rhinology

NOTES What treatment would you begin?

• Immediate surgical debridement of necrotic tissue- The elimination of the


inciting fungal agent is critical in the treatment of invasive fungal rhinosinusitis.
Since lV antifungals are ineffective in necrotic tissue, complete surgical de-
bridement offers the on ly means of disease containment. Therefore, expo-
sure is the key aspect and complete removal of necrotic tissue is critical for
successful surgical therapy
• Systemic antifungal - Consult infectious disease and renal service for assis-
tance with duration and dosage of lipophil ic amphotericin B in this patient
with a cadaveric kidney. Lipophilic amphotericin 8 was designed to reduce
nephrotoxicity of amphotericin 8, which affects up to 80% of patients who
take this medication. The normal dose for lipophilic amphotericin B is 5 mg/
kg/day. For this patient with on ly one kidney and an elevated creatinine level,
this dose may need to be reduced. Duration of use of amphotericin B is
dictated by the response to the anti-fungal, at least several months. Creati-
nine levels should be monitored during amphotericin treatment. The patient
needs to be counseled on the possibi lity of damage to and/or Joss ofher renal
transplant

Surgical techniques:

Highlights and pitfalls:


• Approaches to the pterygomaxillary space are classified as external versus
endoscopic. A brief discussion of the two primary examples of each type of
approach is presented below:

Lateral rhinotomy with Denker's procedure


Highlights:
• This incision was originally described as an approach to the maxillary-sinus
for a medial maxillectomy. It involves a vertical incision that is made be-
tween the medial canthus ligament and nasal dorsum and along the nasal-
cheek groove. The superior border of the incision is the level of the medial
brow and classically extends through the upper lip inferiorly
• The periosteum is elevated off the anterior wall of the maxillary sinus while
respecting the infraorbital nerve
• The Denker's procedure is completed, exposing the inferior orbital floor, the
posterior wall of the maxillary sinus and the ipsilateral nasal cavity. The
Denker's procedure consists of wide anterior maxillary antrostomy, removal
of ascending maxillary process and removal of the inferior half of the lateral
nasal wall. Unlike the medial maxillectomy, the lamina papyracea is pre-
served, minimizing risk to the orbit and to the anterior cranial floor
• In order to expose the pterygomaxi llary space, the posterior wall of the max-
illary sinus is then removed
• The advantage of this approach is the wide exposure, which is critical to
ensure adequate debridement
• The disadvantage is the necessary facial incision

322
Chapter 6 - Rhinology Otolaryngology Clinical Case Studies

PitfaJis: NOTES
• Damage to the medial canthus ligament - The incision should be medial to
the attachment of the medial canthus. However. if avu lsed, the ligament
should be repaired
• Webbing of the soft tissue medial to medial canthus -A Z-plasty can be
incorporated into the incision as it overrides this location to prevent soft tis-
sue webbing

Endoscopic:
Highlights:
• Using typical endoscopic sinus surgety instruments, a medial maxi limy antros-
tomy and total ethmoidectomy are performed. The lamina papyracea is left in-
tact. This allows adequate exposure of the posterior wall of the maxillary sinus
• A window is then created in the posterior antral wall allowing exposure of
the branches of the internal maxi llary artery for hemostasis as needed
• The posterior maxillary wall is then removed endoscopically, allowing expo-
sure of the pterygomaxillary sinus for debridement

Pitfalls:
• The primary disadvantage of this technique is limited exposure, which can
compromise adequate debridement. For this case, this is a serious limitation
• Damage to the lamina papyracea- The lateral border of a total ethmoidectomy
is the lamina papyracea; placing it at risk for violatio_n..If orbital fat is visual-
ized during the procedure, no repair is necessary. However, care needs to be
taken to prevent further damage"at this site

How would you follow this patient?

• After the initial debridement, daily rhinoscopic exams shou ld be performed to


assess need for additional debridement. Any evidence of necrotic tissue should
be treated with aggressive debridement. This often requires a return to the OR
• A post-surgical MRJ can complement rhinoscopic exam to rule out persis-
tent disease. In addition, a MRl with contrast can be used to confirm resolu-
tion of disease. Diseased mucosa is enhanced w.ith gadolinium contrast
• The patient is initiated on sinus irrigation, given that no CSF leak was noted
intraoperatively. This helps to remove blood clots and necrotic tissue, and
allows easier rhinoscopic exams
• The duration of anti-fungals has not been specifically studied. However, many
treat with IV anti-fungals for at least 1 year. Rhinoscopic exams and post-
surgical MRI help guide the length of treatment. In this patient~ creatinine
and chemistries will need to be closely monitored for possible renal failure.
Treatment of invasiv_e fungal sinusitis overrides protection of the kidney.
However, the patient needs to tie counseled about this problem often. If
renal fai lure is imminent, renal dialysis will need to be initiated

Reference:

I. Stevens DA, Kan VL, Judson MA, et al. Practice guidelines for diseases
caused by AspergillusJnfectious Diseases Society of America. Clin Infect
Dis. 2000;30:696-709.

323
Otolaryngology Clinical Case Studies Chapter 6 - Rhinology

NOTES CASE 11 - ALLERGIC RHINOSINUSITIS


Joseph R. Williams, MD

A 37 year-old WF presems with the chief complaint of "sinus all my life.··

HPI: state that you would:

• Obtain a detailed medical history beginning with a history of the present


illness

What additional historical information would you seek? State that you would ask:

• How do you describe your ·'sinus problem?"


• Do you have nasal congestion, post-nasal drip, a "runny" nose, sneezing, or
eye irritation?
• Do you experience fever, chills, facial, or dental pain?
• Are your symptoms seasonal or constant? If seasonal, which season of the
year do you have most of your symptoms?
• Have you had a recent URI?
• Do you have pets?
• Do you work outside or indo6rs?
Do you work in your garden s~asonally?

·The patient reports that she constantly blows yellow mucous from her nose
with alternating blockage of one side or the other. She lives in Texas and
has more trouble in late winter/early sprin'g and the fall. She has no pets.
She ·has not noticed fever or chills and has had no associated pain. She
also reports childhood a~thma which resolved at age 6 or 7. She has used
OTC remedies· "forever" with limited benefit and has taken short courses
of antibiotics which were usually left over from her children s prescrip-
tions. She recently obtained some Claritin from her primary care physician
which benefited her. only minimplly. ·

PMH: Healthy, except for childhood asthma


Allergies: NKDA
Medications: Claritin prn
FH: Mother had "hay fever: ·• 2 toddlers in day care who have
been treated at least twice per year for ear infections and
pinkeye
SH: (-) tobacco/occasional wine (sociably)/CPA
ROS: Itchy ears; occasionally has nonproductive cough without
wheezing

State that you would:

• Perfom1 a complete head and neck PE including vital signs and chest
auscultation

324
Chapter 6- Rhinology Otolaryngology Clinical Case Studies

Pt: NOTES

J!iwls.: Temp: BP: 120180. Pulse: 65, RR: 18


GA: Well-developed, well-nourished, well-dressed WF in NAD
HEENT Nonrender over paranasal sinuses, etc. Darkish discolora-
tion at medial canthi. Slightly retracted right TJ\11 without
effusion. Normal left TM She has a supratip 11asal crease:
boggy, and edematous pink to mauve colored inferior tur-
binares, her nasal septum is straight: no pus or polyps seen.
Moderate improvemem in nasal airways after deconges-
wnt nasal spray applied
Chest: No wheezing or bronchi

What is your differential diagnosis?

• Acute rhinosinusitis
• Chronic allergic rhinosinusitis
• Vasomotor rhinitis

What do you do next?

• Nasal endoscopy
• All.ergy testing

Test results:

• Nasal endoscopy revealed no evidence of purulence or polyps in the


middle meati. The middle turbinates were quite generous in size
• Skin end point allergy testing shows Class Ill reactions to trees, grasses,
and house dust mites (RAST and skin endpoint titration testing are quite
similar with class III reactions as well)
• CT scan of paranasaf sinuses shows minimal mucoperiosteal thicken-
ing in maxillary sinuses: both ostiomeatal windows are patent

Diagnosis:

The diagnosis is chronic allergic rhinosinusitis. This diagnosis is supported


by the chronicity of the problem based on the patients symptoms and the
positive findings on CT scan.

What are the treatment options and their complications? What would you do
and why?

• Daily topical anti-inflammatory nasal steroids


• Topical nasal antihistamines, systemic antihistamines with or without decon-
gestants and/or leukotriene antagonists can be added as needed to control
symptoms
• Allergy immunotherapy should be considered if initial treatment recommen-
dations fail or if the patient prefers this as opposed to daily medications
• Lntraturbinal Kenalog Injections may greatly decrease the need for systemic
medications

325
Otolaryngology Clinical Case Studies Chapter 6- Rhinology

NOTES References:

I. BousquetJ, Vignola A, Campbell A, Michel FB. Pathophysiology of allergic


rhinitis.lntArch Allergy lmmunol. 1996; II 0:207-18.
2. Corren J. Allergic rhinitis: treating the adult. J Allergy Clin Immunol.
2000;1 OS:S61 0--6.
3. Ferguson BJ. Cost-effective pham1acotherapy for allergic rhinitis. Otolaryngol
ClinNorthAm.l998;31:91-IIO.
4. Levenson T, Greenberger P, Pathophysiology and therapy for allergic and non-
allergic rhinitis: an updated review. Allergy Asthma Proc. 1997;18:231-40.

326
Chapter 6 - Rhinology Otolaryngology Clinical Case Studies

CASE 12 - vASOMOTOR RHINITIS NoTES


Joseph R. Williams, MD

A 55 year-old white male presents with the chief complaint of "chronic"


sinus congestion x 5 years.

HPI: state that you would:

• Obtain a detailed medical history beginning with a history ofthe present illness

What additional historical information would you seek? State that you would ask:

• Do you fee l like your symptcms are acute presently or chronic?


• How do you describe "sinus congestion"?
• Are your symptoms seasonal?
• Are your symptoms affected by temperature changes, barometric pressure
changes, perfumes, wind, stress, smoke, dust or ceiling fans?
• Do you have pets?
• Have you ever been allergy-tested?
• Have you had sinus or nasal surgery?

Plv!H: Depression; Heartburn occasionally


Allergies: Penicillin (rash)
Medications: Prozac, Prilosec occasionally
FH: Negative
SH: Quit smoking 5 years ago (1 ppdlyear::;)
3 beers a week
ROS: Mild depression (job was downsized - factory supervisor),
occasional heartburn with occasional coughing spell at
2: 00 a.m.

What would you look for on PE? State that you would :

• Perform a complete head and neck PE including vital signs

He has symptoms of congestion and rhinorrhea year round, but they vary
in severity. He notices increased rhinorrhea, congestion and some sneez-
ing when moving from an air conditioned or heated building to the outside
and vice versa. Weather fronts also precipitate symptoms. He has no pets
and has never had nasal or sinus surgery.

PE:

Yimls.: Temp: 98.6 F. BP: 125185, Pulse: 84


GA: WDWN WM in NAD
HEENT Nontender over sinuses: p olypoid degeneration of
both inferior turbinates
No septal deviation; no pus or polyps seen: clear
mucoid secretions in floor of nose bilaterally

327
Otolaryngology Clinical Case Studies Chapter 6- Rhinology

NOTES Nasal endoscopy: Shows polypoid degeneration of both inferior fur- ·


binates with no septal deviation: the middle meatus
shows no actual polyps and the ostiomeatal com-
plex is without obstruction

What is your differential diagnosis?

• Vasomotor rhinitis
• Allergic rhinitis
• Chronic sinusitis
• Prozac side-effect

What do you do next?

• CT scan
• Trial of topical steroids (Flonase, Nasacort, Rhinocort, Nasonex)
• Trial of Atrovent nasal spray

Test results:

• Minimal improvement after 3 weeks of daily nasal steroids


• CT scan - normal
Allergy tests - negative

Diagnosis:

Perennial non-al(ergic (vasomotor) rhinitis

What are the treatment options and their outcomes? What would you do and
why?

Add topical Astelin nasal- Slight improvement with steroid and Astelin together
• Add antihistamine/decongestants
• lntraturbinal Kenalog injections - Improvement good initially but lasted only
2 weeks
• State that this is a difficult management problem because of poor respon-
siveness to most treatment. The above mentioned options oftopical nasal
steroids and even injectable steroids would generally be the plan of action for
patients with this malady
• In some cases, if there is significant obstruction from hypertrophic turbi-
nates, the turbinates can be resculpted in many differenPways with
microdebridement, laser surgery or somnoplasty. This will improve nasal func-
tion dramatically in most cases
• Also, be aware of aggravating issues such as LPR and antidepressant medi-
cations plus beta blockers. These patients are followed much like the inhal-
ant allergy patients and tend to have seasonal exacerbations as if they were
sensitive

328
Chapter 6- Rhinology Otolaryngology Clinical Case Studies

References: NOTES

I. Scadding, Glenis K. Non-allergic rhinitis: Diagnosis and management. Curr


Opin Allergy Clin Immunol. 2001 ; I ( I ): 15-20.
2. Jordan J, Mabry R. Geriatric rhinitis: what it is and how to treat it. Geriatrics.
j 998;53(6):76-84.
3. Bachert C. Persistent rhinitis-allergic or nonallergic? Allergy. 2004;59(Suppl
76):11-5.
4. Settipane RA, Settipane GA. Nonallergic rhinitis. In: Kaliner MA, ed. Current
review of allergic diseases. Philadelphia: Blackwell Science; 2000, 111-24.

329
Otolaryngology Clinical Case Studies Chapter 6- Rhinology

NOTES CASE 13- FRONTAL SINUS FRACTURE


Joseph Leach, MD

A 35 year-old man comes in after motor vehicle accident with a laceration


over right brow.

HPI: state that you would:

• Obtain a detailed medical history beginning with a history of the present


illness

PMH: . Negative
Allergies: NKDA
Medications: None
FI-i: Negative
SH: Tobacco - I ppd for 15 years
ETOH- 2-3 beers per day
Profession - Construction worker
ROS: Negative

What would you look for on PE? State that you would:

• Perform a complete head and neck PE including vital signs

PE:

The patient is slightly confused, but oriented times three. Alcohol is present
on his breath. The ears are clear. Bruising and swelling are present around
the right eye, but the extraocular muscles are intact. There is no diplopia.
Visual fields are intact. There is no telecanthus. He is leaking thin bloody
mucus out of the nose. The nose and septum are straight and the mucosa is
intact. The OC and OP are clear. His dentition is poor but there is no
evidence of trauma. There are no facial stepo.ffs, but crepitus is present in
right brow area. The neck is in cervical collar. CN II-XII are intact except
for numbness in lower to mid forehead

What is your differential diagnosis?

He has an anterior vs. posterior vs. combined fracture of frontal sinus with
possible involvement of the nasofrontal duct.

What diagnostics would you request? State that you would:

• Request a cranial CT and CT of the facial bones

330
Chapter 6 - Rhinology Otolaryngology Clinical Case Studies

Test results: NoTEs


Axial and coronal CT scans demonstrate opacification of right fron tal si-
nus with a displaced fracture of the frontal s inus. The anterior wall is
displaced about 7 mm and the nasojrontal duct is involved in the fracture.
The posterior_wall is intact.

What do you do next? State that you would:

• Clear the C-spine. Films are taken and the radiologist clears the spine
• Request beta 2 transfe1Tin analysis of nasal discharge

Test results:

• C-spine is cleared
• Beta-2 transferrin is negative

A ring sign will be present when the bloody fluid is dripped onto paper.
The CSF forms an outer halo with the blood confined to ·the cente1~ Copi-
ous mucus can confound the test, howeve1~ A glucose test will show a higher
value for CSF than for mucus. f3ut if the fluid is bloody. interpretation will
be difficult. The best test is a beta 2 transferrin, which is specific for CSF

Diagnosis:

• Laceration of right supratrochlear nerve


• Fracture offrontal sinus involving nasofrontal duct and cribriform plate

What are the treatment options· and their complications? State that you would:

• Admit for elevation of head, observation, bed rest, pain control


• Antibiotics are controversial

What are the surgical considerations?

• Since the nasofrontal duct is occluded, frontal sinus obliteration is the.best


option
• It is optimal to have a 6ft Caldwell radiograph of the sinus with the right side
marked with a coin. This will be used as a template to help locate the frontal
sinus. A templa~e is not critical in the case of frontal sinus fracture, however,
. since edges of the bone fragments are obvious and the limits of the sinus can
be sounded with a probing instrument
• Exposure is best obtained via a bi-coronal approach, unless there is a large
laceration over the sinus
• Elevation is carried out in the subgaleal plane
• Care needs to be taken laterally to avoid injury to the frontal branches of the
facial nerve
• Careful elevation deep to the temporo-parietal fascia or even deep to the
deep temporal fasc ia will protect the nerve

331
Otolaryngology Clinical Case Studies Chapter 6 - Rhinology

NOTES • It is nice to preserve a large pericranial flap, if possible, as the neurosurgeons


may need this to repair any dural defect that may be present. If the bone
fragments are severely comminuted, this may be impossible, however

In a patient this age, callus will form in a couple of weeks, and the bone
will not reduce easily. Swelling maximizes at .J8 hours. The best times to
operate are either within the first 2 days or between 5- 14 days.

Surgical technique:

Highlights and pitfalls:


• After the incision is made and the bone fragments are exposed, they are
oriented and preserved
• Loose fragments are placed on the back table to allow access to the sinus
• Mucosa is carefully removed from the sinus-magnification may be used if
necessary
• The duct is plugged with allograft dermis or a fragment of the patient's
temporal is muscle
• The sinus is obliterated with abdominal fat or hydroxyapatite or bone chips
from the iliac crest
• The bone fragments are then reduced and fixated with microplates or small
w1res
• The incision is closed over a suction drain

In the setting of head and facial trauma, be sure to evaluate for spinal and
intracranial injury before focusing on the facial fracture.

• Carefu lly remove sinus mucosa when obliterating the sinus


• If only the anterior table is fractured, fixation without obliteration is acceptable
• Anterior wall fractures with minimal (< 2 mm) displacement do not need
surgery

Pitfalls:
• Watch out for CSF leaks
• Evaluate the status of the nasofrontal duct
• Warn patients about temporary scalp numbness after the bicoronal incision

What are potential complications?

• Infection of the graft materials may occur


• Malunion of the bone is possible if fixation is not secure
• Hematoma may be a problem if hemostasis is not meticulous
• Frontal branch injury is possible secondary to traction or injudicious cautery
• Years to decades later, a mucocele may develop if removal of mucosa is not
meticulous

332
Chapter 6 - Rhinology Otolaryngology Clinical Case Studies

How would you follow this individual? State that you would: NOTES

• Remove the drain on day I or 2


• Remove sutures about day 7
• Evaluate for infection, undue pain or hematoma, and persistent CSF leak

References:

1. Luce EA. Frontal sinus fractures: guidelines to management. Plast Reconst


Surg. 1987;80(4):500-8.
2. Rohrich RJ, Hollier, LH. Management of frontal sinus fractures-changing
conepts. Clin Plast Surg. 1992; 19(1 ):219-32.
3. Wolfe SA, Johnson, P. Frontal sinus injuries: primary care and management
of late complications. Plast Reconst Surg. 1988;82(5):781-9.

333
Otolaryngology Clinical Case Studies Chapter 6 - Rhinology

NoTES CASE 14-0RBITALABSCESS (SuBPERIOSTEAL ORBITAL ABSCEss)


C. Spencer Cochran, MD

A 7 year-old male presents with a 2-day history of swelling around his left
eye.

HPI: state that you would:

• First obtain a detailed medical history beginning with a history of the present
illness

What additional historical information would seek? State that you would ask:

• Has there been any preceding trauma?


• Has he had recent URI with associated fevers or nasal drainage - clear,
mucoid, or purulent?
• Has he noticed any loss of visual acuity, double vision or blurred vision?

Subject reports he has had symptoms of an URI for the preceding week
with a low-grade temperature initially and nasal congestion. His symptoms
have worsened over· the last two days with a fever of JOJOF and bilateral
purulent rhinorrhea. He denies any gross visual changes. He has not sus-
tained any trauma. Today, for the first time, he has noticed double vision
when he looks to
the left.

PMH: Seas·o nal allergies


Allergies: NKDA
Medical ions: None
FMH: None
Social: + exposure to wbacco smoke by parents
ROS: NC

What would you.look for on PE? State you would:

• Perform a complete head and neck exam including vital signs and nasal
endoscopy

PE:

Yif.tJ.Ls_: Temp: 39.9 C, HR: 110, RR: 30, BP: 108160


GA: WD WN WJ\.1 who appears ill
HEENT Normocephalic/atraumatic PERRLA. Vision is grossly in-
tact. Moderate left-sided proptosis with limitation of lat-
eral and upward gaze. Left periorbital edema with overly-
ing erythema. No scleral injection or chemosis. Nasal exam
reveals bilateral purulent rhinorrhea with injected mucosa.
OP with 2+ enlarged tonsils
Neck: Supple, no palpable lymphadenopathy
New·o : CN 11- XJI intact

334
Chapter 6 -Rhinology Otolaryngology Clinical Case Studies

What diagnostics would you request? State that you would: NoTES

• Ask for a CBC with differential and draw blood cu ltures


• Order axial and coronal CT sinus with and without lV contrast

Test results:

• CBC with differential count - WBC = 19,500 with 80% neutrophi/s


• CT scan of sinuses shows opacification of both maxillary and ethmoid
sinuses, a hypodense convex subperiosteal fluid col1ection di!>placing
the left medial rectus muscle laterally with surrounding edema and de-
hiscence of lamina papyracea

What is your differential diagnosis?

State that the differential should include preseptal cellulitis, orbital cellulitis, sub-
. periosteal orbital abscess, orbital abscess, orbital neoplasm, and trauma (acci-
dental vs. child abuse)

Diagnosis:

Based on the CT scan, the diagnosis is left subperiosteal orbital abscess.


The subperiosteal abscess is an abscess of the medici/ orbit that occurs as
a complication of acute bacterial rhinos inusitis. Urgent surgical decom-
pression is required to prevent visual loss.

What are the _treatment options? What would you do and why?

• State that you would begin IV antibiotic therapy. An antibiotic active against
the common respiratory pathogens, Strep pneumonia, H influenza and M.
cataJThalis should be selected. Ampicillin (12.5 mg/kg) with sulbactam wi ll
provide adequate coverage with the added benefit of a beta-lactamase inhibitor
• Obtain an ophthalmology consult · .
• Recommend surgical intervention after infonning the patient of the potential
risks and complications

What are the surgical routes to the medial orbit?

• Open approach via an external ethmoidectomy with drainage of subperi-


osteal abscess through a curvilinear incision between the medial canthus and
9
dorsum of the nose or through an upper eyelid incision
• Endoscopic approach via uncinectomy and ethmoidectomy

Surgical technique:

Highlights and pitfalls:


• With improved imaging and endoscopic techniques, drainage of a subperi -
osteal orbital abscess can be accomplished via an endoscopic approach with
less morbidity and improved cosmesis compared to open approaches. A rigid

335
Otolaryngology·Clinical Case Studies Chapter 6 - Rhinology

NOTES sinus endoscope is placed within the nasal passage of the affected side. AI,.
uncinectomy is performed which allows visualization of the bulla ethmoidal is.
Next, an ethmoidectomy is performed to allow wide exposure of the lamina
papyracea. The lamina papyracea is removed and the orbital periosteum is
elevated off the lamina until the abscess is encountered. Pus is collected to
examine for anaerobic, aerobic, and fungal organisms. The nasal passage
and abscess site is judic iously inigated. If anatomic constraints preclude an
endoscopic approach, an external ethmoidectomy should be performed
• Postoperative care should include topical and systemic decongestants, IV
antibiotics, and nasal irrigation

Potential complications:

• Trauma to the nasolacrimal duct can result in epiphora - Avoided by limiting


anterior dissection ·
• Trauma to the medial rectus muscles producing permanent diplopia
• CSF leak- Avoided by paying attention to landmarks and avoiding excess
superior or posterior dissection
• The patient should be observed closely for signs of recurrence or inadequate
drainage. CT scans should be repeated as needed

How would you follow this individual?

• In the immediate postoperative period, the patient should be followed with


daily examinations aild serial ophthalmological assessments to determine reso-
lution of ocular symptoms
• IV antibiotics should be continued for 48-72 hours followed by oral amoxicillin/
clavulanic acid ( 15 mg/kg) for 10-14 days, nasal saline irrigation, topical
corticosteroid nasal spray, and topical and systemic decongestants
• Upon discharge, the patient should.be instructed to complete his 10--14 day
course of oral antibiotics and continue nasal saline irrigation and topical nasal
steroid spray for an additional 2 to 3 weeks
• The patient should be seen in an outpatient setting within the first week after
'discharge from the hospital at which time nasal endoscopy and debridement
can be performed if necessary
• Long-term follow-up in outpatient setting with rigid nasal endoscopy and CT
scans to assess for ongoing sinonasal disease

References :

1. Manning SC. Endoscopic management of medial subperiosteal orbital ab-


scess. Arch Otolaryngol Head Neck Surg. 1993; 119(7): 789-91.
2. Younis RT, Lazar RH, Bustillo A, Anand VK. Orbital infection as a compli-
cation of sinusitis: are diagnostic and treatment trends changing? Ear Nose
ThroatJ. 2002; 81(11 ):771-5.

336
Chapter 6 - Rhinology Otolaryngology Clinical Case Studies

CASE 15- EXOPHTHALMOS FROM THYROTOXICOSIS NOTES


lVlichael Yium, MD

A 50 year-old female presents to y our office with a chief compla int of


several months of painless right sided eye swelling.

HPI: state that you would:

• Obtain a detailed medical history beginning with a history of the present illness

PMH: HTN, GERD


PSH: Tubal ligation, C-section x 2
Allergies: NKDA
j\!fedications: Verapamil, rabeprazole
SH: Nonsmoker, nondrinker
ROS: Occasional palpitations,· she has noticed about 15 lb weight
loss over the last 6 months but she reports she has been
trying to lose weight; she does have occasional hot flashes
which she attributes to menopause

What additional historical information would you seek?

• Teri months stated previously


• Slowly progressive stated previously
Is there double vision?
• Is it painful?
• Is it associated with fevers, chills, or night sweats?
• Has she experienced any other systemi'c systems, such as fatigue, tremors,
palpitations, weight loss/gain?
• Has she experienced any history of sinus surgery?
• Any associated nasal complaints - Nasal obstruction, rhinorrhea, anosmia?

The answers to these questions were originally in the HPI:

She tells you she has no pain, no diplopia, no visual acuity changes. No
associated nasal congestion, no rhinorrhea, no anosmia. No fever, no his-
tory of sinusitis, no trauma. No history of sinus surgery.

What do you look for on PE? State you would:

.
• Petform a PE. Stat1 with vital signs and then move on to a head and neck
exam

PE:

J!i1al: Temp: 97.9 F, Pulse: 75, BP: 145/86, RR: 12


GA: WD female in no apparent distress
Ears: Tivf intact bilaterally, no perforations, no retractions
Nose: Clear to anterior rhinoscopy bilaterally
Fiber optic nasal endoscopy: normal nasal cavity and lat-
eral nasal walls bilaterally, no masses or lesions seen
337
Otolaryngology Clinical Case Studies Chapter 6- Rhinology

NOTES Eye: Right eye proptosis of abou1 3 111111 compared to left eye,
EOMI. VA is 20120 with corrective lenses, PERRL. globe is
soft and nontender to palpation
Face: Nontende1~ CN V intact bilaterally
OCIOP: No lesions. no masses palpated
Neck: No LAD, salivary glands feel normal. tracheal midline
Neuro: CN !I-XII intact bilaterally. normal sense of smell
bilaterally

What is your differential diagnosis? What is the most common ca1,1se of unilateral
proptosis?

• Graves' disease with ophthalmopathy


• Orbital neoplasm
• Orbital pseudotumor
• Paranasal sinus neoplasm
• Orbital abscess
Complicated rhinosinusitis
• Sinus mucocele
• Sarcoidosis
• Amyloidosis

The most common cause of unilateral proptosis is Graves'· ophthalmopathy

What diagnostics would you request?

• Labs- Thyroid function tests TSH (Graves' ophthalmopathy is in the work-


ing diagnosis and ·a TSH level is critical·in establishing whether the patient is
euthyroid or hyperthyroid)
• Imaging- CT with contrast of the sinuses to evaluate for orbital neoplasm
• Consultation- Ophthalmology consultation to assess exophthalmos and check
visual acuity

Test results:

• Labs: TSH is measured at 0.1 mUlL. TSH is suppressed in hyperthyroid


states
• Imaging: CT scan with contrast is shown in Figure 1 (at end of case). CT
reveals enlargement of extraocular muscles. with. sparing of the tendi-
nous portions of the muscle. There is no bony erosion. There are no fluid
collections and no masses. The right side is more jnvolved than the left
• Ophthalmology agrees there is moderate exophthalmos, but no optic
neuropathy, and intraocular pressure is normal. Eye movement is not
restricted. The patient has some mild exposure keratopathy

338
Chapter 6- Rhinology Otolaryngology Clinical Case Studies

Figure I: Axial CT scan of orbits/si nuses. Note the hypertrophy of medial recti NOTES
muscles with some proptosis noted on the right side. There is sparing of the
tendinous insertions.

Diagnosis:

State that the diagnosis is Graves' ophthalmopathy.

What are the treatment options and their complications?

• Treatment of the patient's underlying Graves' disease is the first course of action
• An endocrinology consu lt is warranted to primari ly manage the patient's
thyroid disease

Ocular findings in early thyroid ophthalmopathy include:


I. Diplopia
'? Corneal exposure
3. Optic neuropathy
4. Cosmetic defonnity

339
Otolaryngology Clinical Case Studies Chapter 6- Rhinology

N{HES These processes usual ly stabiIize after 6-18 months


Ocular find ings in chronic phase of thyroid ophthalmopathy include:
1. Orbital fibros is
2. Permanent en largement of extraocular muscles

What would you do next?

Treatment of early stage Graves' ophthalmopathy is directed at the patient's


underlying hyperthyroidism. Occasional ly steroids may be indicated for treating
optic neuropathy
• Treatment of hyperthyroidism - PTU vs. radioactive iodine ablation
- PTU: an antithyroid drug
• intrathyroidal action by inhibiting iodine oxidation and organification
to decrease synthesis ofTJ and T4. Extrathyroidal action by inhibit-
ing conversion ofT4 to T3
• Corneal exposure can be treated with ointment, taping the eyelid, and topical
lubrication
• Strabismus is initially corrected with prisms and can be surgically corrected
later if unresponsive to medical therapy
• Optic neuropathy is treated with steroids and possibly surgery if refractory
to medical therapy
• Low dose radiation therapy is often used in combination with steroids and
surgery to treat orbital inflammation

What are the indications for surgery? What are some perioperative considerations?

• Severe exposure keratopathy


• Refractory optic neuropathy
• Severe proptosis

In general, surgery should be delayed until the orbital disease has stabilized with
medical treatment, usually at least 6 months. Orbital decompression should pre-
cede strabismus and :eyelid surgery since decompression alters the position of
the globe.

What are your surgical treatment options?

• Transoral approach to orbital decompression


• Transnasal endoscopic approach to medial decompression (the most com-
mon approach)
• Lateral orbital decompression
• "Balanced" orbital decompression combining both lateral and medial ap-
proaches is preferred (decompression surgery is often performed in con-
junction with an oculoplastics specialist who can assist with the lateral ap-
proach and strabismus issues when necessary)

Medial and lateral decompression is preferred

340
Chapter 6- Rhinology Otolaryngology Clinical Case Studies

Surgical techniques: NOTES

Lateral internal decompression


Highlights and pitfalls:
• Comeal shield to protect the eye
• Lateral canthotomy and cantholysis is performed
• Incise periosteum and reflect medially
• Lateral orbital rim is trimmed with drill and bony resection is carried out
posteriorly and inferiorly to level of infraorbital canal
• Bony resection is carried out superiorly to level of lacrimal fossa
• Temporal is muscle periosteum is left intact so muscle does not prolapse into
orbit
• Hemostasis with bipolar cautery
• Meticulous closure and reconstruction of orbital rim periosteum and lateral
canthal angle to optimize soft tissue aesthetics and avoid ectropion

. Endoscopic medial orbital decompression


Highlights and pitfalls:
• . A transnasal endoscopic approach allows for removal of the inferomedial
orbital wall
• The middle turbinates are medialized using a 4.0 Vicryl suture
• Exposure to the lamina_papyracea is obtained after a middle meatus antros-
tomy, total ethmoidectomy, and sphenoidotomy are performed
• The lamina papyracea is removed using a Cottle elevator, ball-tipped probe,
and through-cutting instruments. The periorbita is left intact until both the
lamina and inferomedial orbital floor are removed. The orbital floor can be
removed medially.to the level of the infraorbital nerve
• Once the bony elements are completely removed, the peri orb ita is incised in
. several areas with a sickle knife, beginning posteriorly and coursing anteri-
orly in longitudinal fashion along the axis of the orbit
• Packing is generally avoided
Postoperative antibiotics and pain medicines are used for about 1 week
The patient follows up in the office about 1 week postop

What are some complications of endoscopic orbital decompression?

State that complications include those routinely associated with endoscopic si-·
nus surgery, including, but not limited to:
• Diplopia
• Vision loss
• Blindness
• CSF leak
• Meningitis
• Stroke
• Intracranial injury
• Persistent proptosis
• Cosmetic deformity
• Complications often occur as a result of disorientation during the operation.

341
Otolaryngology Clinical Case Studies Chapter 6- Rhinology

NOTES Adequate preoperative vasoconstriction and atraumatic techniques provide


an optimum surgical field, so that there are no "blind moves" perfom1ed
• Orbital complications are best avoided by obtaining wide exposure of the
orbital floor and medial orbital wall. A wide middle meatal antrostomy can
provide a good view of the level of the orbital floor, giving the surgeon a
reference point
• Visualizing and understanding the anatomy o~the skull base can help avoid
intracranial compl ications. Some pearls include:
- Avoid aggressive manipulation of tissues superomedial to the middle
turbinate
- The anterior face of the sphenoid sinus is about 7 em from the nasal
openmg
- A "safe zone" to enter the posterior ethmoids is through the basal lamella
of the middle turbinate at the level of the roof of the maxillary sinus
(often seen through a wide midd le meatal antrostomy)
- Using through-cutting instruments to palpate and remove the bony lamella
near the skull base offers the surgeon tactile feedback about the bony
anatomy. While often usefu I for efficient removal of soft tissue in other
areas, .use of powered instrumentation near the skull base is generally
discouraged

How would you follow this individual?

• The patient is seen in the clinic about 1 week postop where nasal hygiene
can be initiated, specifically nasal saline spray/irrigation
• . Although packing is generally avoided, any packing that may have been placed
is removed at this time.
• A second visit about 6-8 weeks following surgery is scheduled to see how
vision and proptosis have stabilized after surgery
Bi-annual or annual visits are then scheduled as indicated

Follow-up with the ophthalmologist is critical to ensure that any remaining


strabismus is corrected either with prisms or with surgical intervention.

Eyelid surgerylblepharoplasty can be performed once the orbital disease


has stabilized and no further decompressions are planned. This is prima-
rily cosmetic, but the psychological impact may be significant for patients
with severe deformities. Blepharoplasty may be performed independently
or in conjunction with strabismus surgery.

References:

1. Kacker A, et al. "Balanced" orbital decompression for severe Graves'


orbitopathy: technique with treatment algorithm. Otolaryngol Head Neck Surg.
2003; 128(2):228-35.
2. Tenis DJ, Levin P. Orbital decompression for Graves' disease. eMedicine
2003 ;http://www.emedicine .comlent/topic 169 .htm
3. Shepard K~ et al. Balanced orbital decompression for Graves' ophthalm-
opathy. Laryngoscope. 1998; 108:1648-53.

342
Chapter 6- Rhinology Otolaryngology Clinical Case Studies

CA.SE 16 - NASAL DERMOID SJNus CvsTs NoTES


Bryan Tigner, lVID

A 6 year-old white male accompanied by parent presents with complaint of


a lump on his nasal dorsum.

HPJ: state that you would:

• Obtain a detailed medical history beginning with a history of the present illness

What additional historical information would you seek? State that you would ask:

• How long has the lump been present?


• Has it ever drained?
• Has it ever been infected and, if so, how often?
• Is there associated nasal airway obstruction?
• Has he had any significant nasal trauma?
• Does he have any other congenital anomalies?
• Has hair grown from the lesion?
• Has the mass increased in size?

His parents state that the child has had a small mass on his nasal dorsum
since infancy. The mass is slightly larger than it was a few years ago. His·
parents aiso state that the child complains -of nasal congestion. A few small
hairs have developed from the mass. His parents deny any history of nasal
trauma and the child has never had meningitis.

PMH: No chronic medical conditions. No oiher congenital


anomalies
Allergies: NKDA
Medications: None
Family Hx: . Negative
Social Hx: He lives with both parents and·younger sibling
ROS: NC

What would you look for on PE? State that you would:

• Perform a complete head and neck exam including vital signs and rigid nasal
rhinoscopy

PE:

YllilLs_: ~ Temp: 98.6 F, Pulse: 104, BP: 100170


GA: Alert, cooperative, NAD
HEENT Nasal exam reveals a punctate lesion on the midline of the
nasal dorsum, proximal to the nasal tip. A thin, milky dis-
charge is noted from the lesion. A few, sparse hairs are
growing from the lesion. The nasal dorsum is widened. Rhi-
noscopy reveals healthy, pink mucosa, and no evidence of
nasal polyps. The septum appears slightly deviated
Neuro: Gait station is normal. CN 11- XIl intact

343
Otolaryngology Clinical Case Studies Chapter 6 - Rhinology

NOTES What is your differential diagnosis?

• A congenital dermoid cyst, glioma, encephalocele, meningocele (sinus) or


hemangioma
• An acquired schwannoma, neurofibroma, fibroma, osteoma. chondroma,
adenoma, carcinoma, lipoma, sebaceous cyst, or mucocele
• Gliomas, encephaloceles, and nasal dermoid sinus cysts are the most com-
mon congenital midline nasal masses. Skin dimpling or a pit is characteristic
of nasal dermoids and usually not present with encephaloceles or gliomas.
Encephalocele may have a positive Furstenberg test, i.e., e!'Jiarge with com-
pression of the jugular veins. Unlike an encephalocele, a glioma does not
have continuity with the CSF

What diagnostic studies would you request? State that you would request:

• MRJ - An MRJ can more clearly define intracranial extension/herniation


and associated anomalies. Therefore, MRJ is study of choice vs. CT scan. A
wide, bifid septum and crista galli with enlarged foramen caecum suggest
intracranial extension
- Gadolinium enhanced, fat-suppressed T,-weighted images should be or-
dered. Dermoids will not enhance. This is important in differentiating
between enhancing lesions such as hemangiomas
• A CTscan can provide information about the bony detail of the nose and
anterior cranial base around the lesion. A CT scan should be considered if
the MRJ suggests abnormal bony anatomy. ACT scan, if perfonned, should
include axial and coronal images with contrast extending from tip of the nose
through the anterior skull base. Widened foramen cecum, bifid crista gall i,
and ipsilateral osseus defect in the cribriform plate are bony skull base findings

Test results:

The MRI shows a cyst deep to the nasal bones. Th,e cyst demonstrated a
light sign.al on T1 weighted images. No intracranial extension was noted.

Diagnosis:

The diagnosis is nasal dermoid cyst. Diagnosis is made from the history of
a congenital lesion that is occasionally draining or infected, and P E find-
ings of a dimpling defect with hair follicles on nasal dorsum from a pit,
nasoglabellar mass, tip fullness, dorsal widening, bifid tip, or lateral or-
bital displacement.

Review of MRI!CT findings are critical to assess for intracranial involve-


ment. 1f intracranial spread is present, coordination between appropriate
surgical teams is necessary.

344
Chapter 6- Rhinology Otolaryngology Clinical Case Studies

What treatment would you suggest'! NoTES

• The usual treatment for nasal dem1oids is surgical excision


• Pollock outlines 4 requirements for successful surgical excision: first, the
approach should allow access to al l midline cysts as well as permit lateral
and medial osteotomies if needed; second, the surgical exposure should al-
low repair of cribriform defects or CSF rhinon·hea should it develop; thirdly,
the approach should facilitate the repair of the nasal dorsum if necessary;
finally, the approach shou ld offer an acceptable cosmetic outcome
• The open rhinoplasty approach meets these 4 criteria and its use is well
documented in the literature. The bifrontal craniotomy provides adequate
exposure of the cribrifom1 plate. A verticomedian incision does not meet
above criteria and thus will not be discussed here
• Complete excision of the na~al dermoid cyst and sinus, if present, is para-
mount in preventing progressive expansion, infection and possible fistula for-
mation. Sessions advocates intraoperative biopsy of stalk on the nasal side of
cranial base. If no dermal component is noted, a craniotomy can be avoided

. Surgical technique:

• External (open) rhinoplasty approach:


A transcolumellar incision is performed
- A standard marginal incision is made and connected to the transcolumellar .
IDCISIOI1
- Then, divide the medical crural attachments from the septum and extend
the dissection posteriorily. This step, combined with elevation of the skin-
soft tissue envelope, allows wide exposure
- The dissection can be carried out to excise the cyst and/or fistula tract
all the way to the skull base. Here, median osteotomy of the nasal bones
is perfom1ed if necessary. The nasal bones are fractured and separated
vertically over the dorsum of the nose at the nasofrontal sutur<e

What are the potential complications'!

• Perioperative infection, including meningitis


• Nasal deformity & nasal obstruction
• CSF leak
• lntracranial injury

.
How would you follow this individual?

• You would see this patient every few weeks post-op and then every 6 months
for a year or 2. Recurrence rates are generally low
• Follow-up MRI at about I year

345
Oto~aryngology Clinical Case Studies Chapter 6 - Rhinology

NOTES References:

I. Denoyelle F, Ducroz V, Roger G, Garabedian EN. Nasal dem1oid sinus cysts


in children. Laryngoscope. 1997;1 07:795-800.
2. Loke DKT, Woolford J. Open septorhinoplasty approach for the excision of
a dermoid cyst and sinus with primary dorsal reconstruction. J Laryngol
Otol. 2001;1 J5:657-9.
3. Pollock RA. Surgical approaches to the nasal dermoid cyst. Ann Plast Surg.
1983; I0:498-50 I.
4. Rohrich RJ, Lowe JB, Schwartz MR. The role of open rhinoplasty in the
management of nasal dermoid cyst. Plast Reconstr Surg. 1999; 104: 1459-
67.

346
Chapter 7- Cosmetic Facial Surgery Otolaryngology Clinical Case Studies

CHAPTER 7/CASE 1 -RHINOPHYMA NoTES


Allison Pontius, MD

A 62 year-old Caucasian male presents with a hist01:v of progressive nasal


enlargement with intermittent nasal swelling and drainage of foul-smelling
material.

HPJ: state that you would:

• Obtain a detailed medical history beginning with a history of the present illness

What additional historical information would you seek? State that you would ask:

• Duration of the symptoms?


• Is this associated with pain?
• History of sun exposure?
-• Previous history of cutaneous neoplasms?
• History of dennatologic diseases?
• Use of facial cosmetic products?

He states that this condition has progressed slowly over the course of the
last 2 years. While the changes of his external nose have not been cornjort- .
able, they have not been particularly painful. He states that he has had
moderate exposure to sunlight, but is careful to wear sun protection for his
rosacea. He has had no other dermatologic conditions and denies a previ-
ous history of cancer. Aside from sunscreen, he wears no cosmetic products.

PMH: HTN, hyperlipidemia, rosacea


Allergies: Penicillin
Medications: Captopril, Lipitor, Retin-A
FH: COR artery disease
SH: Remote history of smoking 1 ppd/20 years, quit 15 years
ago, occasional alcohol use (2 drinks/week), retired from
military service
ROS: Negative

What would you look for on PE? State that you would:

• Perfonn a complete head and neck PE

PE:

A complete head and neck PE is performed. Nasal exam reveals tuberous


enlargement of the lower half of the nose. The skin is irregularly thick-
ened, and the follicles are prominent with foul-smelling inspissated sebum.
Telangiectasias are present on the nasal dorsum and the medial cheeks.
Intranasal endoscopy reveals normal mucosal lining with no evidence of
inflammation or infection. There is partial collapse of the external nasal
valves secondary to the weight of the overlying hypertrophic tissues.

347
Otolaryngology Clinical Case Studies Chapter 7- Cosmetic Facial Surgery

NOTES What is your differential diagnosis?

• Rhinophyma
• Rosacea
• Neoplasm (including basal cell carcinoma, SCC, etc.)
• Cellulitis

Diagnosis:

This is a classic presentation of rhinophyma, especially in light of the


patient :s- past history of rosacea.

Rhinophyma is the last stage of rosacea, predominantly affecting male patients


in the 51" - 7th decades of Iife. The majority of patients with rosacea do not
progress to rhinophyma; however, the patients that do progress develop a thick,
bulbous overgrowth of sebaceous glands and hyperplastic connective tissue.
The nose becomes enlarged and hypervascular, causing severe disfigurement
and occasionally nasal airway obstruction.

Historically, rhinophyma has been associated with alcoholism; however, this


association is unfounded. Several inciting factors for rhinophyma have been
proposed including vitamin deficiencies, infection, invasion by the follicular mite
Demadex folliculorum, stress, and androgenic hormones. The only confirmed
etiological association is with rosacea.

The progression of the disease follows a predictable course. Early in the dis-
ease process, there is an increased size and number of sebaceous glands with
hypervascularity. Later, there is duct dilatation and cyst formation with eventual
fibrous tissue proliferation, giving the appearance .of nodularity.

Spontaneous regression of rhinophyma is rare. Medical therapy is usually only


undertaken in conjunction with surgical resection: Oral antibiotics (metronida-
zole, tetracycline, minocycline, doxycycline, erythromycin1 ampicillin) combined
with either topical metron'idazole or sulfacetamide sodium may be used 2-3
weeks prior to surgery to clear any focal infection. Additionally, abstaining from
alcohol, avoiding stress, pretreatment with steroids, and isotretinoin have been
advocated to reduce erythema and the volume of sebaceous tissue. Surgical
management is the accepted treatment for established rhinophyma and offers
the only chance for a cure.

.
What do you do next?

• The patient should be informed of the diagnosis, etiology, natural history of


the disease process and treatment options
• Advise the patient to pursue surgical management because of the cosmetic
disfigurement, nasal airway obstruction and potential for occult carcinoma

The coincidence of basal cell and less frequently SCC is well established; how-
ever, the true incidence is variable. Most previous studies are small and lack

348
Chapter 7- Cosmetic Facial Surgery Otolaryngology Clinical Case Studies

statistical significance. The rep011ed incidence in the literature of simultaneous NOTES


carcinoma in the setting of rhinophyma varies from 3-30%. Basal cell carci-
noma is the most common malignancy found in the setting of rhinophyma.

What are the different surgical techniques?

• Cryosurgery- Sebaceous glands are readily destroyed by low temperature,


which fom1s the theoretical basis for cryotherapy. This procedure is usually
performed by dematologists and involves the use of a liquid nitrogen spray
gun, with the rhinophyma being treated with 2 freeze-thaw cycles. This tech-
nique has been shown to reduce the bulk of rhinophyma. Resistant areas are
retreated in a similar fashion . Advantages of this technique include its ease
of use, minimal discomfort, hemostasis, and low cost. Disadvantages include
its imprecise nature, inability to control the depth of removal, inability to con-
tour, and hypopigmentation
• Dem1abrasion- Dem1abrasion is usually used as an adjuvant to other rneth-
ods. lts main use is for contouring of the nasal profile. Advantages include low
cost and its contouring ability. Disadvantages include imprecise depth oftissue
. destruction secondary to the bleeding that occurs with this technique. Hemo-
stasis may require the additional use of electrocautery, local anesthesia with
epinephrine, epinephrine soaks, manual pressure, or human fibrin sealant
• Shaw knife -The Shaw knife allows the precision of the conventional scal-
pel with improved hemostasis. Its disadvantages include coagulation ofthe
adjacent epidermis ifleft in contact with the tissue too long and the cumber-
someness of the cord
• Harmonic scalpel -The harmonic scalpel provides a sharp surgical blade
with longitudinal ultrasonic vibration of 50,000 Hz that allows excision of
rhinophymatous tissue with simultaneous hemostasis with a smaller zone of
thermal injury than either the C02 laser or electrocautery. Its disadvantages
include its high cost
• Electrocautery-This technique employs the bipolar electrosurgical unit with
the loop attachment. The cutting mode is set at maximum power and tissQe is
removed in thin layers. Its advantages include less bleeding; however, it has
a high potential for damage to the underlying cartilage, it is imprecise, and the
tissues are slow tore-epithelialize
• Scalpel excision -Cold knife excision allows precise removal oftissue with
preservation ofthe sebaceous gland fundi which pemits spontaneous re-epi-
thelialization. Its advantages include excellent wound healing without adjacent
tissue injury; however, obtaining hemostasis may require adjuvant techniques
• C02 laser- The common technique of C02 laser use involves resection of
the excess tissue with the cutting spot size of 0.2 mm followed by defocusing"
of the laser beam; sculpting and hemostasis is achieved. The depth of vapor-
ization is 0. 1 mm. lts disadvantages include thermal injury to the surrounding
tissue, time, expense, bleeding and operator training
• Argon laser- The basis of the argon laser is that it provides selective coagu-
lation of capillaries secondary to its absorption of the blue-green 488-5 14.5
nm spectrum of oxyhemoglobin. It also diverts coagulation shrinkage of the
hypertrophic connective tissue. Its advantages include no bleeding, mini-
mal patient discomfort, minimal trauma to the underlying tissue and ease of

349
Otolaryngology Clinical Case Studies Chapter 7- Cosmetic Facial Surgery

NoTES operation. Its disadvantages include cost, the need for more than one treat-·
ment, and poor effectiveness for large, deep lesions
• Week blade and argon beam coagulator - This procedure involves Week
blade contouring of the nose after extensive infiltration of local anesthesia.
Following excision, the argon beam coagulator is employed to control bleed-
ing. Its advantages are precision of excision, short operative time, favorab le
wound healing, and control of hemostasis with the addition of the argon beam
coagulator. Disadvantages include the potential for adjacent tissue damage
from the argon beam coagu Iator

Prior to surgical therapy, the patient should be counseled to avoid sun


exposure, apply daily sunscreen with UVA and UVB protection, and to
begin topical treatment with metronidazole cream and oral treatment with
metronidazole or tetracycline at least 3 weeks prior to surgery. If he had
been placed on oral isotretinoin for treatment of rosacea, surgical treat-
ment should be delayed for at least 1 year because of its propensity to
impair re-epithelialization. Topical tretinoin is not a contraindication to
surge1y; however, it should be stopped at least 3- 4 weeks pre-operatively.
Routine laboratory studies, a CXR and an ECG should be obtained.

Surgical technique:

• The surgical procedure begins with the patient placed in a supine position .
and initiation of general endotracheal anesthesia
• Once asleep, pledgers soaked in 4% cocaine are inserted into each nasal
cavity. A nerve block of the infraorbital nerves and the nasociliary nerve is
performed with 0.5% xylocaine and 1:200,000 epinephrine. Additionally, an-
other 10- 15 cc of local ane.sthetic is injected into the rhinophyma. Next, the
patient is prepped and draped in the usual sterile fashion and 10 minutes are
allowed to elapse for the effect of the local anesthetic to take place .
• Loupe magnification is utilized to visualize and preserve the sebaceous gland
fundi to ensure that proper re-epithelialization occurs
• The scalpel is used to tangentially ex,cise the diseased tissue (partial-thick-
ness excision) in a superior to inferior direction. The surgeon·'s finger is placed
into the nasal cavity to provicle tactile perception ofthe thickness of the soft
tissue and to assist in sculpting of the alae and nasal tip
• After major debulking, fine contouring is also performed with the scalpel.
This is followed by dem1abrasion for final contouring and feathering the
edges near the transition to the patient's normal skin
• Hemostasis is obtained with firm, direct pressure. Cautery should be used
very judiciously, as this can create thennal damage to the surrounding tissues.
After hemostasis is obtained the nose is coated with i'nupirocin ointment and
covered with a layer of Xerofom1 gauze ·
• The specimen is sent to the pathologist for permanent diagnosis

What are the treatment options and their complications?

• Scar contracture - Particular attention must be paid to the depth of the


rhinophymatous tissue. Dissection that is too deep leads to scar contracture,
especially in the alar region. If scar contracture occurs, composite graft

350
Chapter 7 - Cosmetic Facial Surgery Otolaryngology Clinical Case Studies

reconstruction should be unde1taken as soon as the problem is identified NOTES


• Hypertrophic scaning- This may occur on the nasal dorsum. If hyper-
trophic scaning develops, weekly Kenalog injections shou ld be undertaken
untiI the scar softens
• Thermal damage- If electrocautery or the CO~ laser is utilized, either pri-
marily or for hemostasis, there is a risk of adjacent tissue and carti lage ther-
mal damage

How would you follow this individual?

• Antibiotics - Postoperatively the patient is given a 5-day prescription for


clindamycin 300 mg q id (cephalexin 500 mg qid in the non-allergic patient)
• Dressing changes - The patient should pe1fonn twice- daily dressing changes
by re-applying a layer of mupirocin ointment followed by a layer ofXeroform
and finally a layer of 4 x 4 gauze. After 5 days the patient should be seen in
the clinic for evaluation. The oral and topical antibiotic treatment is stopped
at this time and daily dressing changes are continued with Xeroform gauze
until re-epithelialization is complete (2-3 weeks)
• · Review pathology report- The final pathology should be available by the
patient's first post-operative appointment and this should be reviewed care-
fully with the patient. If a carcinoma was found within the specimen further
treatment and close follow-up is mandatory. If the margins of the carcinoma
are positive, re-excision should take place as soon as possible or the patient
should be referred to a Moh 's surgeon for further treatment
• Fo!low closely during re-epithelialization process. The patient should be seen
on a weekly basis until re-epithelialization is complete. Following re-epithe-
lialization the patient should apply daily sunscreen to the nose (and entir_e
face) and be evaluated periodically for signs of disease recurrence

References:

1. Redett RJ, Manson PN, Goldberg N, Girotto J, Spence RJ ..Methods and


results of rhinophyma treatment. Plast Reconstr Surg. 200 l; 107: 1115-23.
2. Hoasjoe DK, Stucker FJ. Rhinophyma: review of pathophysiology and treat-
ment. J Otolaryngol. 1995;24:51-6.
3. Plenk HP. Rhinophyma, associated with carcinoma, treated successfully with
radiation. Plast Reconstr Surg. 1995;95:3 :559-62.
4. Dufresne RG, Zienowicz RJ, Rozelle A, Whalen JD. An introduction of the
ultrasonic scalpel: utility in the treatment of rhinophyma. Plast Reconstr Surg.
1996;98: I: 16{}-2.
5. Rex J, Ribera M, Bielsa l, Paradelo C, Ferrandiz C. Surgical management of •
rhinophyma: report of eight patients treated with electrosection. Dermatol
Surg. 2002;28:34 7-9.
6. Acker DW, Helwig EB. Rh inophyma with carcinoma. Arch Dermatol.
1967;95:250-4.
7. Wilson PD. Lymphocytic lymphoma: case reports. Br J Dem1atol.l982; I 07:
45-6.
8. Broadbent NR, Cort OF. Squamous carcinoma in longstanding rhinophyma.
Br J Plast Surg. 1977;30:308- 9.

351
Otolaryngology Clinical Case Studies Chapter 7- Cosmetic Facial Surgery

NoTEs 9. Rees TD. Basal cel l carcinoma in association with rhinophyma. Plast Reconstr
Surg. 1955;16:282-7.
I 0. Sonnex TS, Dawber RPR. Rhinophyma treatment by liquid nitrogen spray
cryosurgery. C1in Exp Dermatol. 1986; 11:284-8.
11. Elliot RA Jr, Hoehn JG, Stayman JW Ill. Rhinophyma: surgical refinements.
Ann Plast Surg. 1978; 1:298-3 01.
12. Riefkohl R, Georgiade GS, Barwick WJ, et al. Rhinophyma: A thirty-five
year experience. Aesthetic Plast Surg. 1983;7:131 -4.
13. Eisen RF, Katz AE, Bohigian RK, Grande OJ. Surgical treatment of rhino-
phyma with the Shaw scalpel. Arch Dermatol. 1986; 122:3Q7- 30.
14. Odou BL, Odou ER. Rhinophyma. Am J Surg. 1961 ;1 02:3-16.
15. Rohrich RJ, Griffin JR, Adams WP Jr. Rhinophyma: review and update.
Plast Reconstr Surg. 2002;1 10: 860-9.
16. Baker TJ, Stuzin JM, Baker TM. TCA P~el s . In: Baker TJ, Stuzin JM,
BakerTM, eds. Facial skin resurfacing. St. Louis: Quality Medical Publish-
ing; 1998, 85.
17. Stucker FJ, Hoasjoe OK. Rhinophyma surgical techniques. In: Bailey BJ,
Calhoun KH, Coffey AR, Neely JG, eds. Atlas of head and neck surgery-
. otolaryngology. Phi !adelphia: Lippincott William,s & Wilkins; 1996, 476.

352
Chapter 7- Cosmetic Facial Surgery Otolaryngology Clinical Case Studies

CASE 2- FACIAL REANIMATION NOTES


Allison Pontius, MD

A 55 year-old male presents to you with a history of a blunt head injury


sustained during a motor vehicle accident approximately 2 years ago.
He has had a complete right-sided facial nerve paralysis since that time
with minimal to no improvement over the enszdng years. He has not sought
surgical correction until this time: however he has been diligent with eye
care and uses artificial tears and lubricates and tapes his eye shut at
night.

HPI:

PMH: HTN, COR artery disease


Allergies: NKDA
Medications: Lisinopril, aspirin
FH: COR artery disease
SH: Smokes 1 ppd, no alcohol use, employed as a security guard
ROS: Negative

What would you ·look for on PE? State that you would:

• Perform a fu ll head and neck examination with special attention to CN

PE:

Pupils are equally round and reactive to light, extraocular muscles are
intact bilaterally, 5 mm scleral show on the right side, 1 mm on the left side,
incomplete eye closure on the right without evidence of exposure keratitis
or corneal ulceration
EAC and TM are clear on the left. On the right a "step off" consistent with
· an old temporal bone fracture is noted, but the TM is normal
Nasal mucosa .z's clear bilaterally with right-sided valve collapse
Midface skin and soft tissue lax.ity
OC and OP are clear
Neck exam is negative for masses or lymphadenopathy
CN II-XII are intact except for the right VI/VI facial nerve paralysis and
subjective complaints of right-sided hearing loss

What is your differential diagnosis?


~

• Right facial nerve paralysis resulting from his history of blunt bead trauma
(likely secondary to a temporal bone fracture)
• Right facial nerve paralysis secondary to another cause-neurologic, infec-
tion, metabolic, neoplastic, toxic, iatrogenic or idiopathic

353
Otolaryngology Clinical Case Studies Chapter 7 -Cosmetic Facial Surgery

NoTES What diagnostics would you request? State that you would:

• Perfom1 electrodiagnostic studies to further elucidate the cause and topographical


location of the injury in order to plan the optimal choice for reanimation
• Audiologic evaluation should be performed as the patient has a suspected
temporal bone fracture
• Imaging:
- CT scan - A temporal bone CT scan should be performed to assess for
a temporal bone fracture. Additional ly, the extra-temporal facia l nerve
can be visualized with a fine-cut CT scan
- MRJ- Gadolinium-enhanced MRl can detect inflammation and edema
of the facial nerve and can differentiate facial nerve tumors or heman-
giomas from Bell's palsy
• Electrodiagnostic testing:
- It evaluates the degree of injury to the facial nerve and the integrity of
the facial musculature

What are the most commonly used electrodiagnostic tests?

• The ma,"Ximum stimulation test, nerve excitability test, ENOG and EMG
- Maximum stimulation test
•. Not useful during the first 72 hours (3 days) after injury
because even with complete proximal transection, the distal
branches will continue to conduct for up to 3 days
1
Performed with the Hilger nerve stimulator
I
Relies on subjective observation oft)1e facial musculature in
response to electrical stimulation. The response on the paretic
side is compared with the normal side
D
This test should only be performed in patients with a complete
Hause-Brackman VINI facial nerve paralysis because the test
is uncomfortable and patients with incomplete paralysis have a
favorable prognosis

- Nerve excitability test:


• Measures the current (in amperage) required to obtain a minimal
visible response at a standard square-wave pulse duration
• Not useful during the first 3 days after injury
a Performed with the Hilger nerve stimulator to measure the
difference in the amperage required to produce a barely visible
twitch on the paretic side compared with that required on the
normal side
• A side-to-side difference of greater than 3.5 rnA indicates a poor
prognosis

- EMG:
• Electrodes are inserted into muscle and the muscle response to
voluntary contraction is measured
• Can determine the existence of functional motor units
• Normal resting muscle exhibits no spontaneous electrical activity
354
Chapter 7 - Cosmetic Facial Surgery Otolaryngology Clinical Case Studies

With denervation. spontaneous fibrillation potentials and positive NoTES


sharp waves are present. It may take 2-3 weeks for fibrillation
potentials to be detected by EMG
• Polyphasic action potentials indicate reinnervation
• Presence of voluntary action potentials indicates at least partial
continuity of the nerve

- ENOG:
• Records muscle response via electrodes after stimulation of the
facia l nerve with a transcutaneous impulse at the stylomastoid
foramen
" Objectively compares muscle compound action potential
ampl itudes (related to intact motor axons) and latencies from the
paralyzed and norrr.al sides
.. Not useful until 72 hours after injury
Commonly used to stratify patients into surgical decompression
versus observation following nerve injury. A 90% degeneration of
the nerve is considered to be the threshold for consideration for
surgical decompression

Diagnosis:

The patient has suffered a complete right-sided facial nerve paralysis re-
·sulting fran: a temporal bo1:7e fracture with disruption suspected at the
geniculate ganglion. Because the injury occurred 2 years ago and the
patient has suffered moderate muscular atrophy, decompression is not likely
to provide significant improvement. At this point the patient will likely ben-
efit the most from a dynamic procedure to reanimate the mouth (temporalis
muscle transposition) and a gold weight implant with an eyelid shortening
procedure to help correct his lagophthalmos and lower lid laxity.

What are the treatment options?

• Primary neuropathy:
- Provides the best chance of rehabilitation (facial muscle movement with
the least synkinesis)
- Optimal timing is immediately following the injury
- Can be performed extratemporally, intratemporally, or intracranially
- Performed via perineural (fascicular) or epineural repair
• Interposition grafting:
- f',4ethod of choice if a tension-free end-to-end anastomosis can not be
performed and motor end plates are intact
- Performed with a nerve cable graft interposed between nerve endings
(great auricular nerve or sural nerve most commonly used)
- Provides resting muscle tone and spontaneous facial expression
• Cross-face grafting:
- This technique has fallen out of favor except as part of a staged free
muscle transposition
- 30-50% of the buccal and zygomatic branches on the nonparalyzed side

J55
Otolaryngology Clinical Case Studies Chapter 7 - Cosmetic Facial Surgery

NoTES are severed and anastomosed to the con·esponding branches on the para- ·
lyzed side
- Theoretically this produces minimal deformity on the normal side wh ile
restoring symmetry and some mimetic function on the paralyzed side:
however, disappointing results have been found
- Can be performed in I or 2 stages
• Nerve crossover:
- Includes XU-VII. Xli-VII jump graft and rarely XI-VII crossover
- Useful when the proximal facial nerve is unavailable but the distal nerve
remains anatomically intact
- Advantages: relatively low degree of technical difficu lty, short time to
movement (4-6 months), one anastomotic suture line and motion that
can resemble mimetic function with practice
- Disadvantages: donor site morbidity, synkinesis, and ipsilateral paralysis
of the tongue musculature (in XII-VII crossover)
- Must be performed prior to atrophy of the facial musculature and distal
facial nerve (within 1 year)
• Dynamic procedures:
- Indicated when the facial nerve is unav~ilable or with atrophic facial
muscles
- The temporalis muscle or masseter muscle are utilized to reanimate the
mouth (with a. gold weight to reanimate the eye)
- The major objectives of lower facial reanimation are to achieve symme-
try at rest, oral sphincteric competence, and facial movement
- Problems with the technique include a bulge over the zygomatic arch,
gradual stretching of the muscle and loss of symmetry at the mouth
• Static procedures: .
- May be used as an adjunctive procedure to enhance facial symmetry by
providing static support
- For lower facial rehabilitation fascia lata; Gore-Tex, or AlloDerm may
be utilized
• Microvascular free tissue transfer:
- Restores some emotional animation in addition to good tone in repose
and some voluntary facial movement
- May be a 1 or 2 stage procedure. The 2 stage procedure uses a cross-
face nerve graft in the first stage and anastomosis to a free-muscle
transfer in the second stage. The single-stage procedure uses a long
nerve pedicle that is grafted to contralateral buccal branches at the time
of the free-muscle transfer
• Adjunctive procedures:
- Browlift
- Facelift
- Functional rhinoplasty to correct nasal valve collapse
- Digastric transposition to correct elevation of the lower lip from loss of
the depressor anguli oris and the depressor labii inferioris
- Cheiloplasty
- Contralateral selective neurectomy
- Selective myectomy
- Botox to the contralateral face

356
Chapter 7- Cosmetic Facial Surgery Otolaryngology Clinical Case Studies

Surgical technique: NoTES

• The method of"dual reanimation'' (addressing the eye and the mouth with
independent surgical procedures) provides the best functional and cosmetic
results. Before any reanimation is performed the patient must undergo a
complete eye exam with assessment of visual acuiry, assessment of the pres-
ence or absence of a Bell's phenomenon, assessment ofte<.ti production with
a Schirmer 's test, evaluation of the lacrimal excretory system with a Jones
test, and measurement of the margin gap (distance between the upper and
lower eyelids on closure)

H.ighlights:
• Lower eyelid procedures:
- Assess the amount of lid laxity with the snap test
- Assess the need for medial lower lid laxity by noting if the punctum is
everted and there is pooling of tears at the inferior lid margin
- Correction of medial lower lid laxity can be performed with either a
Gore-Tex sling procedure or a medial canthoplasty
• Medial canthoplasty:
- Performed under local anesthesia with IV sedation
- The cornea is anesthetized with tetracaine, lubricated, and a corneal
protector is applied
- Mark the incision prior to the injection of the local anesthetic·
- Raise medial superior and inferior flap~ and dissect the tarsal compo-
nents of the superior and inferior canthal tendons
- The lateral portion of the inferior canthal tendon is plicated to the medial
portion of the superior canthal tendon
Excise excess skin prior to closure
• Tarsal strip procedure:
Perform a lateral canthotomy and inferior cantholysis, preserving inser-
tions supplied from the upper lid .
- Separate the anterior lamella skin and muscle from the posterior lamella
Remove the overlying skin and conjunctiva .
- Trim the lateral canthal tendon and suspend it from the periosteum of the
medial surface of the lateral orbital rim superior to Whitnall 's tubercle
• Upper eyelid procedures (gold weight):
- Inject a small amount of local anesthesia into the supratarsal fold and
over the tarsal plate
- Make the incision slightly longer than the implant
- Center the incision over the junction of the medial and middle thirds of
theuppereyel id •
Elevate the skin to the supenor edge of the tarsal plate
Sharply incise the orbicularis and create a pocket over the tarsal plate to
within 2 mm of the lash line
- Place the weight in the pocket so the 2 fenestrations face cephalad
- Suture the implant to the levator aponeurosis over the tarsal plate through
the fenestrations
• Temporal is transposition to the oral commissure:
- The incision extends from the helical crus in a superoposterior direction
to the leve l of the superior temporal line
357
Otolaryngology Clinical Case Studies Chapter 7 - Cosmetic Facial Surgery

NOTES - The incision is carried down to the temporalis fascia


- Divide this layer to expose the deep temporal is fascia immediately over-
lying the muscle
- Using needlepoint cautery, outline the middle third of the muscle from
the temporal line down to the zygomatic arch . Elevate this in a subperi-
osteal plane
- Create a pocket superficial to the SMAS to receive the flap and can-y
this tunnel down to the oral commissure
- Now make an incision at the vermillion border extending to the orbicu-
laris oris muscle
- This incision is now connected with the preauricular incision in the supra-
SMAS plane
- The tunnel should be able to accommodate two fingers so as to be wide
enough to receive the muscle
- The distal end of the temporal is muscle is bisected for a distance of2 em
and figure-of-8 sutures are placed at each end
- The sutures are passed through the tunnel which pulls the temporal is
muscle through to reach the incision at the oral commissure
- The temporal is muscle is sutured to the orbicularis oris or the submucosa
at the oral commissure
- There should be overcorrection, as this resolves in 3-6 months
The defect left by removal of the central temporalis muscle may be filled
with a temporoparietal fascial flap, hydroxyapatite bone cement or a
Silastic implant
- The incision is closed over a suction drain

What are the potential complications/pitfalls?

• Medial canthoplasty:
- Lid malposition
- Extrusion of foreign body (in Gore-Tex sling procedure)
• Tarsal strip:
- Lid malposition
- Entropion
• Gold weight:
- Extrusion
- May not function properly when the patient is supine
- Visib le bump on the upper lid secondary to the profile of the weight
• Temporalis muscle transposition:
- Fullness over the zygomatic arch
- Gradual stretching of the muscle and loss of symmetry at the mouth
- Problems with oral competence
- Temporal fossa defect
- Extrusion/infection of any fore ign materials used in correction of the
temporalis defect
- Need for a revision procedure to tighten the muscle if stretching occurs

358
Chapter 7- Cosmetic Facial Surgery Otolaryngology Clinical 'Case Studies

How would you follow this individual? NOTES

• Immediately post-operatively the patient should be seen at I week to ensure


proper healing and eye protection
• The patient should then be followed periodically after healing is completed
and a good result is ensured
• It is possible the patient may need to continue using eye lubrication if the gold
weight does not provide sufficient eye coverage when supine, or a heavier
weight should be considered
• Additionally any ocular complaints require re-evaluation by an ophthalmologist
• The patient must have long tem1 fol low-up to ensure the eye· remains well
protected and the mouth remains symmetric
• Revision procedures may be necessary

References:

1. Clark JM, Shockley WW. Management and reanimation of the paralyzed


face. In: Papal ID, editor. Facial plastic and reconstructive surgery. New
York: Thieme; 2002,660-85.
2. Pasha R. Facial nerve repair and reanimation. Otolaryngology-head and
neck surgery: clinical reference guide. San Diego: Singular-Thompson Learn-
ing; 2001 ,369- 7 1.

359
Otolaryngology Clinical Case Studies Chapter 7 - Cosmetic Facial Surgery

NoTES CASE 3- CoMPLEX LACERATION INVOLVING THE EYELID AND LACRJMAL


SYSTEM
Allison Pontius, MD

A 33 year-old Hispanic female presents to the emergency department with


a laceration of her eyelid

HPI: state that you would:

• Obtain a detailed medical history beginning with a history .of the present
illness

What additional historical information would you seek? State that you would ask:

What was the mechanism of injury?


• Speed of car within which she was traveling?
• Was there a loss of consciousness?
• Questions to ascertain potential for other significant traumatic injury
Have there been any changes in vision?
• Is there any ocular pain?
• Does the patient experience any diplopia on any field of gaze?

The patient states that she was involved in a motor vehicle accident at ·
which time she received the laceration of her lid. She was the unrestrained
passenger in a car traveling 40 mph. Her car collided with another ve-
hicle from behind. She does not report a loss· of consciousness.

PMH: Negative
Allergies: NKDA
Medications: Oral contraceptives, Ibuprofen
FH: Significant for DM and HTN
SH: Cigarettes ~ ppd, alcohol: 3-4 beers/weekend
ROS: Headache, nausea

What do you look for on PE?

• The PE begins with a thorough multisystem evaluation


• The ABC's (airway, breathing and circulation) are addressed first
• She should be placed in a C-spine collar at this time if it had not been placed
in the field
• Following stabilization and evaluation ofall potentially life-threatening inju-
ries, a secondary survey should be performed with attention to her obvious
mJunes

360
Chapter 7- Cosmetic Facial Surgery Otolaryngology Clinical Case Studies

PE: NoTES

Her facial exam demonstrates multiple facial lacerations of the right fore-
head, temple, brow and eyelid (below). She has a jagged laceration through
the right brow with avulsion of the lateral upper lid with minimal tissue
loss. She does not have any bony step-offs or palpable deformities. She
does not have evidence of enophthalmos, hypothalamus, proptosis, deep-
ening of the upper eyelid sulcus, trismus or hypesthesia of CN V The fa-
cial nerve is intact bilaterally and there is no evidence of traumatic
telecanthus. Extraocular muscles are intact bilaterally and vision is grossly
intact. There is no evidence of a foreign body in either eye.

::<61
Otolaryngology Clinical Case Studies Chapter 7- Cosmetic Facial Surgery

NOTES What is your differential diagnosis?

• Right eyelid trauma with involvement of the right upper lid margin and pos-
sible injury to the eyelid retractors (levator palpebrae superi01is muscle, levator
aponeurosis and MUller's muscle)
• Possible bony orbital/facial injury
• Possible head/intracranial injury
• Possible C-spine injury

What diagnostics would you request?

• C-spine radiographs
• CT scan of the face/orbit (a,-xial scan with coronal reconstructions if per-
formed prior to clearance of the C-spine)
• Complete ophthalmologic exam, including a dilated fundus exam. Informa-
tion must be obtained regarding visual acuity, status of the globe, retina, ante-
rior and posterior vitreous chambers and extraocular muscles (the globe must
be evaluated prior to.treatment of the eyelid!)
• Evaluation of the eyelid and the periorbital structures

Test results:

• C-spine ./ilms demonstrate no abnormalities


• CT scan of the face/orbit reveals no fractures. The globes are imact
• Ophthalmologic examination reveals a normal cornea, globe, and retina.
Normal extraocular motion is noted in all fields of gaze

The primary function of the eyelids is to protect the globe. Multiple layers make
up the eyelids: skin, orbicularis oculi, septum orbitale, eyelid retractors, tarsal
plate and conjunctiva. The eyelid layers can be divided into an anterior and a
posterior lamella by separating the layers of the eyelid into 2 groups. The ante-
rior lamella consists of skin and orbicularis oculi muscle and the posterior lamella
consists of the eyelid retractors, tarsal plate and conjunctiva.

Skeletal support is maintained by the tarsal plates, which are composed of dense
connective tissue. The tarsal plates conform to the configuration of the globe
and keep the conjunctival mucosa in contact with the cornea. They also anchor
the lids to the orbit medially and laterally.

Elevation of the upper lid is controlled by the third CN through innervation of the
levator palpebrae muscle, which inserts into the anterior m~dportion of the tarsal
plates and the overlying skin forming the eyelid crease as it interdigitates with
the orbicularis oculi muscle. Levator muscle relaxation allows the orbicularis·
muscle to close the lids in a blink reflex and during sleep.

MUller's muscle originates from the superior orbit and attaches to the superior
margin of the tarsal plate and contributes approximately 2 mm of retraction to
the upper lid. The lower lid does not undergo retraction but does move during lid
closure. The septum orbitale connects the inferior tarsal margin and the infraor-
bital rim and stabilizes the eyelid during closure.

362
Chapter 7- Cosmetic Facial Surgery Otolaryngology Clinical Case Studies

Tears are secreted primarily by the lacrimal gland which is located in the upper NOTES
lateral orbital quadrant. The lacrimal puncta suction the tears from the cul-de-
sac by a "pumping" action ofthe orbicularis oculi acting through the decussation
of the medial canthal tendon anterior and posterior to the lacrimal sac. The
lacrimal sac is a mucosa lined structure and is located within the lacrimal fossa
anterior to the ethmoid bone. Tears exit the sac into the nasolacrimal duct and
into the inferior meatus, high under the inferior turbinate.

What would you do next?

• After confim1ation that no ocular or bony injuries exist, attention must be


directed at the patient's upper lid injury
• Upper lid lacerations require more immediate attention than lower lid lacera-
tions, since upper lid function is more essential in th.e protection of the globe
from desiccation, sun light and foreign objects
• Perform a thorough exploration of the wound to detem1ine ifthere has been
injury to the eye lid retractors or the lacrimal system. Injury to the upper
eyel id retractors may result in temporary or pem1anent ptosis. Upper eyelid
lacerations that run horizontally above the tarsal plate must be explored for
injury to the eyelid retractors. The presence of prolapsed fat is an important
indicator that the levator may be damaged. Primary identification and repair
of these injuries is preferable because secondary repair after fibrosis and
scarring is usually less successful
• Precise lower eyelid repair is necessary to prevent secondary sequelae such
as eyelid laxity, scleral show, epiphora, scar retraction and ectropion
• Medial canthal injuries may include the lacrimal apparatus. This may result
in disruption of tear flow and epiphora
• Lacerations involving the lacrimal canaliculus or lacrimal sac require identi-
fication and treatment. Repair may be facilitated by the use of surgicalloupes
or the operating microscope
• Medial canthal tendon injuries may result in blunting of the medial canthus
and traumatic telecanthus
• Lateral orbital injuries may involve the lacrimal gland or disruption of the·
lateral canthal tendon. Lateral canthal tendon injuries may result in blunting
of the lateral canthus and .loss of lateral suspensory support of the globe

Diagnosis:

Full-thickness eyelid laceration of the lid margin with minimal tissue loss
Lacerated levator aponeurosis and Miiller s muscle
Disrupted lateral canthal tendon
No evidence of injury to the globe based on ophthalmologic exam
No evidence of intracranial injury or facial fractures on radiographic
studies

Sykes and Dugan propose a graded approach to the treatment of eyelid lacera-
tions in which the type of repair is determined by the location of the injUJy and
the amount of tissue loss.

363
Otolaryngology Clinical Case Studies Chapter 7- Cosmetic Facial Surgery

NOTES • Partial thickness injuries can be closed primarily with local advancement'
flaps if tissue loss is present. If a large amount of tissue loss is present then
it may be necessary to perform a full-thickness skin graft. Ideal donor sites
for the skin graft include the contralateral upper eyelid skin, postauricular
skin, supraclavicular skin and preauricular skin
• Full-thiclmess injuries must first be explored to assess the status of the deeper
layers, including the conjunctiva, tarsal plate, eyelid retractors and septum
orbital e. It is important to restore each of the eyelid layers in order to ensure
the best reconstruction. The conjunctiva should be assessed for lacerations
and reapproximated if necessary with 6-0 fast absorbing sutures. Tarsal plate
injuries also require repair as this layer provides supp011 to the eyelids. Pro-
lapsed fat indicates that the septum orbitale has been violated;.however, this
layer should not be reapproximated because this may lead to future eyelid
retraction, lagophthalmos or ectropion. Muller's muscle and the levator apo-
neurosis must each be repaired to prevent ptosis. If the lid margin is involved
careful reapproximation is required to prevent postoperative eyelid notching.
The eyelid margin should be repaired with a non-absorbable 6-0 suture pass-
ing through the grey Iine, which lies between the anterior and posterior lamellae
. on the eyelid margin
• Full-thickness injuries with up to 25% tissue loss may be repaired primarily;
however, it may be necessary to perform a lateral canthotomy and either an
upper or lower cantholysis to mobilize the lid sufficiently to allow direct clo-
sure of the defect
• Full-thickness defects with loss of greater than 25% of the eyeiid usually
require local flaps. Useful flaps have been described by Tenzel, Cutler and
'Beard, Hughes and Mustarde
• Medial eyelid trauma should always raise the suspicion of an injury to the
medial canthal tendon or lacrimal apparatus. Medial canthal blunting, epi-
phora and traumatic telecanthus are signs of medial canthal disruption. In
repair of medial canthal injuries without associated bony trauma it is neces-
sary to reattach the medial canthal tendon to the posterior superior region of
the posterior lacrimal crest in order to move the lower lid upward and inward
to prevent epiphora ·
• Lacerations to the lacrimal apparatus must be repaired promptly as subse-
quent edema and scarring make future repair difficult. The distal and proxi-
mal ends of the transected canaliculus must be identified. Loupes or the
microscope may be necessary to locate the transected segments. Silastic or
silicon tubing is used to cannulate the upper and lower canaliculus. The tub-
ing is passed through the nasolacrimal duct and tied in the nose and left in
place for 4-6 weeks. Next, the microscope is utilized to perform an end-to-
end anastomosis using 9-0 nylon sutures to bring the canaliculus into align;
ment. Following the repair, 5-0 or 6-0 absorbable sutures are placed in the
surrounding connective tissue to relieve tension on the anastomosis

What would you do and why?

• The patient's right eyel id wounds are copiously irrigated with saline
• A dose of prophylactic N antibiotics (cefazolin) is given. The patient should
also be given a dose of tetanus toxoid

364
Chapter 7- Cosmetic Facial Surgery Otolaryngology Clinical Case Studies

• AII foreign bodies such as glass and gravel are meticulously cleared from the NoTES
wound and minimal tissue debridement is performed
• Photographic documentation is perfom1ed and informed consent is obtained
from the patient. A thorough exploration of the wound is performed and the
findings are documented in the patient's chart

The patient has a fiLII-thickness laceration of the upper eyelid involving


the lid margin with disruption of the conjunctiva, septum orbitale, upper
tarsal plate, eyelid retractors and lateral canthal tendon with minimal loss
of tissue. She also has a jagged laceration through her eyebrow and a
laceration of the temple.

What would you do next?

• Supraorbital and infraorbital nerve blocks are performed with 1% Lidocaine


with 1: I 00,000 epinephrine. Additionally a small amount oflocal anesthetic
may be injected into the wound edges
The· conjunctival laceration is attended to and 6-0 absorbable sutures are
placed in a running fashion with the knot buried so as to avoid irritation.ofthe
cornea
• A 5-0 Vicryl suture is passed in a circular vertical mattress fashion entirely
within the tarsus of the cut edge on either side, burying the knot at the tarsal
border. The suture is left untied until the lid margin sutures are placed
• Two additional 5-0 Vicryl sutures are placed in the tarsus in a similar fashion
• The disrupted edges of the levator aponeurosis are reapproximated to the
superior border of the tarsal .Plate with 6-0 silk sutures in an interrupted,
horizontal mattress fashion
• The levator palpebrae and MUller's muscles are identified and are each
reapproximated with 6.,.0 Vicryl sutures
• The septum orbitale is not reapproximated and it is important to avoid includ-
ing it in the other sutures placed
• The lid margin is repaired with a 6-0 silk suture placed at the mucocutaneous
junction approximately 2-3 mm from the wound edge through the eyelid
. margin. The·wound edges must be everted in order to prevent eyelid notching
• Additional sutures are placed in the grey line and the anterior lash line. These
sutures are tied after the tarsal Vicryl sutures are tied and the lid margin
suture ends are left long to be imbricated under a pretarsal skin silk suture
• The skin is closed with interrupted 6-0 Prolene sutures
• At the conclusion of the repair the eye is irrigated with saline solution and
antibiotic ophthalmic ointment is placed in the eye and over the lid suture
lines. The other incisions in the patient's brow and temple are now attended
to in the usual fashion

What are the potential complications?

• Lid margin notching:


Avoidance requires careful attention to lid margin alignment during initial
repa1r
- Minor notching often improves with time

365
Otolaryngology Clinical Case Studies Chapter 7- Cosmetic Facial Surgery

NOTES - Major notching may require a full-thickness pentagonal resection and ·


repair
• Lagophthalmos:
- Occurs from unrecognized tissue loss, scarring, or incorporation of the
seprum into the superficial wound repair
- The best treatment is prevention
- The skin shortage can be corrected by vertically lengthening the anterior
lamella (skin and musc le)
• Ectropion:
- Can be caused by skin shortage or by vertical traction on the lid margin
- For a minimal ectropion, upward massage of the lid may avoid the need·
for surgical intervention
- Surgical treatment includes full-thickness skin grafts combined with scar
release and horizontal tightening of the eyelid margin
• Hypertrophic scars:
- May improve spontaneously with time
- Can inject with steroids within the first 4-8 weeks after surgery
· • Infections:
- Occur rarely
- Orbital abscess or fistula formation typically indicates an unrecognized
foreign body, which must be removed before proper healing will occur
· • Epiphora (tearing):
- May be of multiple etiologies and each possibility must be investigated
- May be secondary to lid malposition, inadequate lacrimal pump or lacri-
mal drainage duct obstruction
• Traumatic ptosis: _
May be of neurogenic origin (injury to CN III) or myogenic origin (dam-
age to Miiller~s muscle)
- Horizontal wounds, especially if prolapsed fat is seen, are more likely to
have injury to Muller's muscle
- With patient cooperation, the muscle can be identified under local anes-
thesia by having the patient look up and down, and repaired.primarily
- Postoperative residual ptosis should be followed for 6-12 months for
evidence of spontaneous recovery
- There is an increased risk of overcorrection after repair of post-trau-
matic ptosis if good levator function is present

How would you follow this individual? .

• Immediately following the repair, ice packs should be gently placed on the
involved eye
• The head of the patient's bed should be elevated to 45 degrees
• Visual checks should be performed at regular intervals for the next 24 hours
• The skin sutures should be removed in 4-5 days postoperatively to minimize
scarring; however, lid margin sutures should be left in place for approxi-
mately I 0 days
• Following all suture removal the patient should be followed regularly for at
least I year to monitor for potential complications
• Photographic documentation should be obtained at each postoperative visit

366
Chapter 7 - Cosmetic Facial Surgery Otolaryngology Clinical Case Studies

References: NoTES

I. Ne lson CC. Management of eyelid trauma. Aust NZ J Ophthalmol.


I991; I9(4):357-63.
2. Sykes JM. Dugan FM. Evaluation and management of eyelid trauma. Facial
Plast Surg. 1994; 10(2): 157- 71.
3. Young WC, Holt GR, Holt JE. Eyelid trauma. Ear Nose Throat J. 1980;59:59-
72.
4. Tenzel RR, Stewart WB. Eyelid reconstruction by semicircular flap tech-
nique. Ophthalmology. 1978;85: 1164--9.
5. Cutler NL, Beard C. A method for partial and total upper lid reconstruction.
AmJOphthalmol.\955;39:1 - 7.
6. Hughes WL. Reconstruction of the lids. Am J Ophthalmol. 1945;28: 1203-
11.
7. Mustarde JC,. Major reconstruction of the eyelids-functional and aesthetic
c.onsiderations. Clin Plast Surg. 1981 ;8:227-36.
8. Jelks GW, Smith BC. Reconstruction of the eyelids and associated struc-
tures. In: McCarthy JG, ed. Plastic surgery, Vol. 2. Philadelphia: Saunders;
1990.p. 1671-781
9. Beadles KA, Lessner AM. Management of'traumatic eyelid lacerations.
Semin Ophthalmol. 1994;9: 145-51.

367
Otolaryngology Clinical Case Studies Chapter 7- Cosmetic Facial Surgery

NcnEs CAsE 4 - CoMPLEx LrP LAcERATioN


Joseph Leach, MD

A 5 year-old girl comes into rhe ER with a large lip laceration.

HPJ: state that you would:

• Obtain a detailed medical history beginning with a history of the present


illness

What additional historiCal information would you seek? State that you would ask:

• Were there any other injuries other than the ones to the face?
• Was the dog a family pet?
• Is the whereabouts of the dog known?
• How long ago did the accident occur?

Consider rabies. This is unlikely if the dog is a family pet. Has the dog been
acting strangely-foaming at the mouth, etc.? If yes, or if rabies has been re-
ported in area recently, dog should be quarantined. If rabies suspected, dog
should b~ euthanized and examined for rabies, and rabies prophylaxis should be
instituted in the child. ·

She was trying to kiss the neighbors dog when the dog bit her. This hap-
pened about an hour ago. She has been bleeding a lot, and her mother is
holding a bloody cloth over the mouth. S~e has no other known injuries.

PMH: Ask about immunizations. Tetanus is an issue. If not immu-


nized, she will need tetanus immune globulin; if immunized,
she will need a booster shot
Allergies: NKDA
Medications: None
FH: JYo history of bleeding or trouble with anesthetics
SH: NIC
ROS: N/C

What do you look for on PE? State that you would:

• Perform a complete head and neck PE including vital signs

PE:.

She is agitated and crying, and resists your attempts to examine wound.
The laceration appears to extend from the base of columella through the
vermilion and also extends through muscle and mucosa. You are unsure if
there is any missing tissue or not. All teeth appear to be intact.

368
Chapter 7- Cosmetic Facial Surgery Otolaryngology Clinical Case Studies

What diagnostics would you request? State that you would: NoTES

• N/C

What is your differential diagnosis?

• N/C

Diagnosis:

Full thickness laceration of upper lip without loss of tissue

What are the treatment options and their complications? What would you do
and why?

She should be taken to the OR for anesthesia and evaluation. There is some
surrounding crush injury, but all tissue appears to be viable

What are the surgical options?

• Primary repair with minimal debridement

Surgical technique:

• Cleaning:
- Copious amounts of saline are best
- Prep skin around wound with povidone iodine, but it is best not to get
iodine into the wound
- Chlorhexidine is acceptable within wound
• Use either no local anesthetic or a regional block in order to be able to align
vermilion border accurately
Photographs should be taken for documentation
• Use dissolving sutures for the mucosa and muscular layer
• Permanent sutures are usually used for skin, but in a child you might consider
tissue adhesives or dissolving sutures and surgical strips

Highlights: ·
• Complex lip repair in children is best performed under general anesthesia
• Layered c Iasure with exact alignment of vennillion is critical
• Eventual appearance of the scar will not be evident for 6 months to a year

Consider rabies in animal bite wounds and tetanus in all non-surgical


wounds. Local anesthesia may distort tissues.

What are potential complications?

• Scarring, tethering, and infection may occur


• Malalignment of the vennillion may necessitate revision surgery
• Give perioperative and postoperative antibiotics
• Some advocate delay of closure of bite wounds, but data is not convincing if
the wound is not grossly infected

369
Otolaryngology Clinical Case·Studies Chapter 7- Cosmetic Facial Surgery

NOTES How would you follow this individual'? State that you would:

• Recommend a liquid or soft diet as tolerated immediately


• Apply antibiotic ointment to the suture line twice a day
• Return in 1 week for suture removal
• Counsel regarding sun exposure
• Observe for scarring every 1- 2 months
• Reassure parents about redness and swelling as scar matures
• Possible remedies include steroid injections to scar (these are painfu l and
may require general anesthetic), laser treatment for redness, surgical de bulking
of scar tissue, si licone gel sheeting (difficu It to affix, steroid cream ). Derm-
abrasion may be option at 4-8 weeks as scar begins the remodeling phase

What are the phases of scarring?

• Inflammatory (< 4 days)


• Lag (4- 21 days)
• Remodeling (21 days-6 months)

What are the subunits of the upper lip?

• The lip consists of the philtrum (2 philtra! columns, the philtra! dimple and
Cupid's bow)
• The vermillion (vermillion border, wet and dry vermillion and the white roll),
and lateral lip. Relaxed skin tension lines run in a radial direction from lips
• Other areas of camouflage are the melolabial folds, the alar-fac ia l crease
and the nasal sill

What is the anatomy of the upper lip?

• It is composed ofthree layers-the skin, the orbicularis muscle and the mucosa
• A branch of CNV2 provides sensation and motor input is from a buccal
branch of CNVII
• The labial artery is a branch of the facial artery: The artery and nerve enter
deep to the muscle

370
Chapter 7- Cosmetic Facial Surgery Otolaryngology Clinical Case Studies

CASE 5- BLEPHAROPLASTY NoTES


Joseph Leach, MD

A 75-year old woman presenc.s complaining of difficulty seeing out of her


eyes. She states that her field of view is restricted and that her eyes feel
tired and heavy because of "bags ,. in the upper lids. She also thinks that
her eyes make her look older than she really is.

HPI: state that you would:

• Obtain a detailed medical history beginning with a history of the present


illness

What additional historical information would you seek?

• Regarding the eyes, you would like to know if there is any dryness or tearing
- Resection of eyelid skin can make these problems worse after surgery
• Ask about history of glaucoma and cataracts and get an idea of what the
visual acuity is
- Eyelid surgery should not make these problems worse, but an angry
patient might blame you should these problems be discovered later

PMH: She has a history of Type II DM and mild HTN


Allergies: NKDA
Medications: She is on a diuretic and an oral anti-diabetic agent. She uses
aspirin only occasionally. She has ne_ver had a problem with
prolonged bleeding. Her blood sugars and BP_ have been
under good control
FH: No problem with anesthetics o~ prolonged bleeding
SH: There is no tobacco history and ETOH consumption is mini-
mal. She is a widow and a homemaker
ROS: She heals well and forms inconspicuous scars

What would you look for on }>E? State that you would:

• Perform a complete head and neck PE including vital signs


• Pay particular attention to brow position, lid ptosis, symmetry, fat herniation,
globe position, lid laxity, ectropion and scleral show; all are important to evaluate
preoperatively

PE:

This patient is noted to have a symmetrically ptotic brow, with normally set
eyes. There is no eyelid p tosis. There is redundant skin and herniated fat in
both upper and lower lids, but no evidence of undue lower lid laxity, ectro-
pion or scleral show. Her hairline is normal

371
Otolaryngology Clinical Case Studies Chapter 7 - Cosmetic Facial Surgery

NoTES The noll11al brow position is at or above the superior orbital rim. In a male, the
brow is straight, but in a female. the brow arches laterally, peaking between the
lateral limbus and the lateral canthus. The limbus is the junction of the iris and
sclera. When considering a browlift one should assess the thickness of the hair
and the position ofthe hairline. When the brow is manually positioned at an appro-
priate level, the need for upper eyelid skin resection may be reduced or eliminated.

In this position, one should also assess lid ptosis, which would manifest as the lid
margin descending over the pupil in forward gaze. Lid ptosis should be mea-
sured and repaired at the time of blepharoplasty. Often, there is underlying
asymmetry, which should be brought to the patient's attention prior to surgery.
Fat herniation should be distinguished from skin redundancy. Laterally, a ptotic
lacrimal gland may masquerade as hern iated fat. Persons with deep-set eyes
require less skin excision than those with protruding eyes, and they also tend to
get a startled or "hollowed out" appearance if too much fat is removed. Lid
laxity is assessed by a "snap test" whereby the lower lid is pulled anteriorly and
allowed to relax. If the lid does not immediately return to the globe or if it can be
pulled more than 8-1 0 mm forward, laxity is present. Such patients are at risk
for ectropion and scleral show after lower eyelid surgery.

Ectropion is an eversion ofthe lower lid. In these cases, the lacrimal punctum
does not approximate well to the globe, and tearing ensues. Scleral show occurs ·
when sclera is apparent between the lower limbus and lower lid margin. Lower
lid laxity makes one tend to opt for either transconjunctivallower blepharoplasty ·
(which has Jess association with ectropion and scleral show) or suspension of
the lateral canthus. Large, redundant bulges in. the .lower eyelid are known as
"festoons." A bulging medial fat pad may cause a crease between the lower lid ·
and nasal skin kno:wn as a "tear trough" deformity. Lower lid wrinkling may be
due to redundant skin, but may also be due to overactivity of the underlying
muscle, requiring limited muscle excision. ·

What diagnostics would you request? State that you would:

• Take photographs, including views of the brow.elevated, th~ eyes squinting,


the eyes closed and the eyes in relaxed forward gaze
Formal visual field testing is helpful for insurance purposes
• A formal ophthalmologic exam is indicated if there are any uncertainties
about the condition ofthe eyes

Diagnosis:

There is brow ptosis wUh red.J,indant skin and herniated fat in both upper
and lower lids. •

What are the treatment options and their complications? What would you do
and why?

• The patient would likely-benefit from a brow lift, an upper blepharoplasty and
a lower blepharoplasty
• Some patients refuse the brow lift and opt for an upper blepharoplasty alone.

372
Chapter 7- Cosmetic Facial Surgery Otolaryngology Clinical Case Studies

In many cases, this wi ll give an acceptable cosmetic result and should help NoTEs
with the visual fields
• With brow lift, the options in terms of approach are coronal, direct, mid fore-
head, pretrichial or endoscopic
- The coronal approach hides the scar behind the hairline, but may lengthen
the forehead and numb the scalp posterior to the incision. Dissection is
carried out in the subgaleal plane
The direct approach is made right above the brows, while the mid fore-
head approach is via a prominent forehead crease. Both are best suited
to older men with receding hairlines
The pretrichial incision is made at the hairline and dissection is carried
out in the subcutaneous plane. This avoids damage to the sensory nerves
ofthe scalp
Endoscopic brow lift is cnrried out via vertical incisions in the scalp that
avoid transection of the sensory nerves. Skin is not resected, but rather
repositioned posteriorly and superiorly and fixated with sutures, pins or
screws

Surgical technique: ·

• Upper blepharoplasty is usually perforn1ed by excising a fusiform patch of


redundant skin and orbicularis muscle from the upper lid. The lower edge of
the excision is the supratarsal crease. Slight overcorrection is acceptable in
the upper lid, but aggressive removal of skin may cause lagophthalmos. The
incision may extend laterally into a crow's foot, but avoid making the incision ·
medially onto the nasal skin, or webbing will occur. The orbital septum is then
opened, and herniated fat is teased out centrally and medially. The fat is then
clamped, trimmed and the stumps cauterized. Ptotic, herniated lacrimal gland
may be clamped and resected and cauterized laterally. Care must be taken
to avoid the levator aponeurosis
• Lower blepharoplasty may be performed via a subciliary or transconjunctival
incision. The fom1er is generally more effective for removing hypertrophied
muscle, but the latter limits formation of ectropion and scleral show. Fat
removal is carried out in a similar way as upper blepharoplasty. Skin removal
should be more conservative in lower blepharoplasty to avoid lid retraction.
Transconjunctival incisions are either not closed or closed with a single, fine,
dissolving suture with a buried knot. Skin closure is accomplished in I layer
with fine sun1re
• Cold compresses are placed over the eyes intermittently for about 48 hours.
Strenuous activity should be avoided for about 10 days. Contact lenses may
be worn after about a week

Highlights:
• Carefully evaluate for symmetry as the operation proceeds
• Careful hemostasis is critical
• Generally, the upper lid is overcorrected, while the lower lid is undercorrected

Inadvertent cutting of the levator aponeurosis may result in lid ptosis. A subciliary
incision in a patient with lower lid laxity will result in scleral show and ectropion
unless a lid tightening procedure is perforn1ed.

373
Otolaryngology Clinical Case Studies Chapter 7- Cosmetic Facial Surgery

NoTES What are potential complications?

• Lagophthalmos, ectropion and scleral show may occur, but shou ld be avoided
using the techniques discussed above
• All 3 are helped by massage postoperatively
• Minute injections of corticosteroids may be helpful to interrupt scarring
• In severe cases, grafting with skin, cartilage and/or mucosa may be necessary
• Expanding orbital hematoma is an infrequent but dire complication that can
result in blindness. This manifests as a stony hard, proptotic globe with pain
and ecchymosis, possible limitation of gaze and fai ling vision. Treatment in-
cludes lateral canthotomy, evacuation of clot, massage, and stat ophthalmo-
logic consult

How would you follow this individual?

• If pennanent sutures are used, they should be removed 5-7 days after surgery
• Patients should be evaluated for healing, symmetry, visual function and cos-
metic result
• Postoperative photographs should be taken
• Final results are apparent about 2- 3 weeks later

374
Chapter 7- Cosmetic Facial Surgery Otolaryngology Clinical Case Studies

CASE 6- RHYTIDECTOMY NoTEs


Allison Pontius, MD

A 55 year-old female presents to your office seeking rejuvenation of her


facial appearance. She is especially concerned over her facial rhytides
and jowls.

HPI: state that you would:

• Seek to understand the motivating factors behind the patient's request for a
change in her appearance. A patient undergoing a major life change or loss is
not an ideal candidate for cosmetic surgery
• Evaluate the patient's FH to determine the likely speed at which loss of
elasticity of the tissues and the overall aging process is occurring
• Review past history of cosmetic surgical procedures
• Discuss lifestyle and habits, including sun exposure and smoking
• Take acomplete medical history with a discussion of any conditions that may
preclude facelift surgery such as advanced autoimmune disease of the face
or neck, unstable cardiovascular disease, or a full course of radiation to the
face or neck
. • Focus on a history of any bleeding problems, as hematoma is the most com-
mon complication following rhytidectomy
• Review the patient's medications, allergies, previous operations, history of
alcohol or substance abuse, as well as any other medical problems
• Perform a PE, then have a discussion of the patient's candidacy for the
procedure

PMH: Type II DM (controlled with diet and medication), asthma


Allergies: Penicillin .
iV!edications: Glucophage 100 mg bid, aspirin 80 mg q day, albuterol in-
haler, calcium, vitamin C, vitamin E, ginkgo biloba
FH: COR artery disease, DM
SH:. Nonsmoker, drinks socially (2-3 drinks/weekend)
ROS: Negative

The patient states she has noticed a deepening of her wrinkles, especially
the formation ofjowling, over the past 5-l 0 years despite the fact that she
limits her sun exposure and uses sunscreen daily. She has discussed her
concerns over her appearance with her husband who supports her deci-
sion to seek intervention. She has been contemplating facial rejuvenation
surgery for the past few years, but wanted to wait until her children left for
college before pursuing treatment.

What do you look for on PE? State that you would:

• Assess appearance in relationship to her stated age


• Assess signs of photodamage
• Analyze face in thirds, assessing the upper third for ptosis and transverse
creases; the middle third for symmet1y and drooping of soft tissues and

375
Otolaryngology Clinical Case Studies Chapter 7- Cosmetic Facial Surgery

NOTES prominence of the nasolabial fo lds; and the lower third for jowls, jawline, and ·
position of submandibular glands
• Analyze the cervico-mental angle for platysma) banding; this can be better
assessed through having the patient voluntari ly contract the muscle
• Analyze neck for evidence of liposis

PE:

The patient appears her chronological age with mild photodamage and
moderately thick skin. She is thin with a symmetric face with normal facial
function and sensation. The patient :S face should be analyzed in thirds.
The forehead is examined for ptosis and transverse wrinkles. The glabel-
lar area is assessed for transverse and vertical wrinkles. The position of
the eyebrows in relation to the supraorbital ridge is important in determin-
ing whether the patient would also benefit from a brow lift. The mid-face
malar area is assessed for symmetry and drooping of the soft tissues to
assess the need for a midface lift. The prominence of the nasolabial folds
is noted. In the ·lower third of the face it is important to assess the jowls
and jawline as well as any prominence of submandibular glands. The
cervico-mental angle is ·measured and the degree of platysma! banding is
determined by having the patient voluntarily contract the muscle. If there
is a significant amount of supraplatysmal fat present, it may be necessary
to pelform suction assisted lipectomy of the submental area in qddition to
the rhytidectomy.

What is your differential diagnosis?

• Skin laxity
• Jowling
• Rhytides secondary to the aging process and sun exposure

_Diagnosis:

The patient is a good candidate for rhytidectomy. She has appropriate


motivations for surgery and has the support of her family. She has ad-
equate skin elasticity despite having evidence of photodamage. Her jowls
and skin laxity can be effectively treated by rhytidectomy. She does not
have any true contraindications for facelift surgery, such as advanced
autoimmune disease involving the skin of the face, unstable cardiovascu-
lar disease, or a history of a full-course of radiation therapy to the preau-
ricular region or infra-auricular neck. DM in itself is not a contraindica-

tion for surgery. She should be started on a skin care regimen for improve-
ment of the solar damage prior to surgery.

Further assessment and discussion. State that you would:

• Have an honest discussion of what the patient can expect from surgery if
that should follow

376
Chapter 7- Cosmetic Facial Surgery Otolaryngology Clinical Case Studies

• Computer-imaging is an ideal way to show the patient realistic changes she NoTES
can expect from surgery. If computer-imaging is not available, the results
should be demonstrated on the patient in a 3-way mirror
• Obtain medical photodocumentation of the CUITent condition of the patient's
face, neck, and jowls. Photos should include the full-face frontal, full-face
left and right oblique views. and full-face left and right lateral views (all
taken in the Frankfort horizontal line). Additionally, close up pictures of each
auricle with the hair pulled back (without earrings) are taken
• Answer all questions prior to leaving the consultation room, and the entire
procedure should be discussed with its alternatives, risks, and limitations
disclosed
• Financial implications of surgery may be discussed with the office business
manager
• Pre-operative medical exam. If this patient has decided to pursue surgery,
she should have a complete PE by her primary physician. She should have
optimal diabetic and asthma control and a cardiovascular evaluation. She
should be counseled to stop taking aspirin at least 3 weeks prior to surgery.
Additionally, she must stop taking the herbal remedy gink&o biloba and vita-
minE as these also inhibit blood coagulation. She should have an ECG, CXR,
liver functimi tests, renal function tests, electrolyte test, a CBC and coagula-
tion studies prior to surgery .

What are the different surgical techniques?

• Skin-only rhytidectomy:
- This involves subcutaneous undermining of tl~e skin on the cheek and
neck
- This type of face lift is subject to early recurrence of previous problems
as the underlying structures are not tightened
• Skin-and-SMAS rhytidectomy:
- The cheek is undermined as in a skin-only face lift
The SMAS layer is then dissected as a separate flap
- The extent of the dissection under the SMAS varies and may proceed as
far medially as the nasolabial crease
- The zygomatic and masseteric retaining ligaments are released to in-
crease the mobility of the SMAS flap
- The dissected SMAS layer is tightened and secured anterior to the ear
• Deep-plane rhytidectomy:
- Dissection of the cheek is performed under the SMAS layer
- The layer between the SMAS and the skin is not dissected
The composite flap of cheek skin and SMAS is tightened and secured
- May be indicated in heavy smokers, as the undermined cheek and SMAS
flap is thicker and less prone to necrosis
• Mid-face suspension:
- The deep tissues of the mid-face are dissected through a lower blepharo-
plasty or temporal incision
- A suture is placed through the soft-tissue of the cheek and passed up to
the temple
- The mid-face is elevated by tightening the suture and securing it to the
true temporal is fascia

377
Otolaryngology Clinical Case Studies Chapter 7 - Cosmetic Facial Surgery

NOTES • Non-endoscop ic, subperiosteal rhytidectomy:


- The soft tissues of the face are dissected in a subperiosteal plane through
a number of open incisions
• Endoscopic rhytidectomy:
- The face is dissected endoscopical ly at a subperiosteal level
- Once freed from its bony attachments, the face can be elevated and
secured

Surgical technique:

• Design the incision - The face lift incision is marked in the temporal area
around the temporal tuft of hair. The incision continues inferiorl y past the
root of the helix down to the natural crease in the pre-auricular area. The
incision can also be placed posterior to the tragus to avoid any obvious scars.
The incision is continued around the earlobe and then posteriorly onto the
surface of the concha about 2 mm above the post-auricular sulcus. Final ly
the incision is extended along the hairline in the mastoid area
• Local anesthetic- To achieve hemostasis the face is infiltrated with Lidocaine
1% solution with epinephrine 1:200,000
• Elevate skin and SMAS flaps. Begin the diss~ction in a subcutaneous plane
and then address the SMAS as a separate layer. The dissection in the subcu-
taneous plane is begun with a-scalpel and then continued with face-lift scis-
sors with the aid of a·fiberoptic Iighted retractor. The flap ·is elevated across
the malar area,_ inferiorly to the cervical area and superiorly to within 1 em of
the lateral orbital rim. The SivlAS of the face is continuous with the temporo-
parietal fascia superiorly and with the platysma inferiorly.
Depending on the degree of conection that is required at the level of the
cheek, jawline and neck, the SMAS/platysma layer can be simply plicated or
elevated laterally and advanced. The lateral elevation of the SMAS/platysma
flap is begun by first identifying the posterior edge of the platysma muscle
just anterior to the sternocleidomastoid muscle. The platysma is elevated
taking care not to injure .the underlying external jugular vein and great auricu-
lar nerve. The SMAS is dissected first by making a transverse incision at the
level of the inferior border of the zygomatic arch. This incision is continued
with a vertical incision at the pre-tragal level, passing posterior to the man-
dibular angle connecting with the incision made at the platysma muscle.
The SMAS dissection can be canied as far as the anterior edge of the
parotid gland. The buccal branches as well as the marginal mandibul ar
branches of Vll can often be seen as this dissection progresses and should
be carefully preserved. If the dissection is continued to the zygomaticus
major muscle, it is tem1ed an extended SMAS dissection. Once the SMAS/
platysma layer has been elevated, it can be advanced along a cephalo-poste-
rior vector with or without resection of the lateral edge ofthe flap . The flap
is secured in its new position with white braided polyester sutures. This lift
recontours the jawline and lower third of the face
• Address platysma banding - If there is significant platysma banding in the
anterior cervical area, a midline plication of the platysma muscles is per-
formed via a submental incision. A 2-3 em incision is made j ust posterior to
the submental crease and dissection with scissors exposes the platysma

378
Chapter 7- Cosmetic Facial Surgery Otolaryngology Clinical Case Studies

muscles which are then plicated with a running 4:0 Mersilene suture. The NoTEs
platysma muscles can also be transected distal ly to further deepen the cervico-
mental angle. At this time if there is a significant amount of cervical fat.
suction assisted I ipectomy can be used to further contour the submental area
• Re-drape the tlaps - The cervicofacial skin flap is then also redraped in a
cephaloposterior direction similar to the SMAS flap. lt is anchored at 2 points
of maximal tension, anteriorly above the root ofthe helix and posteriorly at
the superior aspect of the postauricu lar incision. In this way there is abso-
lutely no tension at the incision in the preauricular area
• Excise excess skin - The excess skin. in the temporal and mastoid areas is
excised and the skin edges approximated and sutured. In the preauricular
area the excess skin is also excised taking great care not to create any
tension at that level; this is especially true if the initial incision was made
behind the tragus
• Suction draii_ls - A #7 flat Jackson Pratt drain is brought out at the mastoid
level and the incisions are then closed
• Close the incisions- A running 3:0 nylon is used in the mastoid and postauricular
incisions and a running 5:0 nylon in the preauricular area

What are the treatment options and their complications? What would you do
and why?

• Hematoma:
- # 1 complication following rhytidectomy
- Major hematomas requiring re-operation usually occur in the first 12
h-ours post-operatively
- Hematoma formation is heralded by pain and increasing facial edema
- Predisposing factors for hematoma formation include: history ofHTN,
males, rough emergence from anesthesia, postoperative·nausea and :vom-
iting, ingestion of aspirin, NSAIDS, high doses ofvitamin E, Ehlers-Danlos
syndrome
- Delay in treatment can lead to flap necrosis or infection
- Minor hematomas are usually s·een in the first post-operative-week
- Treatment for minor hematomas includes evacuation with an J 8-gauge
needle after liquification, pressure dressing, and antibiotics
- Undetected hematomas can lead to fibrosis, skin puckering and discol-
oration
• Flap necrosis:
- Occurs when blood supply to the distal ends of the random skin flap is
compromised
- Predisposi11g conditions mclude poor flap design, extended subcutaneous
flap elevation, injury to the subdermal plexus, extensive closing tension,
certain systemic medical conditions, and smoking
- The postauricular area followed by the preauricular area is at greatest
risk
Flap necrosis is preceded by venous congestion and flap discoloration-
at this stage frequent massage and an extended course of antibiotics are
instituted
- Treatment of the resultant eschar includes daily peroxide clean ing, lim-
ited debridement and topical antibiotic ointment

379
Otolaryngology Clinical Case Studies Chapter 7- Cosmetic Facial Surgery

NOTES • Nerve damage:


- The great aUJicular nerve is the most common nerve injury in facelift
surgery
- If injury is noted during surgery, the ends should be anastomosed with
epineural repair using 9-0 nylon
- The 2 most common motor nerve injuries are the marginal mandibular
branch and the temporal branch of the facial nerve
- The majority of facial nerve injuries (85%) resolve with time as the
injuries are usually secondary to neuropraxia
- If there is no return of function by 1 year, reconstructive options shou ld
be considered, including brow lift or eyelid reanimation
• Hypertrophic scarring:
- Occurs secondary to excessive tension placed on flap closure
- Most commonly associated with isolated subcutaneous f1ap dissection
- Appears 2-12 weeks post-operatively
- Interval intralesional injections with steroids may help
- Delay excision and primary closure of the hypertrophic scar for at least
6 months
• Incision line irregularities:
- Poor incision planning can lead to loss of temporal tufts ofhair, alopecia,
dog-ear formation, and hair-line stair-stepping pattern
- Hair loss is usually secondary to follicular shock (telogen effluvium), and
reassurance may be given to the patient
- lfthere is no hair recovery by 3-6 months, the alopecia may be excised
and closed primarily with or without the use of micrografts
• Infection:
- Occurs rarely
Treat mild cellulitis with an extended course of antibiotics covering Sta-
phylococcus and Streptococc~ts .
- Abscesses should be treated with incision and drainage, culture and pos-
sibly TV antibiotics
• Earlobe deformity:
- Satyr's ear (devil's ear) can occur if tension is placed on the earlobe at
skin closure
- During the healing period the earlobe is pulled down inferiorly
- The inferior lobe sulcus can be recreated by V-Y plasty; however, this
should be delayed for at least 6 months
• Parotid injUJy:
- Occurs rarely
- Intraoperative injury to the parotid parenchyma should be oversewn with
available SMAS to prevent sialocele or fistula formation
- Postoperative fluid accumulation can be treated with needle aspiration
and pressure dressing

How would you follow this individual?

• The patient should be examined the morning after surgery for drain and
dressing removal
• The patient retums at 1 week for suture removal

380
Chapter 7- Cosmetic Facial Surgery Otolaryngology Clinical Case Studies

• At 2-3 weeks postoperatively the patient returns for re-examination and NoTES
consultation with an esthetician for a skin-care and make-up lesson
• It is imperative to see the patient at I month, 3 months, 6 months, and I year
postoperatively to follow the results

References:

1. Aston S, Beasley R, Thorn C., eds. Grabb and Smith plastic surgery, 5th ed.
Boston : Little Brown; 1997,633---49.
2. Perkins S, Dayan S. Rhytidectomy. ln: Papal 10, editor. Facial plastic and
reconstructive surgery. New York: Thieme; 2002, 153-70.
3. Richards AM. Keynotes on plastic surgery. Oxford UK: Blackwell; 2002,.
296-8.

381
Otolaryngology Clinical Case Studies Chapter 7- Cosmetic Facial Surgery

NoTEs CASE 7 - NASAL SEPTAL REcoNSTRUCTION (SEPTOPLASTY)


Joseph Leach, lVID

A 27 year-old man presents with difficulty breathing out of the nose.

HPI: state that you would:

• Obtain a detailed medical history beginning with a history of the present


illness

What additional historical information would you seek?

• On which side is the obstruction worse?


• Any history of epistaxis?
• ts there any aberration in smell?
• Any history of snoring?
• Does he use any topical decongestants?
• Has he used cocaine in the past?
• Does he have a history suggestive of sinusitis?

The patient states that:

His obstruction is worse out of the left side


• He has occasional 'nosebleeds· out of that side
• His sense of smell is normal
• He snores at night. but awakes well-rested
• He denies a history of sinusitis or allergic rhinitis
• He admits to using OTC nasal sprays
• There is no history of cocaine abuse

Plv!H: Remote trauma occurred - possibly playing basketball.


Allergies: NKDA
Medications: None (other than nasal spray). .
SH: Tobacco - smokes 1 ppd for 10 years,
alcohol - 2 beers/week, software engineer
ROS: NIC

What would you look for ·on PE? State you would:

• Perform a complete head and neck PE including vital signs

PE:

Nose - The septum is deviated to left side with an inferior spw: There is a
75% obstruction on that side. The valves are normal. The turbinates are
red and boggy, but normal in size. No polyps are present. The external
nasal exam is normal. The mucosa is red and dry. The remainder of the
head and neck examination is WNL

382
Chapter 7- Cosmetic Facial Surgery Otolaryngology Clinical Case Studies

What is your differential diagnosis? NOTES

• Deviated septum
• Nasal valve collapse
• Turbinate hypertrophy
• Allergic rhinitis
• Nasal polyposis
• Rhinitis medicamentosa

What diagnostics would you request?

o Radiographs: None

What is the normal angle of the internal nasal valve?

o I 0 degrees

Where is the internal nasal valve?

• It is bounded by the septum and the caudal edge of the upper lateral cartilage

Diagnosis: ·

Deviated septum with rhinitis medicamentosa

. What are the treatment options and their complications? What would you do
next and why?

• Counsel the patient about discontinuing the spray. If surgery is performed


under these conditions, bleeding is more problematic and healing is prolonged
• Oral decongestants, steroid nasal sprays, intranasal steroid injections or a
short course of oral steroids may help the patient wean offvasoconstricting
nasal sprays

Findings of rhinitis medicamentosa include dryness, crusting, bleeding, erosion


of the mucosa and occasionally destruction of the cartilage with perforation
and/or nasal collapse.

What are the surgical options?

• Septoplasty

Surgical technique:

• General or local anesthesia may be used


• Pack the nose with oxymetazoline containing pledgets
• Inject with lidocaine containing a vasoconstrictor
• Killian or hemitransfixion incisions may be used. A Killian incision is more
posterior and can be placed directly over the deviation, but there is a greater

383
Otolaryngology Clinical Case Studies Chapter 7- Cosmetic Facial Surgery

NoTES propensity to tear the flap. The hemitranstixion incision requires somewhat
more dissection, but is easier to close. Be sure to dissect deep to the peri-
chondrium
• Carefully raise mucoperiosteal/mucoperichondrial t1aps on one or both sides
• Remove and/or straighten deviated bone and cartilage. Take care not to
have apposing mucosal tears
• Leave behind an adequate (about 1 em wide) dorsal and caudal carti laginous
strut
• Close with a transseptal suture. Use chromic gut for the incision
• Splints are optional, but have been shown to reduce incidence of re-deviation
• Packing is generally unnecessaty unless bleeding is profuse

Highlights:
• Use a head light and several nasal specula of varying sizes
o Oxymetazoline packs and a vasoconstrictor in the injection help hemostasis
• Mucosal tears are common and may prevent a hematoma

Avoid removing cartilage along the caudal or dorsal border. Avoid apposing
mucosal tears. Pulling deviated portions of the perpendicular plate of the eth-
moid out may inadvertently cause fractures of the cribrif01m plate and cause a
CSF leak.

How do you treat apposing mucosal tears if they occur intraoperatively?


. ~ . ....,7,..·
• Repair with running chromic suture
• · Place a sheet of gelatin film between mucosal layers in the area of tears

What are potential complications?

Saddle nose
• Septal perforation
• Re-deviation
• Some patients have transient upper teeth or palatal pain/numbness as a re-
sult of removing deviated portions of the maxillary crest

How would you follow this individual? State that you would:

• Prescribe postoperative antibiotics and pain medicines


• Nasal saline spray helps
• Petrolatum can prevent build up of scabs around nostrils
• A drip pad may be necessary 2-4 days
• Remove splints within 3-1 0 days
• Clean crusts and scabs under topical anesthesia
• The patient should avoid heavy lifting for 2 weeks
• Follow-up after a month to be sure that the septum remains straight, no
perforations have occurred and the external appearance of the nose has not
changed, and that the patient is breathing well

384
Chapter 7- Cosmetic Facial Surgery Otolaryngology Clinical Case Studies

AAO-HNS clinical indications for septoplasty: NOTES

• History-1 or more required:


- Nasal airway obstruction or difficult nasal breathing causing any of the
fo llowing: mouth breathing; snoring; sleep apnea; or recutTent sinus in-
fection
- Frequent nosebleeds
- Atypical fac ial pain of nasal origin. Positive response to topical anes-
thetic; where deformed septum contacts a turbinate supports but may
not prove septal cause
- Asymptomatic deformity that prevents surgical access to other intrana-
sal or paranasal areas, e.g., sinuses, turbinates
• PE-al! appropriate findings required:
- Description of complete intranasal exam
- Document absence of nasal polyps, tumors, turbinate hypertrophy or
other causes of obstruction
- Identification ofknown or suspected bleeding site ifthe purpose of sur-
gery is. to control epistaxis
- Identification of sinus that is recurrently infected if the purpose of sur-
gery is to control disease
- Description ofNP, OP, hypopharynx and larynx if purpose of surgery is
to treat sleep apnea or snoring
• Tests........:..optional:
- Nasal impedance (rhinometry)
- Rhinomanometry
• Nasal endoscopy
• Postoperative observations:
- Bleeding?
- Pain? .
- Packing or internal splint optional -Is it in desired location?
• Outcome review:
- I week:
• Healing- Did patient require treatment for bleeding, septal hematoma,
or infection?
- Beyond l month:
• Presenting problem -Is it (see history) improved?
• Status of septum- Is septal configuration desirable? Is there a perfo-
ration?

385
Otolaryngology Clinical Case Studies Chapter 7- Cosmetic Facial Surgery

NOTES CASE 8- RHINOPLASTY


Joseph Leach, lYID

A 59 year-old woman presents to your clinic complaining ofdifficulty breath-


ing out of the nose.

HPJ: state that you would:

• Obtain a detailed medical history beginning with a history of the present


illness

What additional historical information would you seel<'?

• Any history of recent or remote trauma?


• Does she have a normal sense of smell?

• Yesterday, she slipped and struck her nose on a railing


• Lost her sense of smell at the time of injury. (Caution -Anosmia may be
attributed to surgery if documentation not performed prior to surgery.)
• Given that she has admitted to a recent injury, ask her to rate her pain
on scale of 1-10

PMH: TIAs, no strokes or heart disease


Allergies: NK.l_)A
Medications: Aspirin
FH: Negative for bleeding abnormalities, no risks for general
anesthesia
SH: No tobacco or alcohol, retired homemaker
ROS: Negative

What would you look for on PE? State that you would:

• Perform a complete head and neck PE including vital signs. (These are
normal)

PE:

The ears, eyes, mouth, throat and neck are normal, but the nose is too
bloody and tender to examine accurately.

What are the treatment options and their complications? What would you do 9

next?

• Spray lidocaine/vasoconstrictor into nose, gently suction out clots, and use
adequate lighting
• Look for a septal hematoma. Probe gently with cotton tipped applicator- is
there ballotable fluid?
• Look for widening of the septum. If a septal hematoma is missed, this may
evolve into a septal abscess which can destroy the carti Iage of the nose and
result in a saddle nose deformity

386
Chapter 7 - Cosmetic Facial Surgery Otolaryngology Clinical Case Studi es

What diagnostics would you request? State that you would request: NoTES
• Radiographs: Plain films show a displaced fracture of nasal pyramid

Test results:

There is no trauma to the orbits or frontal bone. Radiographs are no!


essential to diagnosing a nasal fracture, however

What else would you lil<e to do?

• Document presence of sense of smell - if materials aren ·t available, try a


cup of coffee or a benzoin swab
• Take photographs

What is your differential diagnosis?

• Nasal·fracture
• A nasa-orbital-ethmoid ffacture has been effectively ruled out

There is displacement, but no widening of the septum. There is crepitus and


palpable step-offs in the area of the nasal dorsum.

What are the surgical options?

• Closed reduction ofthe fracture should be attempted. Closed reduction has


about a 60-70% chance of fixing the nose without need for secondary rhino-
plasty. This is best performed within 48-72 hours. She should understand
that there is a somewhat higher risk ofbleeding due to her aspirin intake, but
this will probably not be a major concern since closed reduction is performed
without making incisions
• Some patients may opt ·for more definitive surgery later, even though .it is
more involved, prolonged and expensive

Surgical technique:

• Under general or local anesthesia, a blunt instrument is inserted into nose.


The bones are manipulated into position, and a cast is applied
• Some advocate using general anesthesia so that septoplasty with fracture of
the anterior portion of the perpendicular plate and/or osteotomies of the na-
sal bones may be carried out ·

What are potential complications?

• Re-deviation. Often this is secondary to inadequate reduction that is not


apparent due to swelling in the acute phase
• Saddle nose is possible if the fracture is severely comminuted and the splint-
ing is inadequate

387
Otolaryngology Clinical Case Studies Chapter 7- Cosmetic Facial Surgery

NOTES How would you follow this individual? State that you would:

• Remove the cast in 7-10 days


• You should assess for asymmetry, ability to breathe and sense of smell. In
this case, the patient remains unable to breathe out of the right side, and is
concerned about persistent asymmetry of the nose

There is a 75% obstruction of the airway on the right. Examine the nasal
valves. There should be 10-degree angle at the internal nasal valve, which
is where the caudal border of the upper lateral cartilage and the septum
intersect. Her valves are normal.

What do you do next? State that yGu would:

• Take more photographs


• Discuss rhinoplasty with the patient

Rhinoplasty is best performed when the initial healing is complete and the
swelling has resolved- typically in about 6 weeks. Ask the patient if there
are any other changes she would like made with her nose. This patient
wants to remove a hump in the bridge of the nose, and would also like a
· more upturned appearance to the tip. Since incisions will be made with the
rhinoplasty, you should check with her primary doctor to make sure she
can be off aspirin for about 10 days.

Surgical technique:

• General or local anesthesia may be used


• Septoplasty may be perfonned endonasally or via the open approach
• Hump removal consists of recuding the cartilaginous and bony dorsum using
scalpels, chisels and rasps. This usually leaves an open roof defonnity, re-
quiring that the nasal bones be infractured
• Elevation of the tip is accomplished by resecting parts of the lateral crura (if
the tip is too projected) or by increasing the length of the medial crura (using
struts, sutures or a "steal" technique where the dome is moved laterally)
• After the incisions are closed, splints may be placed and an external cast
applied

Nasal fracture is best diagnosed by palpation of crepitus and deformities. Ra-


diographs are confirmatory only. Closed reduction of nasal fractures is usually
effective, but should be carried out within 48-72 hours for best results. Evaluate
sense of smell and condition of the nasal valves.

Septal hematoma, if missed, may lead to abscess formation and collapse ofthe
nose. Closed reduction may not be sufficient, and formal rhinoplasty or septorhino-
plasty may be necessary later.

388
Chapter 7- Cosmetic Facial Surgery Otolaryngology Clinical Case Studies

How would you follow this individual? State that you would: NoTES

• Remove the cast in 7-10 days


• Assess for asymmetry, ability to breathe and sense of smel l
• Examine the nasal valves and the external appearance
• Make sure any packing and any permanent skin suture is removed

AAO-HNS clinical indications for rhinoplasty:

• History-! or more required:


- Obstruction of breathing (functional)
- Unsatisfactory appearance
- Nasal injury (trauma) causing unsatisfactory breathing and/or appear-
ance
- Nasal birth defect impairing nasal function
- Acquired deformity due to trauma, tumor or infection
• · PE--'-all required:
- External nasal anatomy including description of deformity
Internal nasal anatomy including description of septum and inferior turbi-
nates
- Patency of nasal passages including estimate of percentage o~struction
for each side
• Tests-required and dated within 3 months of surgery:
- Preoperative photographs
• Postoperative observations:
- Skin edema and ecchymosis - Documented observation
- Nasal packing in desired location or removed?
- Pain- Does patient require discharge prescription?
- Bleeding - How managed? Sur~eon infonned?
• Outcome review:
- I week:
• Healing - Did patient require treat~ent for bleeding, infection or ob-
struction?
• Healing of graft donor site when appropriate
- Beyond 1 month:
• Appearance -Appraisal of result comparing postoperative and pre-
operative photographs.
• Airway - Are nasal passages patent?
• Healing of graft donor site when appropriate
• Pain

389

Potrebbero piacerti anche